385

Deja review _emergency_medicine__deja_review_

Embed Size (px)

DESCRIPTION

 

Citation preview

Page 1: Deja review _emergency_medicine__deja_review_
Page 2: Deja review _emergency_medicine__deja_review_

DEJA REVIEWTM

Emergency Medicine

Page 3: Deja review _emergency_medicine__deja_review_

NOTICE

Medicine is an ever-changing science. As new research andclinical experience broaden our knowledge, changes in treat-ment and drug therapy are required. The authors and thepublisher of this work have checked with sources believed tobe reliable in their efforts to provide information that is com-plete and generally in accord with the standards accepted atthe time of publication. However, in view of the possibilityof human error or changes in medical sciences, neither theauthors nor the publisher nor any other party who has beeninvolved in the preparation or publication of this work war-rants that the information contained herein is in every respectaccurate or complete, and they disclaim all responsibility forany errors or omissions or for the results obtained from use ofthe information contained in this work. Readers are encour-aged to confirm the information contained herein with othersources. For example and in particular, readers are advised tocheck the product information sheet included in the packageof each drug they plan to administer to be certain that theinformation contained in this work is accurate and that changeshave not been made in the recommended dose or in thecontraindications for administration. This recommenda-tion is of particular importance in connection with new orinfrequently used drugs.

Page 4: Deja review _emergency_medicine__deja_review_

DEJA REVIEWTM

Emergency Medicine

David H. Jang, MDResident Physician

University of Pittsburgh Affiliated Residency in Emergency Medicine Pittsburgh, Pennsylvania

Tufts University School of MedicineClass 2006

New York Chicago San Francisco Lisbon London Madrid Mexico CityMilan New Delhi San Juan Seoul Singapore Sydney Toronto

Page 5: Deja review _emergency_medicine__deja_review_

Copyright © 2008 by The McGraw-Hill Companies, Inc. All rights reserved. Manufacturedin the United States of America. Except as permitted under the United States Copyright Actof 1976, no part of this publication may be reproduced or distributed in any form or by anymeans, or stored in a database or retrieval system, without the prior written permission of thepublisher.

0-07-159371-3

The material in this eBook also appears in the print version of this title: 0-07-147625-3.

All trademarks are trademarks of their respective owners. Rather than put a trademark sym-bol after every occurrence of a trademarked name, we use names in an editorial fashion only,and to the benefit of the trademark owner, with no intention of infringement of the trademark.Where such designations appear in this book, they have been printed with initial caps.

McGraw-Hill eBooks are available at special quantity discounts to use as premiums and salespromotions, or for use in corporate training programs. For more information, please contactGeorge Hoare, Special Sales, at [email protected] or (212) 904-4069.

TERMS OF USE

This is a copyrighted work and The McGraw-Hill Companies, Inc. (“McGraw-Hill”) and itslicensors reserve all rights in and to the work. Use of this work is subject to these terms.Except as permitted under the Copyright Act of 1976 and the right to store and retrieve onecopy of the work, you may not decompile, disassemble, reverse engineer, reproduce, modify,create derivative works based upon, transmit, distribute, disseminate, sell, publish or subli-cense the work or any part of it without McGraw-Hill’s prior consent. You may use the workfor your own noncommercial and personal use; any other use of the work is strictly prohibit-ed. Your right to use the work may be terminated if you fail to comply with these terms.

THE WORK IS PROVIDED “AS IS.” McGRAW-HILL AND ITS LICENSORS MAKE NOGUARANTEES OR WARRANTIES AS TO THE ACCURACY, ADEQUACY OR COM-PLETENESS OF OR RESULTS TO BE OBTAINED FROM USING THE WORK,INCLUDING ANY INFORMATION THAT CAN BE ACCESSED THROUGH THEWORK VIA HYPERLINK OR OTHERWISE, AND EXPRESSLY DISCLAIM ANY WAR-RANTY, EXPRESS OR IMPLIED, INCLUDING BUT NOT LIMITED TO IMPLIEDWARRANTIES OF MERCHANTABILITY OR FITNESS FOR A PARTICULAR PUR-POSE. McGraw-Hill and its licensors do not warrant or guarantee that the functions containedin the work will meet your requirements or that its operation will be uninterrupted or errorfree. Neither McGraw-Hill nor its licensors shall be liable to you or anyone else for any inac-curacy, error or omission, regardless of cause, in the work or for any damages resulting there-from. McGraw-Hill has no responsibility for the content of any information accessed throughthe work. Under no circumstances shall McGraw-Hill and/or its licensors be liable for anyindirect, incidental, special, punitive, consequential or similar damages that result from theuse of or inability to use the work, even if any of them has been advised of the possibility ofsuch damages. This limitation of liability shall apply to any claim or cause whatsoeverwhether such claim or cause arises in contract, tort or otherwise.

DOI: 10.1036/0071476253

Page 6: Deja review _emergency_medicine__deja_review_

We hope you enjoy thisMcGraw-Hill eBook! If

you’d like more information about this book,its author, or related books and websites,please click here.

Professional

Want to learn more?

Page 7: Deja review _emergency_medicine__deja_review_

Contributors xiAcknowledgments xiiFaculty Reviewers xiiiStudent Reviewers xiv

Chapter 1 INTRODUCTION TO EMERGENCY MEDICINE 1Emergency Medical Service / 1Airway Management / 4Shock / 7Fluids / 12Electrolytes / 13

Chapter 2 NEUROLOGIC EMERGENCIES 25Headaches / 25Seizures / 28Meningitis / 31Cerebral Vascular Accident / 33Vertigo / 37Peripheral Neurologic Lesions / 38Lower Back Pain / 41Syncope / 43Clinical Vignettes / 45

Chapter 3 OPHTHALMOLOGIC EMERGENCIES 47Basic Ophthalmology / 47Trauma of the Eye / 48Infections of the Eye / 51Acute Visual Loss / 54Clinical Vignettes / 57

Chapter 4 ENT AND DENTAL EMERGENCIES 59Acute Otitis Media / 59Otitis Externa (Swimmer’s Ear) / 60Acute Hearing Loss / 61Nasal / 62ENT Infections / 65Dental Emergencies / 73Clinical Vignettes / 75

Contents

For more information about this title, click here

Page 8: Deja review _emergency_medicine__deja_review_

vi Contents

Chapter 5 PULMONARY EMERGENCIES 77Pneumonia / 77Asthma / 79Chronic Obstructive Pulmonary Disease / 80Hemoptysis / 81Pleural Effusion and Empyema / 82Lung Abscess / 83Tuberculosis / 84Spontaneous Pneumothorax / 85Clinical Vignettes / 86

Chapter 6 CARDIOVASCULAR EMERGENCIES 89Acute Coronary Syndrome / 89Congestive Heart Disease and Pulmonary Edema / 93Deep Venous Thrombosis and Pulmonary Embolism / 95Cardiomyopathies / 98Endocarditis / 101Myocarditis / 103Pericardial Disease / 104Valvular Disease / 106Thoracic Aortic Dissection / 112Abdominal Aortic Aneurysms / 114Hypertensive Urgencies and Emergencies / 115Clinical Vignettes / 116

Chapter 7 GASTROINTESTINAL EMERGENCIES 119Esophagus / 119Gastrointestinal Bleeding / 123Peptic Ulcer Disease / 125Appendicitis / 126Gallbladder Disease / 128Pancreatitis / 129Colitis and Ileitis / 131Mesenteric Ischemia / 133Diverticular Disease / 134Hernia / 136Anorectal / 137Diarrhea / 141Clinical Vignettes / 143

Chapter 8 GENITOURINARY EMERGENCIES 145Acute Renal Failure / 145Chronic Renal Failure / 147Nephrolithiasis / 148Urinary Tract Infections / 149

Page 9: Deja review _emergency_medicine__deja_review_

Male Genital Problems / 150Clinical Vignettes / 153

Chapter 9 ENDOCRINE EMERGENCIES 157Hypoglycemia / 157Diabetic Ketoacidosis / 158Thyroid / 159Adrenal / 161Clinical Vignettes / 162

Chapter 10 HEMATOLOGY AND ONCOLOGY EMERGENCIES 165Hematology / 165Oncology / 170Clinical Vignettes / 174

Chapter 11 INFECTIOUS DISEASES 177Influenza and Herpes Viruses / 177HIV/AIDS / 182Sexually Transmitted Diseases / 185Malaria / 187Soft Tissue Infections / 189Gas Gangrene / 191Toxic Shock Syndrome / 194Occupational Postexposure Prophylaxis / 195Infectious Disease Appendices / 196

Chapter 12 PEDIATRIC EMERGENCIES 197High-Yield Pediatric Charts / 197Cardiopulmonary Resuscitation / 198Neonatal/Infant-Specific Conditions / 201Congenital Heart Disease / 205Airway Emergencies / 206Pediatric Gastrointestinal / 211Infectious Disease / 215Child Abuse / 221Clinical Vignettes / 222

Chapter 13 OBSTETRICS AND GYNECOLOGY 225Normal Pregnancy / 225Vaginal Bleeding in Reproductive Women (nonpregnant) / 226Pelvic/Abdominal Pain in NonPregnant Women / 227Ectopic Pregnancy / 228Emergencies during Early Pregnancy / 229Emergencies during Later Pregnancy / 232Emergencies during Postpartum / 234

Contents vii

Page 10: Deja review _emergency_medicine__deja_review_

Vulvovaginitis / 235Pelvic Inflammatory Disease / 236Clinical Vignettes / 237

Chapter 14 TRAUMA 239General Approach / 239Head Injury / 241Neck Trauma / 247Bony Oral-Maxillofacial Injury / 248Spinal Trauma / 251Thoracic Trauma / 254Abdominal Trauma / 259Genitourinary Trauma / 262Orthopedic trauma / 263Trauma in Pregnancy / 267Clinical Vignettes / 268

Chapter 15 ENVIRONMENTAL EXPOSURES 271Burns / 271Electrical, Lightning, and Chemical Injuries / 273Near-Drowning / 276Hypothermia / 277Hyperthermia / 278Altitude Sickness / 280Diving Injuries / 281Bites / 283Rabies / 286Tetanus / 287Insect Bites / 289Clinical Vignettes / 293

Chapter 16 TOXICOLOGICAL EMERGENCIES 297General Approach / 297Over-the-Counter Drugs / 299Prescription Medications / 304Psychiatric Medications / 310Neuroleptics / 314Drugs of Abuse / 316Metals, Chemicals, and Gases / 324Toxicology Supplement / 333

Chapter 17 BEHAVIORAL EMERGENCIES 335Medical Evaluation and Clinical Approach / 335Depression and Suicide / 337Acute Psychosis / 339

viii Contents

Page 11: Deja review _emergency_medicine__deja_review_

Mania / 340Panic Attacks / 341Eating Disorders / 341Dementia and Delirium / 343Intoxication and Withdrawal / 344Psychopharmacology / 345

Index 349

Contents ix

Page 12: Deja review _emergency_medicine__deja_review_

This page intentionally left blank

Page 13: Deja review _emergency_medicine__deja_review_

Contributors

Peter Adler, MDResident PhysicianUniversity of Pittsburgh Affiliated Residency

in Emergency MedicinePittsburgh, PennsylvaniaUniversity of North Carolina School

of Medicine Chapel Hill, NC

Amy E. Betz, MDResident PhysicianUniversity of Pittsburgh Affiliated Residency

in Emergency MedicinePittsburgh, PennsylvaniaUniversity of Pittsburgh Medical

SchoolPittsburgh, PennsylvaniaClass of 2006

Jason Biggs, MDResident PhysicianUniversity of Pittsburgh Affiliated Residency

in Emergency MedicinePennsylvania Jefferson Medical

CollegePhiladelphia, PennsylvaniaClass of 2006

Lawrence Y. Ho, MDResident PhysicianDepartment of OphthalmologyUniversity of PittsburghPittsburgh, PennsylvaniaUniversity of Pittsburgh Medical SchoolPittsburgh, PennsylvaniaClass of 2004

Regan Nichole Ladenburger, MDResident PhysicianUniversity of Pittsburgh Affiliated Residency

in Emergency MedicinePittsburgh, PennsylvaniaUniversity of Kentucky College of Medicine Lexington, Kentucky

Alan Lo, MDResident PhysicianUniversity of Pittsburgh Affiliated Residency

in Emergency MedicinePittsburgh, Pennsylvania

Adam Z. Tobias, MDResident PhysicianUniversity of Pittsburgh Affiliated Residency

in Emergency MedicinePittsburgh, Pennsylvania

Copyright © 2008 by The McGraw-Hill Companies, Inc. Click here for terms of use.

Page 14: Deja review _emergency_medicine__deja_review_

Acknowledgments

To the love of my life Min Young for her support and patience with all things in life;the residents and attendings at The University of Pittsburgh Affiliated Residency inEmergency Medicine, who all have been excellent and continue to push me to betterhelp others.

Copyright © 2008 by The McGraw-Hill Companies, Inc. Click here for terms of use.

Page 15: Deja review _emergency_medicine__deja_review_

Daniel E. Brooks, MDAssistant Professor of Medical Toxicology

and Emergency MedicineDirectorMedical Toxicology ServiceCo-Medical DirectorPittsburgh Poison CenterUniversity of Pittsburgh Medical CenterPittsburgh, Pennsylvania

Robert Frank, MDClinical Assistant ProfessorDepartment of Emergency MedicineUniversity of Pittsburgh School of MedicineAttending Physician Department of Emergency MedicineMercy HospitalPittsburgh, Pennsylvania

Scott Gunn, MDMedical DirectorSurgical Trauma Intensive Care Unit Department of Critical Care Medicine University of Pittsburgh Medical Center

Richard Kaplan, MDClinical Assistant ProfessorDepartment of Emergency MedicineUniversity of Pittsburgh School of MedicineAttending PhysicianDepartment of Emergency MedicineWestern Pennsylvania HospitalPittsburgh, Pennsylvania

Christopher King, MD, FACEPAssociate Professor of Emergency Medicine

and PediatricsDepartment of Emergency MedicineUniversity of Pittsburgh School of MedicineUniversity of Pittsburgh Medical CenterPittsburgh, Pennsylvania

Patrick C. Lee, MDTrauma Service ChiefBaystate Medical Center: Tufts UniversitySchool of MedicineSpringfield, Massachusetts

Thomas P. Martin, MDClinical Assistant ProfessorDepartment of Emergency MedicineUniversity of Pittsburgh School of MedicineAttending Physician Department of Emergency MedicineWestern Pennsylvania HospitalPittsburgh, Pennsylvania

Anthony F. Pizon, MDAssistant Professor of Medical Toxicology

and Emergency MedicineUniversity of Pittsburgh School of MedicinePittsburgh Poison CenterUniversity of Pittsburgh Medical CenterPittsburgh, Pennsylvania

Henry Wang, MD, MPHAssistant ProfessorDepartment of Emergency MedicineUniversity of Pittsburgh School of MedicinePittsburgh, Pennsylvania

Faculty Reviewers

Copyright © 2008 by The McGraw-Hill Companies, Inc. Click here for terms of use.

Page 16: Deja review _emergency_medicine__deja_review_

Michael W. Fill, DOResidentDepartment of Emergency MedicineBotsford General HospitalFarmington Hills, Michigan

David LazarClass of 2007Baylor College of MedicineHouston, Texas

Student Reviewers

Copyright © 2008 by The McGraw-Hill Companies, Inc. Click here for terms of use.

Page 17: Deja review _emergency_medicine__deja_review_

1

C H A P T E R 1

Introduction toEmergency Medicine

EMERGENCY MEDICAL SERVICE

What was the significance of the Highway This act authorized the DepartmentSafety Act of 1966 to the development of of Transportation to provideEmergency Medical Service (EMS) in the funding for improvement of United States? ambulance service and prehospital

provider training, as well as thedevelopment of highway safetyprograms and EMS standards

Which paper helped to bring about the Accidental Death and Disability: TheHighway Safety Act of 1966? Neglected Disease of Modern Society,

which highlighted the dangerous conditions of emergency care in the United States

What important advancements in EMS Development of a curriculum,occurred in First National Conference certification process, and nationalon EMS in 1969? register for the emergency medical

technician (EMT)—ambulance

In what year was EMT recognized as an 1972occupational specialty by the Departmentof Labor?

What are the 15 elements of an EMS 1. Personnelsystem as defined by the Emergency 2. TrainingMedical Services Act of 1973? 3. Communications

4. Transportation5. Facilities6. Critical care units7. Public safety agencies8. Consumer participation9. Access to care

10. Transfer to care

Copyright © 2008 by The McGraw-Hill Companies, Inc. Click here for terms of use.

Page 18: Deja review _emergency_medicine__deja_review_

11. Standardization of patient’srecords

12. Public information and education13. Independent review and

evaluation14. Disaster linkage15. Mutual aid agreement

What are five types of EMS service 1. Public Service (often provided bysystems? the fire department)

2. Hospital-based 3. “Third Service” model, usually a

separate division of the local government

4. “Public Utility” model (a privateambulance company)

5. Volunteer model

What determines which service system is The type of EMS system depends appropriate for a given community? on the needs and the resources

of the community

What are the two general categories of 1. Basic life support (BLS)care provided by EMS systems? 2. Advanced cardiac life support

(ACLS)

What are the three main methods of 1. Ground transportation—ambulancepatient transport? 2. Rotary-wing transportation—

helicopter3. Fixed Wing transportation

What is the average cost of an ambulance $180–$600transport?

Can a patient refuse EMS treatment and/or Yes. A competent, conscious adulttransport? patient may refuse treatment or

transport, but he/she must beinformed of risks when refusing

What are the four levels of EMS training First responder. Requiresand some specific skill sets? approximately 60 classroom hours of

training: Initial scene and patientevaluation; Cardiopulmonaryresucitation (CPR); Basic airwayskills; Hemorrhage control; Spinalimmobilization

EMT—Basic. Requires 100 classroomhours as well as 10 clinical hours:Skills of the first responder; Triageand patient assessment; AED use;Assist patient in taking medications

2 Deja Review: Emergency Medicine

Page 19: Deja review _emergency_medicine__deja_review_

EMT—Intermediate. Requires300–400 hours of classroom andclinical training: Advanced patientassessment; Intravenous lineplacement; Manual defibrillation;Administration of a limited numberof medications

EMT—Paramedic. Requires1000–1200 hours of classroomtraining plus a clinical internship:Electrocardiogram (ECG)interpretation; Needledecompression of a tensionpneumothorax; Needle and surgicalcricothrotomy; Transthoracic cardiacpacing; Administration of selectedmedications

What advancement in communication 911significantly improved the public’s accessto emergency medical services?

What are the responsibilities of the Answering, triaging, and prioritizingEmergency Medical Dispatcher? all calls; Alerting and dispatching

the appropriate unit; Providing pre-arrival instructions

Define unique characteristics of an EMS The EMS medical director is a medical director. physician with a special interest in

and knowledge of the patient care needs in the prehospital environment

What is the definition of a disaster? A disaster is an incident that overwh-elms the response capacities of a com-munity. This occurs when the numberor severity of patients presenting tothe emergency medical responsesystem in a given period of timeoverwhelms the available resources

What are the three phases of a disaster plan? Activation. This includes the initialresponse by EMS and the organizationof an incident command center

Implementation. Componentsinclude search and res-cue, triage,and transport of patients

Recovery.

What are the two phases of a disaster 1. Prehospitaloperation? 2. Hospital

Introduction to Emergency Medicine 3

Page 20: Deja review _emergency_medicine__deja_review_

List the components of the prehospital Triage of patients; Scene control;phase of a disaster operation. Communications; Public health

considerations

The second phase of a disaster operation 1. Development of a central controlfocuses on hospital preparedness. List the centersix components that must be considered 2. Activation of the disaster planduring this phase. 3. Designation of treatment areas

4. Organization of documentation5. Mobilization of security 6. Designation of waiting areas

What is the definition of triage? Triage is the process of classifyinginjured patients into groups accordingto the priority for treatment. The goalis to do the most good for the largestnumber of potential survivors

Describe the four patient triage categories. Dead or unsalvageable.

Critical. These patients are severelyinjured but salvageable and requireimmediate medical attention

Serious. These patients have noimmediate life-threatening injuries

Minor. Often these patients arereferred to as the walking wounded

AIRWAY MANAGEMENT

4 Deja Review: Emergency Medicine

Sedative/Induction Dosage Onset Duration

Etomidate 0.3–0.4 mg/kg 1 minute 3–6 minutesFentanyl 0.5–2 ug/kg 1 minute 30–60 minutesKetamine 1–2 mg/kg 1 minute 15 minutesMidazolam 0.2–0.4 mg/kg 30 seconds 15–20 minutesPropofol 1–2 mg/kg 30 seconds 3–5 minutes

NeuromuscularBlocking Agents Dosage Onset Duration

Succinylcholine 1–2 mg/kg IV <1 minute 3–5 minutes2–4 mg/kg IM

Rocuronium 0.6–1.2 mg/kg <1 minute >30 minutesVecuronium 0.1–0.2 mg/kg 2–3 minutes 30–60 minutes

IV, intravenous; IM, intramuscular.

Page 21: Deja review _emergency_medicine__deja_review_

What are some reasons a patient may need Oxygenation; Prevention/overcomingairway management? airway obstruction; Protection against

aspiration; Assisted ventilation

What are some important things to do Inspect the patient’s mouth (teeth,prior to any airway procedure (assuming palate, tongue, and oropharynx); Askpatient is not in imminent danger)? patient history if possible; Assess for

possible cervical neck injury; Listen forany airway problems (i.e., stridor);Suction any secretions prior toprocedures

List some important causes of Foreign bodies; Trauma (expandingairway obstruction. hematoma); Infections (epiglottis);

Congenital (enlarged airway)

What is the most feared complication Hypoxia; Brain damageof inability to secure an airway in a timelyfashion?What are some important points for each ofthe following airway devices:

Oral airway C-shaped rigid instrument placedinto the mouth; Placed to create apatent airway; Used to preventtongue from falling posterior; Usedin patients with no gag reflex

Nasal airway A nasopharyngeal tube placed into anostril; Typically used to bypassobstructing tongue; Used insomnolent patient with a gag reflex

Bag-valve-mask (BVM) The mainstay of airway management;Inflating bag with a nonrebreathingvalve; Critical to airway management;Two-person use is optimal to avoidair leak

Esophageal tracheal combitube (ETC) Plastic twin-lumen tube withinflatable cuffs; Placed blindly intothe oropharynx; Commonly used asa backup airway to ETT; Typicallyused in the prehospital setting

Laryngeal mask airway (LMA) Tubular oropharyngeal airway;Contains a distal laryngeal mask;Inserted blindly into the oropharynx;Commonly used as backup airway toETT

Endotracheal tube (ETT) Cuffed-tube placed into the larynx;Placed normally by directlaryngoscopy; Considered the goldstandard

Introduction to Emergency Medicine 5

Page 22: Deja review _emergency_medicine__deja_review_

What is rapid sequence intubation (RSI)? The use of special drugs to rapidlysedate and paralyze a patient toallow ETT placement

What is the primary reason for RSI? It allows optimal conditions tosecure an airway

What is a disadvantage of RSI? If unable to intubate, it can result incomplete loss of airway control

What is the most important thing to Always have a back-up airway keep in mind while performing RSI? ready! If ETT fails, the patient will

not be able to breathe (paralytics still in effect)

What are the seven P’s of RSI? Prepare. Have different sized tubesand blades ready; Ensure cuff works;Have back-up airway ready (i.e., LMA)

Preoxygenate. Preoxygenate for about2—5 minutes; Hypoxia develops fasterin children and pregnant women

Position. Flexion of lower neck;Extension at the atlantooccipital joint;This allows direct visualization of thelarynx; Bad positioning commonreason for failure

Premedicate (induction). Induce adeep level of unconsciousness; Agentdepends on situation; Alwayspremedicate prior to paralyzation

Paralyze. Administer neuromuscularblocking agent; Succinylcholine mostpreferred agent

Place ETT. Visualization of vocalcords is critical

Placement confirmation. Look fortube condensation; Bilateral chestrise; Auscultate stomach and lung;Capnography

What is the Sellick’s maneuver? The application of cricoid pressure tohelp prevent aspiration as well as aidin direct visualization of vocal cords

What are some reasons that succinylcholine Rapid onset of action (<45 seconds);is used most often in RSI? Brief duration of action (<7 minutes)

What are some adverse side effects to keep Increases intraocular/intracranial in mind about the use of succinylcholine? pressure; Avoid in hyperkalemic

states (i.e., burns); In rare cases cancause malignant hyperthermia

6 Deja Review: Emergency Medicine

Page 23: Deja review _emergency_medicine__deja_review_

List some alternative paralytics that can be Rocuronium (fast onset, but longer used if succinylcholine is contraindicated. duration of action); Vecuronium

(even longer duration of action)

What are some important points for thefollowing alternative methods:

Cricothyrotomy The primary surgical backup airway;Placement of trach/ETT throughsurgical incision in neck and cricoidmembrane; Contraindicated inchildren < 8

Tracheotomy Longer to perform thencricothyrotomy; Preferred inchildren; Also used in patients withtracheal injury

Digital intubation Index/middle fingers to palpate epiglottis; Typically used in comatosepatients; Success rate is lower thenthat of RSI

Retrograde intubation Use of a guide wire via the cricoid;The guide wire guides the tube viathe cord; Not commonly used inprehospital setting

SHOCK

What is the definition of shock? It is a clinical syndrome that ischaracterized by the body’s inabilityto meet the demands of tissue/organperfusion resulting in decreasedvenous oxygen content and lacticacidosis

What is the initial step in management for Airwayany patient who presents with possible Breathingshock?

Circulation

What are four categories of shock? 1. Cardiogenic2. Hypovolemic3. Distributive4. Obstructive

What are some of the autonomic responses Increase in heart rate (HR) and that occur with shock? contractility of heart; Constriction of

venous capacity vessels; Arteriolarvasoconstriction; Release of vasoactivehormones; Activation of renin-angiotensin system

Introduction to Emergency Medicine 7

Page 24: Deja review _emergency_medicine__deja_review_

What are some important vasoactive Dopamine; Norepinephrine;hormones that are released during a state Epinephrine; Cortisolof shock?

What are two critical organs that the 1. Brainautonomic system tries to preserve blood 2. Heartflow to?

What are some common metabolic Metabolic acidosis (lactic acidosis);derangements that occur with shock? Hyponatremia; Hyperkalemia;

Prerenal azotemia

What are some important elements from Medications (i.e., anaphylaxis); Historythe history that should be considered? of heart disease; History of volume

depletion (i.e., emesis); Neurologicdisease

What is an important way to assess shock Hemodynamic monitoringas well as evaluate therapeutic intervention?

What are some components of ECG; Pulse OX; Central venoushemodynamic monitoring? pressure (CVP)

Name some important early interventions Airway control (intubate if necessary);to consider in a state of shock. Mechanical ventilation (decreases

work of breathing); Aggressive fluidresuscitation; Ensure oxygen delivery(pressors if needed)

What is an important distinction to make Hemorrhagic versus in hypovolemic shock? nonhemorrhagic

What are some important causes of Burns; Gastrointestinal (GI) relatednonhemorrhagic hypovolemia? such as emesis and diarrhea;

Excessive urination (renal saltwasting)

What is the normal circulating volume of 5 Lblood in a normal adult?

What portion of that is plasma and what 3 L of plasma; 2 L of RBCis RBC?

What is the hallmark response for eachof the following categories of hemorrhage:

Class I (about 750 mL) Usually no response in a healthy person

Class II (750–1500 mL) Tachycardia and narrowed pulse pressure; Mild hypotension; Mildchange in mental status

Class III (>1500 mL) Tachycardia and pronounced hypotension; Decline in mentalstatus; Peripheral hypoperfusion

8 Deja Review: Emergency Medicine

Page 25: Deja review _emergency_medicine__deja_review_

Class IV (>2 L) Hemodynamic decompensation is common; Aggressive resuscitation isrequired

What is important to know about children They compensate very well (no and athletes who have acute hemorrhage? tachycardia or hypotension), but can

decompensate very fast soon after,without any warnings

What other etiologies of hypotension Myocardial infarction; Tensionbesides hemorrhage should be considered pneumothorax; Cardiac tamponade;in the setting of trauma? Toxicologic involvement

What is the initial step in management that Airway (ensure patent airway);should be undertaken with acute Breathing (proper ventilation);hemorrhage? Circulation (two large-bore IVs for

fluids)

Why are two large-bore IVs more effective Infusion rate of fluids is muchthan long narrow IVs? faster through short wide tubes

What are commonly used large-bore IVs? 13- or 14-gauge needles

What are some commonly used Isotonic crystalloids; Colloids; Bloodresuscitation fluids?

Name two commonly used isotonic 1. Normal saline (NS)crystalloid fluids used for resuscitation. 2. Lactated ringers (LR)

What are some concerns when large Increased neutrophil activation; LRamounts of isotonic crystalloid fluids may cause lactic acidosis; NS mayare used? cause hyperchloremic acidosis

What are the general guidelines for the Minimal response to 2–3 L of fluids;infusion of blood? Obvious major loss of blood;

Hematocrit of < 16

What are some concerns whenever blood Transfusion reaction; Availability;is given? Infections; Limited storage life

What are some types of blood given? Whole blood; Packed red blood cells(PRBC); Fresh-frozen plasma (FFP);Platelets

What is a concern if too much fluid is Dilutional coagulopathygiven during a resuscitation?

What are some important things to know 50% mortality of those who developabout sepsis? shock; Gram -/+ often common cause

of sepsis; Sepsis is more common inolder adults

What are the most frequent sites of Genitourinary tract; Abdomen; Lunginfection that can lead to sepsis?

What are co-morbid conditions that can Burns; Diabetes mellitus;predispose one to sepsis? Immunosuppressive agents

Introduction to Emergency Medicine 9

Page 26: Deja review _emergency_medicine__deja_review_

What is the definition of bacteremia? Presence of bacteria in the blood

Name the criteria of systemic inflammatory Temperature: <36°C or >38°C;response syndrome (two or more must Tachycardia: >90 beats/min;be met). Tachypnea: >20 breaths/min; WBC:

>12000, <4000 or >10% bands

What is sepsis? Systemic response to infection thatmeet the criteria for systemic inflam-matory response syndrome (SIRS)

What is septic shock? Hypotension with inadequate organperfusion induced by sepsis with another metabolic dysfunction suchas lactic acidosis

What are some possible clinical features ofsepsis in the following organ system:

Respiratory Adult respiratory distress system (ARDS); Pneumonia

Cardiovascular Myocardial depression and tachycardia; Poor response to fluid administration

Renal Acute renal failure due to renal ischemia; Oliguria

Hepatic Cholestatic jaundice; Elevated liverfunction tests (LFTs); Elevatedbilirubin

Endocrine Hyperglycemia is common; Elevatedcortisol and glucagon; Insulinresistance and decreased insulin

Hematology Neutrophilia or neutropenia;Thrombocytopenia; Disseminatedintravascular coagulation (DIC)

What are some key points in the ABCs (aggressive fluid resuscitation);management of sepsis? Not atypical for patients to require

>6 L; Inotropes (i.e., DA) if notresponsive to fluids; Empiric Abx isthe cornerstone; Remove the sourceof infection

What is the definition of cardiogenic Inadequate tissue perfusion due to shock? decrease in cardiac output despite

adequate circulating volume

What is the most common cause of Myocardial infarctioncardiogenic shock?

10 Deja Review: Emergency Medicine

Page 27: Deja review _emergency_medicine__deja_review_

What are some other causes of cardiogenic Mechanical obstruction; Rightshock to be considered? ventricular infarct; Sepsis; Myocarditis

What are some clinical features of Evidence of volume overload (i.e., cardiogenic shock? rales), hypotension, mental status

change, cool/clammy skin, diaphoresis, and jugular venous distension (JVD)

What are some important points for eachof the diagnostic tests commonly used toevaluate cardiogenic shock:

ECG Cornerstone test to diagnose ischemia;Can also detect arrhythmias, drugtoxicity, and electrolyte derangements;Also to detect right ventricular infarct

Chest X-ray (CXR) Commonly show pulmonary edema/effusion; R/O other diseasestates such as a dissection; Normalchest does not rule out shock

Echocardiography Used to assess left-ventricular (LV)function; Color flow Doppler canassess mechanical cause such asvalvular disease; Not typically usedin the emergent setting

What are some laboratory tests to consider Cardiac enzyme; Brain natriureticin cardiogenic shock? peptide (BNP); Arterial blood gas;

Serum lactate

What is the definition of anaphylaxis? Severe hypersensitivity reaction withmultisystem involvement that commonly include airway compromise and hypotension

What is a hypersensitivity reaction? Inappropriate immune response to an antigen

What is an anaphylactoid reaction? Reaction that presents similar to anaphylaxis, but is not IgE mediated and does not require priorsensitization

List some common causes of anaphylactoid Radiocontrast dye; Opiates;reactions. Muscular depolarizing agents

What are the top three causes of serious 1. Medicationanaphylactic reactions? 2. Foods

3. Insects

What are the most common foods Nuts; Milk; Shellfishassociated with serious allergic reactions?

Introduction to Emergency Medicine 11

Page 28: Deja review _emergency_medicine__deja_review_

What is the most common drug implicated Penicillinin serious allergic reactions?

What is the recurrence rate of anaphylaxis Less than 25%for penicillin upon re-exposure?

What is the cross-reactivity of penicillin Less than 10%allergy to cephalosporin?

What is the pathophysiology of Mast cell and basophil degranulationanaphylaxis? due to IgE cross-linking, direct

activation, and complementactivation

What are some clinical features of Diffuse urticaria, rhinorrhea, anaphylaxis? conjunctivitis, nausea, angioedema,

airway compromise such as stridor,and hypotension

What is a general indicator of the severity Faster the onset of symptoms, of a anaphylactic reaction? typically more severe

How is anaphylaxis diagnosed? Clinically—special attention toairway and blood pressure

What is the mainstay in the treatment of Epinephrinesuspected anaphylaxis?

What are some key points in the ABCs; Oxygen, IV fluids, andmanagement of anaphylaxis? epinephrine; Decontamination

What are some commonly used agents for Antihistamines (e. g., diphenhydra-general allergic reactions? mine and ranitidine); Corticosteroids

(e. g., methylpredni-sone); Asthmamedications (e. g., albuterol)

FLUIDS

What percent of the total body weight is 60%comprised of water?

Of the total body water, what percent 2/3makes up the intracellular compartment?

What makes up the extracellular Interstitial fluid and plasmacompartment?

Define the following terms in regards towater regulation:

Osmosis Net movement of water across a selectively permeable membranedriven by a difference in solute concentrations on the two sides of the membrane

12 Deja Review: Emergency Medicine

Page 29: Deja review _emergency_medicine__deja_review_

Osmolality Total number of particles in solution

Semipermeable membrane Allows passage of the solvent, but not solute such as cell membranes

What is the normal serum osmolality? 280–295 mosm/L

Name some important solutes that Chloride, sodium, bicarbonate, andcontribute to serum osmolality? glucose

What is the equation used to calculate the 2 [Na+] + Glucose/18 + BUN/2.8serum osmolality?

What is the osmolal gap? Difference between the measured and calculated osmolality

List some causes of hyperosmolality. Uremia; Increase in serum sodium(no gap); Alcohol ingestion(methanol and ethylene glycol willcause an increase in osmole gap andanion gap acidosis); Ketoacidosis(small gap)

List a cause of hypo-osmolality. Decrease in serum sodium

How much water does an average 2–4 Lhuman adult require each day?

What are the two categories of water loss? 1. Urinary loss (1–2 L/day)2. Insensible loss (i.e., feces and skin)

Name two mechanisms by which the 1. Aldosteronehuman body handles water? 2. Antidiuretic hormone (ADH)

What are some things to know about ADH? Regulates serum osmolality; Acutevolume depletion stimulates ADH;Increased serum osmolality stimulatesADH

Does aldosterone play a significant role in Nomaintaining serum osmolality?

ELECTROLYTES

Hyponatremia

What is the serum sodium level in [Na+] <135 mEq/Lhyponatremia?

What is the serum sodium level in severe [Na+] <120 mEq/Lhyponatremia?

What are some clinical features of Headaches (HA), confusion, and hyponatremia? seizures, but can be asymptomatic

Introduction to Emergency Medicine 13

Page 30: Deja review _emergency_medicine__deja_review_

What is the most feared complication of Cerebral edemasevere hyponatremia?

Name the two most common causes of 1. Syndrome of inappropriate ADH hyponatremia? secretion (SIADH)

2. Decrease in effective circulatingvolume

What is the primary hormone that ADHregulates free water in the body?

What are some triggers that result in Increase in osmolality; Decrease inincreased secretion of ADH? circulating volume

Name an area of the body that mediates Baro receptors in the carotid sinusADH release in response to circulatingvolume?

Name some conditions that can result in True volume depletion (GI bleeding);a decrease in effective circulating volume Exercise-associated hyponatremia;that result in hyponatremia. Heart failure; Cirrhosis; Thiazide

diuretics

What is the mechanism by which The carotid sinus will sense a hyponat-remia occurs in patients with reduced pressure from fall in cardiaccongestive heart failure (CHF) even output and increase ADH releasethough they may have a marked increasein plasma volume?

What is the mechanism by which hypo- Peripheral vasodilation in cirrhosis natremia occur in patients with cirrhosis? that will result in decreased return of

venous blood with a resultant drop in cardiac output

What are some conditions that may be Lung cancer; Drugs; Infectionsassociated with SIADH? (i.e., brain abscess); Traumatic brain

injury

Name two other conditions that 1. Adrenal insufficiencyhyponatremia can also occur in? 2. Hypothyroidism

What are two disorders in which hypo- 1. Primary polydipsianatremia can occur despite normal/low 2. Advanced renal failureADH levels?

Name two causes of primary polydipsia? 1. Psychogenic2. Hypothalmic lesions

Name a cause of hyponatremia with a high Hyperglycemiaplasma osmolality.

14 Deja Review: Emergency Medicine

Page 31: Deja review _emergency_medicine__deja_review_

What are some elements to keep in mind History of fluid loss (i.e., diarrhea);in the history and physical of a patient with Signs of edema (i.e., CHF or cirrhosis);hyponatremia? Signs/symptoms suggestive of

adrenal insufficiency or hypothy-roidism; History that may point toSIADH such as small cell carcinoma

What are three important laboratory tests to 1. Urine osmolalityconsider in differentiating hyponatremia? 2. Plasma osmolality

3. Urine sodium concentration

What is the plasma osmolality in most Reduced (<275–290)hyponatremic patients?

Name one condition where a person may Hyperglycemiabe hyponatremic, but have an elevatedplasma osmolality?

What does a urine osmolality of Inability to excrete free water (i.e.,>100 mosmol/kg typically indicate in SIADH)patients with hyponatremia?

What does a urine osmolality of Primary polydipsia; Malnutrition<100 mosm/kg typically indicate in patientswith hyponatremia?

What is the primary use of urine sodium Helps to distinguish between concentration in elevating hyponatremia? effective volume depletion and other

causes

What is the initial treatment in patients Gradual correction with water who are asymptomatic and have a restriction or administration of plasma sodium concentration above isotonic saline120 mEq/L?

What are four things to consider when 1. Assessing risk of osmotic managing patients with hyponatremia? demyelination

2. Appropriate rate of correctinghyponatremia to avoiddemyelination

3. Determine the best method toraise [Na+]

4. Estimate the sodium deficit ifgiving sodium

What can lead to the development of Rapid correction of severe central pontine myelinolysis? hyponatremia

What are some clinical features of central Dysphagia, dysarthria, quadriparesis,pontine myelinolysis? lethargy, coma, and possible death

At what rate should hyponatremia be No more than 10 mEq/L per daycorrected each day?

Introduction to Emergency Medicine 15

Page 32: Deja review _emergency_medicine__deja_review_

What are some indications for aggressive Acute hyponatremia with severe treatment of hyponatremia? neurologic symptoms such as

seizures

Hypernatremia

What is the serum sodium level in [Na+] >145 mEq/Lhypernatremia?

What is the serum sodium level in severe [Na+] >155 mEq/Lhypernatremia?

What are some causes of hypernatremia in the following groups:

Sodium gain Excessive saline/bicarb adminis-tration; Hypertonic dialysis;Hypertonic feedings

Water loss Decreased water intake; Osmoticdiuresis (i.e., diabetic ketoacidosis[DKA]); Diabetes insipidus (centraland nephrogenic)

What is the most likely cause of hyper- Volume loss natremia in the emergency department?

What is the urine output of healthy Low urine output (<20 mL/hr) and hypovolemic patients? high urine osmolality (>1000

mosmol/kg water)

What is diabetes insipidus (DI)? Failure of peripheral or central ADH response

What are some characteristics of urine of Low urine osmolality (200–300 patients with DI? mosmol/kg); Low urine sodium

(60–100 mEq/kg)

What are some causes of central DI? Pituitary surgery; Trauma; Neoplasm

What is the treatment for central DI? Identify and correct underlying cause; Sodium restriction; May requirevasopressin

What are some causes of peripheral DI? Renal disease; Malnutrition;Hypokalemia

What is the treatment for peripheral DI? Sodium restriction; May requiredialysis

What are some clinical features of Altered mental status such as hypernatremia? confusion, dehydration, and seizures

What is the cornerstone of treatment for Volume repletionhypernatremia due to volume-depletion?

16 Deja Review: Emergency Medicine

Page 33: Deja review _emergency_medicine__deja_review_

What is the formula to estimate body BWD = TBW × ([Na+]/140 − 1)water deficit (BWD)?

What is important to remember with Avoid overly rapid correction due tovolume-replacement for hypernatremia? potential for cerebral edema

Hypokalemia

What is the serum potassium level in [K+] <3.5 mEq/Lhypokalemia?

What is the serum potassium level in [K+] <2.5 mEq/Lsevere hypokalemia?

What are some important causes of hypokalemia in the following conditions:

Renal Renal tubular acidosis, diuretics, Cushing’s syndrome, and hypomag-nesemia

GI condition Emesis, starvation, diarrhea, laxative abuse, and colon cancer

Other Hypothyroidism, and intracellularshift

What are some clinical features of Pronounced weakness, hyporeflexia, hypokalemia, especially if <2.5 mEq/L? ileus, paralysis, and dysrhythmias

What are some characteristic ECG Flat T-waves, U-waves, ST changes of hypokalemia? depression, and prolonged QT

interval

What is a concern if a patient with a Potentiates digoxin toxicityhistory of CHF also has hypokalemia?

What are some key points in the Oral replacement of potassium management of patients who have preferred; Correction of any chronic/subacute hypokalemia? magnesium deficits

What are some key points in the Acute hypokalemia can be life-management of patients with acute threatening; About 40 mEq will raisehypokalemia? [K+] by 1 mEq/L; Give no more than

40 mEq over an hour

Hyperkalemia

What is the serum potassium level in [K+] >4.5 mEq/Lhyperkalemia?

What is the serum potassium level in [K+] >6.5 mEq/Lsevere hyperkalemia?

Introduction to Emergency Medicine 17

Page 34: Deja review _emergency_medicine__deja_review_

What are some important causes of hyper-kalemia in the following conditions:

Renal Renal failure, aldosterone insuffi-ciency, postassium-sparing diuretics,Type IV renal tubular acidosis

Decreased cellular uptake Drugs (i.e., beta-blockers) and diabetic ketoacidosis

Increased potassium level Hemolysis, GI bleeding, and cellular breakdown such as trauma and rhabdomyolysis

What are some clinical features of Lethargy, weakness, hypotension,hyperkalemia? dysrhythmias, and paralysis

What are some ECG changes associatedwith the following degree ofhyperkalemia?

18 Deja Review: Emergency Medicine

Hyperkalemia Level ECG Changes

5.5–6.5 Peaked/large amplitude T-waves6.5–8.0 QRS widening

PR interval prolongation P-wave flattening

>8.0 Ventricular fibrillationSine wave appearance

Treatment Method Mechanism Dose Onset

Albuterol Cellular shifting 10–20 mg (inhaler) 20–30 minutesInsulin and glucose Cellular shifting 15 units of insulin 20–30 minutes

50 g of glucoseSodium bicarbonate Cellular shifting 1 mEq/kg IV 10 minutesKayexalate Excretion 15–30 g PO 1–2 hoursFurosemide w/NS Excretion 40 mg IVHemodialysis ExcretionCalcium gluconate Membrane 10–30 cc IV 1–2 minutes

antagonism

What is an important consideration when Whether there any ECG changestreating hyperkalemia?

What are some treatment options for hyperkalemia?

Page 35: Deja review _emergency_medicine__deja_review_

Hypocalcemia

What is the serum calcium level in [Ca2+] <8.5 mg/dLhypocalcemia?

What is the serum calcium level in severe [Ca2+] <7 mg/dLhypocalcemia?

What are some important causes of Hypomagnesemia; Rhabdomyolysis;hypocalcemia? Hypoparathyroidism; Acute pancre-

atitis with fat necrosis; Vitamin Ddeficiency; Renal failure

What are some clinical features of Typically symptomatic when hypocalcemia? [Ca2+] <6 mg/dL; HTN, paresthesias,

carpopedal spasms, hyperreflexia, seizures

What is Chvostek’s sign? Tapping of facial nerve that results intetany

What is Trousseau’s sign? Carpal spasm that may be elicitedby occluding the brachial artery (i.e., BP cuff)

What are some key points in the Identify and treat the underlying management of hypocalcemia? cause; CaCl2 (10% solution) over 20

minutes if acutely symptomatic

Hypercalcemia

What is the serum calcium level in [Ca2+] >10.5 mg/dLhypercalcemia?

What is the serum calcium level in severe [Ca2+] >12 mg/dLhypercalcemia?

What are some important causes of Malignancy; Vitamin D toxicity; Acutehypocalcemia? osteoporosis; Hyperparathyroidism;

Sarcoidosis

What are some clinical features of hypercalcemia in the following systems:

GI Nausea, emesis, abdominal pain,constipation, and anorexia

GU Renal failure, nephrolithiasis,and polyuria

Chronic Villus Sampling (CVS) Hypertension, dysrhythmias, anddigitalis sensitivity

Central Nervous System (CNS) Confusion, lethargy, weakness,and hyporeflexia

Introduction to Emergency Medicine 19

Page 36: Deja review _emergency_medicine__deja_review_

What are some ECG changes that can be Short QT interval, widening of theseen with hypercalcemia? T-wave, and heart blockWhat are some key points in the Identify and treat the underlying management of hypercalcemia? cause; Decrease bone reabsorption

with bisphosphonates; Furosemidewith normal saline (NS); Avoidthiazide diuretics

Hypomagnesemia

What is the serum magnesium level in [Mg2+] <1.4 mEq/Lhypomagnesemia?

What is the serum magnesium level in [Mg2+] <0.5 mEq/Lsevere hypomagnesemia?

What are some important causes of Pancreatitis; Alcoholism; Malnutrition;hypomagnesemia? Endocrine disorder (i.e., DKA)

What are some clinical features of Similar to hypocalcemia: hypomagnesemia? hypotension, tetany, tremors,

dysrhythmias, hypocalcemia, andhypokalemia

What are some ECG findings that can be Prolongation of PR and QT interval,seen in hypomagnesemia? ST depression, and wide QRS complex

What are some key points in the Identify and treat the underlying management of hypomagnesemia? cause; Check serum potassium and

calcium; Magnesium sulfatereplacement

Hypermagnesemia

What is the serum magnesium level in [Mg2+] >2.2 Eq/Lhypermagnesemia?

What is the serum magnesium level is [Mg2+] >3 m Eq/Lsevere hypermagnesemia?

What are some important causes of Renal failure; Iatrogenic;hypermagnesemia? Adrenal insufficiency

What are some clinical features of Hyporeflexia, weakness, respiratoryhypermagnesemia? depression, hypotension, bradycardia,

and systole in very high levels

What are some ECG findings that can be Extreme ST elevation and T-waves seen in hypermagnesemia? along with prolonged PR and QT

interval

20 Deja Review: Emergency Medicine

Page 37: Deja review _emergency_medicine__deja_review_

What are some key points in the Identify and treat the underlying management of hypermagnesemia? cause; Dialysis for severe serum

levels; Calcium gluconate forconduction problems

Hypochloremia

What is the serum chloride level in [Cl−] <100 mEq/Lhypochloremia?

What is the serum chloride level is severe [Cl−] <70 mEq/Lhypochloremia?

What are some causes of hypochloremia? GI loss such as diarrhea and emesis;Hypokalemic alkalosis

What are some key points in the Identify and treat the underlying management of hypochloremia? cause; NaCl for severe hypochloremia

or hypokalemic alkalosis

Hyperchloremia

What is the serum chloride level in [Cl−] >110 mEq/Lhyperchloremia?

What is the serum chloride level is severe [Cl−] >120 mEq/Lhyperchloremia?

What are some causes of hyperchloremia? Bicarbonate loss; Dehydration

What are some key points in the NS for GI bicarbonate loss;management of hyperchloremia? Bicarbonate for renal bicarbonate loss

Acid and Base Balance

Name three types of acid the body handles 1. Exogenous acidto maintain acid-base balance. 2. Abnormal metabolic pathway

3. Fixed acids

Name two organs that are crucial for 1. Lungsmaintaining acid-base balance. 2. Kidneys

How much volatile acids does the lung 15,000 mg in the form of CO2

excrete each day?

How much nonvolatile acids does the 70 mEq/Lkidney excrete each day?

What are three mechanisms by which the 1. Excretion with ammoniakidneys excrete nonvolatile acid? 2. Excretion with urinary buffers

3. Direct hydrogen excretion

What maintains regulation of hydrogen ion Bicarbonate-carbonic acid systemconcentration on a minute-to-minute basis?

Introduction to Emergency Medicine 21

Page 38: Deja review _emergency_medicine__deja_review_

What are important things to consider in Respiratory status; Volume status;the history of a patient who presents with Medication; Illicit drug useabnormal acid-base status?

What are some important laboratory tests to Arterial blood gas; Electrolytes;consider when evaluating an acid-base Determination of an anion gapdisturbance? acidosis

Briefly give some causes for the followingacid-base disturbance:

Respiratory acidosis Opioids; Respiratory failure;Sedative-hypnotics

Respiratory alkalosis Liver failure; Salicylates; Heartfailure

Anion gap metabolic acidosis Hypoxia; Sepsis; Seizures

Normal anion gap acidosis Renal tubular acidosis; Elevatedchloride

Metabolic alkalosis Volume depletion;Hyperaldosteronism

What else is important to consider in an Existence of a mixed acid-base acid-base disturbance? disturbance

Name two characteristic laboratory 1. pH <7.35findings in metabolic acidosis? 2. HCO3

− <20 mEq/L

What is one of the most common cause of Lactic acidosismetabolic acidosis in the emergency setting?

What are some causes of normal anion gap Renal tubular acidosis; Diarrhea;acidosis (>Cl−)? Extensive fluid resuscitation; Adrenal

insufficiency

What are some causes of an anion gap Methanolacidosis: Uremia

DKA

Paraldehyde

Iron, Isoniazid

Lactic acidosis

Ethylene glycol

Salicylates, strychnine

What is the treatment for metabolic Identify and treat the underlying acidosis? cause; Consider use of sodium bicar-

bonate if pH <7.1 or bicarbonate <5 mEq/L

Name two characteristic laboratory 1. pH >7.45finding in metabolic alkalosis? 2. HCO3

− >26 mEq/L

22 Deja Review: Emergency Medicine

Page 39: Deja review _emergency_medicine__deja_review_

How can one characterize metabolic Chloride-sensitive versus chloride-alkalosis even further? resistant

What are some causes of chloride-sensitive Diuretics; Emesis; Nasogastric suctionalkalosis?

What is the treatment of choice for Normal salinechloride-sensitive alkalosis?

What are some causes of chloride-resistant Mineralocorticoid excess; Primaryalkalosis? reninism; Chronic potassium depletion

What is the treatment of choice for Correction of hypovolemia;chloride-resistant alkalosis? Acetazolamide may help; Administer

potassium as a chloride salt

Name two characteristic laboratory 1. pH <7.40findings in respiratory acidosis? 2. CO2 > 45 mm Hg;

What are some causes of respiratory Neuromuscular disease; CNSacidosis? depression; Chronic obstructive

pulmonary disease (COPD)

How long before full renal compensation 48 hours of steady-state alterationoccurs?

What are some key points in the treatment Identify and treat the underlying of respiratory acidosis? cause; Bronchodilators for COPD/

bronchospasms; Assisting andincreasing ventilation; Oxygentherapy (reduces pulmonary HTN);Drugs to reduce sedation

Name two characteristic laboratory 1. pH >7.4findings in respiratory acidosis? 2. CO2 <35 mm Hg

What are some causes of respiratory CNS (i.e., anxiety); Drugs (i.e., sali-alkalosis? cylates); Hypoxemia

What are some key points in the treatment Identify and treat the underlying of respiratory alkalosis? cause; Respiratory alkalosis rarely

life-threatening; Avoid rapid correctionof PaCO2

Introduction to Emergency Medicine 23

Page 40: Deja review _emergency_medicine__deja_review_

This page intentionally left blank

Page 41: Deja review _emergency_medicine__deja_review_

25

C H A P T E R 2

NeurologicEmergencies

HEADACHES

Cluster Headaches

What is the definition of a cluster Clustering of painful HA over headache (HA)? a period of many weeks, peaks in

about 5 minutes, and may last for an hour

What are some factors associated with Male gender; Smoking; EtOH usecluster HAs?

What are two types of cluster HAs? 1. Chronic—unsustained attacks2. Episodic—occurs in bouts

What are some clinical features of Burning HA on unilateral side, cluster HAs? lacrimation and flushing on affected

side, and Horner’s syndrome

What is Horner’s syndrome? Deficiency of sympathetic activity. The site of lesion to the sympathetic outflow is on the ipsilateral side of the symptoms

What are some findings in Horner’s Ipsilateral ptosis; Anhidrosis; Miosis; syndrome? Enophthalmos

Neurologic Examination

1. Mental status testing 5. Sensory examination2. Higher cerebral function 6. Reflexes3. Cranial nerves 7. Cerebellar testing4. Motor examination 8. Gait

Copyright © 2008 by The McGraw-Hill Companies, Inc. Click here for terms of use.

Page 42: Deja review _emergency_medicine__deja_review_

Name some commonly used medications Sumatriptan; Dihydroergotamine;for abortive therapy. 100% oxygen

Name some commonly used medications Beta-blockers; Tricyclic antidepres-for prophylactic treatment. sants; Calcium channel blockers

Migraine

What are some important things to know Severe headache that afflicts millions;about migraines? Often preceded by sensory warning

sign; Affects women more commonlythan men

What are some of the mechanisms by Vascular structure involvement which migraines occur? (constriction); Serotonergic involve-

ment; Involvement of the trigeminalnerve

What are some features of auras? They will often precede attacks of migraines; Often have visual phenom-ena; Can have motor/sensory distur-bances; Present in about 15% ofmigraines

What are some factors that may provoke Physical activity; Changes in sleepor exacerbate a migraine? cycle; Menstruation; Particular foods

(i.e., chocolate)

What are some clinical features of Pulsating, severe, unilateral headachemigraines? often associated with nausea, emesis,

and photophobia/phonophobia

What are some commonly used Beta-blockers; Tricyclic antidepres-prophylactic medications for migraines? sants; Calcium channel blockers

What are some commonly used abortive Sumatriptan (typically outpatient); therapy for migraines? Metoclopramide/prochlorperazine;

Nonsteroidal anti-inflammatorydrugs(NSAIDs); Dihydroergotamine

What are some other important diagnosis Cluster HA; SAH; Tension HAto consider?

Giant Cell Arteritis aka Temporal Arteritis

What is giant cell arteritis (GCA)? Inflammation of one or more branchesof the external carotid artery

Name three branches of the carotid 1. Temporal arteryartery that are commonly affected by 2. Posterior ciliary arteryGCA. 3. Ophthalmic artery

26 Deja Review: Emergency Medicine

Page 43: Deja review _emergency_medicine__deja_review_

What are some important things to know Rare before the age of 50; Meanabout GCA? age of onset is 70; Females com-

monly more affected

What is the most feared complication of Irreversible blindness; CerebralGCA? vascular accident (CVA)

What rheumatic condition is GCA Polymyalgia rheumaticacommonly associated with?

What are some clinical features of GCA? Unilateral burning headache worse at night often accompanied with tender/pulseless temporal artery,scalp tenderness, jaw claudication,and decreased visual acuity

What is an important diagnostic Erythrocyte sedimentation rate (ESR)laboratory test to obtain in GCA? (Between 50–100 mm/hr)

What study confirms the diagnosis of Temporal artery biopsyGCA?

What are some key points in the Treatment must be started imme-management of GCA? diately; High-dose prednisonel;

Temporal artery biopsy commonlydone

Subarachnoid Hemorrhage

What are some important things to know Accounts for about 10% of CVA; about a subarachnoid hemorrhage (SAH)? Ruptured saccular aneurysms common

cause; Arteriovenous malformation(AVM) is another cause (less common)

What are other causes of SAH? Illicit drug use (especially cocaine);Intracranial arterial dissections;Bleeding diathesis

What are important risk factors for the Cigarette smoking; Moderate todevelopment of aneurysm formation, and heavy alcohol consumption;hence hemorrhage? Hypertension; Family history of SAH;

Antithrombotic therapy

What are some common clinical features of Sudden, severe headache described SAH? as the “worst headache of my life.,”

commonly associated with brief loss of consciousness, seizure, nausea, vomiting, or meningismus

What is a sentinel headache? Sudden and severe headache that often precedes a major SAH by6–20 days: minor hemorrhage

What percentage of patients will manifest Up to 50%a sentinel headache prior to SAH?

Neurologic Emergencies 27

Page 44: Deja review _emergency_medicine__deja_review_

What are some complications to consider in Seizures; Increase in intracranialSAH? pressure (ICP); Hyponatremia;

Vasospasm—ischemia

How is SAH diagnosed? Noncontrast head CT with or without lumbar puncture (LP) afterCT of the head

What are some important points in thefollowing diagnostic tests used for SAH:

Noncontrast head CT Cornerstone for diagnosis of SAH;Sensitivity of head CT is highest earlyon; Less sensitive for minor bleeds

Lumbar puncture Mandatory if there is a strongsuspicion of SAH despite a normalhead CT; Elevated opening pressureis classic; Elevated red blood cell(RBC) count

What is xanthochromia? Pink or yellow tint that represents hemoglobin degradation products, commonly seen 2–4 hours after bleed

When is the optimal time to perform an LP 12 hours after onset of HA is optimal;to detect xanthochromia? Xanthochromia can last for up to

2 weeks

What are the three most common reasons 1. Failure to obtain a CT (know its to miss a diagnosis of SAH? limitations)

2. Failure to obtain an LP3. Attribute HA to other causes like

a migraine

What is the treatment objective in Stabilization; Prevent rebleeding;a patient with SAH? Prevent vasospasms (i.e., nimodipine)

What are some key points in the Airway breathing circulation(ABC);management of SAH? Urgent neurosurgical consultation;

Slowly lower BP (i.e., labetalol); Treatfor pain and emesis; Use nimodipinein consultation with neurosurgery

SEIZURES

What is the definition of a seizure? Uncontrolled rhythmic electrical discharge within the brain that usually, but not always, results in characteristic abnormal movements of the body

28 Deja Review: Emergency Medicine

Page 45: Deja review _emergency_medicine__deja_review_

What is the general incidence of recurrent 1–2%seizures?

What is typically the cause of primary Genetically determined, usually at a seizures (epilepsy)? early age

What are some identifiable causes of Intracranial mass; Vascularseizures? malformation; Infections; Toxicological

(i.e., EtOH); Endocrine (hypoglycemia);Electrolyte

What are some elements in the history to Whether the patient has a history of obtain in a patient who presents with recurrent seizures; The circumstancesa seizure? that led to the seizure; Observed ictal

behavior; Identify potential triggers(i.e., emotions); Loss of bladder/bowel function; Current medication

Define partial seizures. Localized electrical discharge of thecerebral cortex

What are some important points for thefollowing types of partial seizures?

Simple partial No alteration of consciousness;Symptoms based on cortex affected;Visual changes if occipital affected

Complex partial Consciousness is impaired; A simplepartial with mentation affected;Often due to discharge of temporalregion

Define generalized seizures. Global discharge of the cerebralhemisphere

What are some important points for thefollowing types of complex seizures?

Absence (petit mal) Typically very brief, lasting only a fewseconds; Loss of consciousness, butnot postural tone; Will often continueunaware of event; Classically affectsschool-aged children

Atonic Less common type of seizure; Suddenloss of postural tone; May have briefloss of consciousness

Tonic Less common type of seizure;Prolonged contraction of the body;Often will be pale and flush

Clonic Less common type of seizure;Repetitive clonic jerks without tonicelement

Neurologic Emergencies 29

Page 46: Deja review _emergency_medicine__deja_review_

Myoclonic Less common type of seizure; Brief andshock-like movement of extremity;May affect entire body or just one limb

Tonic-clonic (grand mal) Abrupt loss of consciouness; Typicallystarts with tonic (rigid) phase; Oftenclonic phase comes after tonic phase;Loss of bladder/bowel function isnormal; Consciousness returns slowly

What are important elements in the Evaluation of fractures and injuries;physical exam to look for? Particular attention to the head and

spine; Vitals as well as glucose isimportant; A thorough neurologicalexam is crucial

What is Todd’s paralysis? A focal neurological deficit that typically follows a simple-partial seizure that resolves within 48 hours

What are some other conditions that Syncope; Neuromuscular disorders;seizures can be mistaken for? Migraines; Narcolepsy; Pseudoseizure

What are four clinical features that help to 1. Inability to recall attackdistinguish seizures from other causes? 2. Postictal confusion and lethargy

3. Abrupt onset4. Purposeless movement

What are some important diagnostic Anticonvulsant medication level;studies to consider in the evaluation Glucose level very important;of a seizure? Chemistry for electrolyte imbalance;

Complete blood count (CBC) forpossible infection; Toxicology screen;Urinalysis; CT or MRI

What are some indications where First time seizure (absence of fever);imaging of the head is warranted? Seizure pattern that is different; New

focal deficits; Recent head trauma;Use of any anticoagulants; Anysuspicion of meningitis

What is the role of EEG in the evaluation Not typically utilized in the of seizures? emergency department (ED); Typically

done on outpatient basis; Can be usedto classify seizure type

What are some key points in the Ensure intact ABCs; IV/O2/monitor;management of seizures? Accurate diagnostic evaluation should

be the first step

What is typically done for actively seizing Management is expectant most of thepatients? time; Most seizures self-terminate

within minutes; Medication for pro-longed seizure; Gentle firm restraintshould be used; Turn to side to avoidaspiration

30 Deja Review: Emergency Medicine

Page 47: Deja review _emergency_medicine__deja_review_

What is the most common reason that Subtherapeutic anticonvulsant levela person with a seizure disorder has a seizure?

What is typically done for a patient with If a single seizure, the focus is to a seizure disorder who is therapeutic on identify precipitants that may haveanticonvulsant and still has a seizure? lowered the seizure threshold

Name some commonly used Carbamazepine; Phenytoin/anticonvulsants. Fosphenytoin; Valproic acid

What is the definition of status Prolonged or clustered seizures thatepilepticus (SE)? sometimes develop into non-stop

seizures typically >30 minutes

What are some complications of SE? Hypoxia; Hyperthermia; Acidosis;Permanent neural damage

What are some key points in the ABCs—protect the airway inmanagement of SE? particular; Any cause of seizure can

cause SE; Initial laboratory testsshould include glucose, toxicologytest, etc.; Intubation may make itdifficult to monitor SE

What are the three classes of medications 1. Benzodiazepinesused to treat SE? 2. Phenytoin/Fosphenytoin

3. Phenobarbital

How successful is the combination of 70–90%benzodiazepines and phenytoin in controlling seizures?

What medication is commonly used in Intravenous phenobarbitalrefractory cases of seizures?

What is a treatment option for refractory Endotracheal intubation, and EEGstatus epilepticus? monitoring

MENINGITIS

Neurologic Emergencies 31

Cerebrospinal Fluid (CSF)

Normal Viral Bacterial TB Fungal

Protein (mg/dL) <55 <200 >200 >200 >200Glucose (mg/dL) >40 >40 <40 <40 <40WBCs (µL) <5 <1000 >1000 <1000 <500Gram stain negative negative positive negative negativeOpening <170 ~200 >300 ~200 ~300

pressure (mm CSF)

WBC, white blood cell

Page 48: Deja review _emergency_medicine__deja_review_

What is important to know about One of the top 10 infectious causes ofmeningitis? death; Causes over 100,000 deaths

worldwide; Permanent neurologicdeficits are common

What is meningitis? Inflammation of the leptomeninges

What are some common causes? Bacterial; Viral; Fungal; Tuberculosis

What is the mortality of a missed 15–50%diagnosis of bacterial meningitis?

What are the top three causes of meningitis: Depends on the age group

In infants. Streptococcous pneumoniae; Group Bstreptococcous infection; EscherichiaColi

In young adults. S. pneumoniae; Nisseria meningitidis;Haemophilus influenza

In people over 60. S. pneumoniae; Listeria monocytogenes;N. meningitidis/group B streptococcousinfection; H. influenza

In nosocomial infections. Pseudomonas aeruginosa; Other gramnegatives; Staphylococcus aureus

What are some risk factors for Colonization of the nasopharynx;meningitis? Bacteremia (endocarditis/UTI);

Contiguous source (mastoid/sinus);Living in a dormitory or barracks

What are some host factors that can Asplenia; HIV; Complementpredispose to meningitis? deficiency; Long-term steroid use

What is the classic triad of symptoms in Fever; Nuchal rigidity; Mental statusmeningitis? change

What are some other clinical features of Headache; Significant photophobia;meningitis? Nausea and vomiting; Seizures and

focal neurological deficits; Rash

What is Kernig’s sign? Inability to extend patient’s kneedue to pain when leg is flexed withhip at 90°

What is Brudzinski’s sign? Passive flexion of the patient’s neckcauses flexion of both hips

What are three prognostic factors 1. Hypotensionassociated with adverse outcome? 2. Mental status change

3. Seizures

Name two important diagnostic tests 1. LPused to diagnose meningitis? 2. Noncontrast head CT

What is the reason a noncontrast head To rule out intracranial masses CT is done prior to an LP? (elevated ICP)

32 Deja Review: Emergency Medicine

Page 49: Deja review _emergency_medicine__deja_review_

What are some key points in the Empirical treatment should not bemanagement of meningitis? withheld for diagnostic tests; Always

maintain a high index of suspicion;Do not wait for CT/LP to starttreatment

What empiric treatment is commonly used? Ceftriaxone; Ampicillin; Vancomycin;Acyclovir

When is chemoprophylaxis indicated? High-risk contacts of patients with;N. meningitidis; H. influenzae type B

What is the drug of choice for Rifampinchemoprophylaxis?

What role do steroids play in meningitis? Should be given before or with thefirst dose of antibiotics, mostlybeneficial in pneumococcal meningitis

CEREBRAL VASCULAR ACCIDENT

Neurologic Emergencies 33

NIH Stroke Scale

Category Patient Response and Score

1.a. Level of 0 Alertconsciousness: 1 Not alert, but arousable with minimal

stimulation2 Not alert, but requires repeated

stimulation3 Coma

1.b. Ask patient the 0 Answers both questions correctlymonth and age: 1 Answers one question correctly

2 Both questions answered incorrectly

1.c. Ask patient to open 0 Obeys both correctlyand close eyes and fist: 1 Obeys one correctly

2 Does not obey either commands

2. Best gaze: 0 Normal1 Partial gaze palsy2 Forced deviation

3. Vision field testing: 0 No visual field loss1 Partial hemianopia2 Complete hemianopia3 Bilateral hemianopia

(Continued)

Page 50: Deja review _emergency_medicine__deja_review_

What are some important things to note Originates from the vertebrobasilar about the posterior circulation? arteries; Supplies 20% of cerebral

blood flow; The following structuresare supplied:

34 Deja Review: Emergency Medicine

NIH Stroke Scale (Continued)

Category Patient Response and Score

4. Facial paresis: 0 Normal symmetrical movement1 Minor paralysis2 Partial paralysis3 Complete paralysis of one or both

sides of the face

5. Motor function—arm: 0 Normalright arm_____ 1 Driftleft arm_____ 2 Some effort against gravity

3 No effort against gravity4 No movement

6. Motor function—leg: 0 Normalright leg_____ 1 Driftleft leg_____ 2 Some effort against gravity

3 No effort against gravity4 No movement

7. Limb ataxia: 0 No ataxia1 Present in one limb2 Present in two limbs

8. Sensory: 0 Normal1 Mild to moderate decrease in

sensations2 Severe to total loss of sensation

9. Best language: 0 No aphasialook at pictures and 1 Mild to moderate aphasiaread words 2 Severe aphasia

3 Mute

10. Dysarthria: 0 Normal articulationread several words 1 Mild to moderate slurring of words

2 Unintelligible or unable to speak

11. Extinction and 0 Normalinattention: 1 Inattention/extinction to bilateral

simultaneous stimulation2 Severe hemi-inattention to more than

one modality

National Institute of Neurological Disorders and Stroke Health

Page 51: Deja review _emergency_medicine__deja_review_

Brainstem; Upper spinal cord;Medial portion of temporal lobe;Cerebellum; Thalamus; Occipitallobe

What are some important things to note Originates from the carotid arteries;about the anterior circulation? Supplies 80% of cerebral blood flow;

The following structures are supplied;Anterior portion of temporal lobe;Frontoparietal lobes; Optic nerve andretina

What is the circle of Willis? Circle of arteries that supply the brain;creates redundancies in the cerebralcirculation so if one vessel is blocked,blood flow from other vessels can maintain perfusion

Name some important causes of a CVAand some examples:

Ischemic stroke Embolic

Emboli from the heart (i.e., atrial fibrillation); Endocarditis;Plaques from large vessels(i.e., carotid)

Thrombotic

Atherosclerosis; Sickle celldisease; Mycotic aneurysms;Hypercoagulable states; Vasculitis

Hemorrhagic stroke Trauma; AV malformation; Bleedingdisorders; Spontaneous rupture ofberry aneurysm; Transformation ofan ischemic stroke

List some important risk factors of a CVA. Transient ischemic attack (TIA);Hypertension; Cardiac disease;Diabetes; Atherosclerosis;Erythrocytosis; Dyslipidemia

What are some other conditions that may Migraines; Hypoglycemia; Hepaticmimic a stroke with respect to focal encephalopathy; Seizuresneurologic deficits?

What is the definition of a TIA? Blood supply to part of the brainis briefly interrupted, resulting ina transient stroke that lasts onlya few minutes, but may persist upto 24 hours

What is the clinical significance of a TIA? “Red flag” of an impending stroke inevolution

Neurologic Emergencies 35

Page 52: Deja review _emergency_medicine__deja_review_

What are some clinical features for thefollowing stroke syndrome based onthe occluded vessel:

Middle cerebral artery (MCA) Contralateral hemiplegia/ hemianesthesia; Upper extremitydeficit more severe than lowerextremity deficit; Gaze preferencetoward the affected side; Aphasia(dominant hemisphere affected);Constructional apraxia/agnosia (non-dominant hemisphere affected)

Posterior cerebral artery (PCA) Ipsilateral cranial nerve (CN) IIInerve palsy; Contralateral homony-mous hemianopsia, hemisensoryloss, and hemiparesis

Anterior cerebral artery (ACA) Contralateral foot, leg, and arm paralysis; Lower extremity deficitmore severe than upper extremitydeficit; Frontal lobe disinhibition(i.e., abulia)

Cerebellar infarct Nausea, vomiting, ataxia, vertigo,lateralizing dysmetria, and nystagmus

Basilar artery Quadriplegia: severe bilateral signs;Coma; “Locked-in syndrome”—nomotor function except upward gazeof eyes

What is an important consideration in Airway management (i.e., a stroke patient with a depressed level of intubation)consciousness?

What is the NIH stroke scale? Objective way to rapidly assess anddetermine the extent of neurologic deficits of a stroke patient and helps to determine if thrombolytics are needed

Although hypertension is commonly Generally not—lowering the BPassociated with CVA, should it be treated aggressively may worsen the strokein the ED?

What is important to consider in the initial Determine if he/she is a candidate for management of a patient who presents with lytics; Immediate CT scan (i.e., rulea suspected stroke? out bleeds); Establish onset of

symptoms

What are some important guidelines in If symptom onset is within 3 hours;determining if a patient is a candidate for Significant neurologic deficit;thrombolytic therapy? Recommended blood pressure limits;

No contraindications such as recent SAH

36 Deja Review: Emergency Medicine

Page 53: Deja review _emergency_medicine__deja_review_

List some contraindications to the admini- History of structural CNS disease; stration of thrombolytics in acute stroke? Systolic pressure >180 mm Hg;

Significant head trauma in <3 months;History of intracranial hemorrhage;Recent trauma >6 weeks; RecentGI/GU bleeding

VERTIGO

What is the definition of vertigo? Sensation of movement of oneself or the surrounding area most often described as a feeling of spinning

What is the pathophysiology of peripheral Disorders of the ear or CN VIIIvertigo?

How much does peripheral vertigo 85%account for all cases of vertigo?

What are some common clinical features of Sudden onset of intense sensation ofperipheral vertigo? intermittent disequilibrium, nausea

and vomiting, hearing loss/tinnituscommon; nystagmus commonas well

What are some important causes of Benign positional vertigo (BPV); peripheral vertigo? Ototoxic drugs; Otitis media;

Menière’s disease

What is the Dix-Hallpike maneuver Used to diagnose and treat BPV(i.e. Nylen Barany)?

Neurologic Emergencies 37

Central versus Peripheral Vertigo

Central Peripheral

Onset Slow (can be sudden) SuddenSeverity Vague IntenseNystagmus Vertical Horizontal-rotatoryAuditory symptoms No Can have SxPattern Constant IntermittentCNS symptoms Yes NoPrognosis Usually serious Usually benign

Page 54: Deja review _emergency_medicine__deja_review_

What are some key steps in the Dix-Hallpike Sit with patient’s legs extended on maneuver? the examination table; Patient is

brought rapidly from sitting to supine,head slightly extended belowhorizontal, then head is rotated toright and left quickly

What are some treatments commonly used Antihistamine; Antiemetics;for peripheral vertigo? Anticholinergics; Benzodiazepines in

severe cases

Despite intense symptoms of peripheral No—usually can treat on outpatient vertigo, do patients typically require basis, central vertigo is a differentadmission? story

What is the pathophysiology of central Commonly due to lesions of the vertigo? cerebellum or brainstem

What are some clinical features of central Mild, but constant disequilibrium thatvertigo? may present acutely, nausea/vomiting,

vertical nystagmus, and often willhave associated CNS symptoms

What are some CNS symptoms that can be Lateralizing dysmetria, ataxia, associated with central vertigo? dysarthria, scotomata, and blindness

What are some important causes of central Multiple sclerosis; Cerebellarvertigo? tumors; Brainstem infarct;

Vertebrobasilar insufficiency

What is the deposition of patients with Often require admission for further central vertigo? evaluation

PERIPHERAL NEUROLOGIC LESIONS

38 Deja Review: Emergency Medicine

Muscles and Motor Function

Upper extremities Deltoid C5 C6 Hand extensors C6 C7Biceps C5 C6 Finger extensors C7 C8Triceps C6 C7 C8 Finger flexors C7 C8 T1

Lower extremities Quadriceps L2 L3 L4 Dorsiflexors L5Iliopsoas L2 L3 L4 Big toe extensors L4 L5 S1Gluteal L5 S1 Plantar flexors S1 S2Anterior tibial L4 L5 Toe extensors L5 S1

Reflexes Supinator C5 C6 Knee L3 L4Biceps C5 C6 Tibialis post L5Triceps C7 C8 Ankle S1 S2

Page 55: Deja review _emergency_medicine__deja_review_

Myopathies and Myelopathies

What are some defining features of Proximal weakness (i.e., standing up);myopathies? DTRs are typically intact; No alter-

ations in sensation; Often have abnor-mal laboratory test results (i.e., CPK,sedimentation rate, and elevatedWBC)

What are some clinical features for thefollowing types of common myopathies:

Steroid myopathy Long-term steroid use that is associated with muscle weakness and pain

Polymyositis Acute inflammation often leads toproximal muscles weakness andpain; Often have elevated CPK;Patients can also have low-grade fever

Hypokalemic myopathy Typically due to renal tubular acidosis;Often get proximal weakness as well;Consider toluene abuse as well as;Fanconi’s syndrome

What are some clinical features for the following types of myelopathies:

Multiple sclerosis Demyelinating disorder thought to be autoimmune in origin; Often havespinal cord involvement that resultsin upper motor neurons (UMN) signsand bladder/bowel dysfunction;Corticosteroids often used forexacerbations

Syringomyelia Cyst forms within the spinal cord and over time destroys the center ofthe cord; Sensory disruption, especiallyin the hands; Can adversely affectsweating, sexual function, andbladder/bowel control

Epidural mass Can be due to abscesses, metastatic tumor, and epidural hemorrhage;Commonly severe pain and signs ofcord compression (i.e., sensoryalterations)

Dorsal column disorders Commonly due to B12 deficiency orsyphilis; Loss of position sense,vibration, and light touch

Neurologic Emergencies 39

Page 56: Deja review _emergency_medicine__deja_review_

Neuromuscular Junction

What is the most common disorder of the Myasthenia gravisneuromuscular junction?

What is myasthenia gravis? Chronic autoimmune neuromusculardisease characterized by varying degrees of weakness of the skeletalmuscles with no sensory involvement

What is the hallmark of myasthenia gravis? Weakness that is typically first evidentin the eyelids and extraocular muscleswith generalized weakness of thelimbs following

What are some ways that myasthenia Electromyogram; Serology (used withgravis can be diagnosed? clinical picture); Edrophonium test

What are some treatment options in Pyridostigmine; Prednisone; IV gammamyasthenia gravis? globulin, and may be thymectomy

What is the most important complication Respiratory failure (i.e., diaphragm)to consider in myasthenia gravis?

What is myasthenic crisis? Severe weakness from acquired myasthenia gravis (MG) that is severe enough to require intubation often due to dysfunctional deficiencyof acetylcholine (ACh)

What is the treatment of choice for Intravenous immunoglobulin G;myasthenic crisis? Plasmapheresis

What other crisis can also occur with Cholinergic crisismyasthenia gravis?

How does cholinergic crisis commonly When too much acetylcholinesteraseoccur? inhibitors are used that result in an

excess of ACh are received

What are some clinical features of Often cholinergic with muscarinic cholinergic crisis? effects such as excessive salivation

and urination along with severemuscle weakness and possiblerespiratory failure

What treatment is commonly used for Atropinecholinergic crisis?

What is Eaton-Lambert syndrome? Presynaptic disorder of neuromus-cular transmission defined byimpaired release of acetylcholine(ACh) that causes proximal muscleweakness, depressed tendon reflexes,and autonomic changes

40 Deja Review: Emergency Medicine

Page 57: Deja review _emergency_medicine__deja_review_

What does Eaton-Lambert syndrome have Lung cancera high association with?

What complication should patients with Respiratory failure (rare)Eaton-Lambert syndrome be monitored for?

What are other important differentials to Tick paralysis; Botulismconsider in patients with generalized Amyotrophic lateral sclerosis (ALS);weakness? Organophosphate poisoning

Neuropathies

What is the definition of a neuropathy? Disorders of peripheral nerves

What are the three types of nerves that 1. Motor nervesmake up the peripheral nervous 2. Sensory nervessystem (PNS)? 3. Autonomic nerves

What are some clinical features of Mixed sensory/motor involvement peripheral neuropathies? typical; Reflexes usually absent;

Impairment is typically symmetrical/distal

What disorders are commonly associated Diabetes; Uremia; Cancer; with peripheral neuropathies? Hypothyroidism; Tick paralysis;

Guillain-Barré syndrome

What toxins are also commonly associated Organophosphates; Tetanus;with peripheral neuropathies? Heavy metals (i.e., lead); Ethanol

LOWER BACK PAIN

What are some important things to know One of the most common ED about lower back pain (LBP)? complaints; Up to 80% have experi-

enced LBP; LBP is more prevalentbetween the age of 20–40 years; LBPin elderly patients is more concerning

What are red flags in the history of a patient Age >50; History of cancer;that presents with LBP? Constitutional symptoms: fever,

weight loss, etc.; Intravenous drugabuse (IVDA); Recent instrumen-tation; Incontinence; Neurologicaldeficits

What are some findings on physical Positive straight leg raise; Neurologicalexam that is more concerning for serious deficit; Any vertebral point tendernesspathology?

Neurologic Emergencies 41

Page 58: Deja review _emergency_medicine__deja_review_

42 Deja Review: Emergency Medicine

What are some clinical features of thefollowing nerve root involvement:

L3/L4 Diminished or absent knee jerk;Weakness in the quadriceps;Anteromedial thigh and knee pain

L5 There is usually no reflex loss; Footdrop common

S1 Ankle jerk is often diminished or absent; There may be weakness oftoe flexors; Leg pain is often worsethan LBP

What is straight leg raising? Roots may be impinged upon stretch-ing the nerve root causing pain

What are the three classifications 1. Symptoms referable to seriousthat acute LBP can be placed into? conditions

2. Sciatica3. Nonspecific back pain

What is the most common cause of Strain of soft tissue elements in the back pain? back

What is sciatica? Pain radiating in a dermatomal distribution

What are some common causes of sciatica? Herniated disc; Tumor, infection, orhematoma compression; Spinalstenosis

How long does it typically take for Within a monthnonspecific LBP to resolve?

What are some key points in the Appropriate analgesia; Activity asmanagement of nonspecific lower tolerated; Muscle relaxantsback pain?

What are some imaging tests to consider Plain spinal films—concern of fracture;in LBP? CT—superior for vertebral fractures;

MRI—for emergent conditions

List some laboratory tests obtained for CBC, ESR/C-reactive protein (CRP), LBP that are possibly caused by an and urinalysis infection or tumor?

What are some important points to considerin each of the “can’t miss” diagnosis?

Metastasis Often older then 50 with hx of cancer;Often >1 month of weight loss andLBP; Often requires a variety ofimaging tests

Page 59: Deja review _emergency_medicine__deja_review_

Spinal epidural abscess Immunocompromised and IVDA at risk; Often have fever and local spine tenderness; Focal neurological deficitnot uncommon; Broad-spectrum Abx/neurosurgery consult

Disc herniation Common in >30 years with progressive LBP; Sciatica and L4-L5involvement common; Treat conser-vatively; Neurosurgery consult ifevidence of cord compression

Vertebral fracture Often history of trauma or mets;Sudden onset of pain and neurologiclogic deficits; Imaging is importantfor further evaluation

Cauda equina syndrome Often in those with mets or hx of trauma; Incontinence/saddleparesthesias common; MRI test ofchoice; Neurological emergency

SYNCOPE

What is the definition of syncope? Abrupt/transient loss of consciousnessassociated with absence of posturaltone, followed by a rapid and usuallycomplete recovery

List important conditions that should beconsidered for each category:

Cardiovascular Dysrhythmias; Obstruction(i.e., aortic stenosis); Myocardialinfarction

Neurologic Seizure; Subarachnoid hemorrhage;Posterior circulation infarct

Medication Diuretics; Beta-blockers; Nitrates

Miscellaneous Vasovagal; Carotid sinushypersensitivity

What are important elements in the Events prior to the episode; Anyhistory to gather to help determine cause associated pain (HA/chest/abdominalof syncope? pain); Diaphoresis and emesis;

Exertion; Dyspnea

Neurologic Emergencies 43

Page 60: Deja review _emergency_medicine__deja_review_

What are some findings to look for Carotid bruits; Cardiac murmurs;on physical exam? Evidence of bleeding (i.e., GI bleed);

Pulsatile abdominal mass; Adnexaltenderness (i.e. ectopic)

What is an important diagnosis to considerin the following scenario of a patient whopresents with syncope and the followingassociated symptom:

A 21-year-old healthy male presents to Hypertrophic cardiomyopathythe ED after passing out during soccerpractice. Family history is significantfor an uncle who died from suddendeath at 27

A 31-year-old female presents to the Ectopic pregnancyED after a syncopal episode whiletaking care of her kids. Her physicalexam is significant only for right adnexal tenderness

A 17-year-old female with no past Vasovagalmedical history presents to the EDafter passing out while giving bloodat Red Cross. Observers noted sheseemed diaphoretic and nauseousprior to passing out

A 65-year-old male with history of Abdominal aortic aneurysmhypertension, dyslipidemia, and CAD presents after passing out. His physical exam is significant forabdominal tenderness and bruits

A 24-year-old female is brought in by Seizureemergency medical service (EMS) when she was observed to pass out at the mallsoon followed by rhythmic movementsof her extremities. Physical examis significant for lateral tongue bites

A 62-year-old male is brought from Hypoglycemiahome by his wife after he passed out.His history is only significant for HTNand DM. She mentioned he seemeddiaphoretic prior to the event and alsomissed breakfast

What are some considerations in the To separate benign from serious evaluation of syncope? causes; A careful history and physical

is paramount; Initial ECG is also themainstay in evaluation

44 Deja Review: Emergency Medicine

Page 61: Deja review _emergency_medicine__deja_review_

What is an important point to keep in Although most cases of syncope are mind about syncope? benign, syncope may be an initial

symptom of something life-threateningsuch as AAA or SAH

What patients are often admitted for Elderly patients with many comor-syncope? bidities; Syncope with worse HA,

pelvic pain, etc.; Risk for fall andinjury (typically elderly)

Which patients are typically safe to No evidence of structural heart defectsdischarge? Isolated episode of syncope

CLINICAL VIGNETTES

55-year-old-female with an hx of breast Cauda equine syndromecancer presents with radicular pain ofher legs, urinary rentention, and lower back pain; PE: saddle anesthesia andabsent ankle jerk reflexes

71-year-old-female presents with a gradual Alzheimer’s diseasedecline in memory and having increasingdifficulties with normal day-to-day routine,often getting lost when she walks backhome; Head CT: diffuse cortical atrophy

51-year-old-male with hx of heart disease Right MCA cerebrovascular accidentpresents with a sudden onset of left-sidedextremity weakness that has not resolved;PE: flaccidity of left arms and leg alongwith Babinski (+) on left; head CT: normal

16-year-old-male presents with a headache, Epidural hematomanausea, and vomiting soon after beingstruck in the side of the head during a barfight; head CT: lens-shaped, left-sidedhyperdense mass near the temporal bone

31-year-old-female with hx of Guillain-Barré syndromegastroenteritis 1 week ago now presentswith symmetric ascending weakness ofher legs and paresthesias; PE: diminishedreflexes; LP: above normal protein

29-year-old-female presents with slow Multiple scelrosis (MS)onset of paresthesias, diplopia, numbnessof left upper extremity; MRI: discreteareas of periventricular demyelination

Neurologic Emergencies 45

Page 62: Deja review _emergency_medicine__deja_review_

27-year-old-female with a long hx of Migraine headacheheadaches presents with a unilateralheadache, nausea, vomiting, andphotophobia that typically occurs duringher menstrual period; head CT: normal

65-year-old-male with an hx of colon CA Central vertigopresents with an insidious onset ofdisequilibrium and dizziness that hasbeen present for months; PE: verticalnystagmus and ataxia

72-year-old-male presents with loss of Normal pressure hydrocephalusshort-term memory, urinary incontinence,and dementia; PE: wide-based gait; head CT: massively dilated ventricular space

43-year-old-male with a long hx of alcohol Wernicke’s encephalopathyabuse presents with psychosis and ataxia;brain MRI: mamillary body atrophy anddiffuse cortical atrophy

12-year-old is referred to the ED from Absence seizureschool due to frequent brief lapses ofconsciousness with slight limb jerking; PE: during exam, patient again has his brief lapse of consciousness with rapideyeblinking

23-year-old-female is brought in via EMS, Tonic-clonic seizureper report, patient had loss of consciousness(LOC) with loss of postural control followedby tonic phase of contractions with cloniclimb jerking; PE: patient in now awake,but minimally responsive

24-year-old-college student presents with Bacterial meningitisa high-grade fever, headache, and neckstiffness; PE: Kernig’s sign and nuchalrigidity; LP: decreased glucose, increasedprotein, and high polymorphonuclearleukocytes

31-year-old-male presents with unilateral Cluster headacheboring periorbital headache with periodsof multiple headaches alternating withsymptom-free intervals; PE: ipsilateral tearing and conjunctival injection

46 Deja Review: Emergency Medicine

Page 63: Deja review _emergency_medicine__deja_review_

C H A P T E R 3

OphthalmologicEmergencies

47

BASIC OPHTHALMOLOGY

What are the two chambers of the aqueouspart of the eye called?

What is the jelly-like substance in the backpart of the eyeball which provides shapeand is relatively inert?

What are some components that make upthe anterior segment of the eye?

What components make up the fundus ofthe eye?

Please define the following forms:

Anisocoria

Hyphema

Hypopyon

Limbus

Tonopen

What are some important elements in thehistory that should be obtained in anygeneral eye exam?

1. Anterior chamber2. Posterior chamber

Vitreous humor

Cornea; Conjunctiva; Anteriorchamber; Lens; Iris; Ciliary body

Macula; Optic nerve; Retina

Unequal pupil size under equallighting

Red blood cells in the anteriorchamber

White blood cells in the anteriorchamber

Circumferential border of thecornea and white sclera

Pen-shaped device to measureintraocular pressure

History of diabetes or hypertension;Use of contact lenses (i.e., extendedwear); Past visual acuity; Occupation

Copyright © 2008 by The McGraw-Hill Companies, Inc. Click here for terms of use.

Page 64: Deja review _emergency_medicine__deja_review_

What are eight components of the eye examthat should be obtained with all eyecomplaints?

48 Deja Review: Emergency Medicine

1. Visual acuity2. External eye3. Pupils4. Confrontation of visual fields5. Extraocular movement6. Fundus examination7. Anterior segment8. Intraocular pressure

TRAUMA OF THE EYE

Corneal Foreign Bodies

What is important to confirm duringexamination of the eye with regards toa foreign body?

What is the best way to assess foreign bodydepth in the emergency department (ED)?

Can a superficial corneal foreign body be removed in the ED?

What are some key steps in the removal ofa superficial corneal foreign body?

What are some key steps in the removal ofa full-thickness foreign body?

What is an additional concern if a foreignbody is metallic?

Should rust rings be removed in the ED?

A corneal abrasion will be present afterforeign body removal, what are sometreatments for it?

Assess if superficial penetrationversus full-thickness injury

Slit-lamp exam

Yes—under best magnificationavailable

Instill topical anesthetics in botheyes; Use slit-lamp magnification;Can use a 30-gauge needle toremove or a moistened cotton-tippedapplicator; Most superficial objectscan be removed

Do not remove in the ED—shouldbe done by ophthalmology

Metallic bodies can leave behind rustrings that are toxic to the cornea

Can be removed with an ophthalmicburr, but only the superficial layer

Antibiotic ointment; Cycloplegia;Referral to ophthalmology

Corneal Abrasions

What should always be done as part of aneye exam with corneal abrasions?

What is usually done for conjunctivalabrasions?

Check under the eyelids

Erythromycin drops; Ensure noother ocular injuries

Page 65: Deja review _emergency_medicine__deja_review_

Ophthalmologic Emergencies 49

What are some clinical features of cornealabrasions?

What are two common causes of cornealabrasions?

What is typically a limiting factor to doa complete eye exam?

What is an effective way to reduce pain?

What is the optimal way to visualizecorneal abrasions?

What is an effective long-actingcycloplegia for large or very painfulabrasions?

What are some key points in themanagement of corneal abrasions?

What is a particular concern of cornealabrasions from contacts?

What other antibiotic ointment should beadded if concerned about Pseudomonasinfection?

Should patients be sent home with topicalanesthetics for pain control?

Photophobia, tearing, and eye pain

1. Trauma2. Use of contact lenses

Patient is typically in extremediscomfort

Adequate cycloplegia

Fluorescein staining with cobalt-blue lighting

Scopolamine

Adequate pain control withcycloplegias; Erythromycin drops;Abrasion typically heal withoutproblems

Pseudomonas infections

Tobramycin or fluoroquinolonedrops

No—can cause corneal toxicity ifimproperly dosed

Subconjunctival Hemorrhage

What is the mechanism by whicha subconjunctival hemorrhage occurs?

What are some common causes ofa subconjunctival hemorrhage?

What is the treatment of choice fora subconjunctival hemorrhage?

Rupture of conjunctival vessels

Trauma; Hypertension; SuddenValsalva (i.e., coughing)

Nothing—will resolve in1–2 weeks

Chemical Injuries

What is the most important point toremember about ocular chemicalinjuries?

What is considered a more devastatinginjury: acidic or alkali?

What are some common causes of alkaliburns?

True ocular emergency

Alkali burns as they penetratedeeper

Ammonia; Lye; Industrial solvents

Page 66: Deja review _emergency_medicine__deja_review_

50 Deja Review: Emergency Medicine

What is the immediate management ofocular chemical injuries?

When should the copious irrigation bestopped?

What are some long-term complications ofchemical burns?

When should patients be referred toophthalmology?

Assuming there are no corneal clouding oranterior chamber findings, what is thegeneral disposition?

Topical anesthetic; Placement ofMorgan lens; Copious irrigationwith 1–2 L of NS

Once pH of the tears is near normal(7.5–8)

Symblepharon; Cataracts; Scarring/neovascularization of the cornea

Corneal clouding; Epithelial defect

Erythromycin drops; Cycloplegicsfor pain control; Ophthalmologicfollow up within 2 days

Blunt Injuries

What is important to assess after bluntinjury to the eye?

What is an important diagnosis to considerin any blunt trauma to the eye?

What are some clinical features of aruptured globe?

What are common causes of a rupturedglobe?

What is important not to do during an eyeexam if a ruptured globe is suspected?

What are some key points in themanagement of a ruptured globe?

What is a hyphema?

What are some common causes ofhyphema?

What vessel is typically responsible for ahyphema?

What is also important to assess in apatient with a hyphema?

Vision and globe integrity

Ruptured globe

Obvious full-thickness laceration,blindness, flat anterior chamber,irregular pupil, and hyphema

Penetrating injuries (i.e., bullets);Blunt trauma

Checking intraocular pressure (IOP)

Avoid any pressure on the globe;Place a metal eye shield; Updatetetanus status; Consider antibioticuse depending on object;Consultation with ophthalmology

Blood in the anterior chamber ofthe eye

Trauma (blunt or penetrating);Spontaneous (esp. sickle-celldisease)

Iris root vessel

Any other associated trauma suchas a ruptured globe

Page 67: Deja review _emergency_medicine__deja_review_

Why is it recommended to dilate the pupil?

What is an important complication ofhyphema?

What is the general disposition of patientswith hyphema?

How is increased IOP typically treated?

What is an important diagnosis to considerin a patient with blunt trauma andinability to gaze upward?

What is the most frequent site of an orbitalblowout fracture?

How is the diagnosis of an orbital blowoutfracture made?

What is the general disposition of anisolated orbital blowout fracture?

What are other important injuries toexamine for with orbital blowout fractures?

Ophthalmologic Emergencies 51

To avoid pupillary movementwhich may increase bleeding froman iris root vessel

Increased IOP

Consultation with ophthalmology;Elevate patient’s head; Administerdilating agent (i.e., atropine); Treatsignificant IOP increase

Topical beta-blocker; Topical alpha-adrenergic agonists; (IV) Intravenousmannitol

Orbital blowout fracture

Inferior-medial wall

CT (axial and coronal scans)

Referral for surgery within 3–9 days

Hyphema; Abrasions; Traumaticiritis; Retinal detachment

INFECTIONS OF THE EYE

Conjunctivitis

What is common element in the history ofa patient with viral conjunctivitis?

What are some clinical features of viralconjunctivitis?

What is the primary reason the corneashould be stained?

What are some key points in themanagement of viral conjunctivitis?

Preceding upper respiratoryinfection

May initially have one eyeinvolvement with waterydischarge, reddened conjunctiva,and often normal cornea

Avoid missing a herpes dendritickeratitis

Typically self-limiting (1–3 weeks);Highly contagious; Naphcon-A forcongestion/itching; Consider topicalantibiotic in suspected bacterialconjunctivitis

Page 68: Deja review _emergency_medicine__deja_review_

What are some clinical features of bacterialconjunctivitis?

What should be done to avoid missinga corneal abrasion or ulcer?

What is the treatment of choice for patientswith bacterial conjunctivitis?

What is a special consideration for contactlens-wearing patients with bacterialconjunctivitis?

What topical antibiotic should be used forcontact lens-wearers with bacterialconjunctivitis?

What parts of the eye can be affected byherpes simplex virus (HSV)?

What does fluorescein staining typicallyshow with HSV involvement of the eye?

What is an important concern with HSVkeratitis?

What should be avoided with HSVkeratitis?

How is HSV keratitis commonly treated?

What is herpes zoster ophthalmicus(HZO)?

What is Hutchinson’s sign?

What are some clinical features of HZO?

How is HZO commonly treated?

52 Deja Review: Emergency Medicine

Mucopurulent discharge, inflam-mation of eye, and often a historyof exposure to someone with viralconjunctivitis

Fluorescein staining

Broad-spectrum topical antibiotic

Pseudomonas infection

Topical aminoglycoside orfluoroquinolone

Conjunctiva; Cornea; Lids

Linear branching pattern withterminal bulbs

Corneal scarring

Topical steroids

Viroptic drops (i.e., longer if corneainvolved); Erythromycin drops toavoid secondary bacterialinvolvement

Shingles of CN V with involvementof eye

Cutaneous lesions of the tip of thenose

Iritis with pain and photophobiawith possible cutaneous lesions

Topical steroids for iritis; Topicalcycloplegic agents for pain;Consider IV acyclovir; Distinguishfrom primary HSV infection

What is a corneal ulcer? Serious infection involving multiplelayers of the cornea

Corneal Ulcer

Page 69: Deja review _emergency_medicine__deja_review_

What is the pathophysiology of a cornealulcer?

What are some causes of corneal epithelialbreak?

What are some clinical features of a cornealulcer?

What does a slit-lamp exam commonlyreveal in a corneal ulcer?

How are corneal ulcers commonly treated?

Ophthalmologic Emergencies 53

Break in the epithelial layer thatallows bacteria to invade thecorneal stroma

Trauma; Contact lenses; Incompletelid closure

Eye pain, photophobia, tearing, andredness

Staining shows epithelial defectwith underlying infiltrate as well aspossible hypopyon

Topical aminoglycoside or fluoro-quinolone; Topical cycloplegic forpain; Ophthalmology follow-upwithin 24 hours

Periorbital/Orbital Cellulitis

What is periorbital cellulitis?

What are some clinical features ofperiorbital cellulitis?

What is the most common organisminvolved with periorbital cellulitis?

What is the typical management forperiorbital cellulitis without eye involvement?

What is a special concern of periorbitalcellulitis in young children?

What are some key points in themanagement of young children withperiorbital cellulitis?

What is orbital cellulitis?

What are some common organisms toconsider in orbital cellulitis?

What is the most common source of orbitalcellulitis?

What are some clinical features of orbitalcellulitis?

Superficial cellulitis of theperiorbital area

Surrounding area of the eye (i.e.,eyelid) is red, warm, andedematous with no involvement ofthe eye itself

Staphylococcus aureus

Oral antibiotic is sufficient

High risk of bacteremia andmeningitis

Full evaluation with Abx and bloodcultures

Potentially life-threatening orbitalinfection that lies deep to theorbital septum

S. aureus; Haemophilus influenzae(in children); Mucormycosis(in immunocompromised patients)

Paranasal sinus

Fever, pain, extraocular muscle(EOM) impairment, proptosis,decreased visual acuity

Page 70: Deja review _emergency_medicine__deja_review_

What are some key points in themanagement of orbital cellulitis?

54 Deja Review: Emergency Medicine

Admission for full evaluation;CT scan of orbital/nasal area; IVantibiotics

Hordeolum

What is an external hordeolum (stye)?

What is the most common organisminvolved with a stye?

What is the typical appearance of a stye?

What is treatment of a stye?

What is a internal hordeolum known as?

What is a chalazion?

What is the appearance of a chalazion?

What is the treatment of an acute chalazion?

Acute infection of an oil glandassociated with an eyelash

S. aureus

A small pustule at the lash line

Warm compresses with erythro-mycin ointment

Chalazion

Acute or chronic inflammation ofthe eyelid commonly due toblockage of an oil gland

Tender red lump at the lid, cysticmass can occur with recurrentchronic inflammation

Warm compresses with erythro-mycin ointment; Consider doxycy-cline if chronic inflammation

ACUTE VISUAL LOSS

Central Retinal Artery Occlusion

What vessel provides blood supply to theinner retina?

What are some clinical features of centralretinal artery occlusion (CRAO)?

What is a possible warning symptom ofa CRAO?

What is the definition of amaurosis fugax?

What are some important causes of CRAO?

What is the most common cause ofCRAO?

Central retinal artery from theophthalmic artery

Sudden, painless, and profoundmonocular loss of vision

Amaurosis fugax

Loss of vision in one eye caused bya temporary lack of blood flow tothe retina

Giant cell arteritis; Embolus; Sickle-cell disease; Thrombosis; Trauma

Embolus (i.e., atrial fibrillation)

Page 71: Deja review _emergency_medicine__deja_review_

Ophthalmologic Emergencies 55

60–90 minutes

Dislodging the embolus

Initiate treatment as rapidly aspossible; Ocular massage (attemptto dislodge the embolus);Acetazolamide and topical beta-blockers; Immediate ophthalmologyconsultation

How long does it take before irreversibledamage to the retina can occur?

What is the main focus in treatment ofCRAO?

What are some key points in themanagement if CRAO?

Central Retinal Vein Occlusion

What is typically the mechanism of centralretinal vein occlusion (CRVO)?

Name some conditions that are associatedwith CRVO.

What are some clinical features of CRVO?

What is the typical funduscopic finding inCRVO?

What is the typical treatment option forCRVO?

Thrombosis of the central retinal vein

Glaucoma; Hypertension;Hypercoagulable disorders

Acute, painless, and monocularinvolvement with variable visionloss

Diffuse retinal hemorrhage in allquadrants; Optic disc edema

Ophthalmology consultation; Mayconsider giving aspirin

Narrow-Angle Glaucoma

Is a history of glaucoma common inpatients who present with narrow-angleglaucoma?

What is the mechanism by which aqueoushumor is produced?

What is the pathophysiology of narrow-angle glaucoma?

What are some clinical features of narrow-angle glaucoma?

No—patients will typically have anundiagnosed narrow anteriorchamber angle

Aqueous humor is produced in theciliary body from the posteriorchamber which flows through thepupil and into the anterior chamber,where it is reabsorbed

When the pupil becomes mid-dilated, the lens touches the irisleftlet, blocking the flow of aqueoushumor and causing an increase inIOP, causing the cornea to becomeedematous and distorted

Headache, eye ache, cloudy vision,nausea/vomiting, and increasedIOP

Page 72: Deja review _emergency_medicine__deja_review_

56 Deja Review: Emergency Medicine

How high can the IOP be in narrow-angleglaucoma?

What is the typical finding on exam of thepupil?

What is the focus of treating narrow-angleglaucoma?

What are some agents commonly used tosuppress aqueous humor production?

What is another agent to consider that iseffective in lowering IOP?

What agent is commonly used to constrictthe pupil once the IOP has been reduced?

What is the definitive treatment fornarrow-angle glaucoma?

Higher then 50 mm Hg

Mid-dilated and nonreactive

Quickly lowering IOP and decreaseproduction of aqueous humor

Topical beta-blockers and alpha-agonists; Acetazolamide

Mannitol

Pilocarpine (will not typically workduring an acute attack)

Peripheral laser iridectomy

Optic Neuritis

What is optic neuritis?

What are some causes of optic neuritis?

What are some clinical features of opticneuritis?

What test is useful to detect alteration incolor vision?

How is the Red Desaturation test carriedout?

What is a possible finding on funduscopicexam of a patient with optic neuritis?

In what case will the optic disc be normalduring a funduscopic exam?

What is the typical disposition of patientswith optic neuritis?

Optic nerve dysfunction that is themost common cause of acutereduction of vision

Ischemia; Embolus; Nerve com-pression; Multiple sclerosis (Ms);Lupus

Rapid and painful reduction ofvisual acuity, but more commonlyaffects color vision and afferentpupillary defect

Red Desaturation test

Have the patient look at a redobject with each eye individually,the affected eye will often see thered object as pink or lighter

Optic disc is swollen (anteriorneuritis)

Retrobulbar neuritis

Discuss with ophthalmology on theuse of steroids and follow-up

Page 73: Deja review _emergency_medicine__deja_review_

Ophthalmologic Emergencies 57

CLINICAL VIGNETTES

26-year-old male steel worker presents tothe ED with right eye pain and blurringvision; eye exam: small metallic flecks inthe cornea

18-year-old female with no PMH presentswith a 1 week history of tearing,photophobia, and left eye pain. She doeshave a history of sleeping with her contactson; eye exam: fluorescein staining withcobalt-blue lighting

23-year-old male with no PMH presentswith a sudden onset of blood visible in theright eye, patient does not complain of anyvision problems or pain, but is otherwisevery concerned

41-year-old female chemist presents to theED with recent history of lye splashing inher eyes. What is the most crucial aspect inmanagement?

26-year-old male presents to the ED soonafter being hit directly in his right eye witha baseball during a game. He now complainsof pain and blindness; eye exam: irregularpupil, hyphema, and flat anterior chamber

3-year-old female is brought in by hermother due to concerns of bilateral redeyes with watery discharge. Significanthistory includes day care three timesa week

19-year-old college student with eyehistory of contact lens use presents withinflammation of her eyes along withmucopurulent discharge; eye exam:unremarkable for fluorescein staining

55-year-old male presents with inflammationof the eyes with watery discharge for about3 days; eye exam: linear branching patternwith fluorescein staining

21-year-old male with recent eye injurypresents to the ED with left eye pain,photophobia, redness, and tearing for2 days; eye exam: hypopyon and stainingthat shows epithelial defects

Corneal foreign body

Corneal abrasions

Subconjunctival hemorrhage

Copious irrigation

Ruptured globe

Viral conjunctivitis

Bacterial conjunctivitis

HSV keratitis

Corneal ulcers

Page 74: Deja review _emergency_medicine__deja_review_

58 Deja Review: Emergency Medicine

61-year-old diabetic male presents withsurrounding area of redness and edemaaround his left eye that is warm to thetouch; eye exam: normal

24-year-old male presents to the ED due toconcern of a small growth around hisupper eyelash, but otherwise has nochanges in vision; eye exam: remarkablefor a small pustule at the lash line

61-year-old female with an Hx of comorbiddisease (CAD), DM, artrial fibrillation(afib), and cerebral vascular accident (CVA)presents with sudden and painless loss ofvision in her left eye

67-year-old female with history of DMpresents with a pounding headache, cloudyvision, nausea, and eye pain soon aftercoming out from the movies; eye exam:mid-dilated and nonreactive pupil withIOP >50 mm Hg

Periorbital cellulitis

Stye

CRAO

Acute angle-closure glaucoma

Page 75: Deja review _emergency_medicine__deja_review_

C H A P T E R 4

ENT and DentalEmergencies

59

ACUTE OTITIS MEDIA

What are some important things to knowabout acute otitis media (AOM)?

What are some risk factors associated withthe development of AOM?

What is the pathogensis of AOM?

What are the three most common bacterialpathogens involved in AOM?

What are some clinical features of AOM?

What are some complications to consider inotitis media if left untreated?

What is the most reliable sign of AOM?

What is the first-line treatment for AOM?

What are two other drugs to consider inpenicillin-allergic patients?

Most frequent diagnosis in sickchildren; The highest incidencebetween 6–24 months of age; Oftenoccur during winter/spring after anupper respiratory infection (URI)

Age; Day care; Second-hand smoke;Altered host defense

Obstruction of the eustachian tubethat results in a sterile effusion withaspiration of nasopharyngealsecretions into the middle ear thatcan result in acute infection

1. Streptococcus pneumoniae2. Haemophilus influenza3. Moraxella catarrhalis

Examination of the ear often showsdistortion of the tympanic membrane(TM), erythema, decreased motilityof TM on pneumatic otoscopy, andfever

Hearing loss, TM perforation,mastoiditis, lateral sinusthrombosis, and meningitis

Decreased motility of the TM onpneumatic otoscopy

Amoxicillin

1. Erythromycin2. Trimethoprim-sulfamethoxazole

Copyright © 2008 by The McGraw-Hill Companies, Inc. Click here for terms of use.

Page 76: Deja review _emergency_medicine__deja_review_

What are some key points in themanagement of AOM?

What is the definition of bullousmyringitis?

What agents are often associated withbullous myringitis?

What is the treatment for bullousmyringitis?

60 Deja Review: Emergency Medicine

Local heat application for relief;Antibiotic for treatment; Return ifAOM does not improve within48 hours

Inflammation of the TM with bullaethat are present on the TM (typicallymore painful)

Mycoplasma or viral infection

Macrolide antibiotics; TopicalAuralgan for intact TM; ENTfollowup as needed

OTITIS EXTERNA (SWIMMER’S EAR)

What is the definition of otitis externa?

What are some inherent defenses thatcontribute to protection against infection?

What are the two most common organismsassociated with otitis externa?

What are some risk factors that contributeto the development of otitis externa?

What are some clinical features of otitisexterna?

What are some features of severe cases ofotitis externa?

What are some key points in themanagement of otitis externa?

What is the definition of necrotizing otitisexterna?

Inflammation of the externalauditory canal or auricle typicallydue to infection, allergic reaction,or dermal disease

Hair follicles; Tragus and conchalcartilage; Cerumen

1. Pseudomonas aeruginosa2. Staphylococcus aureus

Warm, moist environment (i.e.,swimming); Excessive cleaning;Devices that occlude the auditorycanal

Pain, itching, fullness of ear,redness or swelling of external ear,and cheesy or purulent greendischarge

Complete obstruction of canal dueto edema, auricular erythema,adenopathy, and fever

Clean the canal thoroughly; Controlpain; Topical agents in mild cases(i.e., Cortisporin); Antibiotic inmore severe cases

Serious complication of acutebacterial otitis externa whereinfection spreads from the skin tothe soft tissue, cartilage, and bone ofthe temporal region and skull base

Page 77: Deja review _emergency_medicine__deja_review_

What population is more commonlyaffected by necrotizing otitis externa?

What is the mortality rate of necrotizingotitis externa if left untreated?

What are some clinical features ofnecrotizing otitis externa?

What are some key points in themanagement of necrotizing otitis externa?

ENT and Dental Emergencies 61

Elderly; Diabetic;Immunocompromised

Approaches up to 50%

Otorrhea, pain that is out ofproportion to the exam, granulationtissue at the bony cartilaginousjunction of the ear canal floor, andcranial nerve palsies

Intravenous (IV) antibiotics; ENTconsult; Possible surgical debride-ment; MRI/CT diagnostic test ofchoice to visualize complications ifneeded

ACUTE HEARING LOSS

What are the three components of the ear?

How is hearing loss classified?

What areas of the ear often result inconductive hearing loss if damaged?

What are some important causes ofconductive hearing loss?

What areas of the ear often result insensorineural hearing loss if damaged?

What are some important causes ofsensorineural hearing loss?

What are some common causes of bilateralsensorineural hearing loss?

What are important elements in the examto evaluate acute hearing loss?

1. Outer ear: auricle and ear canal2. Middle ear: TM and ossicles3. Inner ear: cochlea and semicircular

canals

Conductive; Sensorineural; Mixed

External auditory canal; Tympanicmembrane; Middle ear components(i.e., ossicles)

Middle ear effusion; TM perfora-tion; Otitis externa; Foreign bodyimpaction

Cochlea; Auditory nerve; Inner ear

Acoustic neuroma; Viral neuritis;Temporal bone fracture; Presbycusis

Exposure to loud noise; Antibiotics(i.e., aminoglycosides); Nonsteroidalanti-inflammatory drugs (NSAIDs);Loop diuretics

History of vertigo and tinnitus;Cranial nerve examination;Thorough otoscopic exam; CT ifany suspicion of tumor

Page 78: Deja review _emergency_medicine__deja_review_

What two tests are useful to distinguishsensorineural from conductive hearingloss?

What is the Weber test?

In a patient with unilateral conductivehearing loss, in which ear would the soundbe loudest in a Weber test?

In a patient with unilateral sensorineuralhearing loss, in which ear would the soundbe loudest in a Weber test?

How is a Rinne test done?

What are some possibilities with theRinne test?

What are some key points in themanagement of acute hearing loss?

62 Deja Review: Emergency Medicine

1. Weber test2. Rinne test

Tuning fork is struck and placed onthe patient's forehead. The patientis asked to report in which ear thesound is heard loudest

A patient would hear the tuningfork loudest in the affected ear

A patient would hear the tuningfork loudest in the unaffected ear

This is achieved by placing avibrating tuning fork (512 Hz)initially on the mastoid, then nextto the ear and asking which soundis loudest

In a normal ear, air conduction(AC) is better than bone conduction(BC); In conductive hearing loss, BCis better than AC; In sensorineuralhearing loss, BC and AC are bothequally depreciated, maintaining therelative difference of AC > BC

Primarily depends on the cause;Foreign body should be removed;Offending medication should bediscontinued; Tumors requireadmission/consultation

NASAL

Nasal Trauma

What is a common diagnosis in any nasaltrauma?

What are some clinical features of nasalfractures?

What role does x-ray play in the evaluationof uncomplicated nasal fractures?

Nasal fracture

Deformity, nasal swelling, ecchy-mosis, tenderness, or crepitence

Not commonly used

Page 79: Deja review _emergency_medicine__deja_review_

What are some key points in themanagement of uncomplicated nasalfractures?

What are some examples of complicatednasal fractures?

What is the test of choice to furtherevaluate complicated nasal fractures?

What are some possible indications for theuse of prophylactic antibiotics?

What is another major complication ofnasal trauma?

What are some clinical features of a septalhematoma?

What are some key points in themanagement of a septal hematoma?

What is the consequence of failure to draina septal hematoma?

What is the common deformity that occursdue to avascular necrosis of the nasalseptum?

What can occur if the cribriform plate isfractured?

What is the timeline for when this canoccur?

What is a common clinical scenario whenthis can occur?

What diagnostic test can be used to detectcribriform plate fracture?

What are some things to do if one suspectsCSF rhinorrhea?

What is the major concern of CSFrhinorrhea in regard to infections?

What role do antibiotics play in regard toCSF rhinorrhea?

ENT and Dental Emergencies 63

Early reduction if swelling is notsevere; Delay reduction (2–3 days)if severe swelling; Reevaluationafter edema has resolved

Other facial fractures (i.e., orbitalfloor); Nasoethmoid fracture

CT

Use of nasal packing; Laceration ofnasal mucosa; Immunocompromised

Septal hematoma

Bluish-purple swelling of the nasalseptum

Vertical incision of the hematoma;Pack the anterior nasal cavity;Antibiotic coverage (Staph coverage);ENT follow-up

Avascular necrosis; Septal abscess

Saddle-nose deformity

Cerebrospinal fluid (CSF) rhinorrhea

CSF rhinorrhea may not occur untilweeks after the cribriform fracture

Typically occurs in the setting ofa facial trauma followed by clearnasal discharge that can be associatedwith anosmia and headache

Plain radiograph facial series

Keep the patient upright; Avoidcoughing/sneezing; Consulta neurosurgeon

Meningitis

Controversial—use in consultationwith neurosurgery

Page 80: Deja review _emergency_medicine__deja_review_

What age group do nasal foreign bodiesoccur in?

What is the common clinical presentationof a child with a nasal foreign body?

In many cases, can a history of an objectbeing inserted into the nares be recalled?

How is the diagnosis of a nasal foreignbody commonly made?

What are some commonly used methods toremove a nasal foreign body?

What is typically done if the foreign objectcannot be removed?

What are some indications for admissionfor immediate nasal foreign body removal?

64 Deja Review: Emergency Medicine

Children 2–4 years of age

Unilateral foul-smelling nasaldischarge or persistent epistaxis

No

Inspection of nares with nasalspeculum or otoscope

Forceps, wire loops, or right angleprobes; Suction catheter; Positivepressure (i.e., blow via nose)

ENT follow-up within 24 hours(most can be done as outpatient)

Associated infections (i.e., facialcellulites); Sharp objects; Buttonbatteries

Epistaxis

What is more common: anteriornosebleeds or posterior nosebleeds?

What is the most common source ofanterior nosebleeds?

What age group is commonly affected withanterior nosebleeds?

What are some important causes of anteriornosebleeds to consider?

What are some important elements in thehistory to consider with respect to anteriornosebleeds?

What are some important elements in thephysical to focus on?

What simple thing can be done prior tofurther evaluation of nosebleeds?

What are some commonly used methods togain hemostatic control of anteriornosebleeds?

Anterior nosebleeds (90% of cases)

Kiesselbach’s plexus

Children and young adults

Foreign body; Trauma; Nose picking;Blood dyscrasias; Infections

Recurrent; Onset; Duration;Medication; Illicit drug; Underlyingmedical problems

Vitals (i.e., orthostatics); Evidenceof coagulopathy (i.e., bruising);Location (anterior versus posterior)

Apply a topical vasoconstrictor/anesthetic; Pinch nose firmly andkeep head forward

Silver nitrate sticks (cautery);Anterior nose packing; Piece ofhemostatic material (i.e., Gelfoam)

Nasal Foreign Bodies

Page 81: Deja review _emergency_medicine__deja_review_

What is the most common source ofposterior nosebleeds?

What age group is commonly affected withposterior nosebleeds?

What are some important causes ofposterior nosebleeds to consider?

What are some key points in themanagement of posterior nosebleeds?

How is posterior packing commonly done?

What are some important complications ofepistaxis?

ENT and Dental Emergencies 65

Sphenopalatine artery (arterialsource); Woodruff’s plexus (venoussource)

Elderly

Cancer; Coagulopathy

Particular importance on airway;Posterior packing with premadeposterior nasal-packing balloon;Admit with ENT consultation

Use gauze pack with an intranasalballoon device or Foley catheter

Severe bleeding; Airway obstructionfrom bleeding; Sinusitis; AOM

ENT INFECTIONS

Pharyngitis

What is the definition of pharyngitis?

What are some important causes ofpharyngitis?

What is the most common cause ofpharyngitis?

What are some viruses that are commonlyimplicated in pharyngitis?

What are some clinical features ofinfectious pharyngitis?

What are some clinical features of herpessimplex virus (HSV) pharyngitis?

What is the treatment for HSV pharyngitis?

What is the cause of infectiousmononucleosis?

Inflammation of the mucousmembrane of the oropharynx withpotential for airway compromise

Infections; Trauma (i.e., causticingestions); Irritant inhalant

Viral infections

Epstein-Barr virus; Influenza virus;Parainfluenza virus; Adenovirus

Fever, sore throat, dysphagia, andcervical adenopathy

May present with features ofinfectious pharyngitis withgrouped vesicles in the oropharynxthat erode to form ulcers

Acyclovir for immunocompromisedpatients, may benefit other patients(i.e., healthy)

Epstein-Barr virus

Page 82: Deja review _emergency_medicine__deja_review_

What age group is commonly affected byinfectious mononucleosis?

What are some clinical features ofinfectious mononucleosis?

What is an important complication ofinfectious mononucleosis?

What is a common finding on a peripheralblood smear?

What diagnostic test can be used to supportthe diagnosis of infectious mononucleosis?

What are some key points in themanagement of infectious mononucleosis?

List some indications for steroid use ininfectious mononucleosis.

What infectious organisms should beconsidered in a patient with infectiouspharyngitis and a history of orogenitalsex?

What is the significance of pharyngitiscaused by gonorrhea in children?

What are some commonly used antibioticsfor the treatment of pharyngitis caused bygonorrhea?

What other organisms should beconsidered in pharyngitis caused bygonorrhea?

What are two antibiotics commonly used totreat chlamydia?

Is diphtheria a common cause ofpharyngitis?

Who are at risk for diphtheria?

What is the organism responsible fordiphtheria?

66 Deja Review: Emergency Medicine

Young adults (10–26 years of age)

Fever, sore throat, malaise, fatigue,and cervical adenopathy (esp.posterior) with exudative pharyngitisand hepatosplenomegaly

Splenic rupture

Lymphocytosis

Monospot test

Treatment is primarily supportive;Avoid contact sports for a month orso

Neurologic complications (i.e.,encephalitis); Airway compromise;Severe hemolytic anemia

Gonorrhea

Sexual abuse

Ceftriaxone; Ofloxacin; Ciprofloxacin

Chlamydia

1. Macrolides2. Doxycycline

No–not with DPT immunizations,but can still occur for patients whodid not receive DPT immunisation

Really young or old patients; DPTimmunization not up-to-date;Developing countries

Cornyebacterium diphtheriae

Page 83: Deja review _emergency_medicine__deja_review_

What is the pathophysiology of diphtheria?

What are some clinical features ofdiphtheria?

What are the systemic complications ofdiphtheria primarily due to?

What are some important complications ofdiphtheria?

What are some common laboratoryfindings of diphtheria?

What are some key points in themanagement of diphtheria?

What is the typical medical treatment fora patient with diphtheria?

What is the most common cause ofbacterial pharyngitis?

Who are more commonly affected withGroup A streptococcus?

What is the Centor criteria?

What are the four clinical features of theCentor criteria?

How is the Centor criteria used?

ENT and Dental Emergencies 67

Invasive infection that primarilyaffects the throat and nose causingtissue necrosis often producing thecharacteristic pseudomembrane inthe posterior pharynx

Typically toxic-appearing withacute onset of fever, malaise, sorethroat, and hoarse voice. PE:exudative pharyngitis with adherentpseudomembrane in the posteriorpharynx and cervical adenopathy

Powerful exotoxin that primarilyaffects the cardiovascular system(CVS) and central nervous system(CNS)

Airway obstruction; Neuritis;Atrioventricular (AV) block;Myocarditis/endocarditis

Positive culture on Loeffler’s media;Gram (+) rods with clubbing onswab; Complete blood count (CBC)showing thrombocytopenia

Airway, breathing, circulation(ABC) (esp. airway); Respiratoryisolation; Treatment aimed atbacteria and toxin; Considertetanus and diptheria (Td) boosterin close contacts

Diphtheria antitoxin; Penicillin ormarcolide

Group A beta-hemolytic Streptococcus

Young adults during winter

Used to predict group A strepto-coccal (GAS) pharyngitis in adults,therefore help to guide use of Abx

1. Fever2. Absence of cough3. Cervical lymphadenopathy4. Tonsillar exudates

Used in conjunction with a rapidStreptococcus screen whether totreat for Group B streptococcus

Page 84: Deja review _emergency_medicine__deja_review_

What are some commonly used antibioticsto treat GAS?

What role does the use of intramuscular(IM) dexamethasone play ?

What are some important complications ofGAS?

Is the timely treatment of GAS enough toprevent the three mentionedcomplications?

What is rheumatic fever?

What is the Jones criteria?

What is the treatment of choice forrheumatic fever?

What organism can produce pharyngitis inimmunocompromised patients?

List common fungal causes of pharyngitis.

What groups are typicallyimmunocompromised?

What does the physical exam commonlyreveal?

What are two medications that can be usedto treat fungal pharyngitis?

68 Deja Review: Emergency Medicine

Penicillin; Azithromycin (forrecurrent infections); First- andSecond-generation cephalosporin

Often used for severe symptoms;Decreases severity of symptoms;Provides pain relief

Rheumatic fever; Glomerulonephritis;Pharyngeal space infections

All but glomerulonephritis

Nonsuppurative complication ofGAS, it is a serious inflammatorycondition that can affect the heart,joints, nervous system, and skin. Itmost frequently occurs in childrenbetween the ages 6 and 16 years

Used to help diagnose rheumaticfever in conjugation with laboratoryfindings

Major. Carditis; Polyarthritis;Subcutaneous nodules; Erythemamarginatum; Chorea

Minor. History of rheumatic feveror heart disease; Fever; Arthralgias

Penicillin; Steroids for carditis;NSAIDs for arthritis

Fungi

Cryptococcus; Histoplasma; Candida

Diabetics; Chemotherapy recipients;Chronic steroid users; HIV-infected

White/removable plaques on anerythematous base

1. Nystatin swish and swallow2. Systemic fluconazole

Oral and Facial Infections

What is the biggest concern of any abscesswithin the oral cavity?

Airway compromise

Page 85: Deja review _emergency_medicine__deja_review_

What is Ludwig’s angina?

What is a common cause of Ludwig’sangina?

Name the three potential spaces that theinfection can tract to.

Name some commonly involved organismsin Lugwig’s angina.

What are some common clinical features ofLudwig’s angina?

What are the key points in themanagement of patients with Ludwig’sangina?

What are some organisms involved witha masticator space abscess?

What is the pathophysiology of howa masticator space abscess occurs?

What are some clinical features of amasticator space abscess?

What are some key points in themanagement of a masticator space abscess?

Name four potential spaces that canbecome infected in pharyngeal spaceinfections.

Where do retropharyngeal abscesses occur?

In what age groups do retropharyngealabscesses occur?

List the most common pathogens involvedin retropharyngeal abscesses.

ENT and Dental Emergencies 69

Progressive cellulitis of the floor ofthe mouth involving sublingualand submandibular space

Trauma or abscess to the posteriormandibular molars

1. Sublingual space2. Submandibular space3. Submaxillary space

Streptococcus; Staphylococcus;Anaerobic organisms (i.e.,bacteroides)

Patient will often appear sick withodynophagia, dysphonia,dysphagia, drooling, trismus,massive swelling of the floor of themouth, and an elevated tongue

Airway management should be toppriority; Immediate ENT consul-tation; Avoid putting the patient ina supine position; IV antibiotics (i.e.,ampicillin-sulbactam); Admit to ICU

Anaerobes; Streptococcus

Infection secondary to infectionaround third molar or extensionfrom anterior space such as buccalspace

Fever, trismus, and face swelling

Careful attention to airway;Immediate ENT consultation;IV antibiotics (i.e., penicillin)

1. Retropharyngeal space2. Peritonsillar space3. Peripharyngeal space4. Prevertebral space

In the space posterior to thepharynx and anterior to theprevertebral fascia

Most common in children <3 yearsof age

Anaerobes; Group A Streptococcus;S. aureus

Page 86: Deja review _emergency_medicine__deja_review_

What are some clinical features ofretropharyngeal abscesses?

What is the initial diagnostic test of choicefor retropharyngeal abscesses?

What are some findings of the lateral neckfilm that points to a retropharyngealabscess?

What are some significant complications tokeep in mind?

What are some key points in themanagement of retropharyngeal abscesses?

Where do prevertebral abscesses occur?

What are some clinical features ofprevertebral abscesses?

What distinguishing factor can help todistinguish one from the other?

What is a common cause of prevertebralabscesses?

What is the initial diagnostic test of choicefor prevertebral abscesses?

What are some findings of the lateral neckfilm that point to prevertebral abscesses?

What three possible diagnostic tests can beused to confirm prevertebral abscesses?

What are some key points in themanagement of prevertebral abscesses?

70 Deja Review: Emergency Medicine

Patient will appear sick with fever,dysphagia, sore throat, swelling ofneck, unilateral bulge of posteriorpharynx wall, and stridor

Soft-tissue lateral film of neck

Widening of the retropharyngealspace; Displacement of the larynx;Presence of air-fluid level in thespace

Airway obstruction; Invasion toadjacent structures; Sepsis;Aspiration

Careful attention to airway;Immediate ENT consultation forincision and drainage (I&D); IVantibiotics (i.e., ampicillin/sulbactum)

In the space anterior to the cervicalspine and posterior to the prever-tebral fascia

Due to the very close proximity ofthe prevertebral space and retropha-ryngeal space, the clinical featuresare very similar to a retropharyn-geal abscess

Age (prevertebral abscesses morelikely in older patients)

Cervical osteomyelitis

Lateral neck film

Widening of the retropharyngealspace; Displacement of the larynx;Evidence of osteomyelitis of cervicalspine

1. CT2. MRI3. Cervical myelogram (not com-

monly used)

IV antibiotics; Neurosurgicalconsultation; Patient requiresadmission

Page 87: Deja review _emergency_medicine__deja_review_

Where do peritonsillar abscesses occur?

What are peritonsillar abscesses commonlydue to?

What age group are peritonsillar abscessescommon in?

What are some organisms involved withperitonsillar abscesses?

What are some clinical features ofperitonsillar abscesses?

What are some diagnostic studies that canhelp confirm the diagnosis of peritonsillarabscesses?

What are some key points in themanagement of peritonsillar abscesses?

Where do peripharyngeal abscesses occur?

What are some common causes ofperipharyngeal abscesses?

What are some clinical features ofperipharyngeal abscess?

What are some complication ofperipharyngeal abscesses?

What are some key points in themanagement of peripharyngeal abscesses?

ENT and Dental Emergencies 71

Between the superior constrictormuscle and tonsillar capsule

Untreated tonsillitis

Young adults

Usually polymicrobial

Typically a history of sore throat andfever that becomes progressivelyworse and unilateral, can also havetrismus, dysphagia, ear pain, tendercervical adenopathy, and deviateduvula to opposite side

CT; Ultrasound

ABC—ensure airway and hydra-tion; IV antibiotics (i.e., penicillin);ENT consultation for I&D; Culturefor pathogen; If uncomplicated candischarge with 24-hour follow-up

Occur in the space lateral to thepharynx and medial to the masti-cator space

Tonsillar infections; Dental infections

Unilateral neck swelling, fever,neck pain, dysphagia, drooling,cervical adenopathy, and sorethroat

Airway obstruction; Cranial nerveinvolvement; Erosion into carotidsor jugular veins

ABC—ensure intact airway;Admission for further care; ENTconsultation; IV antibiotics

Facial Infections

What is the definition of sinusitis? Infection of the paranasal sinusestypically from a preceding URI

Page 88: Deja review _emergency_medicine__deja_review_

Name four paranasal sinuses?

What is the most commonly involved sinusin sinusitis?

What is the pathophysiology of sinusitis?

Name some pathogens typically involvedin sinusitis?

What are some clinical features ofsinusitis?

What are some diagnostic tests to considerin sinusitis?

What are some key points in themanagement of sinusitis?

What are some complications of sinusitis?

What is the definition of mastoiditis?

Name some pathogens typically involvedin mastoditis?

What are some clinical features ofmastoiditis?

What are some commonly used diagnosticstudies to evaluate mastoiditis?

What are some complications of untreatedmastoiditis?

72 Deja Review: Emergency Medicine

1. Maxillary2. Ethmoid3. Frontal4. Sphenoid

Maxillary

Occlusion of the sinus ostia whichis usually precipitated by a URI orallergic rhinitis, that results in aculture medium ideal for bacterialgrowth and infection

S. pneumoniae; nontypeableH. influenza; S. aureus; M. catarrhalis

Nasal congestion, fever, purulentyellow-green discharge, headache,nasal congestion, tenderness overthe affected sinus, and opacificationof the sinus on transillumination

Diagnosis can typically be made onhistory and physical, but a CT ofthe sinuses can be done

Decongestants; Mucolytics;Analgesics; Antibiotics for severecases or complications

CNS involvement (i.e., meningitis,brain abscesses, etc.), cavernoussinus thrombosis, periorbital/orbital sinus, and surroundingabscess formation

Infection of the mastoid air cellsmost commonly from AOM

S. pneumoniae; nontypeableH. influenza; S. aureus

Posterior auricular tenderness,headache, hearing loss, otorrhea,and abnormal TM

MRI; CT of temporal bone

CNS involvement (i.e., meningitis,brain abscesses, etc.), CN VIIinvolvement, and labyrinthitis

Page 89: Deja review _emergency_medicine__deja_review_

What are some key points in themanagement of mastoiditis?

ENT and Dental Emergencies 73

ENT consultation for possibledebridment; IV antibiotics;Adequate pain control; Admissionfor further care

DENTAL EMERGENCIES

What cranial nerve (CN) provides primarysensation to the face?

What are branching nerves of theophthalmic branch and the area theyinnervate:

Nasociliary nerve

Supraorbital nerve

What are branching nerves of the maxillarybranch and the area they innervate:

Superior alveolar nerves

Posterior

Middle

Anterior

Nasopalatine and greater palatinenerves

Infraorbital nerve (with part of thesuperior alveolar nerve)

What are two commonly used localanesthetics to achieve oral anesthesia?

Name four nerves that are commonlyblocked to achieve anesthesia?

What type of infiltration is commonly usedto achieve individual tooth anesthesia?

What are some important complications ofperforming nerve blocks in patients?

What are the major portions of a tooth?

Trigeminal nerve (CN V)

Ophthalmic branch; Maxillarybranch; Mandibular branch

Dorsal nose and cornea

Forehead and scalp

Maxillary molar

First and second bicuspid

Maxillary central, lateral, andcuspid teeth

Hard palate (along with gingiva)

Midface, maxillary incisors, side ofnose, upper lip, and lower eyelids

1. Lidocaine2. Marcaine (longer acting)

1. Inferior alveolar nerve2. Posterior superior alveolar nerve3. Infraorbital nerve4. Supraorbital nerve

Supraperiosteal infiltrations

Vascular injury; Facial nervedamage (motor paralysis); Neuralinjury

Root; Crown

Page 90: Deja review _emergency_medicine__deja_review_

What key structure keeps the toothanchored into the alveolar bone?

What are some key points in the initialmanagement of an avulsed permanenttooth?

What is the key determinant of theviability of an avulsed tooth?

What is the reason why the root should notbe brushed or wiped?

What are two other management points toconsider?

Are deciduous (primary) teeth typicallyplaced back into the socket?

How are alveolar fractures typically noticed?

What are other possible dental injuriesfrom alveolar fractures?

What is typically done for alveolarfractures?

What are some important points andmanagement for the followingclassification of tooth fractures:

Ellis I

Ellis II

Ellis III

What is typically done for dental caries?

74 Deja Review: Emergency Medicine

Periodontal ligament

Hold tooth by crown and gentlywash root; Place tooth back intosocket; Do not brush root of thetooth; Immediate dental consultation

Time outside the socket

Preserving the periodontalligament is vital

1. Prophylactic antibiotics ifindicated

2. Tetanus status

No—alveolar ankylosis may result

Panorex film or evident on exam

Avulsion or subluxation of tooth;Dental fractures

Immediate dental consultation;Reduction and fixation (via wire);Antibiotics and tetanus whenindicated

Isolated enamel fracture; No pain;Elective treatment

Fracture of enamel; dentin exposed;Sensitive to temperature changes ofhot/cold; Calcium hydroxide pasteover dentin if <14 years of age;Dressing over tooth if >14 years ofage; Dental follow-up in timelymanner

Fracture of tooth with pulpexposure; Pink tinge may be seenon exam; This is a true dentalemergency; Immediately consult adentist and place wet cotton withdental or aluminum foil wrapped ifthere is a delay

Proper pain control and dentistreferral

Page 91: Deja review _emergency_medicine__deja_review_

What is a complication of dental caries toconsider?

What are some clinical features ofa periapical abscess?

What are some key points in themanagement of a periapical abscess?

ENT and Dental Emergencies 75

Periapical abscess

A fluctuant swelling, sharp/severepain when tooth is percussed, andtemperature sensitivity

I&D of the abscess; Antibioticcoverage (may or may not help);Dental referral

CLINICAL VIGNETTES

26-year-old with a recent URI presents witha fever, fatigue, and left ear pain, but isotherwise healthy; PE: left TM showsbullae and is erythematous in appearance

18-year-old male with no past medicalhistory (PMH) presents after beinginvolved in a bar fight and complains of abruise on his leg and some facial pain; PE:ecchymosis of left thigh and nasal swellingwith tenderness and crepitence

31-year-old female presents with a 1-weekhistory of sore throat with low-grade feversand fatigue. Patient mentions her sorethroat is getting progressively worse; PE:exudative pharyngitis with posteriorcervical adenopathy along with left upperquadrant (LUQ) tenderness

3-year-old infant presents with low-gradefever, decreased appetite and mothermentions that he is tugging at his ear; PE:decreased mobility of TM on pneumaticotoscopy

61-year-old female who just recentlyfinished her antibiotics for a UTI presentswith bilateral hearing loss, but is otherwisehealthy; PE: decreased hearing acuity andnormal Rinne and Weber test

3-year-old female was brought in bymother due to purulent drainage from leftnasal passage, but swears that the child didnot place any objects in the nose; PE:general exam was unremarkable

Bullous myringitis

Nasal fracture

Mononucleosis

Acute otitis media

Sensorineural hearing losssecondary to antibiotic use

Nasal foreign bodies

Page 92: Deja review _emergency_medicine__deja_review_

6-year-old male is brought in by mother forpersistent nosebleeds, but is otherwisehealthy with immunizations up-to-date;PE: child was actively picking his noseduring the exam

13-year-old female with a sore throat1 month ago now presents with fevers,joint pain, and what the mom notes as“weird movements”; PE: pain of jointswith movement and subcutaneous nodules

19-year-old male presents with severe leftear pain and complains of decreasedhearing with occasional purulentdischarge; PE: TM could not be visualizeddue to the purulent discharge in theexternal canal

9-year-old female presents with 3-dayhistory of low-grade fevers, sore throat,and fatigue but otherwise healthy; PE:cervical adenopathy with exudativepharyngitis

2-year-old female is brought in by mom forhigh fevers, sore throat, and some swellingof her neck; PE: sick-appearing child withunilateral bulge of posterior pharynx walland stridor

67-year-old female present with epistaxisthat began 2-hours earlier and has notstopped bleeding from conventionalmeans; PE: epistaxis that is refractory to allmethods that are used for anteriornosebleeds

43-year-old male with a recent history ofAOM that was not treated presentsotorrhea, pain around the ear, and amoderate headache; PE: tender posteriorauricular area and distorted TM

24-year-old female presents with a historyof nasal congestion, fever, purulent yellow-green discharge, and headache; PE:yellowish discharge from nose and tendermaxillary sinus

76 Deja Review: Emergency Medicine

Anterior nosebleed

Rheumatic fever

Otitis externa

Streptococcal pharyngitis

Retropharyngeal abscesses

Posterior nosebleed

Mastoiditis

Sinusitis

Page 93: Deja review _emergency_medicine__deja_review_

C H A P T E R 5

PulmonaryEmergencies

77

PNEUMONIA

What are some important things to knowabout bacterial pneumonia?

Name some other important bacterialagents in bacterial pneumonia.

Name some predisposing factors thatincreases susceptibility to bacterialpneumonia.

What are some clinical features of bacterialpneumonia?

What are some common physical findingsin a patient with bacterial pneumonia?

Name the most likely organism for each ofthe following scenarios:

Alcoholic who presents with fever,chills, and productive cough. Chestx-ray (CXR) shows lobar pneumonia

45-year-old male who has been in the ICUfor 2 weeks on vent support developsfever and chills with productive greensputum

It accounts for about 10–15% ofadmissions; Streptococcuspneumoniae is the most commonagent; Most common mechanism isaspiration

Pseudomonas aeruginosa; Hemophilusinfluenza; Staphylococcus aureus;Escherichia coli

Impaired immunity; Impaired gagreflex/mucociliary transport;Iatrogenic (i.e., endotracheal tube);Chest wall dysfunction

Fever, chills, productive cough,purulent sputum, and pleuriticchest pain

Crackles, wheezes, dullness topercussion, egophony, and tactilefremitus

Klebsiella pneumoniae

P. aeruginosa

Copyright © 2008 by The McGraw-Hill Companies, Inc. Click here for terms of use.

Page 94: Deja review _emergency_medicine__deja_review_

63-year-old male with a history of chronicobstructive pulmonary disease (COPD),DM, and debilitation presents withShortness of breath (SOB), fever, anda Chest x-ray (CXR) that shows patchyinfiltrates

36-year-old bird-breeder presents with a 3-day history of high fever, hackingcough, and severe headache

23-year-old farmer presents with a suddenonset of high fever, myalgias, and hackingcough. He mentions he often cleans atone of the slaughterhouses

41-year-old male presents with SOB,dyspnea, and productive cough recallsonset of symptoms after returning froma spa

37-year-old male who typically skinsrabbits presents with high fever, SOB,and hemoptysis

19-year-old patient with AIDS and a cellcount of <200 cells/mm3 presents withfever, nonproductive cough, and dyspnea

What are some diagnostic tests to considerin addition to the CXR?

What are some complications of bacterialpneumonia?

Name some of the most common agents inthe following age group:

Neonates

Children (5 weeks to 18 years)

Adults (18–40 years)

Adults (45 years and older)

What are some commonly used antibioticsin uncomplicated pneumonia?

78 Deja Review: Emergency Medicine

H. influenza

Chlamydia psittaci

Coxiella burnetii

Legionella pneumophila

Francisella tularensis

Pneumocystis carinii

Arterial blood gas (ABG); Sputumculture (typically for high-riskpatients); Blood culture

Abscess formation (esp. S. aureus);Sepsis; Empyema

Group B streptococci, E. coli, andC. pneumoniae

Respiratory syncytial virus (RSV),Mycoplasma pneumoniae,C. pneumoniae, and S. pneumoniae

M. pneumoniae, C. pneumoniae, andS. pneumoniae

S. pneumoniae, H. influenzae,anaerobes, and gram-negatives

Penicillin, macrolides, anddoxycycline

Page 95: Deja review _emergency_medicine__deja_review_

What are some commonly used antibioticsfor those with comorbidities?

Name three common causes of atypicalpneumonia?

What are some clinical features of atypicalpneumonia?

What are some important things to knowabout mycoplasma pneumonia?

What are some complications ofmycoplasma pneumonia?

What is the preferred antibiotic?

Name the most likely organism for each ofthe following scenarios involving viralpneumonia and the preferred treatment:

34-year-old with a history of a kidneytransplant presents with fever, cough,and a CXR showing interstitial infiltrates

2-year-old child presents with a 4-dayhistory of fever, chills, and coryza witha CXR that shows patchy infiltrates

21-year-old male presents with a 2-weekhistory of fever, chills, and nonproductivecough during winter

34-year-old female presents with fever,headache, and myalgia. She primarilyworks with rodents and is from Arizona

Pulmonary Emergencies 79

Fluoroquinolones

1. M. pneumoniae2. C. pneumoniae3. L. pneumophilia

Headache, fever, nonproductivecough, and myalgias

Most common cause of atypicalpneumonia 1–3 weeks incubation;Most common in ages 4–40 years;CXR often show a reticulonodularpattern

Splenomegaly; Aseptic meningitis;Encephalitis; Respiratory failure

Erythromycin; Tetracycline ordoxycycline are alternatives

Cytomegalovirus; Treatment (Tx):Ganciclovir or foscarnet

RSV; Tx: Primarily supportive

Influenza virus; Tx Amantadine

Hantavirus; Tx: Supportive/ribavirin

ASTHMA

What is the definition of asthma?

What are some important things to knowabout asthma?

It is a chronic conditioncharacterized by reversible airwayconstriction typically initiated bya variety of stimuli

More common in children andadolescents; Prevalence is increasing;Asthma-related morbidity is alsoincreasing

Page 96: Deja review _emergency_medicine__deja_review_

80 Deja Review: Emergency Medicine

Name some common triggers of asthma.

What are some of the clinical features ofasthma?

What are some important diagnostic teststo consider in asthma?

What are some key points in themanagement of asthma?

What are the three classes of drugs that arethe mainstay for the treatment of asthmaexacerbation?

What role does noninvasive positivepressure ventilation (NPPV) play?

What are some findings of impendingrespiratory failure?

What procedure should be considered inthe setting of impending respiratoryfailure?

What are some other agents that can beconsidered when the mainstay treatment ofasthma shows little improvement?

What are some important elements toconsider when deciding to admit thepatient?

Allergens; Exercise; Medications;Cold exposure

Dyspnea, cough, and wheezing

Pulmonary function tests (i.e.,PEFR); ABG (if impendingrespiratory failure); CXR (more torule out other conditions); ECG (ifyou suspect ischemia)

Ensure adequate oxygenation;Optimize lung function (i.e.,medication); Identify the cause ofexacerbation

1. Beta-adrenergic (albuterol)2. Anticholinergic (ipratropium)3. Corticosteroids (methylpred-

nisolone)

Impending respiratory failure wherethe patient is able to cooperate

Use of accessory muscles; Cyanosis;Altered mental status (typicallyfrom hypercapnia); No breathsounds (no or very little airflow)

Intubation

Magnesium sulfate; Heliox (heliumto improve airflow); Terbutamine

Social supports; Recent hospital-izations and past intubations;Compliance with medication;Severity of the exacerbation

CHRONIC OBSTRUCTIVE PULMONARY DISEASE

What are the disease elements that makeup chronic obstructive pulmonary disease(COPD)?

Emphysema; Asthma; Chronicbronchitis

Page 97: Deja review _emergency_medicine__deja_review_

What are some important features for eachof the following elements:

Emphysema

Chronic bronchitis

Asthma

What are some important risk factors forthe development of COPD?

What are some clinical features of COPD?

What are some common causes of COPDexacerbation?

What are some of the common clinicalfeatures of the following COPD variant:

Chronic bronchitis (blue bloaters)

Emphysema (pink puffer)

What are some possible findings on CXR?

What are some other diagnostic tests toconsider in COPD?

What two dysrhythmias are common inCOPD?

What are some key points in themanagement of COPD?

Pulmonary Emergencies 81

Irreversible airway destruction

Airway hypersecretion

Hyperactive airway andinflammation

Tobacco use (most common cause);Environmental pollution; Alpha1–antitrypsin deficiency; Cysticfibrosis

Dyspnea, cough, chest tightness,and occasional hemoptysis

Infections; Pulmonary embolism(PE); Congestive heart failure(CHF) exacerbation; Tobacco use

Tend to be heavy set; Normal chestdiameter; Productive wet cough

Tend to be thin; Increased AP chestdiameter; Dyspneic

Increased AP diameter; Overinflation;Presence of bullae

ABGs; ECG (for ischemia ordysrhythmias)

1. Multifocal atrial tachycardia2. Atrial fibrillation

Oxygenation is the cornerstone;Beta-adrenergic agonist;Anticholinergics; Corticosteroidsuse; Abx if signs of infection—purulent sputum

HEMOPTYSIS

What is the definition of hemoptysis? Coughing up of blood thatoriginates from the tracheobronchialtree or pulmonary parenchyma

Page 98: Deja review _emergency_medicine__deja_review_

What are some characteristics ofhemoptysis that help to distinguish it fromhematemesis?

What is the definition of massivehemoptysis?

What are some important causes ofhemoptysis?

What are some important causes of massivehemoptysis?

What are some key points in managementof hemoptysis?

82 Deja Review: Emergency Medicine

Bright red and foamy; Usuallypreceded by a vigorous cough;Lack of food particles

Coughing up of blood thattypically exceeds 50mL in a singleexpectoration or 500mL in a24-hour period

Infection and inflammation (mostcommon); Trauma; CA—especiallybronchogenic; Iatrogenic;Pulmonary embolism

Lung abscess; Bronchiectasis;Tuberculosis

Focus should be on underlyingcause; Ensure stability in case ofmassive hemoptysis as well assecuring airway

PLEURAL EFFUSION AND EMPYEMA

What is the definition of a pleuraleffusion?

What are some of the characteristics of thefollowing types of pleural effusion:

Transudate

Exudate

What are some clinical features of pleuraleffusion?

Which CXR view is more sensitive fordetecting pleural effusion?

What procedure is commonly used toanalyze pleural effusions?

What are commonly used pleural fluidstudies?

An abnormal accumulation of fluidin the pleural space

Increase in hydrostatic pressure;Decreased oncotic pressure; CHF isthe most common cause; Lowprotein infiltrate

Lymphatic blockage; Typically dueto malignancy and infection; Highprotein infiltrate

Pleuritic chest pain, cough, andSOB; PE: dullness to percussionand pleurisy

Lateral decubitus (as little as 5 mLseen)

Thoracentesis

Gram stain and cultures; Pleuralfluid lactate dehydrogenase (LDH)and protein; Serum LDH, protein,and glucose

Page 99: Deja review _emergency_medicine__deja_review_

Name some of the criteria used to supportthat a pleural effusion may be exudative?

Name some important causes oftransudative pleural effusion?

Name some important causes of exudativepleural effusion?

What is the definition of empyema?

What are some common causes ofempyema?

What are some clinical features ofempyema?

What are some diagnostic tests used todiagnosis empyema?

What are some complications of empyema?

What are some key points in themanagement of empyema?

Pulmonary Emergencies 83

Pleural fluid LDH >200 IU/mL;Pleural fluid cholestrol >60 mg/dL;Pleural fluid protein/serum protein>0.5

CHF; Low protein states (i.e.,cirrhosis); Peritoneal dialysis

Bacterial pneumonia (PNA); TB;Malignancy; Connective tissuedisorder (i.e., Systemic lupuserythematosus [SLE])

Collection of pus in the pleural space

Infections (i.e., gram negatives);Aspiration PNA; Iatrogenic (i.e.,chest tube)

Fever, chills, pleuritic chest pain,SOB, fatigue, and weight loss

CXR; Thoracentesis

Loss of lung tissue; Bronchopleuralfistula; Pleural adhesions

Pleural drainage via chest tube;Broad spectrum Abx; Thoracoscopy(controversial)

LUNG ABSCESS

What is the definition of a lung abscess?

What is the most common cause of a lungabscess?

What class of bacteria are typicallyinvolved in a lung abscess?

What are some clinical features of a lungabscess?

What are some important diagnostic teststo consider in a lung abscess?

What are some complications of a lungabscess?

It is a cavitation of the lungparenchyma due to central necrosis

Aspiration

Typically mixed anaerobic andgram (−) bacteria

Weakness, fever, SOB, pleuriticchest pain, putrid sputum, andhemoptysis

Complete blood count (CBC); CXR(shows cavitation); Sputum stain

Empyema; Bronchopleural abscess;Chronic lung abscess

Page 100: Deja review _emergency_medicine__deja_review_

What are some key points in managementof a lung abscess?

84 Deja Review: Emergency Medicine

Abx therapy (Clindamycin pre-ferred); Surgery if cause is a tumoror fistula

TUBERCULOSIS

What are some important points to knowabout tuberculosis (TB)?

What is the pathophysiology of infectionfrom M. tuberculosis?

What is the primary determinant ofwhether the infection is contained or likelyto spread?

What are some factors that are associatedwith an increase in reactivation?

What are some of the clinical features foreach of the following TB states:

Primary

Secondary (reactivation)

Name the four most common sites ofextrapulmonary involvement.

What are some important diagnostic teststo consider in TB?

What is the criteria for a positive PPD test?

The incidence of TB is rising (esp.in AIDS patients); Top cause of infec-tious death worldwide; Transmissionis primarily respiratory

Obligate aerobic rod (acid-faststaining) that is phagocytized bymacrophages, but not killed andallowed to grow (albeit slowly)

Immune status (lifetime risk ofactivation is still 10% in the generalpopulation)

DM; Immunocompromised(i.e., AIDS); Transplant recipient;Malignant disease

Asymptomatic in most patients;Positive TB test primary way todetect; Sometimes Ghon complexon CXR

Constitutional symptoms (i.e.,weight loss); Productive cough;Hemoptysis; Up to 20% haveextrapulmonary features

1. CNS (TB meningitis)2. Vertebral bodies (Pott’s disease)3. Liver4. Psoas muscle

CXR; Sputum (acid-fast bacilli);Purified protein derivative (PPD) test

Less than 5-mm induration forimmunocompromised patients (i.e.,AIDS); Less than 10-mm indurationfor high-risk individuals (IV drugabusers and immigrants from high-risk areas); Less than 15-mminduration in healthy individuals

Page 101: Deja review _emergency_medicine__deja_review_

Pulmonary Emergencies 85

What is a common cause of a false negativePPD?

What is a common cause of a false positivePPD?

What are some key points in themanagement of TB?

What are the main side effects for each ofthe drug used to treat TB (RIPE):

Rifampin

Isoniazid (INH)

Pyrazinamide (PZA)

Ethambutol

Anergy

Infection with a mycobacteriumspecies such as M. avium

Isolation once TB is suspected;Multidrug therapy for more than6 months; Baseline liver/kidneytest and visual acuity

Orange-colored urine, tears, andsaliva; Increase P450 activity

Hepatitis; Neuropathy (givevitamin B6)

Hepatitis; Hyperuricemia; Arthralgias

Optic neuritis; Rash

SPONTANEOUS PNEUMOTHORAX

What is the definition of a spontaneouspneumothorax?

What are some important things to knowabout each of the different type ofspontaneous pneumothorax:

Primary spontaneous pneumothorax

Secondary spontaneous pneumothorax

What are some clinical features ofspontaneous pneumothorax?

What is the diagnostic test of choice?

What are the key points in the managementof spontaneous pneumothorax?

It is collection of air into the pleuralspace (assuming that no trauma isinvolved)

Typically occurs in healthy individ-uals; Most have a history of smoking;Results from rupture of a bleb

Typically will have underlying lungdisease; COPD and asthma mostcommon cause; Usually patients areolder than 45 years

Sudden onset of dyspnea andpleuritic chest pain. PE: decreasedbreath sounds and hyperresonanceon the affected side

CXR

All patients should receive oxygen;Observation and serial CXR ifsmall; If large/expanding it is equalto the chest tube

Page 102: Deja review _emergency_medicine__deja_review_

What is a feared complication ofspontaneous pneumothorax?

What are some clinical features of tensionpneumothorax

What are some key points in themanagement of tension pneumothorax?

What are some more specifics of chestdecompression in the management oftension pneumothorax?

86 Deja Review: Emergency Medicine

Tension pneumothorax

Hypotension, absent breath soundson the affected side, jugular venousdistension (JVD), and tracheadeviation

Immediate chest decompression(14 gauge); Follow with chest tubeplacement

If any evidence of tension pneumoth-orax, immediate needle decom-pression should be done with aneedle placed into the second andthird intercostal space at the anterioraxillary line followed by a chesttube in the fifth intercostal space inthe mid-axillary line

CLINICAL VIGNETTES

10-year-old child with history of allergiespresents with acute respiratory distress witha recent history of chronic coughing, but hasotherwise been healthy; PE: tachypnea,intercostal retractions, and audiblewheezing; CXR: hyperinflation of lung

63-year-old female with a long history ofsmoking comes in via Emergency medicalservices (EMS) in acute respiratory distresswith a recent illness per report of familymembers; PE: hyperresonant chest,decreased breath sounds bilateral; CXR:hyperinflation of lung and small infiltrateof right lower lobe

81-year-old female with a long history ofsmoking presents with a 2-week history ofworsening hemoptysis, but otherwise ishealthy except for a 20-lb weight loss in amonth period; CXR: a spiculated mass isseen on the left side

45-year-old male with an Hx of CHFpresents with SOB and wet cough, butotherwise has been doing well; PE: dullnessto percussion and pleurisy; lateral decubitusCXR: showed dependant fluid collection

Bronchial asthma

COPD exacerbation

Bronchogenic cancer

Pleural effusion

Page 103: Deja review _emergency_medicine__deja_review_

Pulmonary Emergencies 87

35-year-old alcoholic the ED for alcoholicintoxication presents with fever,hemoptysis, SOB, and purulent sputumwho was recently seen in; PE: lung fieldsrelatively clear; CXR: central cavitation

23-year-old lanky male with a smokinghistory presents with sudden onset ofdyspnea and pleuritic chest pain, but isotherwise healthy; PE: an area ofhyperresonance on the left side

31-year-old male with Hx of HIV presentswith hemoptysis and recent weight lossalong with a CD4+ count of <200; PE:cachetic appearance, is actively coughing,but otherwise unremarkable exam

56-year-old male with recent “flu” presentswith a 2-day history of fever, chill,productive cough, and pleuritic chest pain;PE: ill-appearing patient, crackles,wheezes, and dullness to percussion ofright lung

Lung abscess

Spontaneous pneumothorax

Tuberculosis

Community-acquired pneumonia

Page 104: Deja review _emergency_medicine__deja_review_

This page intentionally left blank

Page 105: Deja review _emergency_medicine__deja_review_

C H A P T E R 6

CardiovascularEmergencies

89

ACUTE CORONARY SYNDROME

What is acute coronary syndrome (ACS)?

What are the three clinical presentationsthat cover the ACS spectrum?

What are three non-modifiable risk factorsassociated with development of ACS?

What are four modifiable risk factorsassociated with the development of ACS?

What are some clinical features of thefollowing presentations of ACS:

Stable angina

Unstable angina

It is a continuum of presentationsof coronary artery disease wherethe symptoms are due to myocardialischemia. The underlying cause ofACS is an imbalance betweendemand and supply of myocardialoxygen

1. Unstable angina2. Non–ST-elevation MI (NSTEMI)3. ST-elevation MI (STEMI)

1. Gender2. Age3. Family history

1. Cholesterol2. Hypertension3. Diabetes4. Smoking

Episodic pain that is transient andpredictable, typically reproducibleon exertion and improves with restor use of nitro

New-onset angina that can beexertional or at rest, different fromprevious stable angina, increasedfrequency of attack or increasedresistance to relief such as nitro

Copyright © 2008 by The McGraw-Hill Companies, Inc. Click here for terms of use.

Page 106: Deja review _emergency_medicine__deja_review_

Myocardial infarction (MI)

What are some clinical features of atypicalMI?

What population group can frequentlypresent with atypical symptoms?

What are three important elements inthe patient’s presentation to consider inan MI?

What is the single most importantdiagnostic test to obtain in a patient withsuspected MI?

What are other uses of the initial ECG?

What are some important things to noteabout the use of a ECG?

90 Deja Review: Emergency Medicine

Substernal chest discomfort thatlasts longer than 20 minutes,typically ssociated with nausea,vomiting, dyspnea, diaphoresis,and radiation to arms/jaw/back

Vague chest discomfort/pressure,nausea and vomiting, short ofbreath, confusion, dizziness,abdominal pain, weakness, orsyncope

Diabetics; Women; Elderly;Neurological dysfunction(i.e., cord injury)

1. History and physical2. Cardiac enzymes3. ECG

ECG (within 10 minutes of arrival)

Screening other disease processessuch as pulmonary embolism (PE)and pericarditis

Initial ECG is diagnostic 50% of thetime; Serial ECGs are more usefulfor evolving MI; Comparison witha previous ECG is important

ECG Infarct Region

I: Lateral aVR: V1: Septal V4: AnteriorII: Inferior aVL: Lateral V2: Septal V5: LateralIII: Inferior aVF: Inferior V3: Anterior V6: Lateral

What are some ECG findings in an patientwho presents with a STEMI?

What are some common complications forthe following infarction location:

Inferior

Inverted T-waves; Q-waves; STsegment elevation >1 mm in two ormore contiguous leads; Left bundlebranch block

Increased vagal tone; Bradyarrhymiasare more common; High associationwith right ventricular wall infarct

Page 107: Deja review _emergency_medicine__deja_review_

Cardiovascular Emergencies 91

Greater risk of left ventricular (LV)dysfunction

Greater risk of LV dysfunction(CHF); Conduction abnormalities

Hypotension (preload dependent);Cardiogenic shock

Initial level cannot be used toexclude MI; Serial levels are moreuseful; Detection requires enoughtime/tissue death

Lateral

Anterior

Right ventricular

What are limitations concerning the role ofbiomarkers used in the diagnosis of MI?

Cardiac Enzymes Initial Elevation Peak Return to Baseline

Troponins 2–6 hours 12–16 hours 5–14 daysCK-MB 4–6 hours 12–24 hours 2–3 daysMyoglobin 2 hours 6–8 hours 3–4 days

What is MONA?

What treatment within MONA is clearlyshown to improve morbidity and mortalityof ACS and should always be given(assuming no contraindications)?

What else should be done for all patientswith suspected ACS?

What are some important points for thefollowing treatments used in ACS:

Aspirin (ASA)

Glycoprotein IIb/IIIa inhibitors

It is the initial treatment for allpatients with suspected ACS

Morphine

Oxygen

Nitro

Aspirin

Aspirin

IV-O2-Monitor

Antiplatelet medication; Should begiven within 4 hours of chest painonset; Clearly shown to improveoutcome

Abciximab, tirofiban, and eptifibatide;Platelet inhibitors; Used prior topercutaneous coronary intervention;Also indicated in some cases ofNSTEMI

Page 108: Deja review _emergency_medicine__deja_review_

92 Deja Review: Emergency Medicine

ADP-receptor inhibitors

Heparin

Beta-blockers

Nitroglycerin

Morphine

What is the treatment for choice forSTEMI?

What are some commonly usedthrombolytics in AMI?

What is the most serious complication oflytics?

Which is the preferred reperfusionmodality?

Should thrombolytics be withheld if PCI isanticipated?

Clopidogrel and ticlopidine; Alsoprevents platelets aggregation;Second-line if ASA cannot be used:Reduce risk of recurrence inpatients with recent MI or stroke

Antithrombin III inhibitor; Patientswith ACS (UA/NSTEMI/ STEMI);Decrease reinfarction, deep veinthrombosis (DVT), LV thrombus;Adverse drug reactions includebleeding complications andheparin-induced thrombocytopenia

Improved outcome in acute MI;Should be given in acute MIassuming no contraindications;Should be given within 2–3 hours;Contraindications include high-degree heart block, bradycardia,severe CHF

Decreases preload/dilates coronaryarteries; Should be given in ischemicchest pain; Avoid if hypotensive andif on sildenafil

Decreases anxiety, preload, andafterload; Should be given if painpersists after nitros; Can causehypotension/decrease respiratorydrive

Reperfusion therapy

“Door to balloon time” 90minutes (PCI) or “door to lytics”30 minutes

Streptokinase (not commonly used);Tissue plasminogen activator;Tenecteplase

Intracranial hemorrhage (ICH)

PCI is associated with slightlybetter outcomes, lower incidence ofreinfarction, and death

They should not be withheld iftransfer to a cath lab will be greaterthan 90 minutes despite betteroutcomes with PCI

Page 109: Deja review _emergency_medicine__deja_review_

Cardiovascular Emergencies 93

CONGESTIVE HEART DISEASE AND PULMONARY EDEMA

What is the definition of congestive heartfailure (CHF)?

What are the four classifications commonlyused in CHF:

Class I

Class II

Class III

Class IV

How is congestive heart failure classified?

What are other some clinical features ofleft ventricular failure?

What are some common causes of leftventricular failure?

What is cardiogenic pulmonary edema?

A pathophysiologic state in which,at normal filling pressures, theheart is incapable of pumping asufficient supply of blood to meetthe metabolic demands of the body

Not limited with normal physicalactivity by symptoms

Ordinary physical activity results infatigue, dyspnea, or other symptoms

Marked limitation in normalphysical activity

Symptomatic at rest or with anyphysical activity

While many classification methodsexist (high output vs. low; systolicvs. diastolic), a useful clinicalconstruct is the distinction of leftversus right heart failure

Nocturnal angina, paroxysmalnocturnal dyspnea, orthopnea,fatigue, diaphoretic, and anxious.PE: rales/wheezes, S3 or S4 gallop,tachycardia and tachypnea, andpulsus alternans

Ischemic heart disease (no. 1 cause);HTN; Valvular heart disease; Dilatedcardiomyopathy

Acute presentation of left heartfailure resulting from an imbalancein pulmonary vascular hydrostaticand oncotic forces and leading totransudation of fluid into thepulmonary interstitium

What are important complications toconsider in AMI?

Arrhythmias (esp. ventricularfibrillation); Cardiac rupture;Congestive heart failure; Septalrupture

Page 110: Deja review _emergency_medicine__deja_review_

94 Deja Review: Emergency Medicine

What is the most common cause of rightheart failure?

What are some other common causes ofright heart failure?

What are some common physical examfindings of right ventricular failure?

What are some precipitating factors ofacute pulmonary edema?

Name some common radiographic findingson a CXR in acute pulmonary edema?

Does a normal CXR exclude acutepulmonary edema?

What is β-type natriuretic peptide (BNP)?

How is BNP used clinically?

What is the single most important agent forthe treatment of acute CHF?

Patients with decompensated left heartfailure frequently require assistance inmaintaining adequate oxygenation/ventilation. What are the treatment options:

High-flow via nonrebreather mask

Noninvasive positive pressure ventilation

Left heart failure

Pulmonary hypertension;Pulmonary embolism (PE); Chronicobstructive pulmonary disease(COPD); Right ventricular infarct

Neck vein distension, ascites,dependent edema, andhepatojugular reflux

Myocardial ischemia; High sodiumdiet; Noncompliance withmedications; Dysrhythmias; COPD(chronic cor pulmonale)

Generally an enlarged cardiacsilhouette, pleural effusions,cephalization (vascularredistribution to upper lung fields),and bilateral perihilar infiltrates

No—CXR findings may be delayedup to 12 hours after symptom onset

Cardiac myocytes secrete BNP inresponse to the high atrial andventricular filling pressures

Increasing use as a serum markerfor CHF; Levels of <100 pg/mlreliably exclude acute CHF; Highnegative predictive value with lowBNP

Oxygen

Optimal option to deliver 100%oxygen; Used to maintain adequateoxygen saturation; Commonly usedto avoid hypoxia

Continuous positive airwaypressure (CPAP) and bilevelpositive airway pressure (BiPAP)commonly used; Improvesoxygenation and dyspnea; Earlyuse helps avoid intubation

Page 111: Deja review _emergency_medicine__deja_review_

Endotracheal intubation

What are some key points in themanagement of acute decompensated leftheart failure?

Cardiovascular Emergencies 95

Final pathway if other methods fail;Typically used for the followingconditions:

Cannot maintain PaO2 above60 mm Hg; Obtunded;Progressive increase in CO2;Increasing acidosis

IV-O2-monitor; NTG/furosemide/morphine first-line agents; Carefullymonitor for hypotension as well

DEEP VENOUS THROMBOSIS AND PULMONARY EMBOLISM

What is Virchow’s triad?

What is the biggest risk factor for thedevelopment of deep venous thrombosis(DVT)?

List some other important risk factors forthe development of a DVT.

How does the number of risk factorseffect the likelihood of a DVT?

What determines the clinical presentationof a DVT?

What are some common clinical features ina DVT?

What is the most reliable finding onphysical exam for a DVT?

Can a DVT be diagnosed by physical examalone?

Factors predisposing to vascularthrombosis with risk of pulmonaryembolism:

Hypercoagulability; Vessel wallinjury; Venostasis

Prior history of DVT

Cancer; Pregnancy and postpartum;Recent trauma and surgery; Estrogentherapy; Obesity; Protein C and Sdeficiency

Greater number of RFs = higherrisk of DVT

Degree of occlusion; Location ofocclusion; Extent of collaterals

Unilateral leg swelling, tenderness,edema, discoloration, palpablecord, and Homans’ sign

Unilateral leg swelling with morethan 3-cm difference from the otherleg

Due to variability of presentation, itcannot be used to exclude or makethe diagnosis

Page 112: Deja review _emergency_medicine__deja_review_

96 Deja Review: Emergency Medicine

What are some characteristics of commonlyused ancillary testing for the diagnosis of aDVT:

D-dimer assay

Duplex ultrasonography

MRI

Contrast venography

What are the goals of treatment for a DVT?

What are some commonly usedanticoagulants for a DVT?

What are some indications for the use of aGreenfield umbrella filter?

What are general indications for admissionfor patients with a DVT?

What is the epidemiology of a pulmonaryembolism?

Fibrin degradation product is witha DVT/PE; Other conditions raise itsuch as CA or recent surgery; Moresensitive for proximal clots; Apositive result may require furthertesting

Initial diagnostic test in many cases;Ideal for patients who are pregnant,diabetic, or have a contrast allergynoninvasive; Highly sensitive/specific for proximal DVT; Lesssensitive for deep vein, pelvic, andIVC thrombosis

MRI rarely used in the ED; Highlysensitive/specific for a DVT;Noninvasive but expensive; Candetect pelvic, renal, and calfthrombi; Useful for second/thirdtrimester pregnancy

Once the gold standard diagnostictest; Invasive/painful andrequires contrast; Very highsensitivity/specificity

To prevent PE; Prevent post-phlebitic syndrome

Heparin; Low-molecular-weightheparin (LMWH); Warfarin

Contraindication to anticoagulation;Urgent surgery (cannot anticoagulateprior); Anticoagulation has failed(still clotting)

Limited cardiopulmonary reserve;IV heparin use (contraindications toLMWH); Poor compliance withmedications

Third most common cause of deathin the United States; Most commonpreventable death in the hospitalsetting; Up to 1/3 of PEs areundiagnosed

Page 113: Deja review _emergency_medicine__deja_review_

Cardiovascular Emergencies 97

What is a major source of a PE?

What are some other possible sources of aPE?

What are some risk factors for thedevelopment of a PE?

What is the most common symptom of a PE?

What are some common clinical features ofa PE?

What is the classic triad of a PE?

What are some commonly used screeningtests for a PE?

What are some functions of these screeningtests?

What are some common findings in anarterial blood gas for a patient with PE?

Does a normal A-a gradient, normal PO2,and normal vital signs exclude a PE?

What is the most common CXR finding ina patient with suspected PE?

What are some radiographic abnormalitiesthat can be seen on CXR in a PE?

What is Hampton’s hump?

What is Westermark’s sign?

How common are Hampton’s hump andWestermark’s sign?

What are some common findings on ECGof a patient with a PE?

Venous thrombi from lowerextremities and pelvis

Renal and ovarian veins;Paradoxical left-to-right shunts;Right side of heart

The same as those for DVT

Dyspnea

Pleuritic chest pain, hemoptysis,cough, tachycardia, sweating,elevated temperature, and syncope/hypotension in massive PE

Pleuritic chest pain; Dyspnea;Hemoptysis

ABG; CXR; ECG; D-dimer

Excluding other disease processes;May support the diagnosis of a PE;Should not be used to rule out orrule in PE

PO2 <80 mm Hg; Mild respiratoryalkalosis; Elevated alveolar-arterial(A-a) gradient

No

Normal CXR

Elevated hemidiaphragm;Atelectasis; Small pleural effusion

Triangular density with arounded apex that points towardthe hilum representingpulmonary infarction

Regional oligemia

Rare—if present highly suggestiveof PE

Sinus tachycardia (most commonfinding); Evidence of right heartstrain (S1, Q3, T3); Transientnonspecific ST-T wave changes

Page 114: Deja review _emergency_medicine__deja_review_

98 Deja Review: Emergency Medicine

What are some characterstics of commonlyused testing in PE:

Spiral CT

Ventilation-Perfusion (V/Q) scan

Pulmonary angiography

What are the treatment goals for a PE?

What are some key points in the initialmanagement of PE?

What are some commonly usedanticoagulants for PE?

What are two complications of heparin andLMWH?

What are two commonly usedthrombolytics for PE?

What is an important indication for the useof thrombolytics in the setting of a PE?

Extensively used diagnostic test;Done within minutes/assess otherpossible disease processes; Alsoused if V/Q read as indeterminate;Disadvantage: contrast allergy,radiation, and risk of acute renalfailure

Commonly used as a screening test;Normal V/Q scan virtually excludesPE; Must also look at clinicalprobability; Typically either read asnormal, indeterminate, or highprobability; If indeterminate itimplies further testing (i.e., CTA)may be required

Considered the gold standard forthe diagnosis of a PE; It is invasive,not available everywhere, andcarries a small mortality risk;Complications more common inelderly patients

Prevent recurrent PEs; Eliminateany thrombi in the pulmonaryvasculature

IV-O2-monitor; If present withshock: fluids/ inotrophic agent;Anticoagulation is the cornerstoneof treatment

Heparin; Low-molecular-weight heparin (LMWH)

1. Thrombocytopenia2. Hemorrhage

1. Streptokinase2. Tissue plasminogen activator

(TPA)

Hemodynamic instability

CARDIOMYOPATHIES

What is cardiomyopathy? Disease of the myocardiumassociated with cardiac dysfunction

Page 115: Deja review _emergency_medicine__deja_review_

Cardiovascular Emergencies 99

What are some classifications ofcardiomyopathies?

What diagnostic test is commonly used toevaluate cardiomyopathies?

What is the definition of dilatedcardiomyopathy?

What is commonly associated with dilatedcardiomyopathy?

What are some other important causes ofdilated cardiomyopathy?

What are some clinical features of dilatedcardiomyopathy?

What are some common findings in thefollowing diagnostic tests used for dilutedcardiomyopathy:

ECG

CXR

Echocardiogram

What are the key points in themanagement of dilated cardiomyopathy?

What are commonly used agents inalleviating symptoms of dilatedcardiomyopathy?

What is the definition of restrictivecardiomyopathy?

Dilated cardiomyopathy;Hypertrophic cardiomyopathy;Restrictive cardiomyopathy

Echocardiographic evaluation

Dilatation and impaired contractionof one or both ventricles, affectedpatients have impaired systolicfunction and may or may notdevelop overt heart failure

Viral myocarditis

Idiopathic; Toxins (esp. ethanol/cocaine/lithium); Peripartum;Nutritional deficiencies (thiaminedeficiency)

Signs and symptoms of right andleft-sided heart failure such asexertional fatigue, dyspnea, JVD,orthopnea, and ascites

Poor R-wave progression; Atrial orventricular enlargement; AV block;Atrial fibrillation most commondysrhythmia

Cardiomegaly; Pulmonary venouscongestion

Decreased ejection fraction; Enlargedheart chambers; Mural thrombi;Abnormal ventricle contraction

Alleviation of symptoms;Anticoagulation if mural thrombior in afib

Diuretics, vasodilators, anddigitalis

Nondilated ventricles withimpaired ventricular filling due todiastolic restriction

Page 116: Deja review _emergency_medicine__deja_review_

100 Deja Review: Emergency Medicine

What are some important causes ofrestrictive cardiomyopathy?

What are some clinical features ofrestrictive cardiomyopathy?

What are some common findings in thefollowing diagnostic tests in restrictivecardiomyopathy:

ECG

CXR

Echocardiogram

What are some points in the managementof restrictive cardiomyopathy?

What is the definition of hypertrophiccardiomyopathy?

What is the most common cause ofhypertrophic cardiomyopathy?

What is the most common presentingsymptom of hypertrophic cardiomyopathy?

What are some other clinical features ofhypertrophic cardiomyopathy?

Amyloidosis; Endomyocardialfibrosis; Hemochromatosis; Type IIglycogen storage disease

Similar to constrictive pericarditis:often will have symptoms of right-sided CHF with exercise intolerancebeing very common. PE: abnormalheart sounds (S3/S4 gallop),dependent edema, andrales/wheezes

Commonly show afib; NonspecificST-T wave changes; Low voltage

Cardiomegaly can be seen; Mayinitially show a normal heart

Normal systolic function;Thickened wall; Atria size is greaterthan ventricle size

Commonly use diuretics/digitalisfor relief; Vasodilators may deceaseafterload; Diagnosis is confirmedwith biopsy

Left ventricular hypertrophywithout dilation that often resultsin impaired diastolic relaxationand can result in decreased cardiacoutput

50% is autosomal dominantinherited

Dyspnea on exertion

Syncope, dysrhythmias (afib mostcommon), ischemic chest pain, andsudden death (esp. from ventricularfibrillation. PE: systolic ejectionmurmur especially with valsalva,rapid biphasic carotid pulse, andprominent A wave of neck veins

Page 117: Deja review _emergency_medicine__deja_review_

What are some common findings in thefollowing diagnostic tests:

ECG

CXR

Echocardiogram

What are some components in themanagement of hypertrophiccardiomyopathy?

What treatment is reserved for severelysymptomatic patients who fail medication?

Cardiovascular Emergencies 101

Afib and PVCs are common;Changes in anterior, inferior, orlateral leads; Left ventricularhypertrophy (LVH)

Typically normal

LVH especially with septalhypertrophy; Small left ventricularchamber

Beta-blockers are the mainstay forsymptom relief; Calcium (channelblocker in select patients);Amiodarone for ventriculardysrhythmias; Avoid inotropicagents; Anticoagulation for afib

Septal myomectomy

ENDOCARDITIS

What is the definition of endocarditis?

What is the pathophysiology ofendocarditis?

What are some risk factors of endocarditis?

Name two sites that commonly allow entryof bacteria in endocarditis.

What are some classes of organismsinvolved in endocarditis?

What are the top three cause ofendocarditis in the following situations:

IVDA or immunocompromised

Localized infection of theendocardium that is typicallycharacterized by vegetations

Any injury to the endocardium canresult in platelet-fibrin complexthat can be colonized by organismssuch as bacteria or fungus

Prosthetic valves; Intravenous drugabuse (IVDA); Any acquired orcongenital valvular lesions;Indwelling lines (i.e., shunts orcatheters); Hemodialysis orperitoneal dialysis

1. Oral cavity2. Genitourinary tract

Bacteria (most common); Fungi;Viruses; Rickettsiae

Streptococcus species; S. aureus;Gram-negative bacteria

Page 118: Deja review _emergency_medicine__deja_review_

102 Deja Review: Emergency Medicine

Normal valves

Prosthetic valves

What are some important things to knowabout right-sided endocarditis?

What are some important things to knowabout left-sided endocarditis?

What are some clinical features ofendocarditis?

What are Janeway lesions?

What are Osler’s nodes?

What are some important diagnostic teststo consider in evaluating endocarditis?

Summarize the management ofendocarditis?

What factors decide which antibioticregiment to use?

Streptococcus viridans; S. aureus;Enterococci

Staphylococcus (coagulasenegative); Streptococcus viridans;S. aureus

Commonly involves the tricuspidvalve; Typically an acutepresentation; Very common inIVDA; S. aureus most commonagent

Commonly involves the mitralvalve; More common in valvulardefects; S. viridans and S. aureusmost common

Typically nonspecific such asfever, fatigue, weight loss,neurologic complaints, and chestpain. PE: heart murmur, seedingto other sites such as lung (PNA),cutaneous signs (i.e., petechiae),and eye findings (i.e., conjunctivalhemorrhages)

Nontender and small erythematous/hemorrhagic nodules in the palms orsoles, which are pathognomonic ofinfective endocarditis. The pathologyis due to a type III hypersensitivityreaction

Painful, red, raised lesions on thefinger pulps that are indicative ofsubacute bacterial endocarditis(can be seen elsewhere such assystemic lupus erythematosus

Blood culture: positive in mostcases; CBC; CXR; ESR/C-protein:often elevated; Echocardiography:often show vegetations

Empiric Abx typically after culturesdrawn; Most patients are typicallyadmitted

The stability of the patient;Resistance of the organisminvolved; Acuteness of thepresentation

Page 119: Deja review _emergency_medicine__deja_review_

Cardiovascular Emergencies 103

MYOCARDITIS

What is the definition of myocarditis?

Name some examples for the followingcauses of myocarditis:

Viruses (most common cause)

Bacteria

Parasites

Drugs/Toxins

Systemic diseases

What are some clinical features ofmyocarditis?

What history is common to those whopresent with myocarditis?

What are some common findings for thefollowing diagnostic studies that may beused to evaluate myocarditis:

ECG

Echocardiography

Inflammation of the muscles of theheart, often due to infection that isalso often associated with acutepericarditis

Coxsackie A and B; Poliovirus; CMV

N. meningitidis; Beta-hemolyticstreptococcus; C. diphtheriae

Chagas disease; Trichinosis;Toxoplasmosis

Cocaine; Inhalants; Methyldopa

Lupus; Kawasaki syndrome;Sarcoidosis

Highly variable depending ondegree of cardiac involvement thatcan range from chest pain, signs ofheart failure, to dysrhythmias andtachycardia. PE: S3/S4 gallop,pericardial friction rub (if pericarditispresent), and various murmurs

Preceding viral illness in manycases

Any type of dysrhythmias may bepresent; Low-voltage QRS;Nonspecific ST-T wave changes

Dilated chambers; Focal wallmotion abnormalities

What are some commonly used antibioticsfor patients with endocarditis?

What are some conditions that require theuse of prophylaxis for endocarditis?

Aminoglycoside (i.e., gentamicin);Vancomycin; Rifampin

Prosthetic heart valves; Anyacquired or congenital valvularlesions; Any congenitalmalformation; Hypertrophiccardiomyopathy

Page 120: Deja review _emergency_medicine__deja_review_

What is the primary presentation ofpericardial disease?

What are the two most common causes ofpericardial disease?

List some other important causes ofpericardial disease.

What are some clinical features ofpericarditis?

What physical finding is pathognomonicfor percarditis?

What is the best way to elicit a pericardialfriction rub?

What is the most common ECG finding?

104 Deja Review: Emergency Medicine

The principal manifestations ofpericardial disease are pericarditisand pericardial effusion

1. Infections (i.e., Coxsackie virusesA and B)

2. Idiopathic

Rheumatologic disease (i.e., lupus);Cancer (i.e., metastatic); Radiation;Cardiac injury (i.e., post MI);Medication (i.e., hydralazine)

Sharp inspirational chest pain thatis relieved when leaning forward,low-grade fever, and dyspnea

Pericardial friction rub

Sitting and leaning foward

Sinus tachycardia (dysrhythmiasare rare)

CXR

Cardiac enzymes

In what case should one suspectmyocarditis?

How is myocarditis confirmed incombination with clinical history?

What are some components in themanagement of myocarditis?

Typically normal; May showcardiomegaly; May also showpulmonary edema

Unlike AMI, they will rise and fallslowly; Elevations of cardiactroponin I or T more common thanCK-MB

A young healthy male who presentswith unexplained cardiacabnormalities, especially if recenthistory of viral infections

Endomyocardial biopsy

Primarily supportive; Abx ifbacterial cause is suspected; Avoidsteroids/NSAIDs in early course;IVIG may be useful in pediatricpatients, especially with Kawasakisyndrome; Intensive care unit insevere cases

PERICARDIAL DISEASE

Page 121: Deja review _emergency_medicine__deja_review_

Cardiovascular Emergencies 105

What are some common findings for thefollowing diagnostic studies that may beused to evaluate percarditis:

CBC

ESR/CRP

Cardiac enzymes

ECG

CXR

Echocardiography

What are some key points in themanagement of percarditis?

What are some guidelines to admit patientswith pericarditis?

What is the most serious complication ofpericardial disease?

What is Beck’s triad?

What are some other common clinicalfeatures of cardiac tamponade?

What are some common ECG findings incardiac tamponade?

What are some important differentials toconsider in patients with JVD andhypotension?

Often show an elevated whitecount

Typically elevated due toinflammation

May be mildly elevated; Oftenincrease in setting of myocarditis

Diffuse ST-segment elevation;Reciprocal ST segment depressionin aVR and V1; PR segmentdepression; Diffuse T waveinversion—late finding

Typically normal; May showenlarged silhouette if pericardialeffusion >200 mL

Test of choice to evaluate effusion;Echo can also assess cardiacfunction; Can detect as little as 15mL of effusion

Treat the underlying cause; Paincontrol with NSAIDs commonlyused; Monitor for tamponade andtap if needed

Serious underlying cause (i.e., MI);Severe pain refractory tomedication; Most can be managedon a outpatient basis

Cardiac tamponade

Jugular venous distension (JVD);Hypotension; Muffled heart sounds

Dyspnea, narrow pulse pressure,pulse paradoxus, and tachycardia

Low QRS voltage; Total electricalalternans (beat-to-beat alternatingpattern)—not always present

Cardiac tamponade; Tensionpneumothorax; Massive pulmonaryembolism

Page 122: Deja review _emergency_medicine__deja_review_

106 Deja Review: Emergency Medicine

What are some causes of tricuspid stenosis?

What is important to note about tricuspidstenosis?

What are some common clinical features oftricuspid stenosis?

What is the most common dysrhythmiaassociated with tricuspid stenosis?

What are some common findings for thefollowing diagnostic studies that may beused to evaluate tricuspid stenosis:

CXR

ECG

What are some key points in themanagement of tricuspid stenosis?

Endocarditis secondary to IVDA;Rheumatic fever; Congenitaltricuspid atresia; Carcinoidsyndrome

Tricuspid stenosis often coexist withother valvular disease (i.e., mitralstenosis)

Systemic venous congestion, fatigue,and dyspnea in some cases. PE:diastolic murmur, ascites, and JVD

Atrial fibrillation

May show an enlarged right atrium

Tall and pointed P-waves; Afib ifpresent

Treat for afib (rate control/anticoagulate); Antibioticprophylaxis when indicated

VALVULAR DISEASE

Tricuspid Stenosis

Tricuspid Regurgitation

What are some causes of tricuspidregurgitation?

What are some common clinical features oftricuspid regurgitation?

Rheumatic fever; RV dilation due topulmonary HTN; Infectiveendocarditis; Trauma

Dyspnea on exertion, fatigue,anorexia, peripheral edema, andJVD. PE: holosystolic murmurand palpable left ventricularheave

What is the gold standard to diagnosecardiac tamponade?

What are some key points in themanagement of cardiac tamponade?

Echocardiography

Immediate pericardiocentesis ifunstable; Aggressive fluidresuscitation; Inotropic agents (i.e.,dopamine)

Page 123: Deja review _emergency_medicine__deja_review_

Cardiovascular Emergencies 107

What are some common findings for thefollowing diagnostic studies that may beused to evaluate tricuspid regurgitation:

CXR

ECG

What are some key points in themanagement of tricuspid regurgitation?

May show an enlarged rightatrium/ventricle; Pulmonaryvasculature often normal

Right atrial and ventricularhypertrophy; Incomplete rightRBBB; Afib if present

Treat for afib (rate control/anticoagulate); Adequate control offluid overload and failure symptoms;Surgical intervention for structuraldeformity

Mitral Stenosis

What are some causes of mitral stenosis?

What are some clinical features of mitralstenosis?

What are some common findings for thefollowing diagnostic studies that may beused to evaluate tricuspid regurgitation:

CXR

ECG

What are some key points in themanagement of mitral stenosis?

Rheumatic fever (>90% of cases);Left atrial myxoma; Congenital

Dyspnea on exertion, hemotysis,fatigue, othopnea, and palpitations.PE: early diastolic opening snap,palpable diastolic thrill, and loudS1

Pulmonary congestion; Left atrialenlargement

P mitrale (left atrial enlargement);Afib if present

Treat for afib (rate control/anticoagulate); Diuretics forpulmonary congestion; Abxprophylaxis when indicated

Chronic Mitral Regurgitation

What are some causes of chronic mitralregurgitation?

Rheumatic fever; Connective tissuedisorder; Mitral valve prolapse;Infective endocarditis

Page 124: Deja review _emergency_medicine__deja_review_

108 Deja Review: Emergency Medicine

What are some clinical features of chronicmitral regurgitation?

What are some common findings for thefollowing diagnostic studies that may beused to evaluate chronic mitral regurgitation:

CXR

ECG

What are some key points in themanagement of mitral regurgitation?

Dyspnea on exertion and orthopnea,but even with severe MR, most areasymptomatic unless LV failure,pulmonary HTN, or afib. PE: S1 isdiminished, S3/S4 gallop, and leftparasternal heave

May show an enlarged leftatrium/ventricle; Pulmonaryvasculature often congested

Left ventricular hypertrophy; Leftatrial enlargement; Afib if present

Treat for afib (rate control/anticoagulate); Adequate control offluid overload and failure symptoms;Abx prophylaxis when indicated

Acute Mitral Regurgitation

What are some causes of acute mitralregurgitation?

What structures associated with the mitralvalve can be damaged?

What are some clinical features of acutemitral regurgitation?

What are some common findings for thefollowing diagnostic studies that may beused to evaluate acute mitral regurgitation:

CXR

ECG

What are some key points in themanagement of mitral regurgitation?

Myocardial infarction; Trauma;Infective endocarditis

Papillary muscle; Valve leaflet;Chordae tendineae

Dyspnea on exertion andorthopnea, but will often present asfulminant CHF and symptoms ofthe cause of the rupture (i.e. MI).PE: S1 is diminished, S3/S4 gallop,and left parasternal heave

Often have a normal cardiacsilhouette; Evidence of severepulmonary edema

Often show sinus tachycardia; Mayalso show evidence of MI, if thecause

Oxygen and afterload reduction;Adequate control of fluid overloadand failure symptoms; Emergentconsult with CT surgery

Page 125: Deja review _emergency_medicine__deja_review_

Cardiovascular Emergencies 109

What are some important things to knowabout mitral valve prolapse?

What are some causes of mitral valveprolapse?

What are some clinical features of mitralvalve prolapse?

What are some complications to considerfor mitral valve prolapse

What are some common findings for thefollowing diagnostic studies that may beused to evaluate aortic stenosis:

CXR

ECG

What are some key points in themanagement of mitral valve prolapse?

Most common valvular heart disease;More common in young females;Present in up to 10% of population

Idiopathic; Associated with tissueconnective disorder; Autosomaldominant congenital disorder

Palpitations, syncope, chest pain, orcan be asymptomatic. PE: high-pitched late systolic murmur or latesystolic click

Sudden death (very rare); CHF(due to severe regurgitation);Embolization

Typically normal unless severeregurgitation

Typically normal; May show T-wave changes in inferior lead; Mayshow QT prolongation

Abx prophylaxis when indicated(usually if with injury); Beta-blockers for chest pain/dysrhythmias; Anticoagulation forsuspected embolization

Aortic Stenosis

What are some causes of aortic stenosis?

When do patients generally becomesymptomatic with aortic stenosis?

What are some clinical features of aorticstenosis?

Congenital bicuspid valve;Rheumatic heart disease; Calcificaortic disease

Most are asymptomatic until verylate in the disease—valve openingdecreases <1 cm

Syncope, chest pain, dyspnea onexertion, sudden death, andsymptoms of heart failure PE: harshsystolic murmur (crescendo-decrescendo), narrow pulse pressure,and diminished carotid upstroke

Mitral Valve Prolapse

Page 126: Deja review _emergency_medicine__deja_review_

110 Deja Review: Emergency Medicine

What are some common findings for thefollowing diagnostic studies that may beused to evaluate aortic stenosis:

CXR

ECG

What are some key points in themanagement of aortic stenosis?

Aortic calcification; Left ventricularenlargement; Poststenotic dilatationof the aorta

Left ventricular hypertrophy; Leftor right BBB

Symptomatic patients referredfor either valve replacement orvalvuloplasty; Admit patients inCHF; Abx prophylaxis whenindicated

Chronic Aortic Regurgitation

What are some causes of chronic aorticregurgitation?

What are some clinical features of chronicaortic regurgitation?

What are some common findings for thefollowing diagnostic studies that may beused to evaluate chronic aorticregurgitation:

CXR

ECG

What are some key points in themanagement of chronic aortic regurgitation?

Rheumatic heart disease; Connectivetissue disorder; Bicuspid valve;Infective endocarditis; Teritiarysyphillis

Dyspnea on exertion, orthopnea,fatigue and palpitations. PE: S1 isdiminished, wide pulse pressure,high-pitched decrescendo blowingmurmur, and displaced PMI

Often have cardiomegaly;Pulmonary root congestion; Aorticroot dilation

Left ventricular hypertrophy;Sometimes an LBBB can be seen

Adequate control of fluidoverload and failure symptoms(treat as CHF); Abx prophylaxiswhen indicated

Acute Aortic Regurgitation

What are some causes of acute aorticregurgitation?

Aortic dissection; Trauma; Infectiveendocarditis

Page 127: Deja review _emergency_medicine__deja_review_

Cardiovascular Emergencies 111

What are some clinical features of acuteaortic regurgitation?

What are some common findings for thefollowing diagnostic studies that may beused to evaluate acute aortic regurgitation:

CXR

ECG

What are some key points in themanagement of acute aortic regurgitation?

Severe dyspnea on exertion, signsof heart failure, and chest pain. PE:low blood pressure, tachycardia,normal pulse pressure, midsystolicflow murmur, and low CO

Often have a normal cardiacsilhouette; Evidence of pulmonaryedema

Often show sinus tachycardia; Leftventricle strain; Nonspecific ST-Twave change

Determine cause and treat; Adequatecontrol of fluid overload and failuresymptoms; Emergent consult withCT surgery for valve replacement

Prosthetic Valves

What are two types of prosthetic valvescommonly used?

What are some important points regardingmechanical valves?

What are some important points regardingbioprosthetic valves?

What is the most serious complication ofprosthetic valves?

What are some other complications ofprosthetic valves?

1. Mechanical valves2. Bioprosthetic valves (porcine or

bovine)

Typically made from carbon alloys;Most mechanical valves last20–30 years and metallie noise canbe heard; Life-long anticoagulationrequired; Greater hemolysis thantissue valves

Can be human, bovine, or porcinetissue; Typically last <10 years;Closure noise similar to nativevalves; Anticoagulation required insome situations; Less hemolysisthen mechanical valves

Thromboembolic events

Structural failure; Bleeding;Embolization; Hemolytic anemia;Valvular obstruction (from thrombus)

Page 128: Deja review _emergency_medicine__deja_review_

112 Deja Review: Emergency Medicine

THORACIC AORTIC DISSECTION

What is the epidemiology of thoracic aorticdissection (TAD)?

What is the pathophysiology of TAD?

What are two factors that determine therate of dissection propagation?

What is the biggest risk factor for thedevelopment of TAD?

What are some other important risk factorsof TAD?

What are the two major classificationsystems used to classify TAD based onlocation of dissection?

What is the Stanford classification:

Type A

Type B

What is the Debakey classification:

Type I

Type II

Type III

Subtype IIIA

Subtype IIIB

What is the mortality rate for untreatedTAD once the dissection begins:

1 day

2 days

2 weeks

1 month

Males are affected more thanfemales; Most patients affected areover 50 years; TAD are morecommon than AAA

Degeneration of the aortic media, orcystic medial necrosis, that leads to atear in the aortic intima. Propagationof the dissection to various areas (i.e.,coronary artery is the feared concern)

1. Blood pressure2. Rate of ventricular contraction

Uncontrolled blood pressure

Connective tissue disorders—Marfan’s; Congenital heartdisease; Turner’s syndrome;Infections (i.e., syphilis); Drugsthat raise BP (i.e., cocaine);Trauma

1. Stanford2. Debakey

Ascending aorta

Descending aorta

Ascending aorta and part distal aorta

Ascending aorta only

Descending aorta only

Dissection above the diaphragm

Dissection below the diaphragm

33%

50%

75%

Approaches 90%

Page 129: Deja review _emergency_medicine__deja_review_

Cardiovascular Emergencies 113

What is the character of chest pain in TAD?

What are some other clinical features ofTAD based on the location of thedissection?

What are some important physical findingsthat help to establish the diagnosis ofTAD?

What is the clinical significance of a“silent” TAD?

What are some important points for thefollowing initial tests that should be undertaken:

CXR

ECG

Name four studies that are commonly usedto confirm the diagnosis of TAD?

What is the test of choice at mostinstitutions as it is noninvasive,inexpensive, and fast?

What is important initial management forany patient with suspected TAD?

Chest pain that is abrupt andmaximal at onset, migrates as thedissection progresses that istypically described as tearing withradiation to jaw/arm/back

Abdominal pain (mesentericischemia); Flank pain/GUsymptoms (< renal flow); CVA(dissection of carotid artery); MI(dissection of coronary artery);CHF; Syncope; Spinal cord deficits

Focal neurological deficits, a20 mm Hg extremity BP difference,and unequal or absent pulsesbetween extremities

“Silent” TAD is not that uncommonand must be distinguished fromMI/CVA as the use of lytics wouldbe disastrous

Should be done immediately andupright; CXR will almost beabnormal in TAD; Mediastinalwidening (>8 cm) common; Othercommon findings include loss ofaortic knob, deviation of trachea,effusion, etc.

Will be abnormal in most cases;Changes seen in MI is common; LVhypertrophy is common as well;Inferior wall MI most commonpattern

1. MRI2. Aortography3. Transesophageal

echocardiography4. CT

TEE (CT is done in most caseswhen TEE cannot be done)

Control of BP (i.e., nitroprusside);Control HR (i.e., beta-blocker);Avoid anticoagulants/lytics

Page 130: Deja review _emergency_medicine__deja_review_

114 Deja Review: Emergency Medicine

ABDOMINAL AORTIC ANEURYSMS

What are some important things to knowabout abdominal aortic aneurysms (AAA)?

What is the diameter of the aorta that isconsidered pathologic?

What is the pathophysiology of AAA?

What are some risk factors for thedevelopment of AAA?

What are some clinical features of AAArupture?

What is the most common misdiagnosis ofAAA rupture?

What are some important physical findingsthat help to establish the diagnosis of AAArupture?

What are some important points for thefollowing diagnostic tests commonlyutilized for AAA:

Plain abdominal film

Involve all layers of the aorta; MostAAA occur below the renal arteries;Ruptured AAA is an emergency

Diameter >3.0 cm is generallyconsidered aneurysmal

Aortic aneurysms are caused by aprogressive weakening of the aorticwall which results in a dilatation. Theaneurysm will grow progressivelylarger and eventually rupture if it isnot diagnosed and treated

Age (most occur in >70 years);Male gender; History of smoking;Hypertension; Family history infirst degree relatives; History ofCAD or PVD

Classic presentation is suddenonset of severe abdominal, back, orflank pain that may be associatedwith syncope. Pain can radiate tothe testicles/labia as well

Kidney stone

A ruptured AAA will often havea tender pulsatile mass in theepigastric area, bruits, and signs ofdistal extremity ischemia

Not very accurate for AAA; Mayshow aneurysmal calcification;Does not confirm/excludediagnosis

What are some key points in the treatmentof TAD?

Immediate CT consultation; Ifhypotensive—small fluid bolus;Ascending dissection = surgery;Descending dissection = medical;Pain control with narcotics

Page 131: Deja review _emergency_medicine__deja_review_

Cardiovascular Emergencies 115

Ultrasonography

CT contrast with contrast

What is the initial management for anypatients with suspected AAA rupture?

Can be utilized on unstablepatients; Inexpensive, fast,sensitive; Can only detectaneurysm, not leaks; Limited byadipose tissue and gas

Very accurate and sensitive; Canalso detect other abnormalities;Negative: IV contrast and longstudy test; Not to be used onunstable patients

IV O2-monitor; Aggressive fluidresuscitation; Type and cross for5–10 units; ECG; Immediatesurgical consultation

HYPERTENSIVE URGENCIES AND EMERGENCIES

What is the definition of hypertensiveurgency?

What is the most common cause ofhypertensive urgency?

What is a consequence of aggressive bloodpressure reduction?

What are some key points in themanagement of hypertensive urgency?

What is the definition of hypertensiveemergency?

What is the pathophysiology ofhypertensive emergency?

Name two characterstics of hypertensionencephalopathy?

Severe hypertension (oftendefined by systolic blood pressure≥180 mm Hg and/or diastolicblood pressure ≥120 mm Hg)without any evidence of end-organdamage

Nonadherence to antihypertensives

CVA (due to fall below autoreg-ulation)

Slowly bring down the patient’sBP; Ensure patient’s compliance tomedicines

Marked increase in blood pressure,generally ≥180/120 mm Hg, withevidence of end-organ damage

Initial response is arterial andarteriolar vasoconstriction,autoregulatory process bothmaintains tissue perfusion at arelatively constant level.Increasingly severe hypertensionwill result in failure ofautoregulation

It is reversible and acute in onset

Page 132: Deja review _emergency_medicine__deja_review_

116 Deja Review: Emergency Medicine

23-year-old female with no past medicalhistory presents with unilateral left legswelling soon after a trip to Mexico, butotherwise has been well; PE: unilateralswelling of left calf with a (+) Homans’sign

45-year-old male with a history of DM andHTN is brought over via EMS for alteredmental status, confusion and minimalresponse, and they mention patient has along history of poor medicationcompliance; PE: BP is taken at 240/180,with otherwise unremarkable PE

71-year-old female with Hx of HTNpresents with syncope whenever she exertsherself, but otherwise no other complaints;PE: harsh systolic murmur (crescendo-decrescendo), narrow pulse pressure, anddiminished carotid upstroke

23-year-old male with Hx of IVDA presentswith fever, fatigue, and weight loss for thepast 2 weeks; PE: heart murmur, petechiae,and conjunctival hemorrhages

18-year-old male with no PMH presentswith a sudden syncopal episode duringsoccer practice, but otherwise is nowfeeling fine in the ED; PE: rapid biphasiccarotid pulse and systolic ejection murmur;ECG: left ventricular hypertrophy

61-year-old male with Hx of DM and HTNpresents with “chest pressure” for about2 hours associated with dyspnea,diaphoresis, nausea, and radiation of painto jaw; PE: unremarkable exam exceptpatient is anxious; Labs: elevated cardiacenzymes; ECG: ST-depression in inferiorleads (II, III, and aVL)

Deep vein thrombosis

Hypertensive emergency

Aortic stenosis

Infective endocarditis

Hypertrophic cardiomyopathy

NSTEMI

CLINICAL VIGNETTES

What are some clinical features ofhypertensive encephalopathy?

What are two eye findings to look for witha patient who present with hepaticencephalopathy?

Confusion, severe headache, focalneurologic deficits, or coma

1. Papilledema2. Hypertensive retinopathy

Page 133: Deja review _emergency_medicine__deja_review_

Cardiovascular Emergencies 117

55-year-old female with Hx of breast cancerpresents with pleuritic chest pain anddyspnea on exertion for 3 days, but isotherwise stable; PE: tachycardia, butotherwise unremarkable exam; CXR: clearfields except for regional oligemia

51-year-old male with Hx of uncontrolledhypertension presents with tearing chestpain with radiation to the back that hasbeen refractory to nitro; PE: 20 mm Hgextremity BP difference, and unequal orabsent pulses between extremities

21-year-old female presents with CP thatshe describes as sharp and more painful ondeep inspiration, but relieved when sheleans forward; PE: friction heard oncardiac exam; ECG: depressed PR intervaland diffuse ST segement elevation

73-year-old male with history of DM, HTN,and smoking presents with a sudden onsetof abdomen pain with radiation to the leftflank as well as his testicles; PE: pulsatilemass in the epigastric area as well asabdominal bruits

56-year-old male with Hx of DM and HTNpresents with CP. Patient mentions that hepreviously had CP only when he did anymoderate activity and was was relieved byrest and his nitro, but now he gets his CPwhen he is at rest, but it still does not lastmore then 5 minutes or so

71-year-old female with no cardiac historypresents as a transfer from an outsidehospital for chest pain. She mentions thather CP started about 3 hours ago and saysit is substernal associated with diaphoresisand nausea; PE: unremarkable; ECG 3-mmST-elevations in V1–V4

31-year-old female who recently arrivedfrom South America presents with chestpain and recalls that it started about a weekafter her cold; PE: S3/S4 gallop with apericardial friction rub; ECG: low-voltage

Pulmonary embolism

Thoracic aortic dissection

Pericarditis

Aortic abdominal aneurysm

Unstable angina

Anterior STEMI

Myocarditis

Page 134: Deja review _emergency_medicine__deja_review_

This page intentionally left blank

Page 135: Deja review _emergency_medicine__deja_review_

C H A P T E R 7

GastrointestinalEmergencies

119

ESOPHAGUS

Anatomy

What are some important anatomic pointsto know about the esophagus?

What are the major nerves of the extrinsicnervous system that innervate theesophagus?

What are the two major plexuses that arefound within the esophagus?

What are the three layers of the esophagus?

What is the clinical significance of the lackof serosa?

Name the three anatomical constrictionswithin the esophagus that may representpoints of obstruction?

It is 25-cm in length; Upper third isstriated muscle; Lower two-thirdsis smooth muscle

Vagus nerve; Sympathetic fibers;Spinal accessory nerve

1. Meissner’s plexus2. Auerbach’s plexus

1. Inner mucosa2. Submucosa3. Muscle layer

Any compromise of the submucosawill lead to diffuse rapidmediastinitis

1. Upper esophagus sphincter (UES)2. Lower esophagus sphincter (LES)3. Level of the aortic arch

Dysphagia/Odynophagia

What is the definition of dysphagia?

What is the definition of odynophagia?

It is a subjective experience thatranges from the inability to swallowto the sensation of food “stuck” inthe esophagus

The sensation of pain whenswallowing

Copyright © 2008 by The McGraw-Hill Companies, Inc. Click here for terms of use.

Page 136: Deja review _emergency_medicine__deja_review_

What are some important elements in thehistory to obtain in dysphagia?

What are some important points for thetwo categories of dysphagia:

Transport dysphagia

Transfer dysphagia

List some common anatomical problemsthat may result in dysphagia.

List some common neuromuscularproblems that may result in dysphagia.

What are some key points in the managementof dysphagia?

What is a common cause of odynophagia?

What are some clinical features ofodynophagia?

What are two main causes of odynophagia?

List some examples that may causeinfectious esophagitis?

List some examples that may causeinflammatory esophagitis?

What are some key points in themanagement of esophagitis?

120 Deja Review: Emergency Medicine

Whether it is acute versus chronic;Dysphagia to food or liquids (orboth); Intermittent versus progressive

Problem typically lies at theesophagus; Often patient willcomplain of a sticky sensation;Commonly due to anatomicalproblems

Problem typically is at theoropharynx; Difficulty in transfer offoul bolus to esophagus; Commonlydue to neuromuscular problems

Strictures (i.e., radiation injury);Malignancy; Webs; Diverticula

Achalsia; Spasms; Neurologicalinsults (i.e., stroke)

Ensure the patient is stable; Oftenrequires various tests (i.e., EGD,barium swallow, etc); A carefulhistory is paramount

Esophagitis

Pain on swallowing and chest pain(important to distinguish fromcardiac pain)

1. Infectious2. Inflammatory

Candida; Herpes simplex virus(HSV); Cytomegalovirus (CMV);Aphthous ulceration

Medication (i.e., nonsteroidal anti-inflammatory drugs [NSAIDs] andantibiotics); GERD

If chest pain, distinguish from cardiacorigin; Monitor for bleed, perforation,and obstruction; Typically managedon a outpatient basis

Gastroesophageal Reflux Disease

What is the definition of gastroesophagealreflux disease (GERD)?

Reflux of stomach acid typicallyfrom transient relaxation of LES ora weak LES

Page 137: Deja review _emergency_medicine__deja_review_

What are some complications of GERD?

Name some major causes of GERD?

What are some clinical features of GERD?

What are some things that may exacerbateGERD?

What are some key points in themanagement of GERD?

Gastrointestinal Emergencies 121

Esophageal erosions; Esophagealstrictures; Barrett’s esophagus;Esophageal cancer

Decrease in esophagealmotility (achalsia); Prolongedgastric emptying (obstruction);Transient decrease in LES tone(diet)

Dysphagia, odynophagia,heartburn, asthma exacerbation,and presen- tation that may besimilar to heart ischemic (squeezingpain, pain radiation, andnausea/vomiting

Meals are often a major factor;Medication; Supine position

Avoid triggers (i.e., eating beforebed); H2-blockers and proton-pumpinhibitors (PPI)

Esophageal Perforation

What are some causes of esophagealperforation?

What is the most common cause ofesophageal perforation?

What is Mallory-Weiss syndrome?

What are some clinical features of Mallory-Weiss syndrome?

What are some risk factors for Mallory-Weiss syndrome?

What is Boerhaave’s syndrome?

What are some of the clinical features ofBoerhaave’s syndrome?

What are complications of Boerhaave’ssyndrome?

Chest trauma; Iatrogenic(endoscopy); Swallowing (object);Sudden increase in intra-abdominalpressure such as emesis

Iatrogenic

It is a partial thickness tear alongthe esophagus

Mild self-limiting upper GIbleeding, dysphagia, andodynophagia

Hiatal hernia; EtOH abuse;Esophagitis

It is perforation of the esophagus

Severe tearing chest pain that oftenradiates to the back/neck. PE:mediastinal crunch and epigastrictenderness

Mediastinitis (high mortality);Sepsis

Page 138: Deja review _emergency_medicine__deja_review_

What are some diagnostic tests used inBoerhaave’s syndrome and their typicalfindings:

Chest x-ray (CXR)

Esophagram (water-soluble contrast)

What are some key points in themanagement of esophageal rupture?

122 Deja Review: Emergency Medicine

Widened mediastinum, left pneu-mothorax, left pleural effusion, andmediastinal emphysema

Leakage of content into the medi-astinal area

Aggressive fluid resuscitation;Intravenous (IV) antibiotics;Surgical consult

Swallowed Foreign Body

What are some important things to knowabout a swallowed foreign body?

What type of foreign bodies are mostcommonly swallowed by children?

What type of foreign bodies are mostcommonly swallowed by adults?

What portion of the esophagus do mostobjects get lodged in children?

What portion of the esophagus do mostobjects get lodged in adults?

What are some clinical features of aswallowed foreign body?

What is the diagnostic test of choice in aswallowed foreign body?

What are three complications of foreignbody impaction?

What is the probability that a foreign bodywill pass completely once past thegastroesophageal junction?

What is typically done for proximalimpactions of the esophagus?

80% of complaints are in the pediatricpopulation; Most ingestions do passthrough the GI tract withoutintervention or problems; Typically1500 die per year from objectingestion

True foreign objects such as coins

Food impactions more common

Cricopharyngeal area

Distal portion of the esophagus

Dysphagia, foreign body sensation,gagging, emesis, and possiblerespiratory distress

Plain films with at least two views;Endoscopy (diagnostic andtherapeutic); Esophagogram (ifperforation is suspected)

1. Obstruction2. Esophageal perforation3. Esophageal strictures

90%

Removal of object via endoscopy

Page 139: Deja review _emergency_medicine__deja_review_

What are some commonly usedmedications to help with passage of distalesophagus impaction?

About what percentage of foreign bodiesthat are lodged and cannot be removedrequire surgical intervention?

What are some key points in themanagement of a swallowed foreign bodies?

Gastrointestinal Emergencies 123

Nifedipine; Sublingual nitroglycerin

1%

Most can be managed expectantly;Lodged sharp objects mandateremoval; Most foreign bodies arecleared in 2–3 days; Swallowedbatteries also mandate removal

GASTROINTESTINAL BLEEDING

What are some important epidemiologicinformation about GI bleeding?

What are some factors that are associatedwith high mortality in GI bleeding?

What defines upper GI bleeding?

What are some important things to knowabout each of the important causes ofupper GI bleeding:

Gastric and esophageal varices

Peptic ulcer disease

Mallory-Weiss syndrome

What are some other less common causesof upper GI bleeding?

Common, but potentially life-threatening; Upper GI bleeding ismore common in elderly males;Mortality rises with age

Advanced age; Coexisting organdisease; Hemodynamically unstable;Repeated hematemesis/hematochezia

Bleeding that is proximal to theligament of Treitz

Commonly from portal hypertension;Very high rebleed rate and mortalityrate; Comprise small number ofupper GI bleeds

Includes gastric, and duodenalulcers; Most common cause ofupper GI bleed; Gastric ulcershigher rebleed rate than duodenalulcers

Longitudinal tear of esophagus;Classically hematemesis followingretching; Seizures and coughing arerisk factors

Arteriovenous malformation;Malignancy; Aortoenteric fistula

Page 140: Deja review _emergency_medicine__deja_review_

What is the most common cause ofapparent lower GI bleeding?

What is the most common cause of actuallower GI bleeding?

What are other some common causes oflower GI bleeding?

What are some important elements toobtain in a patient who presents with GIbleeding?

What are some elements on the physicalexam to consider?

What are some laboratory data to considerin GI bleeding?

What are some diagnostic studies toconsider in GI bleeding?

What is the most accurate test to perform inupper GI bleeding?

Why is EGD evaluation useful in upper GIbleeding?

What are some important managementpoints with patients who present with GIbleeding?

What role does somatostatin or octreotideplay in GI bleeding?

124 Deja Review: Emergency Medicine

Upper GI bleeding

Hemorrhoids

Inflammatory (i.e., inflammatorybowel disease

Neoplasm (i.e., colon cancer)

Other (i.e., hemorrhoids)

Vascular (i.e., arteriovenousmalformation [AVM])

Anatomical (i.e., diverticulosis)

Characterize the bleeding;Changes in bowel habits andweight loss; Retching andvomiting; History of medication(i.e., NSAIDs); Alcohol use;Ingestion of bismuth or iron

Vitals (i.e., decreased pulse pressure);Stigmata of liver disease (i.e.,jaundice); Abdominal examination;Rectal exam

Type and cross-match blood;Complete blood count (CBC);Coagulation studies; Liverpanel; Chem-7 (i.e., BUN can beelevated)

CXR and abdominal films (low yield);Endoscopy (EGD) and colonoscopy;Scintigraphy

EGD

Diagnostic and therapeutic such asband ligation of esophageal varices

GI bleeding is potentially life-threatening; Immediate resuscitation(fluids and blood); Neogastric (NG)tube placement is important

Effective in reducing bleedingfrom esophageal varices andpeptic ulcers, as effective asvasopressin without the adversedrug reaction

Page 141: Deja review _emergency_medicine__deja_review_

Should patients with upper GI bleeding beplaced on a proton pump inhibitor?

What role does balloon tamponade play inGI bleeding?

Gastrointestinal Emergencies 125

Omeprazole shown to reducerebleeding, need for surgery withPUD, and reduce transfusionrequirement

Sengstaken-Blakemore tube cancontrol variceal hemorrhage

PEPTIC ULCER DISEASE

What is the definition of peptic ulcerdisease (PUD)?

What are the two most common causes ofPUD?

List some other predisposing factors for thedevelopment of ulcers.

What are some important things to knowabout the following types of ulcers:

Gastric ulcers

Duodenal ulcers

Stress ulcers

What conditions are H. pylori usuallyimplicated in?

What are some common diagnosticmethods used to identify H. pylori?

What are some clinical features of PUD?

PUD is a chronic disease that istypically caused by defects in themucosal barrier most commonlyalong the lesser curvature of thestomach and duodenum

1. NSAIDs2. Helicobacter pylori

Zollinger-Ellison syndrome;Cigarette smoking; Long-termsteroid use; Stress

Damage is from mucosalbreakdown; H. pylori is found inover 75% of cases; Pain is typicallyshortly after eating

Damage is usually from acidhypersecretion; H. pylori is found inover 90% of cases; Pain is typically2–3 hours after meals

Commonly due to acute trauma/CNS tumors; Usually located onfundus/body of stomach; Verycommon cause of gastric bleeding

PUD; Lymphoid hypertrophy;Adenocarcinoma of stomach;Gastric lymphoma

Serology; Endoscopy (i.e., rapidurea); Urea breath test

Epigastric pain that is vague anddescribed as “burning” oftenrelieved by food

Page 142: Deja review _emergency_medicine__deja_review_

What are some important diagnostic teststo consider in PUD?

What are some key points in themanagement of uncomplicated PUD?

What is the importance in the eradicationof H. pylori?

What are three drugs commonly used astriple therapy in the eradication ofH. pylori?

What are some complications of PUD?

What are some key points in themanagement of the followingcomplications of PUD:

Upper GI bleeding

Perforation

Gastric outlet obstruction

126 Deja Review: Emergency Medicine

Endoscopy; Barium-contrast x-ray

Avoid exacerbating factors (i.e.,NSAIDs); Antacids—symptomaticrelief; H2-blockers (cimetidine);Proton pump inhibitors (omeprazole);Eradication of H. pylori

Reduces the recurrence of PUD;Reduces need for suppressivetherapy

1. Macrolide (clarithromycin)2. Tetracycline3. Omeprazole

Bleeding; Perforation; Outletobstruction

Volume replacement and transfuseif needed; Administer PPI or H2-blocker; Nasogastric (NG) tubedrainage; GI or surgery consult insevere case

Monitor for peritoneal signs; X-rayevidence of free air; IV fluids/ABx/NG tube drainage; Surgicalconsultation

Healed ulcer scar that blockspyloric outlet; Endoscopy;Upright abdominal plain film; IVfluids/NG tube suction andadmit

APPENDICITIS

What is the definition of appendicitis?

What are some important things to knowabout appendicitis?

It is inflammation of the appendixdue to obstruction of the outlet

Most common cause of emergentsurgery; Highest incidence in males10–30 years of age

Page 143: Deja review _emergency_medicine__deja_review_

What is the pathophysiology ofappendicitis?

What are some common causes ofappendiceal obstruction?

What are the most common symptoms ofappendicitis?

What are some common signs ofappendicitis?

How common is the classic migratory painwith associated symptoms in appendicitis?

What is the concern in a patient withsuspected appendicitis who has a suddendecrease in pain followed by a dramaticincrease in pain?

What other conditions can appendicitismimic?

What are some important diagnostic teststo consider in appendicitis?

What are some key points in themanagement of appendicitis?

Gastrointestinal Emergencies 127

Obstruction of the lumen that leadsto intraluminal distension, venouscongestion, and eventually ischemiafollowed by perforation (bacterialinvasion common)

Fecalith—most common; Enlargedlymphoid follicles; Tumors;Adhesions

Abdominal pain (periumbilicalthen right lower quadrant);Anorexia; N/V (should occur afterpain); Fever and chills

Abdominal tenderness with/withoutrebound; Rosving’s sign (RLQpain when pressing left lowerquadrant [LLQ]); Psoas sign(Passive extension of right hip thatcauses RLQ pain); Obturator sign(Passive internal rotation of flexedhip causes RLQ pain); Cervicalmotion tenderness (seen in pelvicinflammatory disease)

Found in up to 2/3 of patients withappendicitis

Perforation

Nephrolithiasis; Pelvic inflammatorydisease; Right upper quadrant painin pregnant women; Ectopicpregnancy

Pregnancy test—rule out ectopicpregnancy; Complete blood count(CBC)—elevated WBC is typical;Plain abdominal films—may showfecalith; CT with IV and rectal/oralcontrast—first choice; Ultrasound(U/S)—useful for children/pregnantwomen

NPO and IV fluids; Pain control;Early surgical consult if suspicion ishigh; If surgery—prophylacticantibiotics (Abx)

Page 144: Deja review _emergency_medicine__deja_review_

What is the definition of cholecystitis?

What are some important things to knowabout cholecystitis?

What are some risk factors for thedevelopment of cholelithiasis (hencecholecystitis)?

What are some important points for each ofthe following types of gallbladder disease:

Calculous cholecystitis

Acalculous cholecystitis

Ascending cholangitis

Gallstone ileus

Emphysematous cholecystitis

What are some clinical features ofcholecystitis?

What is Murphy’s sign?

128 Deja Review: Emergency Medicine

Acute inflammation of thegallbladder that is commonlycaused by obstruction at the neck ofthe gallbladder or cystic duct

It is more common in females; Mostcases (>90%) due to cystic stones;One of the most common indicationfor surgery

Obesity; Female; Rapid weight loss;Advanced age; Cystic fibrosis;Long-term TPN use

It is the most frequent variant; Mostcommon cause of pancreatitis

It makes up about 5–10% ofcases; More common in elderly,DM, and sepsis; Perforation andgangrene are more common

Extending infection into the liver;Charcot’s triad: fever/jaundice/RUQpain; Reynold’s pentad: Charcot’sTriad plus shock/∆MS; Requiresrapid surgical intervention

Uncommon cause of bowelobstruction; Gallstone erodesthrough the gallbladder andimpacts in bowel near the cecum;More common in elderly females

Rare infection of the gallbladder;Agents usually include anaerobes/gram (–)

RUQ pain, fatty food intolerance,gallstone risk factors, N/V, fever,and tachycardia

It is increase in pain and temporarycessation of breathing when directpressure is applied to RUQ whenthe patient takes a deep breath

GALLBLADDER DISEASE

Page 145: Deja review _emergency_medicine__deja_review_

What are some important diagnosticstudies to consider in cholecystitis?

What are the typical findings in thefollowing imaging modalities in theassessment of cholecystitis:

Ultrasound

Biliary scintiscanning (HIDA)

CT

What are some key points in themanagement of cholecystitis?

What are some general criteria for admission?

Gastrointestinal Emergencies 129

CBC—elevated WBC is typical;Liver function test (LFT)—enzymesand alkaline phosphate may beelevated; Amylase/lipase—increasedif pancreatitis; Abdominal plainfilms: typically normal

The study of choice where commonfindings include presence of gall-stones, gallbladder wall thickening(>5 mm), pericholecystic fluid,and dilated common ducts

Typically used if U/S results areindeterminate and clinical suspicionis high; Positive results typicallyshow lack of isotopes in thegallbladder

It is not any more sensitive orspecific when compared to U/Sand exposes patient to significantamount of radiation

NPO/IV fluids/NG tube ifneeded; Broad-spectrum Abx;Surgical consult; Pain control

Fever, significant abdominal pain,elevated WBC; Complications(i.e., ascending cholangitis);Cholecystectomy (usually within72 hours)

PANCREATITIS

What is the definition of pancreatitis?

What are the two most common causes ofacute pancreatitis?

What are some other causes of acutepancreatitis?

It is acute inflammation of thepancreas

1. Alcohol abuse2. Bile duct disease (gallstone)

Surgery; ERCP; Hyperlipidemia;Hypercalcemia

Page 146: Deja review _emergency_medicine__deja_review_

What are some clinical features of acutepancreatitis?

What is Grey Turner’s sign?

What is Cullen’s sign?

What do these two signs point to?

What is chronic pancreatitis?

Give some important features of thefollowing diagnostic tests:

Amylase

Lipase

CBC

What is Ranson’s Criteria?

On admission

After 48 hours

What are some complications of acutepancreatitis?

130 Deja Review: Emergency Medicine

Epigastric pain typically afteringestion of EtOH or a fatty meal,N/V, low-grade fever, andtachycardia

Bluish discoloration of the flank

Bluish discoloration near theumbilicus

Although not common, theyindicate the presence ofhemorrhagic pancreatitis

It is progressive, irreversiblestructural changes due to repeatedbouts of acute pancreatitiscommonly due to EtOH

Amylase is also found in otherorgans; 1.5 above upper limit pointsto pancreatitis

More specific and as sensitive asamylase; Lipase is found primarilyin the pancreas; It is reliable andinexpensive

Low hematocrit points tohemorrhagic pancreatitis; HighWBC is common

It is a set of prognostic factors thatcorrelate with mortality based onthe number of prognostic signs thatare met

Age >55 years; Hyperglycemia>200 mg/dL; Leukocytosis>16,000 per mm3; LDH >350 IU/L;AST >250

PO2 <60 mm Hg; Calcium <8 mg/dL;Hct >10% drop; Base deficit>4 mEq/L; Sequestration >4 L offluid; BUN >5 mg/dL

Abscess; Hemorrhagic; Fluidsequestration; Acute respiratorydistress syndrome (ARDS)

Page 147: Deja review _emergency_medicine__deja_review_

What is the definition of crohn’s disease(CD)?

What is the epidemiology of CD?

What are some clinical features of CD?

What are some extraintestinal manifestations of the following organsystems in CD?

What are some complications of CD?

What is the diagnostic test of choice forCD?

What are some key points in themanagement of CD?

What are some indications for admissionfor CD?

Gastrointestinal Emergencies 131

Crohn’s disease is a chronic,recurrent inflammatory disease ofthe intestinal tract (primarily theileum and colon)

Greater incidence in whitesbetween the age of 16–40, morelikely to affect Jews and a positivefamily history in up to 20%

Recurrent abdominal pain, fever,and diarrhea with weight loss. RLQthat mimic appendicitis is also notuncommon

Dermatology. Pyoderma gan-grenosum; Erythema nodosum

Ophthalmic. Iritis; Conjunctivitis;Uveitis

Rheumatology. Ankylosing spondyli-tis; Arthritis

Vascular. Arteritis; Thromboembolicdisease; Vasculitis

Hepatobiliary. Gallstones;Pericholangitis

Strictures; Perforation; Perianalcomplications; Abscess; Fistulas

Colonoscopy with histologicalsample

IV fluids and NG tube; Steroids toreduce inflammation; Azathioprine—steroid sparing; Metronidazole forperianal complications; Infliximabmay help in severe cases

Acute complications; Unable tokeep PO; Severe exacerbation

COLITIS AND ILEITIS

Crohn’s Disease

Ulcerative colitis

What is the definition of ulcerative colitis(UC)?

Chronic inflammatory disease ofthe colon that always has rectalinvolvement

Page 148: Deja review _emergency_medicine__deja_review_

132 Deja Review: Emergency Medicine

What is the pathophysiology of UC?

What is the epidemiology of the UC?

What are some clinical features of thefollowing degrees of UC:

Mild disease

Severe disease

What is the diagnostic test of choice forUC?

What are some complications of UC?

What are some key points in themanagement of mild/moderate attack ofUC?

What are some key points in themanagement of severe UC?

Mucosa/submucosa inflammationwith sparing of the serosa withcontinuous involvement unlikeCrohn’s disease

Greater incidence in whitesbetween the age of 16–40, morelikely to affect Jews and a positivefamily history in up to 20%

No systemic symptoms; Less than4 bowel movements per day; Fewextraintestinal symptoms

Systemic response (F/C, weightloss, etc.); Greater than 4 bowelmovements per day; Extraintestinalsymptoms

Colonoscopy

Toxic megacolon (more common inUC); Perforation; Obstruction;Perianal abscess and fistulas; Coloncarcinoma; Hemorrhage

Sulfasalazine—mainstay therapy;Mesalamine/olsalazine—second line;Corticosteroid—supplement; Avoidantidiarrheal agents; Azathioprine/cyclosporine—if steroids fail

IV fluids and NG tube; Broad-spectrum Abx; Monitor forhemorrhage/toxic megacolon;Surgical consult

Pseudomembranous enterocolitis

What is the definition ofpseudomembranous enterocolitis?

What is the pathogenic species responsiblefor pseudomembranous enterocolitis?

What antibiotics are commonly associatedwith the proliferation of C. difficile?

Inflammatory bowel diseasecharacterized by yellow exudativepseudomembranous plaque overnecrotic colon

Clostridium difficile

Clindamycin; Ampicillin;Cephalosporins

Page 149: Deja review _emergency_medicine__deja_review_

What is the pathophysiology of C. difficileassociated pseudomembranousenterocolitis in relation to Abx use?

What are some common clinical features ofpseudomembranous enterocolitis?

What is the general time frame for thedevelopment of pseudomembranousenterocolitis after Abx use?

What is the diagnostic study of choice?

What are some key points in themanagement of pseudomembranousenterocolitis?

What role do antidiarrheal drugs play inmanagement?

Gastrointestinal Emergencies 133

Abx use alters normal gut flora andallows C. difficile to propagate

Profuse watery diarrhea withcrampy abdominal pain, stool mayhave blood, and fever

Generally 7–10 days after Abxuse, but can occur weeks afterdiscontinuation

C. difficile toxin in stool

IV fluids and electrolyte balance;Discontinue the offending agent;Oral metronidazole is first-line;Oral vancomycin if metronidazoledoes not work

None—they can worsen symptomsand increase likelihood of toxicmegacolon

MESENTERIC ISCHEMIA

What is the pathophysiology of mesentericischemia?

What are some important things to knowabout mesenteric ischemia?

What are some key points in the followingcauses of mesenteric ischemia:

Nonocclusive

Acute occlusion

Mesenteric arteries that do notdeliver enough blood to the smallor large intestine, typically due tosudden occlusion or decreasedcardiac output (CO)

Commonly affects elderly withCVS disease; Mortality rate 50%once infarction occurs

Typically due to reduction in CO(i.e., CHF); Account for up 25% ofall cases; Commonly affects criticallysick/elderly; Presentation is moresubtle and insidious

Typically due to embolization(i.e., afib); Accounts for themajority of cases; Common insevere atherosclerotic patients;Presentation is acute, sudden, anddramatic

Page 150: Deja review _emergency_medicine__deja_review_

Venous thrombosis

What are some causes for the followingcauses of mesenteric ischemia:

Nonocclusive

Acute occlusion

Venous thrombosis

What are some clinical features ofmesenteric ischemia?

What are some commonly used diagnostictests?

What is the diagnostic test of choice insuspected mesenteric ischemia?

What are the general key points in themanagement of mesenteric ischemia?

What are the indications for surgicalintervention in mesenteric ischemia?

134 Deja Review: Emergency Medicine

Typically due to hypercoagulablestate; Often have history of deepvein thrombosis/Pulmonaryembolism (DVT/PE)

Hypotension (i.e., sepsis); CHF;Hypovolemia

Recent MI; Atherosclerotic heartdisease; Dysrhythmias (esp. afib)

History of DVT/PE;Hypercoagulable

Vague abdominal pain that is outof proportion early in the course,sudden severe pain if cause is acute,guaiac positive stool, N/V, andperitoneal signs late in the course ifinfarction occurs

CBC—often elevated white count;Arterial blood gas (ABG)—metabolicacidosis is common; Plain films—often normal; CT/US—not the firstline choice; Lactate level

Angiography

IV fluids and NG tube for decom-pression; Broad-spectrum Abx;Look for underlying cause andcorrect them; Use of papaverine fordiagnostic study

Necrotic bowel requiring resection;Revascularization; Evidence ofperforation (peritoneal signs)

DIVERTICULAR DISEASE

What is the definition of a diverticula? Sac-like herniations of the mucosain the colon typically due to anincrease in intra-luminal pressureoften from lack of fiber

Page 151: Deja review _emergency_medicine__deja_review_

What are some important things to knowabout diverticular disease?

What are the two main complications ofdiverticular disease?

What are some clinical features ofdiverticulosis?

What are some key points in themanagement of diverticulosis?

What is the definition of diverticulitis?

What are some clinical features ofdiverticulitis?

What are some complications ofdiverticulitis?

How commonly do paitents withdiverticulitis present with RLQ pain?

What other differential should be consideredin those who present with RLQ pain?

What are some commonly used diagnostictests in diverticulitis?

What studies are contraindicated during anacute attack of diverticulitis?

What are some key points in themanagement of diverticulitis?

Gastrointestinal Emergencies 135

Direct correlation with incidenceand age; High in patients whoconsume low fiber; Common causeof painless lower GI bleed

1. Diverticulosis2. Diverticulitis

Hallmark is self-limiting painlessrectal bleeding that is typicallybright red or maroon, although asmall percentage have massivelower GI bleed

Ensure that there is no massiveGI bleeding; Bleeding is typicallyself-limited; Diagnosis requirescolonoscopy; Increase in fibermay reduce future attacks;Avoidance of seeds—not reallyproven

Microperforation of diverticulathat result in an inflammatoryresponse that is typically walled offby pericolic fat

LLQ pain present for a few days isthe hallmark, N/V, diarrhea, andchanges in urinary symptoms

Abscess formation; Fistula;Obstruction; Perforation

Less than 5%—more common inAsians

Appendicitis

CBC—leukocytosis; Ab plain film—examine for complications; CT—test of choice to evaluate extent ofdisease

Colonoscopy and contrast studies

IV fluids and NPO; NG tube insuspected obstruction; Broad-spectrum Abx; Surgical consult ifsuspected complications

Page 152: Deja review _emergency_medicine__deja_review_

What is the definition of a hernia?

Define the following possibilities for ahernia:

Reducible

Irreducible/Incarcerated

Strangulated

What are some important points for thefollowing types of hernia:

Femoral hernia

Direct inguinal hernia

Indirect inguinal hernia

Umbilical hernia

What are some clinical features of a hernia ?

What are some key points in themanagement of a hernia?

136 Deja Review: Emergency Medicine

It is the protrusion of any body partout of its natural cavity primarilydue to inherent weaknesses(congenital) or acquired (surgery)

The contents can be returned to itscavity

Unable to reduce—no vascularcompromise

Vascular compromise of herniation

Protrudes below the inguinal ring;More common in females; Highfrequency of incarceration

Directly via the floor of Hesselbach’striangle; Incidence increases withage; Rarely incarcerates

Protrudes via the internalinguinal ring; More common inmen; More common in youngerpopulation; High frequency ofincarceration

Represents a congenital defectin newborns; Most will closeby 3 years of age; Rarelyincarcerates

Palpable bulge that often can bedetected on exam and can be sorewhen pressed, but rarely painfulunless incarcerated

If hernia is recent, can try toreduce; If suspected necrosis, donot reduce; Incarcerated = surgeryconsult; Strangulation = surgeryand Abx; Reducible = considerelective surgery

HERNIA

Page 153: Deja review _emergency_medicine__deja_review_

What is the definition of a hemorrhoid?

What are some risk factors for thedevelopment of hemorrhoids?

What are some important points for thefollowing types of hemorrhoids:

Internal hemorrhoids

External hemorrhoids

What are some common clinical featuresfor the following types of hemorrhoids:

Internal hemorrhoids

External hemorrhoids

What are some key points in themanagement in the following types ofhemorrhoids:

Internal hemorrhoids

External hemorroids

What are some indications for surgicalintervention?

Gastrointestinal Emergencies 137

Dilated internal or external hemor-rhoidal venous plexus

Straining; Increase in portal pressure(i.e., cirrhosis); Constipation; Lowfiber diet; Pregnancy

Originate above the dentate line;Relatively insensitive area—no/little pain; Rarely palpable—painlessbleeding common; Visualized at2, 5, and 9 o’clock positions

Originate below the dentate line; Inwell-innervated area, often painful;Usually can be visualized

Painless bright red blood perrectum, most common cause oflower GI bleed in youngerpopulation

Tender palpable mass often due tothromobosis

Often resolves on its own; Increasedietary fiber and fluids; Stoolsofteners, bulk laxatives, and sitzbath; Refractory bleeding = IVfluid/ packing/surgery

Analgesics/sitz bath/stoolsofteners; Acute thrombosis:excision of clots

Refractory bleeding or pain;Incarceration/strangulation

ANORECTAL

Hemorrhoids

Page 154: Deja review _emergency_medicine__deja_review_

138 Deja Review: Emergency Medicine

Anorectal Abscess

What is the definition of an anorectalabscess?

Name some potential spaces where ananorectal abscess can occur.

What are some other causes of an anorectalabscess?

What is a common complication of ananorectal abscess?

What are some common clinical features ofan anorectal abscess?

What are some key points in themanagement of an anorectal abscess?

What are the indications for the use ofAbx?

Abscess that typically develops inone of the potential spaces near therectum/anus most often due toobstruction of glands at the base ofthe anal crypts

Perianal, intersphincteric, andischiorectal

Inflammatory bowel disease;Radiation injury; Cancer; Trauma; TB

Fistula formation

Dull aching pain that is worse withbowel movements and relieved after,sometimes palpable mass on exam,fever, and obvious discomfortwhenever patient sits

Simple perianal abscess = EDdrainage; Most require surgicalintervention; Most individuals donot require Abx

DM, immunocompromised patient,and valvular heart disease

Anal Fissure

What is the definition of an anal fissure?

What are some important points about ananal fissure?

What are some clinical features of an analfissure?

What are some key points in themanagement of an anal fissure?

Linear tears of the epitheliumwithin the anal canal, typically dueto recurrent diarrhea or passage oflarge hard stools

Majority are located in the posteriormidline; Number 1 cause of painfulrectal bleeding; IBD and TB areother causes

Severe pain with defecation, oftenwith a history of constipation, andlinear tear of the posterior midlineon exam. Pain is so intense, patientsoften try to avoid defecation

Symptomatic relief to allow healing;Analgesic topical, sitz bath, anddietary fiber; Refractory cases oftenrequire excision

Page 155: Deja review _emergency_medicine__deja_review_

What is the recurrence rate, even withtreatment?

Gastrointestinal Emergencies 139

Up to 50%

Fistula-In-Ano

What is the definition of a fistula-in-ano?

What is fistula-in-ano often caused by?

What conditions are fistula-in-anoassociated with?

What are some of the clinical features offistula-in-ano?

What is the primary treatment for fistula-in-ano?

Abnormal communication betweenanus and the skin

Commonly from ischiorectal orperianal abscess

TB; Cancer; IBD

Persistent blood-stained purulentdisharge or an abscess if it becomesblocked

Surgical incision

Anal Foreign Bodies

How do anal foreign bodies usually occur?

What important distinction must be madein regard to location of the foreign object?

What age group commonly present withanal foreign bodies?

What are some important points to knowabout anal foreign bodies?

What are some clinical features of analforeign bodies?

What are some commonly used diagnostictests for anal foreign bodies?

What are some key points in themanagement of anal foreign bodies?

Placement of object into anus;Transit from GI foreign body

Whether the object is below or abovethe rectosigmoid junction (difficult tovisualize and remove if above)

20–30 years of age (anal eroticism)

Often present late due to embar-rassment; Suspected in psychiatricpatients with anal symptom;Attempted self-extraction = risk ofperforation

Anal pain, bleeding, pruritus, andF/C with rigid abdomen inperforation

Abdomen x-rays; Upright CXR ifperforation suspected; Rigidproctosigmoidoscope

ER removal if the object if lowriding; Retractors, snares, forcepsmay be used; Serial observationafter removal; Surgical consult ifevidence of perforation

Page 156: Deja review _emergency_medicine__deja_review_

What is the definition of proctitis?

What are some clinical features ofproctitis?

What are some common causes of proctitis?

What is the diagnostic study of choice toevaluate proctitis?

What are some complications of proctitis?

What are some key points in themanagement of proctitis?

140 Deja Review: Emergency Medicine

It is inflammation of the rectalmucosa within 15 cm of the dentateline that typically affect adult males

Passage of blood and mucus,tenesmus, and abdominalcramping

Idiopathic; Infectious (HSV-1 and-2);Radiation; Ischemia

Proctosigmoidoscopy

Fistula; Fissures; Strictures

Analgesic relief; Abx if cause isinfectious (i.e., HSV-1); Sitz bath forrelief

Rectal Prolapse

What is the definition of rectal prolapse?

What is the pathophysiology of rectalprolapse?

What are some common causes of rectalprolapse?

What two age groups are commonlyaffected with rectal prolapse

What are some clinical features of rectalprolapse?

What are some complications of rectalprolapse?

What are some key points in themanagement of rectal prolapse?

It is when rectal mucosa or full-thickness rectal tissue slides outsidethe anal orifice

Initially begins as an internalprolapse that progresses to anexternal prolapse outside the analorifice

Straining (i.e., constipation);Weakness of the pelvic floor;Neurologic disorder

1. Pediatric (up to 90% resolve ontheir own)

2. Elderly (most require correctivesurgery)

Fecal incontinence, painless masson exam, and rectal bleeding

Ulceration; Bleeding; Necrosis

Most rectal prolapses can be reduced;Emergent surgery if evidence ofischemia; Stool softeners if reductionis successful

Proctitis

Page 157: Deja review _emergency_medicine__deja_review_

What is the definition of a pilonidal sinus?

What is pathophysiology for thedevelopment of a pilonidal sinus?

What are some clinical features of apilonidal sinus?

What are some key points in themanagement of pilonidal sinus?

Gastrointestinal Emergencies 141

Abscess that forms at the superioredge of the buttock in midline

Ingrowing hair that penetrates theskin and induces a foreign bodyreaction

Recurrent pain and purulentdischarge

Incision and drainage of abscess;Surgical intervention for excision;Consider Abx if immuno-compromised

What is the definition of diarrhea?

What are some important causes of diarrhea?

List common parasite-induced diarrhea:

What are some important things to knowabout viral-induced diarrhea?

What are some clinical features of viral-induced diarrhea?

What are some common modes oftransmission?

What are some key points in themanagement of viral-induced diarrhea?

Loose watery stools that occurmore than three times per day thattypically is self-limited, but canlead to dehydration and electrolyteimbalance

Infection (bacterial/viral/parasitic);Food intolerance; Medicationreaction; Intestinal disease (i.e.,celiac disease); Functional boweldisorder (i.e., IBS)

Giardia lamblia; Entamoeba histolytica;Cryptosporidium; Necator americanus

Causes the majority of all acuteepisodes; Norwalk and rotavirusmost common; Peak during wintermonths; Adenovirus also common

Low-grade fever, vomiting, diarrhea,mild abdominal cramping, andsometimes an upper respiratoryinfection (URI) prodrome beforehand

Sick contact; Contaminated food

Treatment is supportive; Ensureadequate hydration; Typically self-limited

Pilonidal Sinus

DIARRHEA

Page 158: Deja review _emergency_medicine__deja_review_

What are some important points to knowabout bacteria-induced diarrhea?

What are some examples of invasivebacteria?

What are some examples of toxin-producing bacteria?

What does a wet mount of stool typicallyshow?

What are some important points andtreatment for the following bacterial-induced diarrhea:

Vibrio cholera

Vibrio parahemolyticus

Staphylococcus aureus

Escherichia coli serotype O157:H7

E. coli (enterotoxigenic)

142 Deja Review: Emergency Medicine

Accounts for about 25% of acutediarrhea; Classified as invasive ortoxin producing

Salmonella; Shigella; Vibrio;Campylobacter

Bacillus cereus; Staphylococcus aureus;Clostridium difficile

Fecal leukocytes (typically + withbacteria); WBCs (use of methyleneblue)

Typically from contaminated water/seafood; Incubation about 5 days;Profuse watery diarrhea is thehallmark; Tx: IV hydration andAbx-fluoroquinolone

Invasive bacteria—typically frombad seafood; Range from mild toexplosive diarrhea; Tx: Supportivecare; usually self-limiting

Number 1 common cause of food-related diarrhea; Presentationfrom preformed toxins; Often inprotein-rich food such as meat;Incubation in hours; Tx: Supportive;usually self-limiting

Common cause of hemorrhagiccolitis; Often from contaminatedbeef and milk; Incubation inabout a week; Diarrhea, vomiting,and severe abdominal pain;Associated with HUS; Tx:Supportive—typically a week toresolve

Very common cause of traveler’sdiarrhea; Often in contaminatedfood and water; Presents likeV. cholera; Tx: Supportive; Abx canshorten course

Page 159: Deja review _emergency_medicine__deja_review_

Shigella

Salmonella

Campylobacter

Gastrointestinal Emergencies 143

Includes S. flexneri and S. dysenteriae;Highly infectious and usually fromfecal-oral; High-grade fever, bloodymucoid stool, and abdominal painis common; Tx: Typically resolvein a week, highly infectious, andsupportive care

Includes S. typhi and S. typhimurium;Often from contaminated food orpets; Immunocompromised patientsmost at risk; Variable presentation(i.e., typhoid fever); Tx: Mild casessupportive care; more severe casesmay require Abx

Very common cause of bacterialdiarrhea; Often in food (poultry)and water; More common in thepediatric population; Incubation isabout 4 days; Fever, HA, abdominalpain, and watery bloody stool; Tx:Abx in severe cases; Associated withHUS and Guillain-Barre syndrome

CLINICAL VIGNETTES

31-year-old male with long history ofalcohol abuse presents with progressivedifficulty in swallowing which wasinitially to foods only, but now to liquids;PE: unremarkable exam

29-year-old female who recently finishedher course of tetracycline presents withodynophagia, but is otherwise healthy;PE: unremarkable exam

4-year-old child is brought in by hermother due to recent onset of dysphagiaand gagging, otherwise the child ishealthy with no other complaints; PE:unremarable exam, clear oropharynx

21-year-old male with no PMH presentswith abdominal pain that was initiallyaround the umbilicus, but now hasprogressed to the RLQ associated withnausea and vomiting after the onset of pain;PE: RLQ tenderness and (+) Rosving’s sign

Esophageal carcinoma

Inflammatory esophagitis

Swallowed foreign body

Appendicitis

Page 160: Deja review _emergency_medicine__deja_review_

61-year-old elderly male with arthritispresents with epigastric pain that is oftenrelieved by intake of food, but is otherwisehealthy; PE: epigastric tenderness, but norebound

45-year-old female with an Hx of recentERCP presents with epigastric painradiating to her back associated withnausea and emesis; PE: epigastrictenderness; Labs: elevated lipase

18-year-old female with a history ofbulimia presents with chest pain withdysphagia that occurred soon after her boutof emesis; PE: unremarkable exam

34-year-old obese female presents with RUQpain along with fever and nausea, patienthas a history of gallstones; PE: fever,tachycardia, RUQ tenderness, and yellowishsclera on examination of eye; Labs: elevatedalkaline phosphate and LFTs

81-year-old female with an Hx of HTN,afib, and DM presents with a sudden onsetof diffuse abdominal pain along withnausea and vomiting; PE: pain out ofproportion on exam, guaiac positive stool,and rebound; Labs: elevated lactate

24-year-old female presents with 2 days oflower GI bleeding and describes the toiletbowl as being bright red after each bowelmovement, other then a history ofconstipation, patient is otherwise healthy; PE:unremarkable exam and guaiac positive stool

56-year-old male presents with LLQ painwith nausea, vomiting, and urinary changesfor 2 days; PE: LLQ tenderness and norebound on exam; Labs: elevated white count

31-year-old male with recent dischargefrom hospital now presents with diffusewatery diarrhea and crampy abdominalexam: PE: low-grade fever and mildtenderness of abdomen

31-year-old male with history of HIVpresents with tenesmus, abdominalcramping, and passage of blood and mucusfor 3 days; PE: tenderness on rectal exam

144 Deja Review: Emergency Medicine

Peptic ulcer disease

Pancreatitis

Mallory-Weiss syndrome

Ascending cholangitis

Mesenteric ischemia

Internal hemorrhoids

Diverticulitis

Pseudomembranous enterocolitis

Proctitis

Page 161: Deja review _emergency_medicine__deja_review_

145

C H A P T E R 8

GenitourinaryEmergencies

ACUTE RENAL FAILURE

What is the definition of acute renal It is deterioration of renal function failure (ARF)? that results in accumulation of waste

and loss of internal homeostasis

What are some key physiology points Kidney receives 25% of the cardiac about the kidney? output; Outer medulla is susceptible

to hypoxia; With decreased renal bloodflow (RBF), increased susceptibilityto toxins

What is the primary way to assess renal Glomerular filtration rate (GFR)function? (via creatinine clearance)

What are some important things to knowabout each in the following setting:

Community-acquired ARF The majority of cases are reversible;Mortality rate is less then 10%; Themost common cause is hypovolemia(pre-renal)

Hospital-acquired ARF Mortality rate can be higher than50%; Many typically have othercomorbidities; Most common causeis iatrogenic (intrinsic)

What are three types of acute-renal failure? 1. Prerenal2. Renal3. Postrenal

What are some important causes of Hypovolemia (i.e., diuretics/ prerenal azotemia? dehydration); Third space

sequestration (i.e., pancreatitis);Sepsis; Decreased cardiac output

Copyright © 2008 by The McGraw-Hill Companies, Inc. Click here for terms of use.

Page 162: Deja review _emergency_medicine__deja_review_

What is the typical urine status in patients Oliguric; Avid reabsorption of sodiumwith prerenal azotemia? and water; BUN/creatanine (BUN/

Crea) ratio of 20:1; U/A typicallyshows no evidence of damage;Fractional excretion of sodium <1%

What are some important causes of renal Acute tubular necrosis; Thrombosis;azotemia? Glomerular disease; Vascular disease;

Acute interstitial nephritis

What are common causes of acute tubular Ischemia—most common; Pigmentsnecrosis (ATN)? (i.e., myoglobin); Nephrotoxic agents

What are some common nephrotoxic Contrast dye; Nonsteroidal anti-agents? inflamnatory drugs (NSAIDs);

Angiotensin-converting enzymeinhibitors; Antibiotics (i.e., penicillin)

What is the typical urine status in patients Inability to concentrate urine (dilute);with renal azotemia? Have evidence of damage (i.e., casts);

High urine sodium (>40 mEq/L)

What are some important causes of Ureteral obstruction (i.e., stones);postrenal azotemia? Bladder obstruction; Urethral

obstruction (i.e., strictures)

What are some important tests to Urinalysis; Ultrasound; Postvoidconsider to differentiate the type of ARF? residual urine; Urine and serum

Na and creatinine; Urine osmolality;Urine eosinophil

What are some key points in treatment foreach of the following causes of ARF:

Prerenal Rapid volume replacement; Find thecause of hypoperfusion and correct it;Initial fluid administration of isotonicsaline is appropriate in most cases

Renal Increase the urine flow; If cause isa nephrotoxic agent, remove it;Maintaining balance of fluid/electrolytes; Dialysis if indicated

Postrenal Relieve obstruction; Catheter untilobstruction is relieved

What are some important points for eachof the following complications of ARF:

Hypocalcemia It is common in setting of ARF;Typically asymptomatic; Intravenous(IV) calcium chloride if symptomatic

146 Deja Review: Emergency Medicine

Page 163: Deja review _emergency_medicine__deja_review_

Hypermagnesium It is common in setting of ARF;Typically asymptomatic

Hyperkalemia Potentially the most life-threatening;Death due to cardiac dysrhythmias;Important to obtain serum K and ECG;Treat (IV glucose/insulin, bicarb, etc.)

Metabolic acidosis It is also common in the setting of ARF;Typically asymptomatic

What are some indications of dialysis in Hyperkalemia; Uremia (i.e., encephalo-the setting of ARF? pathic); Creatinine >10 mg/dL or

BUN > 100 mg/dL; Clinicallysignificant fluid overload/acidosis;Particular nephrotoxins (i.e., ethyleneglycol)

CHRONIC RENAL FAILURE

What is the definition of chronic renal The irreversible and gradual loss of failure (CRF)? renal function that results in inability

to regulate homeostasis and concen-trate urine

What are the two most common causes of 1. DiabetesCRF? 2. Hypertension

What are some other causes of CRF? Glomerulonephritis; Polycystickidney disease; Alport syndrome

What are some important things toknow for each of the following stagesof CRF:

Stage I Decreased renal function <50% GFR;At least 1/2 of renal function is gone;Homeostasis and excretion intact

Stage II Renal insufficiency with 20–50% GFR;Mild anemia due to decreasederythropoietin (EPO); Mild azotemia

Stage III Renal failure with 5–20% GFR;Severe anemia; Azotemia; Electrolyteimbalance (i.e., hyperkalemia)

Stage IV Renal failure <5% GFR; Multipleorgan system effects

What is the treatment for CFR? Kidney transplant; Peritonealdialysis; Hemodialysis (also for ARF)

Genitourinary Emergencies 147

Page 164: Deja review _emergency_medicine__deja_review_

What are some complications associated Infection of vascular access;with hemodialysis? Thrombosis of the vascular access;

Hemorrhage

NEPHROLITHIASIS

What is the definition of nephrolithiasis? Supersaturation of a mineral within the ureters that result in urinarychanges and ureter spasms

What are some important things to know More common in males between 20–45;about nephrolithiasis? There is a hereditary predisposition;

Over 90% of stones <5 mm will pass;Recurrence can be as high as 50%

What is the most common type of kidney Calcium oxalate (about 75% of all stone? stones)

What are some possible causes of calcium Hyperparathyroidism; Sarcoidosis;stone? Neoplasm

What are some important things to note for each of the following stone types:

Struvite stone After calcium stones, the next most common; Radiopaque; Associatedwith urea-splitting Proteus

Uric acid stones Next common after calcium and struvite; Radiolucent; Common inpatients with gout and leukemia

Xanthine stones Rare; Radiopaque; Associated withmethylxanthine/theophylline

Cystine stones Radiopaque; Familial associated

What are some clinical features of Unilateral flank pain that is often nephrolithiasis? colicky, can also have pain in the

back with radiation to the groin (labia/testicles), urinary symptoms (hematuria, dysuria, etc.), and nausea/vomiting (N/V)

What is another important diagnosis to Abdominal aortic aneurysmconsider in a person who presents for thefirst time with flank pain and is elderlywith history of uncontrolled HTN?

What are some important laboratory teststo consider and common findings:

Complete blood count (CBC) Usually normal

148 Deja Review: Emergency Medicine

Page 165: Deja review _emergency_medicine__deja_review_

U/A Hematuria (can be absent in upto 25%); Urinary pH >7.6 (suspectProteus)

Urine culture Positive if infection is present

BUN/Crea To assess renal function

What are some important diagnostic tests CT: diagnostic study of choice;to consider? Intravenous pyelogram (IVP): for

anatomical/functional assessment;Ultrasound (U/S): for pregnant women and children

What are some key points in the Proper fluid hydration; Narcoticmanagement of nephrolithiasis? with ketorolac (optimal pain control);

Antiemetic for sustained emesis

What are some common indications for Evidence of active infection admission of a patient with nephrolithiasis? (fever/pyuria); Inability to tolerate

oral intake; Stone >5 mm (unlikely topass on its own); Renal insufficiency

URINARY TRACT INFECTIONS

What is the definition of a urinary tract Presence of bacteria in the urinaryinfection (UTI)? system

What are some important things to know One of the most common bacterial about UTI? infections; 50% of women will have

at least one UTI; Sexual activityincreases risk of UTI

What are the three most common 1. Escherichia coli (up to 80% of allorganisms associated with UTI? UTIs)

2. Chlamydia3. Staphylococcus saprophyticus

What are some clinical features of UTI? Dysuria, urge to urinate, increased urination frequency, nocturia, and suprapubic heaviness (should not have systemic effects such as fever)

What are some other differentials to cons- Pelvic inflammatory disease (PID); ider in a woman who presents with UTI? Vulvovaginitis

What are some features of a complicated Resistant species; Male; Children orUTI? elderly; Pregnant female; Associated

condition such as pyelonephritis;Underlying anatomical abnormalityof the GU system

Genitourinary Emergencies 149

Page 166: Deja review _emergency_medicine__deja_review_

List some methods used to collect a Catheterization; Midstream cleanproper urine sample. catch; Suprapubic aspiration (infants)

What are some common microscopic Pyuria (>10 WBC/HPF in womenfindings in a U/A of a patient with UTI? and 1–2 for for men); Significant

bacteriuria

When is it appropriate to obtain a urine Infants; Men; Pregnant females; culture? Associated complications

What are some key points in the Bactrim or nitrofurantoin are first-line;management of a UTI? Fluoroquinolone if UTI is complicated;

Treatment time ranges from 7–14 days;Pyridium if dysuria is intolerable

What is another important consideration Rule out STDs in sexually active in a patient who presents with a UTI? patients

What is the definition of pyelonephritis? It is infection of the kidney most commonly as a result of a UTI withascending infection

What are some risk factors for the Recurrent UTI; Immunocompromiseddevelopment of pyelonephritis? person; Vesicoureteric reflux

List the classification of pyelonephritis. Acute pyelonephritis; Chronicpyelonephritis (chronic infection);Reflex nephropathy (typicallyobstruction)

What are some clinical features of High-grade fever and chills in the pyelonephritis? setting of a UTI typically with

flank/back pain and nausea/emesis

What are some complications of Chronic pyelonephritis; Perinephricpyelonephritis? abscess; Sepsis; ARF

What are some key points in the If mild and can tolerate PO—management of pyelonephritis? fluoroquinolone may discharge with

follow-up; Low threshold to admit ifelderly or if severe

What are some indications for admissions Uroseptic; Children and elderly; in patients with pyelonephritis? Unable to tolerate PO and persistent

emesis; Immunocompromised

MALE GENITAL PROBLEMS

What are the three cylindrical bodies of 1. Two corpora cavernosathe penis? 2. Corpus spongiosum

What is the primary blood supply of the Internal pudendal artery to the penis? superifical/deep penile arteries

What is the average size of the testis? 4–5 cm × 3 cm

150 Deja Review: Emergency Medicine

Page 167: Deja review _emergency_medicine__deja_review_

What are the two investing layers of the 1. Tunica albugineatestis? 2. Tunica vaginalis

What are some important components of Visual inspection; Palpation of thethe physical exam? scrotum for fluid; Milk the penis

for discharge; Rectal exam (checkprostate); Check for inguinalhernias

Common Genital Infections

What are some common organisms Gonorrhea and chlamydia (most responsible for urethritis? common); Trichomonas vaginalis;

Ureaplasma urealyticum

What are some clinical features of Discharge and dysuria, but can beurethritis? asymptomatic

How is the diagnosis of urethritis usually Gram stainmade?

What is the treatment of choice? Directed against gonorrhea (i.e., ceftriaxone); Directed againstchlamydia (i.e., doxycycline);Metronidazole if suspected trichinfection

What is orchitis? Inflammation of the testis

What are some common causes of orchitis? Systemic infections (i.e., mumps);Direct extension such as epididymitis

What are some clinical features of orchitis? Testicular swelling and pain that typically does not include urinarysymptoms

Does mumps-induced orchitis require No—typically resolves treatment?

What is the key point in the management Urology consultation; It is rare whenof orchitis? compared to torsion/cancer

What are some common etiologic causes Usually gram (–) bacteria such as of acute bacterial prostatitis? E. coli, Proteus, and Pseudomonas

What are some common clinical features Urinary symptoms (i.e., dysuria), of acute bacterial prostatitis pelvic/back pain, systemic signs of

infection such as fever/chills; PE:swollen/tender prostate

What should one be careful not to do Massaging the prostate during a rectal exam?

What will urinanalysis commonly show? Evidence of cystitis

Genitourinary Emergencies 151

Page 168: Deja review _emergency_medicine__deja_review_

What are some key points in the Antibiotic therapy; Analgesics, stoolmanagement of acute bacterial prostatitis? softeners, and hydration; Urology

consultation if evidence of urinaryretention

What are some common causes of penile Herpes simplex, chancroid, syphilis, ulcers? and granuloma inguinale

How is the diagnosis of syphilis Positive VDRL or RPR confirmed bycommonly made? Treponema-specific tests

What is the antibiotic of choice for Penicillin, doxycycline, and syphilis? tetracycline

What is the cause of a chancroid? Haemophilus ducreyi

What is the antibiotic treatment of choice A macrolide (i.e., azithromycin)for a chancroid?

What is the cause of granuloma inguinale? C. granulomatous

What are some antibiotics commonly used Doxycycline or trimethoprim-to treat granuloma inguinale? sulfamethoxazole (TMP-SMX)

What is Fournier’s gangrene? Known as idiopathic scrotal gangrene

What are some common clinical features Often febrile and toxic with a painfulof Fournier’s gangrene? erythematous penis/scrotum

What groups are more commonly affected Elderly; Diabetics;with Fournier’s gangrene? Immunocompromised people

What are some common etiologic causes Typically mixed: E. coli, Streptococcus,of Fournier’s gangrene? Bacteroides fragilis, etc.

What are some key points in the Broad-spectrum antibiotics; Urologicmanagement of Fournier’s gangrene? consult for debridement; Supportive

management

What is phimosis? Inability to retract foreskin behind the glans

What is the most common cause of Chronic infection of the foreskinphimosis? that results in scarring

What can be done if phimosis appears to Dorsal slit of the foreskin and be causing vascular compromise? circumcision for definite treatment

What is paraphimosis? Inability to pull the foreskin over the glans

What is the primary concern of Vascular compromiseparaphimosis?

What can be done in an emergent situ- Dorsal slit and circumcision for ation if vascular compromise is evident? definitive treatment

What age group is epididymitis more Young adultscommon in?

152 Deja Review: Emergency Medicine

Page 169: Deja review _emergency_medicine__deja_review_

What are some clinical features of Gradual onset of unilateral testicular epididymitis? pain, dysuria, fever, and tenderness

of epididymis on exam

What is Prehn’s sign? Relief of testicular pain byelevating it

What are some common etiologic causes E. coli, Pseudomonas, and Chlamydiaof epididymitis?

What are some common diagnostic studies CBC; Urethral culture and gram stain;to consider in epididymitis? Urinanalysis

What are some key points in the Antibiotic coverage (i.e., Ciprofloxacin);management of epididymitis? Stool softeners; Analgesics with ice

What age groups are commonly affected Bimodal: neonates and 12–18 years with testicular torsion? of age

What are some important elements in the Recent physical exertion (i.e., sports/history of a patient who presents with sex); History of testicular pain withtesticular torsion? relief after; History of cryptorchidism

What are some clinical features of Acute onset of unilateral testicular testicular torsion? pain often with nausea/vomiting;

PE: affected testicles are high riding with loss of cremasteric reflex

What diagnosis can testicular torsion be Epididymitisconfused with?

What is the diagnostic test of choice for Color Doppler ultrasoundtesticular torsion?

What are some key points in the Urgent urology consult for surgery;management of testicular torsion? Surgery within 6 hours: 80–100%

salvage; Analgesics prior to surgery;Salvage rate is 20% after 10 hours and 0% after 24 hours

CLINICAL VIGNETTES

43-year-old female with PMH of afib Renal infarctpresents with a sudden onset of left flankpain and hematuria Abdominal CT: wedge-shaped lesion of the left kidney

24-year-old female presents with dysuria Urinary tract infection and increased frequency of urination,patient is sexually active; PE: suprapubictenderness; U/A: (+) nitrate and leukocyte esterase

Genitourinary Emergencies 153

Page 170: Deja review _emergency_medicine__deja_review_

31-year-old male presents with sudden Nephrolithiasis onset of right flank pain along with nausea, vomiting, and hematuria; PE:right CVA tenderness and in severe pain;U/A: (+) blood; U/S: shows righthydronephrosis

26-year-old male presents with hemoptysis, Goodpasture’s syndromedark urine, and general fatigue for 3 days;PE: unremarkable exam; Labs: anti-GBMantibodies and urine that shows blood

81-year-old male with DM presents to the Fournier’s gangreneER via EMS with fever and appears sicklooking; PE: unremarkable except an erythematous penis that is very tender to the touch with evidence of a prior wound in the scrotum

14-year-old male with a recent history of Poststreptococcal glomerulonephritissore throat presents with low-urine outputand swelling of lower legs; PE: periorbitaledema; Labs: elevated BUN/Crea andurine that shows blood

25-year-old female presents fever, chills, Pyelonephritisand left flank pain for about 2 days; PE: left CVA tenderness; U/A: (+) nitrate and leukocyte esterase

21-year-old male with no PMH presents Bacterial prostatitis with fever, dysuria, and pelvic/back pain;PE: remarkable for a tender and swollen prostate

82-year-old male with a long history of Bladder cancersmoking presents with frank blood onurination, also with recent weight loss: PE: unremarkable exam; U/A: gross blood

64-year-old male presents with a 2-week Benign prostatic hyperplasia (BPH)history of nocturia, urinary hesitance, andweak stream during urination, otherwisehealthy; PE: rectal exam showed diffuselyenlarged prostate; Labs: normal prostate-specific antigen (PSA)

18-year-old male with PMH of undescended Testicular torsiontestis presents with sudden onset of righttesticular pain associated with nausea andvomiting; PE: tender/swollen right testiclewith (–) cremasteric reflex

154 Deja Review: Emergency Medicine

Page 171: Deja review _emergency_medicine__deja_review_

61-year-old female with a history of long- Chronic renal failurestanding hypertension and DM presents with altered mental status; Labs: significant for a potassium of 6, BUN of99, creatinine of 7 with a GFR <5%

21-year-old male presents with a gradual Epididymitisonset of unilateral testicular pain, fever,and dysuria for about 4 days, patient doesadmit to having unprotected sex; PE:tenderness of the penis on exam that is relieved when raised

Genitourinary Emergencies 155

Page 172: Deja review _emergency_medicine__deja_review_

This page intentionally left blank

Page 173: Deja review _emergency_medicine__deja_review_

157

C H A P T E R 9

EndocrineEmergencies

HYPOGLYCEMIA

What is the glucose blood level where <50 mg/dLmanifestations of hypoglycemia typicallyoccur?

What are some common clinical features Tachycardia, tremulousness,of hypoglycemia? diaphoresis, mental status change,

seizures, focal neurologic deficits that can mimic a stroke

What are some elements of the history to Medications; Medical problems;obtain in a patient who is hypoglycemic? Fasting/fed state preceding

List some hormones released during Glucagon; Epinephrine; hypoglycemia. Growth hormone; Cortisol

List common causes of fasting Islet cell tumor; Myxedema; hypoglycemia. Adrenocortical insufficiency;

Extrapancreatic tumor

List common causes of post-prandial Hyperinsulinism; Fructose intolerance;hypoglycemia. Galactemia

What is the most common cause of Insulin and sulfonylureashypoglycemia in diabetics?

List other medications that commonly Beta-blockers; Ethanol; Salicylate; cause hypoglycemia. Cimetidine

What is Whipple’s triad? Diagnostic features of insulinoma ofthe pancreas:1. Symptoms and signs of

hypoglycemia2. Blood sugar levels below 50 mg/dL3. Recovery from an attack following

the administration of glucose

Copyright © 2008 by The McGraw-Hill Companies, Inc. Click here for terms of use.

Page 174: Deja review _emergency_medicine__deja_review_

What are some key points in the Monitor glucose (glucometer oftenmanagement of hypoglycemia? not reliable below 50 mg/dL);

Intravenous (IV) D5W continuousdrip; Intramuscular (IM) glucagonand IV dextrose if patient is unre-sponsive; If patient is awake, oralfeeding is preferred

What are some indications to admit a Admit if overdose on insulin or oralpatient who is hypoglycemic? hypoglycemics; Patients suspected

of having fasting hypoglycemia forevaluation

DIABETIC KETOACIDOSIS

What is the definition of diabetic Diabetic ketoacidosis is a state of ketoacidosis (DKA)? absolute or relative insulin deficiency

aggravated by ensuing hyperglycemia,dehydration, and acidosis-producingderangements

What population is DKA primarily Predominately type 1 diabeticsseen in?

What are some metabolic derangements Relative or absolute lack of insulin;that occur with DKA? Excessive stress hormones (i.e.,

cortisol); Overproduction of free fattyacids

What are three commonly seen ketone 1. Acetacetatebodies? 2. Beta-hydroxybutyrate

3. Acetone

Which ketone body is not measured in Beta-hydroxybutyrateserum?

List important precipitating factors of Infection (esp. PNA and UTI); Lack ofDKA? insulin; Trauma; Surgery; Myocardial

Infarction (MI) and cerebral vascularaccident (CVA)

What are the clinical symptoms of DKA Volume depletion; Degree ofprimarily due to? hyperosmolality; Metabolic acidosis

What are some clinical features of DKA? Nausea, vomiting, and abdominalpain; partial motor seizures, visualchanges, lethargy, obtundation,and coma; fruity breath andhyperventilation

158 Deja Review: Emergency Medicine

Page 175: Deja review _emergency_medicine__deja_review_

What are some commonly used diagnostic Complete blood count (CBC); Chem-7;tests in DKA? Serum ketones; Calcium/magnesium/

phosphorus; ECG—for changes inserum potassium

What particular electrolyte is important to Potassiummonitor in DKA?

What are some important confirmatory pH: <7.3; Bicarb: <10 mEq/L; Serumlaboratory results in DKA? acetone: 2:1 ratio; Serum glucose:

>350 mg/dL

What are some key points in the ABCs and IV-O2-monitor; Correctmanagement of DKA? fluid losses, often require up to 5 L;

IV infusion of low-dose insulin; Earlypotassium replacement; Consider useof bicarbonate (if pH <7.0)

What are some complications that may CHF from aggressive fluid resusci-occur when treating DKA? tation; Hypokalemia from not

replacing potassium; Hypoglycemiafrom not monitoring glucose; Alkalosisfrom too much bicarbonate

What is the primary cause of mortality in Sepsiselderly patients with DKA?

What is alcoholic ketoacidosis? Accumulation of ketones in the blood,caused by excessive alcohol consump-tion and lack of food intake

What are some clinical features of Nausea, vomiting, and abdominal alcoholic ketoacidosis? pain; partial motor seizures, visual

changes, lethargy, obtundation, and coma

What are some common laboratory High anion gap acidosis; Serumfindings in alcoholic ketoacidosis? glucose— <200 mg/dL; Hypokalemia;

Serum EtOH low or not present

What are some key points in the Large volume fluid replenishment;management of alcoholic ketoacidosis? Early potassium replacement; Thiamine

prior to glucose administration;Insulin typically not indicated

THYROID

Myxedema Coma (Hypothyroid)

What is an important point to know True emergency with up to 45% about myxedema coma? mortality

Endocrine Emergencies 159

Page 176: Deja review _emergency_medicine__deja_review_

What is the normal physiologic mechanism Hypothalamus—TRH; Anteriorof thyroid hormone production? pituitary—thyroid-stimulating

hormone (TSH); Thyroid—T3 and T4

What is primary hypothyroidism? Intrinsic failure of the thyroid gland—most common cause

What are some common causes of primary Partial thyroidectomy; Radioactivehypothyroidism? ablation; Autoimmune (i.e.,

Hashimoto’s thyroiditis); Iodinedeficiency; Medications (i.e., lithium)

What is secondary hypothyroidism? Hypothyroidism due to dysfunction of the pituitary or hypothalamusgland

What are some common causes of Pituitary tumor; Sarcoidosis; secondary hypothyroidism? Sheehan’s syndrome

What are some common clinical features Cold-intolerance, hypoventilation, of hypothyroidism? fatigue, constipation, weight-gain,

memory loss, irregular menstruation,scaly skin, and muscle cramps

What is the definition of myxedema coma? A rare and severe form ofhypothyroidism typically due toundertreatment/undiagnosed

Who is commonly affected with Elderly womenmyxedema?

What is the most common cause for the Physiological stressor (i.e., infections)progression of myxedema to myxedemacoma?

List some other common causes of Trauma; CHF; Medications (beta-myxdema coma? blockers)

What are some important diagnostic tests TBG, TSH, and free T4; CBCto consider in the evaluation of myxedema (possible infection); Chem-7coma?

What are some key points in the Supportive measures; Correctionmanagement of myxedema coma? of electrolyte disturbances;

Vasopressors for hypotension;Thyroxine IV pushed slowly;Hydrocortisone for adrenalinsufficiency; Antibiotics forunderlying infection; Search forunderlying trigger

What is the disposition for patients who Generally admitted to ICU;present with myxedema coma? Require endocrinologist consult

160 Deja Review: Emergency Medicine

Page 177: Deja review _emergency_medicine__deja_review_

Thyroid Storm

What is the definition of hyperthyroidism? Elevated level of thyroid hormones can result in clinical manifestationsranging from mild to severely toxic with resultant morbidity and mortalityfor affected patients

What is the most common form of Grave’s diseasehyperthyroidism?

What is Grave’s disease? Autoimmune condition in whichautoantibodies are directed againstthe TSH receptor resulting in increaseof thyroid hormone

What are some common clinical features Heat intolerance, palpitations, fatigue,of hyperthyroidism? increased bowel movements, moist

skin, insomnia, tremulous hands, andCNS hyperactivity

What are some characteristic physical Exophthalmus, tachycardia, andfindings in a patient with hyperthyroidism? palpable goiter

What is thyroid storm? Thyroid storm is a decompensated state of thyroid hormone–induced severe hypermetabolism involving multiple systems

What are four diagnostic criteria used to 1. Temperature >37.8°Cdiagnose thyroid storm? 2. Central nervous system (CNS)

symptoms (i.e., obtundation)3. Cardiovascular (tachycardia,

dysrhythmias)4. Gastrointestinal (i.e., diarrhea)

What are some common triggers of thyroid Infection; Grave’s disease; Trauma;storm? MI

What are some key points in the Supportive measures; Antithyroidmanagement of thyroid storm? medication (i.e., propylthiouracil);

Iodine (suppresses release of T3/T4);Treat any other complications (i.e., afib);Glucocorticoids and propranolol oftenused

ADRENAL

What are the two major regions of the 1. Adrenal medullaadrenal gland? 2. Adrenal cortex

What major hormones does the adrenal Catecholaminesmedulla produce?

Endocrine Emergencies 161

Page 178: Deja review _emergency_medicine__deja_review_

What three major hormones does the 1. Aldosteroneadrenal cortex produce? 2. Glucocorticoid

3. Androgens

What are some common causes of primary Infections (i.e., TB); Infiltrativeadrenal insufficiency (Addison’s disease)? (i.e., metastatic); Medications;

Idiopathic atrophy

What is the name of the syndrome of Waterhouse-Friderichsen syndromebilateral adrenal hemorrhage secondary tomeningococcemia?

What are some common causes of Pituitary tumor; Head trauma;secondary (pituitary dysfunction) adrenal Infections; Infiltrative diseaseinsufficiency? (i.e., sarcoidosis)

What is the most common cause of adrenal Iatrogenic steroid use (chronic)suppression?

What are some clinical features of adrenal Weak, fatigable, lethargic, postural insufficiency? hypotension/syncope secondary to

aldosterone deficiency, nausea, abdominal pain, and emesis

What are characteristic laboratory findings Hyponatremia and hyperkalemia;in adrenal insufficiency? Hypoglycemia; Azotemia

What is adrenal crisis? Patients who have underlying chronic adrenal insufficiency and are exposed to any stress

What are some common stressors that can Infections; Trauma; Surgery;put a patient into adrenal crisis? Pregnancy

What are some common clinical features Typically weak and very ill appearing, of adrenal crisis? gastrointestinal (GI) affects (i.e.,

diarrhea), hypotension, delirium,and possible seizure

What are some key points in the IV fluids with dextrose; Glucocorticoidmanagement of adrenal crisis? (dexamethasone)

CLINICAL VIGNETTES

28-year-old female presents with heat Grave’s diseaseintolerance, fatigue, increased bowelmovements, palpitations, and tremulous hands for months, but otherwise doingwell; PE: exophthalmus, tachycardia,palpable goiter; Labs: increased TSH, reduced T4 and T3 levels

162 Deja Review: Emergency Medicine

Page 179: Deja review _emergency_medicine__deja_review_

27-year-old male presents with weight Addison’s diseaseloss, progressive weakness, nausea; PE: hyperpigmentation of skin; Labs:hyponatremia and hyperkalemia

41-year-old obese female presents with Cushing’s diseaseirregular menses, HTN, and increase in weight along with visual changes; PE:buffalo hump, hirsutism, and increase inBP; Labs: increased ACTH and suppressionwith high-dose dexamethasone test

21-year-old male presents with 2-week Diabetes insipidushistory of polyuria and polydipsia; U/A:urine osmolality <200 mosm/kg,hypernatremia, and urine specific gravityof <1.005

18-year-old male with history of type Diabetic ketoacidosisI diabetes mellitus presents with diffuseabdominal pain, nausea, and vomitingalong with confusion; PE: shallow rapidbreathing; Labs: glucose >300 andmetabolic acidosis

34-year-old male is brought in by EMS for Hypoglycemiaaltered mental status and only knows that the patient is on sulfonylureas; PE: tachycardic, diaphoresis, andtremulousness

81-year-old female currently taking Myxedema comathyroid hormones presents via EMS withobtundation; PE: hypothermia,bradycardia, hypoventilation, cold non-pitting edema of legs; Labs: free T4 and T3 levels are low

Endocrine Emergencies 163

Page 180: Deja review _emergency_medicine__deja_review_

This page intentionally left blank

Page 181: Deja review _emergency_medicine__deja_review_

165

C H A P T E R 1 0

Hematology andOncology Emergencies

HEMATOLOGY

What is hemostasis? It is an intrinsic balance between thrombosis and excessive bleeding

What are the major components of Platelets; Vascular integrity; hemostasis? Coagulation factors; Fibrinolysis

What are some components for bleeding Abnormal platelet function or count;disorders? Missing factors in the coagulation

cascade; Excessive fibrinolysis;Inflammation of blood vessel walls

What are some important points for thefollowing laboratory tests used to evaluatehemostasis:

Bleeding time (BT) Normal time 3–8 minutes; Measuresintegrity of platelet function; IncreasedBT in von Willebrand’s (vWF) diseaseand uremia; Aspirin and nonsteroidalanti-inflammatory drugs can affectBT

Platelets Normal is 150,000–400,000 per mm3;Decreased count: disseminated intravascular coagulation (DIC), uremia, idopathic thrombocytopenic purpura (ITP), etc.; Increased count:consider malignancy; Less than50,000 per mm3: post-traumaticbleeding; Less than 20,000 per mm3:life-threatening bleeding possible

Prothrombin time Normal time is 10–12 seconds;Measures extrinsic (factor VII)pathway; Normal PTT but elevated

Copyright © 2008 by The McGraw-Hill Companies, Inc. Click here for terms of use.

Page 182: Deja review _emergency_medicine__deja_review_

PT: factor VII deficiency; Coumadin/Vitamin K/liver disease: < factor VII

Internationalized normalized PT ratio: normal value 1; Monitorratio (INR) anticoagulation in Coumadin; INR

2–3 for most patients (i.e., afib); INR2.5–3.5 for patients with mechanicalvalves

Partial thromboplastin time (PTT) Normal time is 25–35 seconds;Measures integrity of intrinsic path-way; Elevated in heparin use andhemophilia

Bleeding Disorder

What is one of the oldest hereditary Hemophiliableeding disorders?

What are two types of hemophilia and 1. Hemophilia A: lack of factor VIIItheir associated factor deficiency? 2. Hemophilia B: lack of factor IX

What is the more prevalent form? Hemophilia A

What are some important elements in the Hematomas; Hemarthrosis; Prolongedbleeding history of hemophilia? bleeding from dental procedures;

Spontaneous hematuria; Epistaxis

What are characteristic laboratory findings Prolonged PTT; Normal BT, PT,in hemophilia? and platelets

What is hemophilia A known as? Classic hemophilia

What are some important points to know Sex-linked recessive disorder; about hemophilia A? Deficiency of factor VIII; Intracranial

hemorrhage major cause of death

What are some key points in the Infusion of Factor VIII; Desmopressinmanagement of hemophilia A? (DDAVP); Cryoprecipitate (not used

often)

What is hemophilia B known as? Christmas Disease

What are some important things to know Sex-linked recessive disorder;about hemophilia B? Deficiency of factor IX; Comprises

about 15% of all hemophilias

What are some key points in the Factor IX concentrate; Fresh frozenmanagement of hemophilia B? plasma (FFP)

What is vWF? Autosomal dominant with either quantitative or qualitative disorder in vWF

166 Deja Review: Emergency Medicine

Page 183: Deja review _emergency_medicine__deja_review_

What is the primary defect in each of thefollowing forms of vWF disease:

Type I A low level of the vWF factor;Mildest and most common form

Type II Qualitative disorder of vWF

Type III Virtual absence of vWF;Most serious, but rare form

What are some common clinical features Mucocutaneous bleeding is the of vWF disease? defining feature (i.e., epistaxis)—

bleeding is milder then hemophilia A

What are classic laboratory findings in a Increased BT; Increased PTT; Normalpatient with vWF disease? platelet count and function; Normal

PT

What are treatment options available for FFP; Cryoprecipitate; DDAVPvWF disease?

Platelet Disorders

What is the most common platelet Thrombocytopeniadysfunction?

What are some important causes of Decreased platelet production; thrombocytopenia? Increased platelet destruction;

Increased splenic clearance

What are some examples for the following Decreased platelet production. causes of thrombocytopenia? Aplastic anemia; Radiation;

Myelofibrosis

Increased platelet destruction. Sepsis;Thrombotic thrombocytopenic pur-pura (TTP); HELLP syndrome

What are some clinical features of Mucocutaneous bleeding (i.e., thrombocytopenia? epistaxis), hematuria, menorrhagia,

and GI bleeding

What are common laboratory findings in Low platelets and increased BT; PTa patient with thrombocytopenia? (INR) and PTT will be normal

What are a few indications for platelet Platelet <50,000 per mm3 and majortransfusion in a patient with bleeding; Platelet <20,000 per mm3

thrombocytopenia?

What is the most feared complication of ICHthrombocytopenia?

What is the most common hemorrhagic ITPdisease in the pediatric population?

Hematology and Oncology Emergencies 167

Page 184: Deja review _emergency_medicine__deja_review_

What are some important things to know Typically occurs in patients between about ITP in children? 2 and 8 years of age; Generally self-

limited and resolves in weeks; Oftentriggered by viral infections

What are some key points in the Treatment primarily supportive;management of ITP in children? Transfusion: major bleeding/platelet

count; Consider use of steroids—dexamethasone

What age group typically manifests with Females between 25 and 40 yearsITP refractory to standard treatment of of agesteroids?

What are some treatment options for cases Platelet transfusion;of ITP that are refractory to steroids? Immunosuppressive drugs;

Splenectomy

What is a platelet disorder that has a very TTPhigh mortality rate if left untreated?

What are some common clinical features Thrombocytopenic purpura, hemolyticof TTP? anemia, mental status change, fever,

and renal disease

What is the treatment of choice for TTP? Fresh frozen plasma; Plasmapheresis

What are other treatment options to Steroids; Splenectomy; Heparinconsider in TTP?

What treatment is generally avoided in Platelet transfusionTTP?

What is disseminated intravascular A life-threatening disorder that is a coagulation (DIC)? characterized by: Depletion of

platelets; Depletion of coagulationfactors; Small vessel occlusions;Fibrinolysis; Hemolytic anemia

What are some important causes of DIC? Sepsis; Trauma; Drug reactions; Snakebites; Cancer

What are some common clinical features Bleeding, petechiae, thrombosis, andof DIC? possible gangrenous changes

What are some classic laboratory findings PT (INR) and PTT increased; in a patient with DIC? Decreased platelet count; Decreased

fibrinogen; Increased thrombin time;Increased D-dimer

What are some key points in the Important to find the underlying management of DIC? cause; Intravenous (IV) fluids; Packed

red blood cells (PRBC) as needed; Ifserious hemorrhage—consider: FFP,platelets, and cryoprecipitate

168 Deja Review: Emergency Medicine

Page 185: Deja review _emergency_medicine__deja_review_

Sickle Cell Disease

What are some important features of Characterized by abnormalsickle cell disease? hemoglobin—HbS; High prevalence

in African Americans

What are some characteristic features of RBC sickling responsible for majorityHbS? of symptom; Sickled RBCs are more

easily hemolyzed; RBCs sensitive tohypoxia (i.e., sickling)

What is sickle cell trait? Occurs when a child inherits HbS from one parent and HbA fromanother parent, so most RBCs will contain both types

What are some clinical features of sickle Generally asymptomatic; Spontaneouscell trait? bleeding; Decreased ability to concen-

trate their urine; Laboratory evaluationis normal

What are some complications of sickle cell Splenic infarction; Vaso-occlusivetrait? crisis; Death

Are these complications common? Rarely occur unless extreme hypoxia

What is sickle cell anemia? HbS that is inherited from both parentswhere most RBCs have only HbS

What are some clinical features of Anemia; Jaundice; Hand-footsickle cell anemia? syndrome (swelling of foot/hand);

Frequent infections; Vision problems

What infections are patients with sickle Pneumonia; Meningitis; Sepsis;cell anemia more susceptible to? Osteomyelitis

What particular pathogens are those with Salmonella; Haemophilus influenzae;sickle cell anemia more prone to? Streptococeus pneumoniae

What are some complications that occur Aplastic crisis; Vaso-occlusive crisiswith sickle cell anemia? (i.e., pain crisis); Acute chest syn-

drome; Cerebral vascular accident(CVA); Renal papillary necrosis;Priapism

What are some key points in the Analgesics; IV hydration; Oxygen;management of patients who present with Antibiotics if suspected infectionsickle cell crisis?

Hematology and Oncology Emergencies 169

Page 186: Deja review _emergency_medicine__deja_review_

ONCOLOGY

Hypercalcemia (Secondary to Malignancy)

What is important to know about Common life-threatening disorderhypercalcemia secondary to malignancy? associated with cancer

What are some important causes of Parathyroid hormone (PTH) (i.e., hypercalcemia associated with malignancy? squamous cell lung carcinoma);

Osteoclast-activating factor; Bonedegradation (metastasis to bones)

What are some clinical features of Fatigue; Nausea and vomiting;hypercalcemia associated with malignancy? Constipation; Back pain; Hypertension

What are some diagnostic studies to Calcium and phosphorus; Alkalineconsider? phosphatase; Chem-7 (chloride and

potassium); Albumin; ECG (shortenQT interval); PTH

What is the mainstay treatment to quickly IV normal saline and furosemide;reduce ionized calcium? Magnesium and potassium;

Bisphosphonates (i.e., pamidronate);Steroids

Tumor Lysis Syndrome

What is tumor lysis syndrome (TLS)? Constellation of metabolic disturbancesthat may be seen after initiation ofcancer treatment

In what types of cancer does TLS occur? Occurs in patients with rapidlyproliferating, and treatment-responsive tumors

When is TLS commonly seen? Most often is seen 48–72 hours afterinitiation of cancer treatment

What is the pathophysiology of TLS? Rapid tumor cell turnover results in release of intracellular contents into the circulation which can inundaterenal elimination

What are some common laboratory Hyperkalemia (first derangement); findings in patients who have TLS? Hyperuricemia; Hyperphosphatemia

(hypocalcemia)

What are some complications of TLS? Dysrhythmias (hyperkalemia); Uratenephropathy; Acute renal failure;Neuromuscular instability; Metabolicacidosis

170 Deja Review: Emergency Medicine

Page 187: Deja review _emergency_medicine__deja_review_

What is the most common cause of acute Hyperuricemiarenal failure in the setting of TLS?

What is the mainstay treatment for Allopurinol; IV fluids; Alkalinizationhyperuricemia? of urine

What are some key points in the IV fluids; Hemodialysis in life-management of TLS? threatening situations; Serial chem-7

with calcium and phosphorus; Treathyperuricemia

Syndrome of Inappropriate ADH Syndrome

What malignancies are commonly Small cell lung cancer—most common;associated with SiADH Brain; Prostate; Pancreas

What are some diagnostic criteria of Hyponatremia (serum sodium SIADH? <135 mEq/L); Inappropriately con-

centrated urine; Clinical euvolemia

What are some common clinical features Mental status change, weakness, of SIADH? dizziness, and seizures/coma in severe

cases: severity is determined by rateof sodium loss

What are some diagnostic tests to obtain Urinalysis, urine sodium/osmolality;in SIADH? Chem-7 and serum osmolality

What are some key points in the Water restriction; Furosemide;management of SIADH? 3% saline given slowly for severe cases

Spinal Cord Compression

What is spinal cord compression? Spread of cancer to the spine and tissues around the spinal cord that may result in compression of the cord:oncologic emergency

What is the initial presenting feature Back pain (commonly thoracic)of spinal cord compression?

What are some other clinical features of Sensory deficits; Lower extremityspinal cord compression? weakness/paralysis; Urinary inconti-

nence; Urinary retention

What are some malignancies commonly Lung cancer (most common); Breastassociated with spinal cord compression? cancer; Prostate cancer; Multiple

myeloma

What is the diagnostic test of choice for MRIspinal cord compression?

Hematology and Oncology Emergencies 171

Page 188: Deja review _emergency_medicine__deja_review_

What are some other diagnostic tests to Plain films; CT; Myelographyconsider? (considerable complications)

What are some key points in the Steroids to reduce edema andmanagement of spinal cord compression? inflammation; Radiation and neuro-

surgical intervention

Superior Vena Cava Syndrome

What are some important things to Gradual compression of the SVC;know about superior vena cava SVCS is associated chiefly withsyndrome (SVCS)? malignancy; Bronchogenic CA

accounts for more than 80% of casesof SVCS

What are some other causes of SVCS Thrombosis (central venousaside from malignancy? instrumentation); Infectious causes

(i.e., tuberculosis/syphilis);Lymphoma

What are some clinical features of SVCS? Venous distension of face/upperextremity, facial flushing, headache,JVD, cough, and dyspnea

What are some important diagnostic tests CXR: Mass or widened mediastinum;to consider in SVCS? Thoracic CT: test of choice; Histological

sample: important for therapy

What are some key points in the ABCs: rarely present acutely; Tissuemanagement of SVCS? diagnosis for palliative therapy;

Elevation of head provides some relief

Adrenal Insufficiency

What are two important hormones 1. Aldosteroneproduced by the adrenal cortex? 2. Cortisol

What are some clinical features of adrenal Patient is often hypotensive with insufficiency? dehydration and may present with

vasomotor collapse as well asweakness

What are common laboratory findings in Hyperkalemia; Hyponatremia;adrenal insufficiency? Hypoglycemia; Hypercalcemia

What are some common causes of adrenal Malignant melanoma; Breast cancer;insufficiency in the setting of malignancy? Lung cancer; Chronic steroid with-

drawal

172 Deja Review: Emergency Medicine

Page 189: Deja review _emergency_medicine__deja_review_

What are some key points in the Ensure intact ABCs; Aggressivemanagement of adrenal insufficiency? volume replacement; Treat the

underlying cause of crisis; Administerhydrocortisone; Administer fludro-cortisone acetate

Malignant Pericardial Effusion

What is the pathophysiology of malignant Normally is lubricated by a very smallpericardial effusion? amount of serous fluid, malignant

involvement of the pericardium maybe primary (less common) or second-ary to spread from a nearby or distantfocus of malignancy

What are some clinical features of patients Often asymptomatic, but mostwith malignant pericardial effusion? common symptom is dyspnea, and

can include cough, chest pain, andhypotension. PE: JVD, pulsusparadoxus, distant heart sound, andpericardial friction rub

Name some malignancies commonly Leukemia; Breast cancer; Lung cancer; associated with malignant pericardial Melanomaeffusion.

What are commonly used diagnostic ECG. Low-voltage QRS complexes;tests and possible findings. Electrical alternans; ST-segment ele-

vation and T-wave inversion

CXR. Massive effusions = large cardiacshadow; Pleural effusion/mediastinalwidening/mass

CT. As little as 10 ml of pericardialfluid can be seen as a irregular contourof the cardiac silhouette

Echocardiography. Test of choice(highly specific and sensitive); Usedfor guiding needle pericardio-centesis

What are some key points in the Supportive care (IV fluids/inotrope management of malignant pericardial if needed); Pericardiocentesis iseffusion? definitive treatment; Tamponade can

be the presenting symptom

Hematology and Oncology Emergencies 173

Page 190: Deja review _emergency_medicine__deja_review_

CLINICAL VIGNETTES

8-year-old male presents with a long Hemophilia Ahistory of intermittent epistaxis alongwith prolonged bleeding whenever hegoes for any dental procedures, patient’smother is now concerned since patientis having recent hematuria; Labs:prolonged PTT, but normal PT timeas well as platelet count

21-year-old female with no known von Willebrand’s diseasePMH presents to the ER with concernof nose bleeding that has become frequent;Labs: abnormal PTT and BT, but normalplatelet count and PT

6-year-old male is brought in by his Idiopathic thrombocytopenicmother due to concerns of episodes of purpurasudden nose bleeding about a week afteran upper respiratory illness (URI),patient is otherwise healthy; Labs:CBC otherwise unremarkable exceptfor platelet count of 5,000

17-year-old female with a recent snake DICbite presents with hypotension, confusion,fever, and gingival bleeding; Labs:increased INR and PTT time, decreasedplatelets with decreased fibrinogen andincreased thrombin time

23-year-old AA male with sickle cell Vaso-occlusive crisisdisease presents with a recent coldand excruciating pain in his limbs;PE: unremarkable

71-year-old female with a history of Hypercalcemiauntreated squamous cell lung cancerpresents with fatigue, constipation, andback pain; ECG shows shortenedQT interval

81-year-old male with recently treated Tumor lysis syndromecancer presents with weakness, flank pain, dysuria, and abdominal pain; Labs:elevated potassium, LDH, BUN/creatinine, and uric acid

174 Deja Review: Emergency Medicine

Page 191: Deja review _emergency_medicine__deja_review_

65-year-old female with history of HTN, Spinal cord compressionbreast cancer, and CAD is concerned withrecent onset of urinary retention and lowerextremity weakness; PE: bilateral legweakness

76-year-old male with history of Superior vena cava obstructionbronchogenic cancer now presents withdyspnea and cough for about 1 monthwhich is becoming more progressive;PE: obvious venous distension of faceand JVD

56-year-old female with breast cancer in Malignant pericardial effusionthe past presents with chest pain anddyspnea for the past 2 weeks which isgetting progressively worse; PE: distantheart sound, pulsus paradoxus, and JVD;ECG: low-voltage QRS complex

Hematology and Oncology Emergencies 175

Page 192: Deja review _emergency_medicine__deja_review_

This page intentionally left blank

Page 193: Deja review _emergency_medicine__deja_review_

177

C H A P T E R 1 1

Infectious Diseases

INFLUENZA AND HERPES VIRUSES

Influenza Virus

What are influenza viruses? Single-stranded RNA viruses thatfall within the orthomyxovirus familywith three types—A, B, and C

Name two surface glycoproteins that are 1. Hemagglutinin (H)responsible for the pathogenicity 2. Neuraminidase (N)of the influenza virus.

What is antigenic drift? Minor mutations in the RNA genomethat code for N or H molecule causinga change in antigenicity

What is antigenic shift? Occurs when a host is infected withtwo different influenza viruses,producing a new virus with littleantigenic similarity to the old

When does the flu generally occur The fall and springin the United States?

What are some clinical features of the flu? Headache, fever, chills, myalgia andmalaise often with rhinorhea, sorethroat, enlarged cervical lymph nodes,and a dry cough

What is the typical time course for the flu? Fever that lasts for 2–4 days withrapid recovery, although cough andmalaise may last longer

What are some complications of an Secondary bacterial pneumonia;influenza infection? Pneumonitis; Croup; Chronic

obstructive pulmonary disease(COPD) exacerbation; Reye’ssyndrome (if ASA used)

Copyright © 2008 by The McGraw-Hill Companies, Inc. Click here for terms of use.

Page 194: Deja review _emergency_medicine__deja_review_

Name two medications currently 1. Rimantadineapproved for the treatment of influenza A. 2. Amantadine

What are some points with regard Should be started within 48 hoursto rimantadine and amantadine? of symptoms; Amantadine is renally

cleared; Rimantadine is hepaticallycleared

Name two medications approved for Zanamivir and Oseltamivirtreatment of influenza A and B?

What is the flu vaccine? It is made annually and containstwo strains of influenza A and onestrain of influenza B

Which groups should receive the influenza Anyone with cardiopulmonaryvaccine? disease; Immunocompromised

patients; Healthcare workers; Patientsover 65

Herpes Virus

What are some important facts about the They are an ubiquitous group ofherpes virus family? DNA viruses; Ability to remain in a

host as a lifelong latent infection thatcan reactivate; Commonly transmittedby close contact

What is the pathophysiology of herpes Infects and replicates in epithelialsimplex virus (HSV) exposure? cells, causing lysis of the cell leading

to an inflammatory response and thecharacteristic HSV rash

What is the general appearance of Clusters of small, thin-walleda HSV rash? vesicles on a erythematous base

What are some clinical features of Primarily caused by HSV-1, but canoral HSV? be caused by HSV-2 that range from

asymptomatic to pharyngitis orgingivostomatis with fever andcervical adenopathy

How is oral HSV diagnosed? Typically made clinically, althoughviral cultures can be used (takes days)

What is the oral distribution of the lesions? All over the mouth

What is the recurrence rate of oral lesions? Vary from 60–90%, but recurrencestend to be milder

What are some triggers for an Stress; Trauma; SunburnHSV recurrence?

178 Deja Review: Emergency Medicine

Page 195: Deja review _emergency_medicine__deja_review_

What role does acyclovir play in oral HSV? Shown to shorten course if givenwithin 72 hours of symptoms andcan be used as prophylaxis insevere cases

What are some important points Majority caused by HSV-2; Recurrentabout genital herpes? lesions can cause intrauterine infec-

tions; C-section if active lesions arepresent during a pelvic exam

What is the most common manifestation of Ulcerative keratitisocular HSV?

What is the most feared complication Recurrent infections leading toof ocular HSV? blindness

What are some clinical features of Herpetic vesicles on the conjunctivaocular HSV? or the lid margin and fluorescein

staining that shows dendriticulcerations

What are some key points in the Consultation with ophthalmology;management of ocular HSV? Administration of IV acyclovir; Avoid

the use topical steroids

What is one of the most common viral HSV encephalitis (usually HSV-1)encephalitis?

What portion of the brain is typically Temporal lobesinvolved?

What are some clinical features of HSV Often a viral prodrome which mayencephalitis? be followed by HA, fever, altered

mental status, and even focal seizures

What does a lumbar puncture often show? Nonspecific—elevated WBC countwith an increase in mononuclearcells

What is the test of choice for diagnosing PCRHSV encephalitis?

What is the treatment of choice for Intravenous acyclovirsuspected HSV encephalitis?

What is an HSV infection of the finger Herpetic whitlowknown as?

What is the concern of a patient who is Dissemination or severe HSVimmunocompromised with HSV? infection

What are some complications of HSV in Proctitis, esophagitis, colitis, andan immunocompromised patient? pneumonitis

What is the cause of chickenpox? Varicella-zoster virus (VZV)

Infectious Diseases 179

Page 196: Deja review _emergency_medicine__deja_review_

What is some important epidemiologic Chickenpox is the primary infection;information about VZV? Zoster (shingles) is reactivation of

VZV; Prior to vaccine, over 90% ofprimary infections occurred to those<10 years

What is the dermatologic hallmark of Skin lesions in various stageschickenpox? throughout the body

What are some clinical features of Prodrome of fever, HA, and malaisechickenpox? followed by clear vesicles on an

erythematous base which eventuallyscab over

What are some serious complications of Cerebellar ataxia; Pneumonitis;chickenpox? Encephalitis

Who is oral acyclovir recommended for? Patients older then 14 years of age;Patients on chronic ASA therapy

Who should receive IV acyclovir? Patients suffering from varicellaencephalitis/pneumonitis

What is herpes zoster (shingles)? Reactivation of latent VZV infectionwith a lifetime incidence of 25%,especially in the elderly

What are some clinical features Vesicular lesions similar toof shingles? chickenpox in a single dermatome

that may persist for up to a month

What is herpes zoster ophthalmicus Involvement of the ophthalmic(HZO)? branch of cranial nerve (CN) V

which can threaten vision and alsocause a lesion on the tip of the nose(Hutchinson’s sign)

What is the most common complication of Postherpetic neuralgiashingles?

What are some clinical features of Severe pain and occasionalpostherpetic neuralgia? involvement of the anterior horn

cells leading to transient weakness

What is the initial treatment for Systemic analgesia such as narcotics;postherpetic neuralgia? Carbamazepine may work as a

second-line treatement

What is the role for the use of antivirals If used within 72 hours, may decreasein shingles? the duration of the disease

What is the primary cause of infectious Epstein-Barr virus (EBV)mononucleosis?

How is EBV typically spread? Close contact such as kissing, EBVcannot survive outside the host forlong

180 Deja Review: Emergency Medicine

Page 197: Deja review _emergency_medicine__deja_review_

What is the pathophysiology of infectious 1–2 month incubation period wheremononucleosis? the EBV replicates in B lymphocytes

resulting in the production of anti-EBVantibodies and heterophil antibodies

What are some clinical features of Fever, HA, exudative pharyngitis,infectious mononucleosis? splenomegaly, atypical lympho-

cytosis, and bilateral cervicallymphadenopathy

What are some complications of infectious Splenic rupture; Thrombocytopenia;mononucleosis? Autoimmune hemolytic anemia;

Meningitis; Encephalitis

How is the diagnosis of EBV typically Clinical features of EBV along withmade? atypical lymphocytes and (+) monospot

test are generally confirmatory

What are some epidemiologic features Ubiquitous virus found worldwide;of CMV? Causes primary infection and often

exists as a latent infection; Not easilyspread by casual contact

What are some clinical features of Often asymptomatic in healthyCMV infection? people, but can appear as flu-like

symptoms such as fever, chills, andmyalgia

When should CMV infection be suspected Mononucleosis-like illness, butin healthy adults? heterophil antibody negative

What are some complications of Guillain-Barré syndrome; Hepatitis;CMV infection in healthy individuals? Hemolytic anemia; Pneumonitis;

Thrombocytopenia

In what population group can CMV HIV; Transplant recipientsbe particularly devastating?

What is the most common CMV CMV retinitisinfection in patients with advanced HIV?

What are some CMV infections to Hepatitis; Colitis; CNS diseaseconsider in transplant recipients?

What is the most serious CMV infection CMV pneumoniain transplant patients?

What are some ways in which transplant Blood products or transplant organ;patients can contract CMV infection? Reactivation of latent infection

When does CMV infections most Within 3 months of transplantationcommonly occur in transplant recipients?

What are two medications are used in 1. GanciclovirCMV infections? 2. Foscarnet

Infectious Diseases 181

Page 198: Deja review _emergency_medicine__deja_review_

HIV/AIDS

What are some important points about Cytopathic retrovirus of the lentivirusthe HIV virus? family; Very labile outside the body;

Two major subtypes: HIV-1 and HIV-2;Selectively attacks CD4+ T-cells

What are some risk factors for the Intravenous drug use; Verticaldevelopment of HIV infection? transmission; Unprotected sex

What is the most common presentation Fever, pharyngitis, fatigue, rash, andof acute HIV infection? headache

Why is the diagnosis of HIV infection The nonspecific presentation, whichinitially difficult? often resembles flu-like symptoms

What is seroconversion? Detectable antibodies in response toHIV that usually occurs between4–8 weeks, but can be delayed forup to a year

What is the average time frame from 8–10 yearsinitial HIV infection to the developmentof AIDS?

What are some conditions that Kaposi’s sarcoma; Pneumocystismay indicate AIDS? carinii pneumonia; Brain

toxoplasmosis; Cyptococcosis;Mycobacterium avium complex; CD4+

T-cell count <200 cells/µL

What is the standard and most common Detection of the antibodies to theway to diagnosis HIV infection? virus by Western blot assay or ELISA

What are two useful things to know when 1. CD4+ T-cell counta patient with HIV presents to the ED? 2. HIV viral load

What are some numbers to keep in mind CD4+ T-cell count of <200 and HIVabout CD4+ T-cell count and HIV viral load >50,000 is often associatedviral load? with progression to AIDS-defining

illness and an indication to startantiretrovirals

What are some differentials to keepin mind about HIV-infected patientswho present with fever based on CD4+

T-cell count:

CD4+ T-cell count >500 Cause of fever similar to healthypatients who are non-immunocompromised

CD4+ T-cell count 200–500 Early bacterial respiratory infection

182 Deja Review: Emergency Medicine

Page 199: Deja review _emergency_medicine__deja_review_

CD4+ T-cell count <200 P. carinii pneumonia; M. aviumcomplex; CMV; M. tuberculosis

What is the most common cause of serious CMVopportunistic viral disease in HIV-infectedpatients?

What is the most common cause of fever Drug fever; Neoplasmthat is noninfectious in origin?

What is an important diagnosis to keep Infective endocarditisin mind about HIV-infected patientswith a history of intravenousdrug abuse (IVDA)?

Name the three most common causes of 1. Toxoplasma gondiiCNS disease in HIV-infected patients? 2. AIDS dementia

3. Cryptococcus neoformans

What are some clinical features that are Altered mental status, seizures,indicative of CNS disease? headache, and focal neurologic

deficits

What should an ED evaluation include for CT of the head and LP, especially ifHIV-infected patients who present the CD4+ T-cell count <200 cells/µLwith neurologic symptoms?

What is important to know about Most common cause of focaltoxoplasmosis in patients with AIDS? encephalitis

What is the treatment of choice for Sulfadiazine; Pyrimethamine;patients with suspected toxoplasmosis? Decadron for brain swelling/edema

What should be given for HIV-infected Trimethoprim-Sulfamethoxazolepatients who have positive toxoplasmosis (TMP-SMX)antibodies and CD4+ T-cell count<100 cells/µL?

What are some presenting symptoms of Diffuse meningoencephalitis;cryptococcal CNS infection? Focal cerebral lesions

How is the diagnosis of cryptococcal CNS CSF cryptococcal antigen, culture, andinfection commonly made? staining with India ink; LP will often

have a very high opening pressure

What is the preferred treatment for Amphotericin B; Fluconazolepatients with cryptococcal CNS infection?

What are some other important CNS Bacterial meningitis; CMV; HSV;infections in consider? Neurosyphilis; TB

What is the most frequent and serious CMV retinitisocular opportunistic infection ofHIV-infected patients?

What is the treatment of choice for Ganciclovirpatients with CMV retinitis?

Infectious Diseases 183

Page 200: Deja review _emergency_medicine__deja_review_

What are some important pulmonary Bacterial pneumonia; CMV infection;infections to keep in mind with TB; Pneumocystis carinii pneumoniaHIV-infected patients? (PCP); C. neoformans; Neoplasms

Which disease is the most serious P. carinii pneumoniacomplication and common cause of deathin HIV-infected patients?

What are some clinical features of PCP? Nonproductive cough, fever, andshortness of breath with diffuseinterstitial infiltrates on CXR

What is the medication of choice TMP-SMX; Pentamidine isothionatefor PCP?

What are some clinical features of Fever, hemoptysis, weight loss,TB in HIV- infected patients? night sweats, and anorexia

What is the CD4+ T-cell count where CD4+ T-cell count 200–500 cells/µLTB is more common?

Does a negative PPD test in an No—can be negative due toHIV-infected patient rule out TB? immunosuppression

What is a common treatment option INH and pyridoximinefor HIV-infected patients with TB?

What are some common oral/esophageal Oral candidiasis (most common);complaints in HIV-infected patients? HSV; Oral hairy leukoplakia

What is the most frequent GI complaint Diarrheain HIV-infected patients?

What are some common causes of diarrhea Shigella; Isospora belli; E. coli;in HIV-infected patients? Cryptosporidium

What are some common generalized Seborrheic eczema; Pruritus; Xerosiscutaneous conditions in HIV-infectedpatients?

What is the appearance of Kaposi’s Painless dark papules/nodules thatsarcoma? do not blanch

What are some other important causes HSV; Zoster; Scabies; Syphilisof skin lesions to consider?

What are some important treatment goals Prolongation and improvement of life;for HIV-infected patients? Reduction of viral load; Improved

CD4+ T-cell count; Maintain drugregiment with minimal ADR

What are three main classes of drugs used 1. Protease inhibitorsin the treatment of HIV? 2. Nucleoside reverse-transcriptase

inhibitors3. Nonnucleoside reverse-

transcriptase inhibitors

184 Deja Review: Emergency Medicine

Page 201: Deja review _emergency_medicine__deja_review_

SEXUALLY TRANSMITTED DISEASES

What are some important elements to Pregnancy status; Sexual practice;establish when evaluating a patient for Evaluate for sexual abuse; Evaluatesexually transmitted diseases (STDs)? for domestic violence

What infection commonly coexists with Chlamydia trachomatisgonorrhea?

What are some facts about chlamydial Common cause of nongonococcalinfections? infection; Often asymptomatic in

patients

What are some clinical features of Urethritis, dysuria, vaginal discharge,chlamydial infections? and proctitis

Name two important complications of 1. Pelvic imflammatory disease (PID)chlamydial infections in females if 2. Infertilityleft untreated.

Name some antibiotics commonly Azithromycin; Doxycyclineused to treat nongonococcalurethritis/cervicitis?

What are some clinical features of Males tend to have dysuria and puru-gonococcal urethritis/cervicitis? lent penile discharge while females

tend to have more nonspecific symp-toms such as lower abdominal pain

What are some factors that contribute to Poor detection method; Subclinicalcomplications of gonococcal infection? presentation (esp. females)

How common is disseminated gonococcal About 5%infection if left untreated?

What are some clinical features of Fever, malaise, skin lesions on andisseminated gonococcal infection? erythematous base, and asymmetric

arthralgias

What is the standard for the diagnosis of Cervical or urethral culturegonococcal infection?

Name the antibiotics commonly used Ceftriaxone or Ciprofloxacinto treat gonococcal urethritis/cervicitis?

What is important to keep in mind about Increasing resistance in certainusing fluoroquinolones for gonococcal areas like California and Asiainfections?

Name five diseases that are characterized 1. Syphilisby genital lesions? 2. HSV

3. Lymphogranuloma venereum4. Granuloma inguinale5. Chancroid

Infectious Diseases 185

Page 202: Deja review _emergency_medicine__deja_review_

What is the causative organism of syphilis? Treponema pallidum; The spirocheteenters the body through the mucousmembrane or non-intact skin

What are the three phases of syphilis andsome important points regardingeach phase:

Primary Hallmark: painless chancre; Incubationperiod is about 3 weeks; Lesionstypically disappear after a month

Secondary Nonspecific symptoms: fever andmalaise; Rash that starts at the trunkand moves towards the palms/soles;Often resolves spontaneously

Tertiary (latent) CVS/CNS involvement is charac-teristic; Granulomatous lesions arecommon; Meningitis, dementia, tabesdorsalis, and thoracic aneurysm morelikely

What are some methods for diagnosing RPR, VDRL, and dark-fieldsyphilis? microscopy

What is the treatment of choice for Penicillin; Doxycycline for penicillinsyphilis? allergy

What is the causative agent of chancroid? Haemophilus ducreyi

What are some clinical features of Painful genital ulcer andchancroid? lymphadenitis/abscess/periadenitis

if left untreated

What are some other infections to consider Syphilis, HSV, and HIVin patients with chancroid?

How is the diagnosis of chancroid typically Typically made clinicallymade?

What are some antibiotics commonly used Azithromycin; Ceftriaxone; to treat chancroid? Ciprofloxacin

What is the causative agent of C. trachomatislymphogranuloma venereum (LGV)?

What are some clinical features of LGV? Painless primary chancre of shortduration, lymphadenopathy, andsystemic effects such as fever,arthralgias, and erythema nodosa

How is the diagnosis of LGV commonly Culture and serologic testsmade?

What is the treatment of choice for LGV? Doxycycline

186 Deja Review: Emergency Medicine

Page 203: Deja review _emergency_medicine__deja_review_

What is the causative agent of granuloma Calymmatobacterium granulomatisinguinale?

What are some clinical features Subcutaneous nodules on penis orof granuloma inguinale? labia/vulva area after incubation

which progresses to a painless ulcer-ative lesion with a “beefy” appearance

How is the diagnosis of granuloma Difficult to culture; visualization ofinguinale commonly made? Donovan bodies on tissue biopsy is

characteristic

What is the treatment of choice for Doxycycline and bactrim; granuloma inguinale? Ciprofloxacin and azithromycin

What is the causative agent of Human papillomavirus (HPV)genital warts?

What is the typical appearance of HPV? Flesh-colored papules or cauliflower-like projection that is often painless

How is HPV commonly diagnosed? Often clinically, but can be done withPCR

What is an important long-term Cervical cancercomplication of HPV to consider?

What HPV types are commonly associated HPV type 16 and 18 with cervical cancer?

What are risk factors associated with Increasing number of partners; Earlyacquisition of HPV? age of first sexual intercourse

What is the primary reason for treatment Removal of visible warts forof HPV? cosmetic reasons

What are some treatment options Cyrotherapy; Surgical removal;for visible lesions of HPV? Podophyllin resin

Is there a vaccine for HPV? Yes—a quadrivalent vaccine thatprotects against HPV types 6, 11, 16,and 18. These four are responsiblefor 70% of cervical warts and 90% ofgenital warts

MALARIA

What is the primary vector for the Anopheles mosquitotransmission of malaria?

Name four species that are responsible for 1. Plasmodium vivaxmalaria? 2. Plasmodium ovale

3. Plasmodium malaria4. Plasmodium falciparum

Infectious Diseases 187

Page 204: Deja review _emergency_medicine__deja_review_

Which species of Plasmodium is the most Plasmodium falciparumdeadly form of malaria?

Name some locations in the world where Caribbean; Middle East; Centralmalaria transmission primarily occurs? America; Indian subcontinent

Name two species of Plasmodium that can 1. Plasmodium vivaxlie dormant for months and cause clinical 2. Plasmodium ovalerelapse?

What form of the parasite is injected into Sporozoitesthe bloodstream when a mosquito takesits bloodmeal?

What form of the parasite invades the Merozoiteserythrocytes?

What are some clinical features of malaria? Nonspecific: fever, HA, myalgia,and malaise; PE: splenomegaly andtender abdomen in acute infections

What are some common laboratory Elevated ESR and LDH, mildlyfindings in malaria? abnormal kidney and liver function

What is the clinical hallmark of malaria? Recurrent febrile paroxysm thatcorresponds to the hemolysis ofinfected erythrocytes

What are some complications of malaria Immune-mediated glomeru-if left untreated? lonephritis; Splenic enlargement or

rupture; Hemolysis; Noncardiacpulmonary edema

What is cerebral malaria? Most common with infection fromPlasmodium falciparum: coma, delirium,seizures, and somnolence with up toa 25% mortality

How is the diagnosis of malaria made? Giemsa-stained thick and thin bloodsmear—with the first smear beingdiagnostic in 90% of cases

What are two important questions to 1. Is Plasmodium falciparumaddress when viewing a blood smear? responsible?

2. Degree of parasitemia? (>3% is bad)

What is the drug of choice for the Chloroquinetreatment of malaria due to any speciesaside from Plasmodium falciparum?

What drug is recommended for the Primaquinedormant form of Plasmodium vivax andPlasmodium ovale?

What can the use of primaquine in a Hemolytic anemiapatient with G6PD deficiency cause?

188 Deja Review: Emergency Medicine

Page 205: Deja review _emergency_medicine__deja_review_

What is the treatment for malaria caused Doxycycline and quinine with orby Plasmodium falciparum? without pyrimethamine-sulfadoxine

What treatment option is available Exchange transfusionfor patients with extensive parasitemia?

SOFT TISSUE INFECTIONS

Cellulitis

What is the definition of cellulitis? Bacterial invasion of the skin thatleads to a local soft tissue inflam-matory reaction

What groups does cellulitis more Elderly; Immunocompromisedcommonly occur in? patients; Diabetics; Peripheral

vascular disease

Name the two most common groups of Streptococcus; Staphylococcusbacteria that are involved with cellulitis.

What organism is becoming more common Community-acquired methicillin-as a cause of cellulitis, especially among resistant Staphylococcus aureusteam athletes, prison inmates, and (CA-MRSA)military personnel?

Name a common cause of cellulitis Haemophilus influenzaein children?

What are some clinical features of Induration, pain, erythema, andcellulitis? warmth; PE: fever, leukocytosis, and

lymphadenopathy as systemicinvolvement

When is the use of soft-tissue radiography If a foreign body is involved as aor ultrasound recommended? cause

What is elephantiasis nostra? Recurrent attacks that can lead todermal fibrosis, epidermal thickening,permanent swelling, and impairmentof lymphatic drainage

What are some treatment options for Macrolide; Amoxicillin-clavulanate;cellulitis in an otherwise healthy adult? Dicloxacillin

What is an exception to outpatient Cellulitis of the head or neck wheretreatment of simple cellulitis? they should be admitted for IV

antibiotics or immunocompromisedpatients with evidence of rapidlyspreading cellulitis

What is erysipelas? Superficial cellulitis with involvementof the lymphatic system

Infectious Diseases 189

Page 206: Deja review _emergency_medicine__deja_review_

What organism is the most common Group A Streptococcuscause of erysipelas?

What are some common ways that Ulcers; Infected dermatoses; Toe-weberysipelas occurs? intertrigo

What are some clinical features Abrupt onset of symptoms thatof erysipelas? include high fever, chills, and nausea

and with progression of the infectionthat leads to a shiny, red, and hotplaque; PE: bullae, purpura, andsmall areas of necrosis can be seen

What is a possible complication of Necrotizing fasciitiserysipelas that should be considered?

What are some treatment options of Penicillin G; Amoxicillin-clavulanate;erysipelas? Imipenem in severe cases; Marcolide

for penicillin allergy

Cutaneous Abscesses

What are some factors that contribute to Lower pH of 3–5; Constantthe skin’s protective function? desquamation of epidermis; Skin

continually shedding bacteria

Name some ways in which abscesses Abrasions or lacerations; Puncture;can develop. Bites

How do abscesses typically start? Local cellulitis

Name some organisms commonly Staphylococcal species; Streptococci;involved with cutaneous abscesses? Bacteroides

What is a common site of abscesses when Hair folliclesStaphylococcus species are involved?

What is folliculitis? Bacterial invasion of a hair follicle thatcauses inflammation

What is a deeper invasion of the soft tissue Furuncle (i.e., boil)surrounding a hair follicle known as?

What is a carbuncle? Several furuncles that coalesce to forma large area of infection that containsinterconnecting sinus tracts

What is sufficient to treat most cases of Warm compressesfolliculitis and boils?

What are some clinical features of Tenderness, erythema, and swellingcutaneous abscesses? with an area that may show indura-

tion and fluctuance

What are some clinical features that may Fever, lymphadenitis, and localized

190 Deja Review: Emergency Medicine

Page 207: Deja review _emergency_medicine__deja_review_

indicate systemic involvement? lymphadenopathy

What is typically done with cutaneous Incision and drainage (I&D)abscesses if it is fluctuant?

Are antibiotics commonly recommended I&D is sufficient in most casesalong with I&D?

If the abscess is not fluctuant and an I&D Treat with antibiotics as cellulitiscannot be done, what is recommended?

Of all the perirectal abscesses, which is Perianal abscessesthe only one that can be drained safelyin the ED?

When are antibiotics recommended in Overlying cellulitis;the case of cutaneous abscesses? Immunocompromised patients

What is a particular concern in patients Bacterial endocarditiswith underlying structural heart disease?

What are some high-risk cardiac conditions Prosthetic valves; Hypertrophicwhere prophylactic antibiotic coverage cardiomyopathy; History of bacterialmay be considered? endocarditis; Acquired valvular

dysfunction

GAS GANGRENE

What agent is commonly implicated as a Clostridium speciescause of gas gangrene?

Name two species of Clostridium that are 1. Clostridium perfringens (80–90%identified as causing gas gangrene? of cases)

2. Clostridium septicum

What is important to note about clostridial It is a rapidly progressive and seriousmyonecrosis? disease that threatens both life and

limb and it is the deepest of thenecrotizing soft tissue infections

What is the primary pathophysiologic Production of various exotoxin,mechanism by which the Clostridium α-toxin in particular, that causes aspecies cause myonecrosis? variety of problems such as tissue

necrosis, cardiodepressant, andhemolysis

In what environment does Clostridium Anaerobic environments that canspecies thrive? occur after injury

Aside from direct inoculation from an Hematogenous spreadopen wound, name another route of entry?

Infectious Diseases 191

Page 208: Deja review _emergency_medicine__deja_review_

In what group is hematogenous spread Immunocompromised patientsmore common?

What is the incubation period of gas Around 3 daysgangrene once inoculation occurs?

What is the most common presentation of Pain out of proportion to physicalgas gangrene in the early stages? findings

What is the hallmark of gas gangrene? Sepsis with gas production

What are some other clinical features Low-grade fever, tachycardia, irritable,of gas gangrene? confused; PE: area may have edema

with crepitance, brownish discolorationwith a malodorous discharge

What can radiographic studies show in Gas within the involved areathe case of gas gangrene?

What are the four hallmarks for thetreatment of gas gangrene:

Resuscitation Aggressive fluid resuscitation; Avoidvasoconstrictors if possible

Surgical debridement Mainstay for the treatment of gasgangrene; Early removal of theinfected area is crucial; Debridementmay range from fasciotomy toamputation

Antibiotic therapy Includes penicillin; Ceftriaxone andmacrolides as alternatives; Updatetetanus status as indicated

Hyperbaric oxygen (HBO) Initiated soon after debridement;Therapy consists of 100% oxygen at3 atm of pressure for 90 minutes withthree dives in the first 24 hours and2 per day for 4–5 days

What is the most common cause of gas Mixed infections with both aerobicgangrene that is nonclostridial? and anaerobic organisms

Does the presentation of nonclostridial Not reallygas gangrene differ much from onecaused by clostridial species?

What are some species of bacteria involved Enterococcus; Bacteroides; Bacillus;with nonclostridial gas gangrene? Staphylococcus

What are some treatment differences for Broad-spectrum antibiotic crucial;nonclostridial gas gangrene when HBO still utilizedcompared to clostridial gas gangrene?

192 Deja Review: Emergency Medicine

Page 209: Deja review _emergency_medicine__deja_review_

Necrotizing Cellulitis/Fasciitis

What is necrotizing cellulitis? Superficial form of necrotizing softtissue infection limited to the skinand subcutaneous fat

What are some conditions associated with Surgery; Trauma; Malignancy;necrotizing cellulitis? Diabetes

What is the most common bacteria causing Clostridial speciesnecrotizing cellulitis?

What are some clinical features of Erythema and pain is the mostnecrotizing cellulitis? common complaint; PE: may show

blebs or vesicles

What are some key points in the Surgical debridement is crucial, butmanagement of necrotizing cellulitis? extensive soft tissue removal not

needed; Broad-spectrum antibiotics

What is necrotizing fasciitis more “Flesh-eating bacteria”commonly known as?

What is necrotizing fasciitis? Widespread necrosis that commonlyinvolves the fascia and subcutaneoustissue, but not underlying muscle aswith myonecrosis

What are some major predisposing factors Peripheral vascular disease; Diabetes;for necrotizing fasciitis? Intravenous drug use

What are two forms of necrotizing 1. One caused solely by group Afasciitis? streptococcus (GAS)

2. Other caused by mixed organisms,which is the most common formof necrotizing fasciitis

Why does necrotizing fasciitis have the Bacterial tissue toxins causeability to spread so quickly? inflammation and thrombosis that

leads to an environment favorablefor bacterial growth and rapidspread along the fascial plane

What is the most common presenting Pain out of proportion of the examcomplaint for patients with necrotizingfasciitis?

What is indicated in all suspected cases of Early surgical consultationnecrotizing fasciitis?

What is the treatment of necrotizing Similar to that of gas gangrene withfasciitis? focus on resuscitation, antibiotic

use, surgical debridement, andHBO

Infectious Diseases 193

Page 210: Deja review _emergency_medicine__deja_review_

What are some differences between GAS While very similar in presentation andnecrotizing fasciitis when compared to treatment, GAS necrotizing fasciitisnecrotizing fasciitis from mixed organisms? tends to be more rapidly progressive

with greater likelihood for bacteremiaand TSS

TOXIC SHOCK SYNDROME

What is the etiologic cause of toxic shock Staphylococcus aureussyndrome (TSS)?

What are some risk groups of TSS? Menstruating women; Postoperativestaphylococcal wound; Persons whohave undergone nasal surgery

What are some clinical features of TSS? Sudden onset of fever, chills, vomiting,diarrhea, muscle aches and rash;Desquamation, particularly on thepalms and soles can occur up to2 weeks after onset

What is particularly worrisome about TSS? Rapid progression to severe hypoten-sion and multisystem dysfunction

What is the most crucial aspect in the Aggressive management of circulatorymanagement of TSS? shock

What are some key points in the Identify and treat source of infection;management of TSS? Culture all sites; Remove all

foreign bodies; Prompt antibiotictherapy

How is streptococcal toxic shock syndrome More aggressive form of TSS thatdifferent from TSS caused by S. aureus? often develops in association with

severe skin infection

What group of streptococcus is responsible Group Afor this form of TSS?

What are some clinical features of Similar to TSS caused by S. aureus,streptococcus TSS? many will have signs of soft-tissue

infection with pain

What are some key points in the Aggressive exploration/debridementmanagement of TSS from streptococcus? of soft-tissue infection; Early circulatory

support; Prompt antibiotic therapy

194 Deja Review: Emergency Medicine

Page 211: Deja review _emergency_medicine__deja_review_

OCCUPATIONAL POSTEXPOSURE PROPHYLAXIS

What are three infections that are 1. Human immunodeficiency viruscommonly evaluated in an occupational (HIV)postexposure such as needle sticks? 2. Hepatitis B virus (HBV)

3. Hepatitis C virus (HCV)

Give some examples of potential Contact with mucous membranesinfectious sources? with infectious material; Percutaneous

injury

Name some examples of potential Blood; CSF; Semen; Amniotic fluid;infectious bodily fluids. Pleural fluid

What are some things to do when Obtain a thorough history thatevaluating a patient who is exposed includes the circumstance, exposureto potentially infectious material? type, etc.; Wash the wound with water

and soap; Assess immune status ofpatient

Can HBV be transmitted by contact with Yes—HBV can survive in dried bloodenvironmental surfaces?

What is the risk of developing hepatitis Less then 5%if the blood source is HBsAg(+) anHBeAg(−)?

What is the risk of developing hepatitis About 25%if the blood source is HBsAg(+) anHBeAg(+)?

What are some factors to consider in the HBV vaccination status of the patient;treatment of HBV? Immunity of the patient; HBV status

of the source

What is the postexposure prophylaxis Hepatitis B immune globulin (HBIG);for HBV? Vaccination series: Hepatitis B vaccine

at the time of evaluation, at 1 month,and at 6 months

Is pregnancy a contraindication for HBV? No

What is the risk of seroconversion from 1–2%an HCV(+) source?

What is the prophylaxis currently available None availablefor HCV?

What is the probability of transmission ofHIV from a single exposure in thefollowing situations:

Vaginal intercourse 0.1–0.2%

Infectious Diseases 195

Page 212: Deja review _emergency_medicine__deja_review_

Anal intercourse 0.1–4%

Percutaneous exposure 0.3%

What is a basic postexposure prophylaxis Zidovudine; Lamivudine orregiment for HIV exposure? Combivir

INFECTIOUS DISEASE APPENDICES

Pregnancy Categories

A Generally acceptable. Controlled studies show no adverse effect to fetusB Use may be acceptable. Animal studies show no risk, but human studies not

availableC Use with caution only if the benefits outweigh the riskD Use only in life-threatening emergencies, possible risk to fetusX Do not use in pregnancyNA Information not available

196 Deja Review: Emergency Medicine

Antibiotic Use in Pregnancy

Cephalosporins Category B: generally safePenicillin Category B: generally safeMetronidazole Category B: generally safeQuinolones Category C: can cause arthropathy in the fetusNitrofurantoin Category B: avoid in third trimester due to

possible hemolytic anemiaTetracycline Category D: can stain teeth and boneSulfonamides Category B: in first and second trimester

Category D: can cause kernicterusClotrimazole Category B: generally safe

Page 213: Deja review _emergency_medicine__deja_review_

197

C H A P T E R 1 2

PediatricEmergencies

HIGH-YIELD PEDIATRIC CHARTS

Vital Signs

Age Heart Rate Respirations Systolic Blood Pressure

Newborn 90–180 30–60 50–706 months 85–170 24–40 65–1051 year 80–140 20–40 70–1103 years 80–130 20–30 75–1146 years 70–120 18–25 80–1158 years 70–110 18–25 85–12010 years 65–110 16–20 90–13012 years 60–110 14–20 95–13515 years 55–100 14–20 100–14018 years 50–90 14–18 105–150

Temperature Conversion

F 106 105 104 103 102 101C 41.1 40.6 40 39.4 38.9 38.3F 99 98.6 98 97 96 95C 37.2 37 36.7 36.1 35.6 35

Copyright © 2008 by The McGraw-Hill Companies, Inc. Click here for terms of use.

Page 214: Deja review _emergency_medicine__deja_review_

CARDIOPULMONARY RESUSCITATION

Name some important risk factors associated with cardiopulmonary arrestfor each of the following categories:

Fetal Congenital infection; Acidosis;Prematurity or postmaturity; Thickmeconium

Maternal Poor prenatal care; Illicit substanceabuse; Premature rupture of mem-branes (PROM); Infections (i.e., HIV)

Intrapartum Placenta abruption/previa; Cordprolapse; Maternal shock; C-section

What are some important things to know Pediatric intubation slightly differsabout pediatric intubations? from adult; Important to know the

anatomic differences; Also knowpotential complications

198 Deja Review: Emergency Medicine

Rapid Sequence Intubation Protocol

Premedicate:Atropine <10 kg 0.1 mg

>10 kg 0.01 mg/kgLidocaine 1 mg/kg

Sedation:Etomidate 0.3 mg/kg

Paralysis:Rocuronium 1 mg/kg

Seizures

Medication Dose Route

Diazepam 0.1–0.3 mg/kg IV/IO0.5 mg/kg initially Rectal0.25 mg/kg

Lorazepam 0.05–0.1 mg/kg IVPhenytoin 20 mg/kg IVPhenobarbital 20 mg/kg IV

IV, intravenous; IO, intraosseous.

Page 215: Deja review _emergency_medicine__deja_review_

What is a Broselow tape? Quick length-based reference forpediatric resuscitation that includestube size and pediatric medicationdosing

What are some key things to know foreach of the following anatomicalvariations:

Larynx More anterior and superior thanadults; Angle for intubation is moreacute; Straight blade (Miller) ispreferred; Infant tongue is largerrelative to mouth size

Trachea Much shorter compared to adults;Intubation of right bronchus is morelikely; Dislodgement of tube is morelikely

Cricoid ring Narrowest region of the airway

What is the formula used to calculate Tracheal tube size = 4 + age/4tracheal tube size for children?

What is the backup airway of choice in Percutaneous transtracheal ventilationchildren less than 12 when intubationfails?

What are some important points about Not a definite airway; Will progres-percutaneous transtracheal ventilation? sively get hypercapnia; Typically

useful for <1 hour

What is the normal rate of breathing ineach of the following age group:

Neonates About 50 breaths/minute

Infants and children <8 years 20 breaths/minute

Children >12years 12 breaths/minute

What is an important point with regard to Make sure it is volume-limited andmechanical ventilation of children? hyper-ventilation in the setting of

acute herniation

What are some key points in an infant Avoid the Heimlich maneuver; Usewho presents with complete airway back blows and chest compressions;obstruction due to a foreign body? Avoid blind finger sweeps

What are the two preferred routes of Intravenous (IV) and intraosseous (IO)vascular access?

What is the primary purpose of establishing Administration of medications; FluidIV/IO access in children? resuscitation; IO access is not usually

effective for significant volumeresuscitation

Pediatric Emergencies 199

Page 216: Deja review _emergency_medicine__deja_review_

What are some key points in vascular The preferred site is the largest vein;access? Peripheral access should be attempted

first; Central access can havesignificant complications

What are some key points in IO access? Typically performed if peripheralaccess fails; IO is easier and fasterthen central access; Anteromedialproximal tibia is the preferred site

What is an important point about central Should only be attempted if peripheralaccess in children (typically <6 years)? and IO access fails and by an experi-

enced provider

What is an alternative way to deliver Tracheal routemedication?

What are some key points about tracheal Only useful for specific drugs;medication administration? Typically use 2–3 × the amount

of IV; Switch over to IV once available;Pharmokinetics are less reliable

List commonly used medications that LEAN: Lidocaine, epinephrine,can be given though the tracheal route. atropine, and naloxone

What is a commonly used route for Cannulation of the umbilical arteryvascular access in newborns?

Name some commonly used medicationsin pediatric resuscitation and theirprimary indications:

Epinephrine Used for primary arrests in children;Can be given IV/IO/tracheal route

Atrophine Not used in acute resuscitation; Can beused for increased vagal stimulation;Used as premedication prior tointubation

Adenosine Used for supraventricular tachycardia(SVT) in pediatric patients

Lidocaine Used to blunt ICP increase inintubation

What are the two most common causes of 1. Respiratory arrestcardiac arrest in children? 2. Hypovolemic shock

What are two most common 1. Asystoledysrhythmias in pediatric arrest? 2. Bradyarrhythmias

200 Deja Review: Emergency Medicine

Page 217: Deja review _emergency_medicine__deja_review_

NEONATAL/INFANT-SPECIFIC CONDITIONS

What are the main categories of the Activity (muscle movement)APGAR score: Pulse

Grimace

Appearance

Respirations

What is the definition of the APGAR score? It is a 0–10 point scale that is assignedto newborns at 1 and then 5 minutesthat is used to evaluate the newbornand has prognostic functions as well

What are the key parameters to monitor in Respiratory status; Heart ratedeciding if resuscitation should be done?

List some important things to do during a Suction airway of secretions; Maintainneonatal resuscitation. temperature; Supplemental oxygen;

Cardiopulmonary resuscitation(CPR) (HR <60 beats)

What is meconium? Thick green substance that linesthe intestines of the fetus and isnot typically released as a bowelmovement until the first few days oflife

Is it possible for meconium to be released Yes—increased risk of aspirationinto amniotic fluid prior to deliveryof fetus?

What is the feared complication of Respiratory distress (esp. thickmeconium aspiration? meconium); Meconium aspiration

pneumonia (PNA)

In what circumstances can meconium be Meconium found in amniotic fluid;bad for the fetus/newborn? Consistency of meconium is thick/

green

What is the mortality rate if a newborn 30–50%has aspirated thick meconium?

What are some key points in the treatment Bulb suction mouth/nose duringof meconium staining? delivery; Meconium-stained fluid

and respiratory depression = trachealsuctioning

What is esophageal atresia? Birth defect where the esophagus is segmented and cut off

How common is esophageal atresia? Occurs 1 in 4000 live births

Pediatric Emergencies 201

Page 218: Deja review _emergency_medicine__deja_review_

What other congenital defect does Tracheoesophageal fistulaesophageal atresia typically occur with?

What is the biggest risk factor for the Prematuritydevelopment of esophageal atresia?

What are some clinical features of Coughing/choking when feeding isesophageal atresia? attempted, recurrent aspiration PNA,

and inability to pass a catheter intothe stomach

What is the treatment for esophageal Surgical correctionatresia?

What is the definition of necrotizing Condition with varying degrees ofenterocolitis? intestinal necrosis most common in

premature newborns with low birthweight

What are some risk factors associated with Prematurity; Infections; Hypertonicnecrotizing enterocolitis? feeding solutions

When does necrotizing enterocolitis First 2 weeks of lifetypically occur in the newborn?

What is the most common physical Abdominal distension with gastricfinding in a newborn who presents with retentionnecrotizing enterocolitis?

What are some other clinical features in a Bloody stools, bilious emesis, and newborn with necrotizing enterocolitis? abdominal wall redness and/or

tenderness

What are some important clinical features Bouts of apnea, temperature changes,of a newborn who may be septic? lethargy, and metabolic acidosis

What are some complications of necrotizing Necrosis of bowel; Perforation;enterocolitis? Sepsis

What are some diagnostic tests to consider Plain films (classic finding is pneu-in a newborn with necrotizing enterocolitis? motosis intestinalis); Cultures (stool,

urine, blood, and CSF)

What are some key points in the NPO and NG tube; IV fluids/Abx;management of a newborn with Surgical consultationnecrotizing enterocolitis?

What is an omphalocele? A defect in the umbilical wall withherniation of intestinal contentcovered in a peritoneal sac

What is important to know about Many of them are associated withomphalocele? other congenital defects

What is a gastroschisis? A defect in abdominal wall withherniation of intestinal contentwithout peritoneal sac

202 Deja Review: Emergency Medicine

Page 219: Deja review _emergency_medicine__deja_review_

What are some complications of Obstruction; Strangulation; omphalocele and gastroschisis? Hypovolemia; Death

What are some key points in the NG tube for GI decompression;management of omphalocele and Do not attempt to reduce the mass;gastroschisis? Cover in saline-soaked sterile gauze;

IV fluids and Abx for prophylacticcoverage; Surgical correction required

What is the definition of diaphragmatic A congenital defect due to devel-herniation? opmental failure of a portion of the

diaphragm that allows herniation ofstomach/intestines

Which are more common? Right-sided Left-side far more common thanherniation or left-sided herniation? right side

What is important to know about As with most congenital defects, therediaphragmatic herniation? are typically other defects as well,

with GI/GU abnormalities andcongenital heart defects being fairlycommon

What are some clinical features of Symptoms referable to herniation ofdiaphragmatic herniation? GI tract into the chest: emesis and

respiratory distress as well as bowelsounds over the chest wall

What are some common radiographic Displacement of mediastinal contents findings in diaphragmatic herniation? (heart); Loops of bowel in the chest;

Lack of distinct diaphragmatic margin

What is an important consequence of Hypoplastic lungdiaphragmatic herniation on the lung?

What are some key points in the NG tube for GI decompression; NPOmanagement of diaphragmatic herniation? and IV fluids; Surgical correction

What are some different types of neonatal Myoclonic; Tonic-clonic; Focal clonicseizures?

What is the most common cause of Simple febrile seizureseizures in children?

What are some important things to know Up to 5% of children are affected;about febrile seizures? Commonly occur between 3 months

and 5 years

What are some clinical features of a febrile Rapidly ascending fever; Generalizedseizure? seizure less than 15 minutes duration;

No focal neuro deficit

What is another important diagnosis to Meningitisrule out in the setting of febrile seizure?

Pediatric Emergencies 203

Page 220: Deja review _emergency_medicine__deja_review_

What important diagnostic test should be Lumbar puncturedone for suspected meningitis?

What are some key points in the Lower the fever; Treat the underlyingmanagement of febrile seizures? cause of fever; Seizure prophylaxis is

not recommended

What is another important seizure to Generalized tonic-clonic seizureconsider in children?

What are some clinical features of a tonic- Bilateral hemisphere involvementclonic seizure? with motor involvement and alter-

nations in consciousness

When should one consider use of a Focal neuro deficit; Signs of increasedhead CT? ICP; Suspected child abuse/head

trauma

What are some key points in the Most will terminate on their own;management of tonic-clonic seizures? Benzodiazepines are the mainstay;

Phenytoin and phenobarbital aresecond-line agents

What is the preferred benzodiazepine due Lorazepamto its long half-life and least effect onrespiratory depression?

What are some important causes of Hypoxia; Drug withdrawal;neonatal seizures to consider? Electrolyte imbalance; Metabolic

(i.e., hypoglycemia); CNS infections;Neoplasm

List the TORCHS infections. TOxoplasmosis

Rubella

Cytomegalovirus

Herpes

Syphilis

What are some key points in the Airway, breathing, circulation (ABCs);management of neonatal seizures? Correct easily reversible conditions

(e.g., hypoxia); Monitor associatedproblems (e.g., acidosis);Anticonvulsant therapy

What is the drug of choice in the Phenobarbitalmanagement of neonatal seizure?

What are other classes of drugs used if Benzodiazepine; Phenytoinphenobarbital fails?

What are some reasons that benzodiazepine Profound respiratory depression;is not first-line treatment in neonates as Displacement of bilirubin fromopposed to adults? albumin

204 Deja Review: Emergency Medicine

Page 221: Deja review _emergency_medicine__deja_review_

CONGENITAL HEART DISEASE

Is cyanosis ever normal in the newborn? Yes, but only within the first 1/2 hourof life

What are the physical findings in a Bluish tongue, peripheral extremity,newborn with central cyanosis? and mucous membrane

What is the amount of unsaturated Greater then 5 gHgb in a newborn with centralcyanosis?

At what point is central cyanosis If it persists for greater thanpathologic? 30 minutes

What are some important causes of Primary lung disease; Cyanoticpersistent central cyanosis in the newborn? heart disease; Methemoglobinemia

What are the five “Ts” of cyanotic heart Tetralogy of Fallot; Tricuspid atresia;disease that result in central cyanosis due Transposition of the great vessels;to right-to-left shunt? Truncus arteriosus; Total anomalous

pulmonary venous return

What particular agent is important to Prostaglandin E1

maintain the patency of the ductusarteriosus in newborns with congenitalheart defects such as transposition of thegreat vessels?

What is a useful test to do to distinguish Administer 100% oxygen and watchright-to-left shunts from other causes of oxygen saturation, if it fails tocentral cyanosis such as sepsis? improve, it points to a right-to-left

shunt

What is the most common cyanotic Tetralogy of Fallotcongenital heart disease in children?

What are the four anatomical abnormalities 1. Pulmonary artery stenosisin tetralogy of Fallot? 2. Right ventricular hypertrophy

3. Ventricular septal defect (VSD)4. Overriding aorta

What are some common findings for eachdiagnostic test used with tetrology of Fallot:

ECG Right ventricular hypertrophy; Rightaxis deviation

CXR Decreased pulmonary vasculature;Boot-shaped heart

CBC Compensatory polycythemia

Pediatric Emergencies 205

Page 222: Deja review _emergency_medicine__deja_review_

What are two most common non-cyanotic 1. VSDcongenital heart defects? 2. Aortic stenosis

What are some clinical features of aortic Typically not detected until laterstenosis? in life: congestive heart failure (CHF),

chest pain (CP), and syncope

What are some complications of congenital Sudden death (2° dysrhythmias);aortic stenosis? Endocarditis

What are some clinical features of VSD? Determined by the size of VSD: rangesfrom asymptomatic to heart failure

What is the most common cause of CHF in Congenital heart diseaseneonates and children?

What are some clinical features of CHF? Rhonchi, rales, hepatomegaly, failureto thrive, and feeding difficulty

What are some other causes of CHF aside Sepsis; AV malformations; Severefrom congenital heart disease? anemia; Hypoplastic left heart

syndrome; Infectious myocarditis

What are some key points in the Search for and correct underlyingmanagement of CHF in neonates/children? cause; Supplemental oxygen; Use of

digoxin and furosemide when needed

AIRWAY EMERGENCIES

Upper Airway

What is epiglottitis? Epiglottitis is a life-threatening condi-tion that occurs when the epiglottis—a small cartilage “lid” that covers thewindpipe—swells, blocking the flowof air into the lungs

What is the most common cause of H. influenzaeepiglottitis?

What are some other causes of epiglottitis? Burns from hot liquids; Direct traumato throat; Various infections

What age group is epiglottitis most 2–6 years of ageprevalent?

Is HIB epiglottitis common today? No—since the introduction of HIBvaccine, it is not commonly seen. It ismore common in immigrants andunvaccinated children

What are some clinical features of Typically will be ill-appearing, stridorepiglottitis? and drooling with the child leaning

forward is a classic picture

206 Deja Review: Emergency Medicine

Page 223: Deja review _emergency_medicine__deja_review_

What is the diagnostic test of choice in the Lateral neck film—typically showsevaluation of epiglottitis? enlarged epiglottis

What are some key points in the Ensuring intact airway is paramount;management of evaluation of epiglottitis? ENT should be consulted in severe

cases; Low threshold for intubation;IV Abx (third generation cephalosporincommon); Typically ICU admissionfor monitoring

What is croup? Inflammation of the upper airwaythat leads to a cough that sounds likea seal bark, particularly when a childis crying

What is the most common cause of croup? Viral (parainfluenza being mostcommon)

What age group is croup most prevalent? Around 2 years of age (in the fall-winter)

What are some clinical features of croup? Bark-like cough worse at night is thehallmark, upper respiratory infection(URI) prodrome, stridor, and hoarse-ness with a low-grade fever

What role does a lateral neck film play? To rule out epiglottitis (although rare)

What are some key points in the manage- Typically resolves in a week; Abx notment of croup? used—since viral most of the time;

Cool mist and hydration; Steroidsshould be given to help resolve; Admitif refractory to tx (persistent stridor)

What role does racemic epinephrine Used for children who have restingaerosol play? stridor and more severe respiratory

distress

What is bacterial tracheitis? Diffuse inflammatory process of thelarynx, trachea, and bronchi withadherent or semiadherent mucop-urulent membranes within the trachea

What are some clinical features of bacterial Often will present as croup, buttracheitis? defining feature is that child will not

respond to standard croup tx and willoften get quite sick

What is the most common pathogen Staphylococcus aureusimplicated in bacterial tracheitis?

What are the key points in the Ensure an intact airway; IV hydrationmanagement of bacterial tracheitis? and Abx directed against staph; ENT

consult is usually recommended;Admit to ICU for monitoring

Pediatric Emergencies 207

Page 224: Deja review _emergency_medicine__deja_review_

What is a retropharyngeal abscess ? Infection in one of the deep spaces ofthe neck with potential for airwaycompromise

What are two ways in which the 1. Direct inoculation via traumaretropharyngeal space can become infected? 2. Spread from infection

What are some common infections that can URI, otitis, pharyngitis, and sinusitislead to a retropharyngeal abscess?

What age group is a retropharyngeal 6 months to 5 years of ageabscess most common?

What are some complications of a retropha- Airway compromise the mostryngeal abscess? important; Abscess rupture; Spread

of infection (i.e., sepsis)

What are some common clinical features Typical picture is an ill-appearingof a retropharyngeal abscess? child who is drooling and cannot

tolerate PO and will often have aneck mass

What are some important diagnostic Lateral neck film-retropharyngealtests to consider in a retropharyngeal swelling; Make sure child is inabscess? inspiration during film; CXR-inspect

for possible mediastinitis; CT is studyof choice

What are some key points in the ENT involvement for incision andmanagement of a retropharyngeal abscess? drainage (I&D) of abscess; Abx; ICU

monitoring in severe cases

What is the most common cause of Foreign body aspirationaccidental home death in young children?

What are some common clinical features in Stridor is common if the obstructionforeign body aspiration? is higher; Respiratory wheezing if

obstruction is lower; Suspect animpacted object in the airway ifrecurrent PNA

What is the most common location of Right mainstem bronchusforeign bodies?

What are some common diagnostic tests CXR and MRI can be used to evalu-used in foreign body aspiration? tate; Bronchoscopy is diagnostic and

therapeutic

Lower Airway

What is bronchiolitis? Acute infectious disease of the lowerrespiratory tract

What is the pathophysiology of Narrowing of the bronchi/bronchiolesbronchiolitis? typically due to inflammation of

epithelial cells

208 Deja Review: Emergency Medicine

Page 225: Deja review _emergency_medicine__deja_review_

What is the most common cause of Viral-RSV most commonbronchiolitis?

What is a common history element in most Sick contact or at day carepatients who present with bronchiolitis?

What age group is most commonly affected Infants of 2 months to 2 yearswith bronchiolitis?

What are some clinical features of Typically URI-like symptoms beforebronchiolitis? progression to lower respiratory tract

symptoms of wheezing, SOB, andpossible cyanosis

What are common diagnostic findings in Patchy atelectasis; Hyperinflationa CXR? of lungs; Air trapping

What airway disease do many children Asthmawith bronchiolitis later develop?

What are some key points in the Supportive care; Ensure propermanagement of bronchiolitis? hydration; Abx not indicated—viral

infection; A trial of bronchodilatorsmay be warranted

What are some indications for admission Respiratory distress, extreme tachyp-for bronchiolitis? nea; Hypoxia; Inability to take PO;

Poor home care

What is the most common chronic disease Asthmaof the pediatric population?

What are some important points The prevalence is rising in theabout asthma? United States; Mortality from asthma

is also rising; Accounts for largeamount of ED visits

What is the definition of asthma? It is a chronic inflammatory disordercharacterized by increased respon-siveness to a variety of stimulithat results in reversible airwayconstriction/obstruction

List important triggers for asthma Any upper respiratory infection;exacerbation. Inhaled irritants (i.e., smoke);

Medication; GERD; Cold environment;Exercise

What is the pathophysiology for asthma Triggers that result in an IgE-mediated exacerbation? response that leads to inflammation

and bronchial smooth muscle contrac-tion, this eventually results in airwayedema and movement of inflammatorycells. The end result is increasedairway resistance

Pediatric Emergencies 209

Page 226: Deja review _emergency_medicine__deja_review_

What are some clinical features of asthma Respiratory distress, increased work ofexacerbation? breathing, tachypnea, tachycardia,

and in some, only a chronic cough

Is it reassuring if no wheezes can be heard No—may represent total cessation ofon exam of an asthmatic with exacerbation? airflow

What is PEFR? Peak expiratory flow rate—typicallymeasured before and after treatmentto assess effectiveness

What are some important points for eachof the following categories of exacerbation:

Mild exacerbation Oxygen saturation above 95% onroom air; PEFR >80%; Mild wheezingon exam; Able to speak in fullsentences

Moderate exacerbation Oxygen saturation in low 90s; PEFR50–80%; Wheezing via expiratoryphase; Difficulty in speaking

Severe exacerbation Oxygen saturation <90% in room airPEFR <50%; Typically using accessorymuscles; Can only speak one or twowords at a time

What are risk factors associated with poor Prior intubation or ICU admission;outcome in asthma exacerbation? Greater than three hospitalizations per

year; Use or cessation of oral steroids;Significant comorbid disease (CAD);Lower socioeconomic status

What are some key points in the ABCs—particular with O2 adminis-management of asthma exacerbation? tration; B2-agonist is the mainstay

treatment; Anticholinergic used insevere cases; Steroids

What are some important points for each ofthe following used medications in asthma:

B2-agonist The mainstay treatment: nebulizeror inhaler; Primary effect is on smallairway; Albuterol most commonlyused; IV use only in very sickpatients

Anticholinergic Ipratropium most commonly usedagent; Primary effect is on largeairways; Added to B2-agonist in moresevere cases; Atropine not used dueto side effect profile

210 Deja Review: Emergency Medicine

Page 227: Deja review _emergency_medicine__deja_review_

Steroids Shown to prevent progression andrelapse; IV and oral equally effective;Should be continued on steroids onced/c

Leukotriene modifiers Inflammatory mediators used in out-patient; No role in acute managementof asthma

Magnesium sulfate Has bronchodilator properties; Usedin acute exacerbations as second-linetreatment; Not particularly effectivein mild exacerbations

Ketamine Induction agent with mild bron-chodilator effects; Recommended ifintubation is to be done

Heliox Mixture of helium-oxygen (80:20);Helps decrease work of breathing;May help in severe exacerbations

What are some considerations for Failure to improve after treatment inadmission in a patient with asthma the ED; Poor home care; History ofexacerbation? ICU/intubation for asthma

What are the general guidelines to safely Good patient follow-up; 3–4 hours isdischarge a patient from the ED? usually enough to show improvement

with medication

PEDIATRIC GASTROINTESTINAL

Appendicitis

What are some clinical features of Typically have diffuse periumbilicalappendicitis? pain that eventually leads to N/V and

RLQ pain. Will often have a low-gradetemperature as well

What are some commonly used diagnostic Kidney-ureter-bladder (KUB): rarelytests in appendicitis? shows a fecalith; CT with contrast:

test of choice; U/S: operator-dependent

What are some findings in a child with High-grade fever, high WBC, andperforation of the appendix? symptoms over 2 days as well as

diffuse abdominal pain and peritonealsigns

What are some key points in the IV fluids and NPO; Broad-spectrummanagement of appendicitis? Abx prior to surgery; Surgical consult

Pediatric Emergencies 211

Page 228: Deja review _emergency_medicine__deja_review_

Pyloric Stenosis

What is the definition of pyloric stenosis? It is hypertrophy of the pylorus withgastric outlet obstruction

What age group is commonly affected with Male newborns between 2–4 weekspyloric stenosis?

What are some common clinical features Nonbilious projectile vomiting isof pyloric stenosis? the hallmark with failure to thrive

and sometimes a palpable rightupper quadrant (RUQ) mass canbe felt

What are some tests used to diagnose U/S and upper GI seriespyloric stenosis?

What are some key points in the NPO and IV fluids; Prompt surgicalmanagement of pyloric stenosis ? correction

Incarcerated Hernia

What age group do incarcerated hernias Under 1 year of agetypically occur in?

What are some clinical features of Emesis with a palpable scrotal/incarcerated hernias? inguinal mass

What other conditions are in the Hydrocele; Torsion of testicles;differential diagnosis for incarcerated Undescended testishernia?

What are some key points in the Manual reduction, then outpatientmanagement of incarcerated hernias? surgery; If any evidence of ischemia =

immediate surgical reduction

Intestinal Obstruction

What is the clinical hallmark of intestinal Emesis with abdominal pain andobstruction? distension

What are some important causes of Hernias; Intussusceptions;intestinal obstruction? Congenital atresia

What are some common findings on Dilated loops of bowel with air-fluid abdominal plain films? levels

What are some key points in the NPO, IV fluids, and NG tube; Surgicalmanagement of intestinal obstruction? intervention required

212 Deja Review: Emergency Medicine

Page 229: Deja review _emergency_medicine__deja_review_

Intussusception

What are some important things to know Number 1 common cause of obstruc-about intussusception? tion in children; Most common age

group: 3 months to 5 years; Ileocolicintussusception most common; Morecommon in males

What are some clinical features of Emesis, colicky pain, and red jellyintussusception? stools as well as possible mental status

change. PE: may palpate a sausage-shaped mass

What is the most prominent feature of Periods of intense abdominal painabdominal pain in intussusception? followed by periods of no pain

What are some important points for eachof the following diagnostic tests:

Abdominal plain films May show abdominal mass in RUQ;May show dilated bowel with air-fluidlevels; Free air in perforation

Barium enema/air-contrast It the test of choice to detect intussus-ception; Therapeutic: reduces in mostcases; BE may show coiled-springappearance

What is the next step to be taken if BE Surgical interventionor air-contrast fails to reduce theintussusception?

What is the recurrence rate after a As high as 10% in the first 24 hourssuccessful BE or surgical reduction?

Meckel’s Diverticulum

What is the definition of Meckel’s A Meckel’s diverticulum is a remnantdiverticulum? of structures within the fetal digestive

tract that were not fully reabsorbedbefore birth and leads to a pouchwith GI tissue

What remnant of tissue from the prenatal Gastric tissue most commondevelopment of the digestive system isfound in Meckel’s diverticulum?

What is the “rule of 2’s” in Meckel’s Peak age of symptoms is 2 yearsdiverticulum? of age; Affects 2% of the population;

2 inches in length; Two times morelikely in males

Pediatric Emergencies 213

Page 230: Deja review _emergency_medicine__deja_review_

What are some clinical features of Painless bleeding from rectum, N/V,Meckel’s diverticulum? and sign of obstruction if a volvulus

develops

What are some other considerations in Anal fissures; Juvenile polyps;an infant with painless bleeding? Infection

What are three complications of Meckel’s 1. Inflammation that mimics appen-diverticulum? dicitis

2. Bleeding—can be massive3. Obstruction—volvulus or

intussusception

What is the diagnostic study of choice for Meckel’s isotope scanningMeckel’s diverticulum?

What are some key points in the Remove if heavy bleeding or pain; management of Meckel’s diverticulum? Surgical intervention if sign of

obstruction

Volvulus

What is the definition of a volvulus? A form of obstruction typically dueto malrotation of the bowel duringembryonic development

What age group is more commonly Greater then 90% present <1 yearaffected with a volvulus? of age

What are some clinical features of a Failure to thrive, anorexia, intermittentvolvulus? apnea, emesis (bilious) with abdominal

distension

What is a feared complication of a Gangrene with perforationvolvulus if not promptly treated?

What are some important points for eachof the following diagnostic tests:

Obstructive series Gastric/duodenal distension (doublebubble); Relative paucity of lowerGI gas

Upper GI contrast series Study test of choice; “Bird-beak"obstruction at proximal duodenum

Ultrasonography and CT scanning Use is more as adjunctive tests;Definitive diagnosis rests on uppergastrointestinal (UGI) study

What are some key points in the NPO, IV fluids, and NG tube; Surgicalmanagement of a volvulus? intervention

214 Deja Review: Emergency Medicine

Page 231: Deja review _emergency_medicine__deja_review_

INFECTIOUS DISEASE

Bacteremia and Sepsis

What is the pathophysiology of fever? Typically due to exogenous substance(antigens/bacterial wall components)that result in the release of pyrogensthat in turn result in PG production,this acts on the hypothalamus to raisethe hypothalamic set point

What are some common manifestations of Chills, shivering, peripherala raised hypothalamic set point? vasoconstriction, and behavioral

activities (using blankets) that resultin elevation of body temperature

What area of the hypothalamus regulates Ventromedial preoptic area;body temperature? Periventricular nucleus

What are some common methods to Oral; Axillary; Rectal; Tympanicmeasure temperature?

What method is the most accurate and thus Rectalshould be used whenever possible?

What are some risks for serious bacterial Infants: rectal temp (>38° C) andinfection that may not be obvious in the leukocytosis; Neonates withpediatric population? hypothermia (<36° C); Fever with

a low white count (<5k); Fever witha petechial rash

What are some commonly used drugs to Ibuprofen; Acetaminophentreat fevers?

What role does aspirin play in the Should be avoided due to associationtreatment of fever from viral illnesses? with Reye’s syndrome (it is effective

and used commonly in some parts ofthe world)

What is Reye’s syndrome? It affects all organs of the body but ismost harmful to the brain and theliver, causing an acute increase ofpressure within the brain and, often,massive accumulations of fat in theliver and other organs

What is the most common preceding factor? Viral illness (i.e., chicken pox)

What are some clinical features of Reye’s Recurrent vomiting, listlessness,syndrome? personality changes such as irritability

or combativeness, disorientation orconfusion, delirium, convulsions, andloss of consciousness

Pediatric Emergencies 215

Page 232: Deja review _emergency_medicine__deja_review_

What other conditions is Reye’s syndrome Meningitis, diabetes, drug overdosecommonly mistaken for? poisoning, and encephalitis

What is the most common cause of Brain herniation from swellingmortality in Reye’s syndrome?

What is the treatment for Reye’s syndrome? Treatment is primarily supportivewith care focusing on reducing brainswelling

What is occult bacteremia? It is fever with positive blood culturesin a child who does not have a majorsource of infection

What are the three most common 1. S. pneumoniae—by far the mostorganisms responsible for occult commonbacteremia? 2. N. meningitidis

3. Salmonella species

What age group is most susceptible Between 6 months and 2 years of ageto infection?

What is the reason for this? Infants <6 months typically havematernal antibodies which decreaseleaving infants more susceptible tillthe age of 2, when they eventuallydevelop their own

What is the definition of sepsis? It occurs when bacteria, which canoriginate in a child's lungs, intestines,urinary tract, or gallbladder, maketoxins that cause the body's immunesystem to produce various cytokinesthat act on many targets in the body

What are the three most commonorganisms responsible for sepsis in thefollowing age group:

Neonates Group B Streptococcus; Listeria mono-cytogenes; E. coli

Infants S. pneumoniae; H. influenzae;N. meningitidis

What are some clinical features of an Ill-appearing, lethargic, periods ofinfant who is septic? apnea and bradycardia, failure to

thrive, and often hypothermic(<36ºC)

What is the standard workup for CBC, blood cultures, U/A with urineneonates/infants who may be septic? cultures, stool cultures, CXR, and LP

216 Deja Review: Emergency Medicine

Page 233: Deja review _emergency_medicine__deja_review_

Meningitis

What is the definition of meningitis? It is a serious CNS infection of themeninges with often devastatingresults in infants and young childrenif not treated early

What are two common sources of 1. Hematogenous spread—mostinfections in meningitis? common

2. Direct spread from a contiguousfocus

Why is the diagnosis of meningitis more The classic signs/symptoms (stiffelusive in infants? neck/HA/fever) are often not present

What are some clinical features of Lethargy, decreased oral intake,meningitis in infants (<4 months)? irritability, fever or hypothermia,

seizure, and bulging fontanelle

What are the three most commonorganisms responsible for meningitisin the following age group:

Neonates Group B streptococcus; L. monocy-togenes; E. coli

Infants/young children S. pneumoniae; H. influenzae;N. meningitidis

What are other important causes of Viral; Fungal; TB; Asepticmeningitis to consider aside from bacteria?

What are some key points in the IV broad-spectrum Abx without delay;management of meningitis? LP to diagnose and tailor Abx therapy;

Antiviral tx if suspicious of herpes

What role do steroids play in the treatment They may play a role in reducingof meningitis? neurologic sequelae if given early

Otitis Media

What is the definition of otitis media? Infection of the middle ear with acuteonset, possible presence of middleear effusion, and signs of middle earinflammation

What is the pathophysiology of Obstruction of the eustachian tubeotitis media? that result in a sterile effusion with

aspiration of nasopharyngealsecretions into the middle ear thatresult in acute infection

Pediatric Emergencies 217

Page 234: Deja review _emergency_medicine__deja_review_

Why do otitis media occur more Infants and younger children havefrequently in children? shorter and more horizontal

eustachian tube then adults

Name four of the most common pathogens 1. S. pneumoniaethat cause otitis media? 2. H. influenzae

3. Moraxella catarrhalis4. Group A streptococcus

What are some clinical features of Exam of ear often show distortion ofotitis media? tympanic membrane (TM), erythema,

decreased mobility of TM on pneu-matic otoscopy, fever, poor feeding,and child pulling at ear

What are some complications to consider Hearing loss, TM perforation,in otitis media if left untreated? mastoiditis, lateral sinus thrombosis,

and meningitis

What are the main Abx used to treat Amoxicillin is the mainstay followedotitis media? by TMP-SMX or macrolide as second-

line treatement

When should the fever and symptoms Within a few days after Abx is startedbegin to subside?

Pneumonia

What age group is most commonly affected Incidence is greatest in 6–12 monthswith pneumonia (PNA)? of age

What is the primary mode in which Typically from aspiration of infectiousPNA occurs? particles, such as from a preceding

URI

What are some important elements in the Comorbid conditions; Age; Sickhistory of a child with PNA? contact (i.e., day care); Immunizations

What is the most common cause of PNA in Viruses—RSV being most commonchildren (not neonates)?

What are some common bacterial pathogens Mycoplasma; S. pneumoniae; that cause PNA in infants/children? C. trachomatis; H. influenzae

What are some clinical features of PNA in Often will have a preceding URI,infants/children? cough, fever, and tachypnea are

common

What are some important diagnostic Pulse ox (hypoxia), CBC and bloodstudies to consider in PNA? cultures are often ordered, CXR, and

sputum stain

What is the more likely cause of PNA in Viral; Chlamydial; Mycoplasmawhich the CXR shows diffuse interstitialpattern?

218 Deja Review: Emergency Medicine

Page 235: Deja review _emergency_medicine__deja_review_

What is the more likely cause of PNA Bacterialin which the CXR shows lobarinvolvement?

What are some key points in the Bacterial PNA require specific Abxmanagement of PNA? coverage; Viral PNA typically require

supportive care; Persistent PNA =possible foreign body aspiration inchildren

What are indications for admission in an Respiratory distress; Ill-appearing;infant/child who presents with PNA? PNA complications (i.e., empyema);

Hypoxia; Outpatient Abx failure;Social reasons (poor care at home)

Pertussis

What is the causative agent of pertussis Bordetella pertussis(whooping cough)?

What are some important things to know Highly infectious (via respiratoryabout pertussis? droplets); Incubation time is about

10 days; Mortality is highest in firstfew months

What group is commonly affected Nonimmunized childrenby pertussis?

Can adults who received vaccination Yes—does not confer life-longagainst pertussis still develop it later immunityin life?

What is the three-stage illness of pertussis:

Catarrhal URI prodrome that last for about2 weeks; Highly infectious at thisstage

Paroxysmal Paroxysmal coughing spells; Emesisis common with the coughing; Canlast up to 1 month

Convalescent Residual cough that can last formonths

What is the characteristic finding on CBC WBC that can be as high as 50k;in a patient with pertussis? Lymphocytosis is common

What are some commonly used tests to Bordet-Gengou medium; PCR;diagnose pertussis? Antibody staining

What is the Abx of choice to treat pertussis? Erythromycin (can also be given toclose contacts of patients withpertussis)

Pediatric Emergencies 219

Page 236: Deja review _emergency_medicine__deja_review_

What is the typical pertussis vaccine Before age 7, children should get regiment? five doses of the DTaP vaccine; These

are usually given at 2, 4, 6, and15–18 months of age and 4–6 yearsof age

What are some complications of pertussis? PNA; Seizure; Brain death fromhypoxia

Urinary Tract Infection

What are some important things to know They are fairly common in theabout UTI in infants/children? pediatric population; More common

in males during infancy; Infants/children have few specific symptoms

What is the mechanism of UTI in Ascending infection from perinealinfants/children? contaminants is common, but

hematogenous spread is morecommon in neonates

What is an important consideration in Structural problem in the GU tract;infants less then 1 year of age who have Vesicoureteral refluxrecurrent UTIs?

How common is urosepsis in infants 30%1–3 months in age?

What are some clinical features of neonates Irritability, emesis, diarrhea, poor oralwith UTIs? intake, and possible septic (as children

get older, their sx become more specificfor UTI-dysuria and frequency)

What is the most common pathogen in E. coliUTIs in this age group?

What are some possible causes of UTIs Meatal stenosis; Phimosis;in male children? Paraphimosis

What are some complications of UTIs? Pyelonephritis; Urosepsis; Renalscarring; Renal failure

What are three optimal ways to collect 1. Midstream collectionurine for a U/A? 2. Suprapubic aspiration

3. Bladder catheterization

What are the typical U/A findings Pyuria: >10 WBCs/HPF; Bacteriuria:that suggest UTI? >100k CFU/mL

What is another diagnostic test that Urine cultureshould be obtained for females <3 yearsand males <1 year?

What are the indications for Abx use? Symptomatic with pyuria/bacteriuria;Any evidence of pyelonephritis

220 Deja Review: Emergency Medicine

Page 237: Deja review _emergency_medicine__deja_review_

What are commonly used Abx in the Trimethoprim-sulfamethoxazoletreatment of UTIs? (TMP/SMX); Amoxicillin; Third

generation cephalosporins

What are commonly used radiographic Renal ultrasound; Voidingstudies to further evaluate UTIs? cystourethrography; IVP

CHILD ABUSE

What are three common types of abuse in 1. Sexual abusechildren? 2. Neglect

3. Physical abuse

How common is sexual abuse in children? Upto 25% of all females sexuallyabused; Upto 10% of all malessexually abused

In what percentage of sexual abuses Upto 90% (most often family/relatives)is the perpetrator known to the victim?

What are some findings on physical exam Vaginal discharge; Sexually transmittedthat is suggestive of sexual abuse? disease; Scarring/tearing of the hymen;

Anal fissures

Do sexually abused children always show No—up to 50% may present normallyevidence of abuse on exam?

What are important laboratory tests to Culture for gonorrhea and chlamydia;conduct in a child who is sexually abused? Syphilis; HIV testing

Are health-care providers required to Yesreport sexual abuse?

What are some physical findings in a child Poor hygiene, evidence of failure towho is suffering from neglect? thrive such as low weight for age,

alopecia, and avoidance

What is an important consideration in a Suspect physical abusechild who is suffering from neglect?

What are some important things to do if A skeletal survey for abuse; Reporta child is suffering from neglect? to the proper agencies; Child is

typically admitted

What does the skeletal survey usually AP and lateral views of skull/chest/consist of? pelvis/spine, and extremities

What are the most common causes of death Head and abdominal injuryin children who are physically abused?

What are red flags in a child’s history that Inconsistent history from caregivers;should raise the suspicion of physical History that does not match PE;abuse? Pattern injuries such as choke marks;

Bruises in certain areas like buttocks

Pediatric Emergencies 221

Page 238: Deja review _emergency_medicine__deja_review_

What are common injury patterns Posterior rib fractures; Cigarette burns;associated with physical abuse? Skull fractures; Healing fractures that

were not treated; Spiral fractures ofextremities

What is shaken baby syndrome? Type of inflicted traumatic braininjury that happens when a baby isviolently shaken

What are some reasons why a baby is more Weak neck; Proportionally largersusceptible to being shaken? head

What are the characteristic injuries that Subdural hematoma; Retinaloccur in shaken baby syndrome? hemorrhages/detachment; Spinal

neck/cord damage; Fracture of ribsand bones

What are some clinical features of shaken Extreme irritability, lethargy, poorbaby syndrome? feeding, breathing problems, convul-

sions, vomiting, and pale or bluishskin

What age group is shaken baby syndrome Typically infantsmost common in?

What are some important diagnostic tests to CBC/coags—to assess for coagu-consider in suspected physical abuse? lopathy; Skeletal survey; Imaging

studies such as CT or MRI

What is essential to do in all cases of Report to police and proper agencies;suspected physical abuse? Must not allow child to go back

home

CLINICAL VIGNETTES

A newborn is noticed to be apneic and Esophageal atresia choking whenever feeding is attemptedfor the past week, the newborn’s historyis only significant for prematurity

A 1-week-old newborn who was born Necrotizing enterocolitispremature is brought in the ER due to concerns of recent abdominal distensionwith bilious emesis; PE: abdominaltenderness

A 5-week old is brought in due to periods Diaphragmatic herniationof breathing difficulty as well as bouts ofemesis for about a month; PE: remarkablefor bowel sounds heard over the leftanterior chest

222 Deja Review: Emergency Medicine

Page 239: Deja review _emergency_medicine__deja_review_

A 1-year-old is brought in by his frantic Simple febrile seizuremother due to a sudden onset of ageneralized seizure that occurred aboutan hour ago; PE: low-grade temperature,but otherwise unremarkable PE

A 3-year-old ill-appearing female is brought Epiglottitisin by her mother for high-fever, historyis significant for recent immigration tothe United States from China; PE:ill-appearing child leaning forwardwith drooling and stridor

A 4-year-old child presents with low-grade Otitis mediafever, HA, and decreased oral intake;PE: erythema and decreased motility ofright tympanic membrane

A 2-year-old male is brought in with a 1-week Crouphistory of a URI, now presents with abark-like cough particularly worse atnight; PE: child otherwise appears welldespite the cough

An alarmed father brings in his 2-year-old Foreign body aspirationson due to a sudden onset of wheezing,but is otherwise well; PE: unremarkable

An 8-year-old child with a long history of Asthma exacerbation allergies is brought in by her mother dueto difficulty in breathing soon after soccerpractice; PE: bilateral wheezing

A 3-year-old female is brought in by her Intussusceptionconcerned mother who mentions that herchild has intense periods of colickyabdominal pain with periods of no painas well as red jelly stools

A 3-year-old female presents with a 2 day Appendicitishistory of nausea, emesis, fever, andirritability; PE: diffuse abdominal pain;Labs: elevated WBC

A 3-year-old ill-appearing male with a Retropharyngeal abscessrecent history of sinusitis now presentswith a high-grade fever and the inability toswallow; PE: Child is drooling and asmall mass can be felt on the neck, lateralneck film: retropharyngeal swelling

A 3-week-old male is brought in by her Pyloric stenosismother with concerns of ability to keep

Pediatric Emergencies 223

Page 240: Deja review _emergency_medicine__deja_review_

any nutrition down, she mentionswhenever the patient eats, he soon hasprojectile vomiting; PE: a nontenderRUQ mass can be felt

A 2-year-old male is brought in with Intestinal obstructiona 2-day history of abdominal pain anddistension with the inability to tolerateany feedings; abdominal films: dilatedloops of bowel with air-fluid levels

A 2-year-old male presents with painless Meckel’s diverticulumbleeding with nausea and vomiting,but otherwise has no other medicalproblems; PE: unremarkable

A 3-month-old child is brought in by her Meningitismother with lethargy, irritability, fever,and decreased oral intake that has beenongoing for about 2 days; PE: bulgingfontanelle

A 5-year-old male is brought in by her Child abusemother for a fall from his bed last night,his medical history is significant for threeother fractures to various other areas ofthe body; PE: fracture of the left clavicle

224 Deja Review: Emergency Medicine

Page 241: Deja review _emergency_medicine__deja_review_

C H A P T E R 1 3

Obstetrics andGynecology

225

NORMAL PREGNANCY

What are some physiologic changes thatoccur to each of the following systemduring normal pregnancy:

Respiratory

Cardiovascular

Gastrointestinal

Genitourinary

Hematology

Endocrine

Increase in tidal volume, minuteventilation, O2 consumption, andrespiratory rate along with adecrease in total lung capacity

Increase in circulating volume, heartrate (HR), and cardiac output (CO)with a 20% decrease in BP duringfirst trimester

Gastroesophageal reflux disease(GERD) very common, cholestasis,hemorrhoids, and nausea/vomiting

Increase in renal blood flow,glomerular filtration rate (GFR),kidney size, and urinary stasis;decrease in BUN/Crea

Increase in plasma volume, decreasein hematocrit (Hct), decrease inWhite blood cell (WBC) counts, andincrease in coagulation factors

Increase in glucose level, progeste-rone, estrogen, T3/T4 (euthyroid),thyroid-binding globulin, andprolactin

Copyright © 2008 by The McGraw-Hill Companies, Inc. Click here for terms of use.

Page 242: Deja review _emergency_medicine__deja_review_

Uterus

Dermatology

What are some important points to knowabout human chorionic gonadotropin(hCG)?

What are some conditions that can result ina positive pregnancy test?

What are some common causes of veryhigh levels of beta human chorionicgonadotrophin (β-hCG)?

226 Deja Review: Emergency Medicine

Weight will increase from 80 g to1,000 g and volume will increasefrom 10 mL to 5,000 mL

Hyperpigmentation of nipples,abdominal midline, and face; palmarerythema, and spiderangiomata

Detected as early as 9 days afterfertilization; Doubles every 2 daysearly in pregnancy; Very low falsenegative rate (<1%); Peaks at about10 weeks gestational age

Intrauterine pregnancy; Ectopicpregnancy; Recent abortion;Trophoblastic disease; Germ celltumors

Multiple gestations; Advanced age;Ovarian cancer; Trophoblasticdisease; Germ cell tumors

Define the following types of vaginalbleeding?

Abnormal bleeding

Dysfunctional uterine bleeding (DUB)

Menorrhagia

Metrorrhagia

Menometrorrhagia

What are some important elements togather in the history of anyone whopresents with vaginal bleeding?

What are important elements to gather inthe sexual history of a patient?

Vaginal bleeding outside one’sregular cycle

Abnormal vaginal bleeding due toanovulation

Excessive bleeding or cycles >7 days

Irregular vaginal bleeding

Excessive irregular bleeding

Menstrual history; Last menstrualperiod (LMP); Age of menarche;Any pattern of abnormal bleeding;Vaginal discharge; If they arepregnant (always do a pregnancytest)

Number of sexual partners in thepast; Contraception use and type;History of venereal disease (HIV,PID, Hep)

VAGINAL BLEEDING IN REPRODUCTIVE WOMEN (NONPREGNANT)

Page 243: Deja review _emergency_medicine__deja_review_

Obstetrics and Gynecology 227

What are some important causes of vaginalbleeding to consider in reproductive femaleswho are not pregnant?

What are some important causes of vaginalbleeding in menopausal women?

What are some important elements in thephysical to perform?

What are some key points in managementof vaginal bleeding in reproductive non-pregnant women?

Pregnancy; Exogenous hormone use;Coagulopathy; Thyroid dysfunction;Polycystic ovary syndrome;Leiomyomas; Adenomyosis

Endocervical lesions; Endometrialcancer; Exogenous hormone use;Atrophic vaginitis

A thorough vaginal exam; Examinefor possible GI or GU bleed

Make sure patient is not unstable(bleeding); Rule out pregnancy;OCP are often effective to controlbleeding; NSAIDs are also effectivein management

PELVIC/ABDOMINAL PAIN IN NONPREGNANT WOMEN

Pregnancy test

It is a surgical emergency; Oftenwill have a history of cysts ortumors; Exercise or intercourseoften precede pain; Often suddenonset of unilateral pelvic pain; U/Sand early surgical consult isimportant

They may twist, bleed, or rupture;Sudden unilateral pelvic pain iscommon; Must distinguish frompossible ectopic; U/S is an importantdiagnostic tool

Very common cause of cyclic pain;Most common in the third decadeof life; Often due to ectopicendometrial tissue; Often can get anormal pelvic exam

What is the single most important test todo on a female who presents withpelvic/abdominal pain?

What are some important points to knowabout each of the following causes ofpelvic pain in nonpregnant women:

Adnexal torsion

Ovarian cysts

Endometriosis

Page 244: Deja review _emergency_medicine__deja_review_

Adenomyosis

Leiomyomas (fibroids)

228 Deja Review: Emergency Medicine

Often present with dysmenorrhea;Most common in the fourth decadeof life; Pelvic can show a symmet-rical large uterus; Analgesic andhormonal tx often help

It is a smooth muscle tumor; Mostcommon in fourth decade of life;Typically estrogen-growth responsive;U/S will often detect fibroids;Analgesic and hormonal tx oftenhelp as well

What must be ruled out in any female who presents with pelvic/lower abdominal painor syncope?

What are some important points to knowabout ectopic pregnancy (EP)?

What are some major risk factors for EP?

What is the classic triad for the clinicalpresentation of EP?

What are some clinical features of aruptured EP?

What are some less common clinicalfeatures of a ruptured EP?

What is the differential diagnosis for asuspected EP?

What is the single most important test todo on any female of child-bearing age?

How does a pregnancy test work?

Qualitative pregnancy tests are positive atwhat level?

Ectopic pregnancy

Leading cause of first-trimester death;Implantation of fertilized egg outsidethe uterus; Most EPs occur withinthe fallopian tube

Pelvic inflammatory disease; Use ofintrauterine device; History of tubalsurgery; Exposure to diethylstilbe-strol (DES) in utero

Pelvic pain, spotting, andamenorrhea

Rebound tenderness, hypotensive,and adnexal mass

Syncope, unexplained shock,tenesmus, or shoulder pain

Ovarian rupture/torsion, abortion,and surgical abdomen

Pregnancy test

Pregnancy tests rely on thedetection of β-hCG, humanchorionic gonadotropin isa hormone produced by thetrophoblast

β-hCG is >20 mIU/mL in urine;β-hCG is >10 mIU/mL in serum

ECTOPIC PREGNANCY

Page 245: Deja review _emergency_medicine__deja_review_

What is a concern of doing a urinepregnancy test?

If the bedside urine pregnancy test isnegative, but EP is still a consideration,what is the next step?

How is the definitive diagnosis of EP made?

What is the primary purpose of U/S?

If U/S shows an IUP, is EP now excluded?

What should be noted about transabdominalultrasound (TA)?

What are some findings on ultrasound thatmay be suggestive of an EP?

What is the discriminatory zone?

What is the discriminatory zone of TA U/S?

What is the discriminatory zone of TV U/S?

What is the preferred medical managementfor EP?

What is the mechanism of MTX?

What are some things to keep in mindabout the use of methotrexate?

What is the most common surgical methodfor EP?

What are the key points in themanagement of EP?

Obstetrics and Gynecology 229

Dilute urine can be false-negative,especially early in pregnancy

Quantitative serum test should bedone

Surgery; Visualization duringlaproscopy; Ultrasound

Determine if there is an intrauterinepregnancy (IUP)

No—should consider heterotopicpregnancy

Less invasive; Wider field of viewand easier orientation; Requires afull bladder; Transvaginal if TA isnot diagnostic

Echogenic adnexal mass; Free pelvicfluid

The level of β-hCG at which an IUPcan be visualized by U/S

β-hCG >6000 mIU/mL

β-hCG >1500 mIU/mL

Methotrexate (MTX)

Inhibits dihydrofolic acid reductase:Interferes with DNA synthesis,cellular respiration, and repair

Surgical tx may be needed if MTXfails; MTX use should be in con-junction with close follow-up

Laparoscopic salpingostomy

Patient should go to the OR ifunstable; Medical approach is pre-ferred to surgery; Alloimmunizationcan occur—give Rhogam

EMERGENCIES DURING EARLY PREGNANCY

What are some factors associated withpregnancy-related death?

Poor prenatal care; Unmarried;Advanced maternal age; Minorityrace

Page 246: Deja review _emergency_medicine__deja_review_

230 Deja Review: Emergency Medicine

Name some leading causes of pregnancy-related death?

Name some common causes of firsttrimester bleeding?

Pulmonary embolism (PE); HTN(i.e., stroke); Hemorrhage

Abortion; Ectopic pregnancy;Gestational trophoblastic disease;Cervical infection

Abortion

What is the definition of spontaneousabortion (SAB) or miscarriage?

What is the most common cause of SAB?

What are some risk factors associated withSAB?

What are some clinical features of SAB?

What is the most common method ofsurgical evacuation in the first trimester?

What is the most common method ofsurgical evacuation in the secondtrimester?

Name the different types of abortion andtheir treatment:

Threatened abortion

Inevitable abortion

Incomplete abortion

Complete abortion

Missed abortion

The loss of pregnancy prior to20 weeks or delivery of a fetus<500 g

Chromosomal abnormalities

Poor prenatal care; Advancedmaternal care; Infections

Vaginal bleeding, cramping, andabdominal pain

Dilation and curettage (D&C)

Dilation and evacuation

Vaginal bleeding with no cervicaldilation; Tx: verify live fetus andbed rest

Vaginal bleeding with cervicaldilation; No expulsion of productsof conception (POC); Tx: surgicalevacuation

Partial expulsion of POC; Tx:typically admit for D&C

Complete expulsion of POC; Tx:none

Death of fetus and retained POC;Tx: surgical evacuation of POC

Gestational Trophoblastic Disease

What is gestational trophoblastic disease(GTD)?

Rare neoplasm of the trophoblasticcells that produce hCG

Page 247: Deja review _emergency_medicine__deja_review_

Obstetrics and Gynecology 231

Name three types of hydatidiform molesfor each description:

Karotype of product is 69XXY due to twosperms that fertilize egg, fetal parts arepresent

Karotype of product is 46XX due tosperm that fertilizes an egg with noDNA, no fetal parts

GTD that becomes malignant,penetrates the myometrium, and canpotentially metastasize

What are some clinical features of GTD?

What diagnostic abnormalities are typicalof GTD?

What are some key points in managementof GTD?

What is an important complication toconsider in GTD?

What are some key points in themanagement of choriocarcioma?

Incomplete mole

Complete mole

Invasive mole

Vaginal bleeding, hyperemesisgravidarum, and HTN

Very high hCG (>100,000), U/S thatshows absence of fetal heart and“snowstorm” appearance

D&C and monitor hCG (shouldtrend down); Also monitor forpossible metastasize (rare); Most dowell after removal

Choriocarcinoma

Chemotherapy that typicallyachieves almost 100% remission

Hyperemesis Gravidarum

What is hypermesis gravidarum (HEG)?

What are some important points toconsider in HEG?

What is an important consideration foranyone who presents with HEG?

What are some key points in themanagement of HEG?

It is excessive nausea and vomitingthat leads to dehydration/electrolyteimbalance

It affects about 2% of all pregnancies;The presence of abdominal pain isunusual; Associated with weightloss and ketosis; Severe cases requireadmission

Gestational trophoblastic disease

NPO and IV fluids; Antiemetics;Refractory cases may requiretermination

Page 248: Deja review _emergency_medicine__deja_review_

232 Deja Review: Emergency Medicine

How is hypertension defined duringpregnancy?

Name four types of hypertension that canoccur during pregnancy?

What are some risk factors that determineHTN in pregnancy?

What is the definition of preeclampsia?

What is believed to be the cause ofpreeclampsia?

What are important diagnostic tests used todiagnose preeclampsia and their typicalfindings:

Blood pressure

Urine protein collection

What are some clinical features ofpreeclampsia?

What is the definition of severepreeclampsia?

What are some other abnormal laboratoryfindings in severe preeclampsia?

What is the definition of eclampsia?

What are the key points in the manage-ment of severe preeclampsia and eclampsia?

It is over 140/90 or a 20 mm Hgincrease in systolic pressure or 10 mm Hg increase in diastolicpressure

1. Chronic hypertension2. Transient hypertension3. Preeclampsia4. Eclampsia

Multiple gestations; Nulliparity;Age >40; Obesity; GTD

It is HTN after 20 weeks withproteinuria

Disturbed blood flow to theplacenta

More than 140/90 (even onereading merits a workup)

Urine protein concentration of0.1 g/L in two random collectionsor 0.3 g/day in a 24-hour collection

Headache, edema, abdominal pain,and visual disturbances

Blood pressure of >160/110 andmore than 5 g/day of protein in theurine

Thrombocytopenia and elevatedliver function tests (LFTs)

It is essentially preeclampsia withthe presence of seizures from 12 weeks to 1 month after delivery

Magnesium sulfate for seizureprophylactic; Control HTN withmethyldopa; Induce labor iffetus/mother unstable; Delivery isdefinitive cure

EMERGENCIES DURING LATER PREGNANCY

Hypertensive Emergencies

Page 249: Deja review _emergency_medicine__deja_review_

What is the definition of abruptioplacentae (placental abruption)?

What are some risk factors associated withplacental abruption?

What are some clinical features of aplacental abruption?

What are some complications of a placentalabruption?

How is placental abruption typicallydiagnosed?

What are the key points in themanagement of a placental abruption?

Obstetrics and Gynecology 233

It is separation of the placenta fromthe uterine wall

HTN; Trauma; Cocaine use;Advanced maternal age; Multiparity

Third trimester bleeding, painfulcontractions, and fetal distress

Fetal or maternal death;Disseminated intravascular coagu-lation (DIC); Hypovolumic shock

U/S

Admit and resuscitation if in shock;C-section if fetus/mother unstable;Induction if stable; Rhogam isindicated

What is the definition of HELLPsyndrome?

Hemolytic anemia, Elevated LFT,and Low Platelets

Abruptio Placentae

Placentia Previa

What is the definition of placentia previa?

What are some risk factors for placentiaprevia?

What are some clinical features of placentiaprevia?

What is important to keep in mind duringan exam?

What are some complications of placentiaprevia?

What are the key points in themanagement of placentia previa?

Implantation of the placenta over thecervical os (total, partial, or marginal)

Multiparity; Advanced maternal age;Smoking

Late pregnancy painless bleeding

Avoid a pelvic exam until an U/S isdone

Preterm delivery; Hypovolemicshock

Resuscitation if in shock; Rhogamwhen indicated; C/S if unstable orfetus is mature

Premature Rupture of Membranes

What is the definition of prematurerupture of membranes (PROM)?

Spontaneous rupture of membranesbefore labor. If it occurs preterm, itis PPROM

Page 250: Deja review _emergency_medicine__deja_review_

How is PROM diagnosed?

What are some key points in the managementof PROM?

234 Deja Review: Emergency Medicine

Gush of fluid, positive pool ferning,or nitrazine test

Induction of labor if failure toprogress in 24 hours and Abx ifchorioamnionitis is suspected(increased WBC count, fever, anduterine tenderness)

What is the definition of preterm labor(PTL)?

What are some risk factors for PTL?

What is the most common cause ofmortality in PTL?

Name some commonly used tocolytics?

What are the purposes of tocolytics?

What are the key points in themanagement of PTL?

Onset of labor prior to 37 weeks

PROM; Infection; Preeclampsia;Multiple gestations; Tobacco use

Lung immaturity

Magnesium sulfate, indomethacin,and terbutaline

Delay labor to allow administrationof steroids for lung maturation

Empiric Abx, hydration, tocolysis,and steroids if fetus less than 34 weeks

What are some important things to knowabout each emergency and their treatment:

DVT/PE

Postpartum hemorrhage

Leading cause of maternal death;Greatest risk is first few weeksafter labor; Commonly will haveSOB, CP, or shock; Tx: heparin orlow molecular weight heparin(LMWH)

Related to one fourth of all post-partum deaths; Most occur withinthe first 24 hours; Consider uterineatony/rupture and inversion; Tx:if rupture = OR; atony = oxytocin;inversion = manual reduction

Preterm Labor

EMERGENCIES DURING POSTPARTUM

Page 251: Deja review _emergency_medicine__deja_review_

Postpartum infection

Peripartum cardiomyopathy

Amniotic fluid embolus

Obstetrics and Gynecology 235

Most common postpartum compli-cation; Fever, tenderness, anddischarge (foul odor); Tx: drainage,debridement, and Abx

Present similar to CHF (DOE, cough,CP); Echo will show massivelydilated chambers; Poor prognosis ifno cause is found; Tx: diuretics andfluid restriction

Very acute onset and high mortality;Typically permanent neurologicalsequelae; Tx: supportive with highO2 and monitor for DIC

What is the definition of vulvovaginitis?

What are some clinical features ofvulvovaginitis?

What are some differentials to consider?

List some important points and treatmentfor each of the following:

Candida albicans

Trichomonas vaginalis

Gardnerella vaginalis

Genital herpes

It is inflammation of the vulva/vagina

Discharge, itching, and odor

Infection; Foreign body; Allergiccontact; Atrophic vaginitis

Dysuria, dyspareunia. and itchingcommon; Wet prep of KOH todetect (shows hypae); Tx: topical-azole drugs or nystatin

High association with gonorrhea; Isalmost always sexually transmitted;Associated with adverse outcomesin pregnancy; Slide prep willshow teardrop trichomonads; Tx:metronidazole

Is almost always sexually trans-mitted; Commonly have malodorousdischarge; Associated with PROMand endometritis; Tx: metronidazole

Commonly caused by HSV-2serotype; Neonatal infection can bedevastating; Commonly have painfululcers; Avoid normal delivery ifactive lesions; Tx: acyclovir orvalacyclovir

VULVOVAGINITIS

Page 252: Deja review _emergency_medicine__deja_review_

Foreign body

Contact vulvovaginitis

236 Deja Review: Emergency Medicine

Very common in children andadolescents; Often have malodorousdischarge; Children tend to inserttissues and objects; Adolescentstend to leave tampons; Can growE.coli/anaerobes if left too long; Tx:remove object

Contact dermatitis due to irritant(i.e., tights); Typically have erythemaand edema; Commonly have super-imposed infection; Tx: R/O infec-tion, remove irritant, and steroidsin severe cases

PELVIC INFLAMMATORY DISEASE

What is the definition of pelvicinflammatory disease (PID)?

What are the two most common causes ofPID?

What is the pathophysiology of PID?

What are some immediate complications ofPID?

What are some long-term complications ofPID?

Name some risk factors of PID.

What are some clinical features of PID?

What is the minimum CDC criteria for thediagnosis of PID?

What are some other diagnostic criteria forPID?

What are some common pelvic examfindings in PID?

What is the name of the condition of RUQtenderness and jaundice in the setting of PID?

It is a wide spectrum of infectionsof the upper female genital tract

1. Neisseria gonococcus2. Chlamydia trachomatis

It is an infection that starts at thecervix and vagina and ascends upthe genital tract

Salpingitis; Endometritis; Tubo-ovarian abscess

Infertility; Chronic pain; Ectopicpregnancy

Multiple sexual partners; History ofSTD; Frequent douching; Sexualabuse

Lower abdominal pain, vaginalbleeding or discharge, dyspareunia,but can also be asymptomatic

Cervical motion tenderness; Lowerabdomen, adnexal, or uterinetenderness

Fever; WBC >10,000/mm3; ElevatedCRP or ESR; Cervical infection withN. gonorrhea or C. trachomatis

Cervical motion, uterus, andadnexal tenderness

Fitz-Hugh-Curtis Syndrome

Page 253: Deja review _emergency_medicine__deja_review_

What are some important points in themanagement of PID?

What are some criteria for admission?

Obstetrics and Gynecology 237

Rule out ectopic pregnancy; Cervicalswab for culture and stain; Empirictreatment for gonorrhea/chlamydia;Patient education

Ovarian abscess; Unable to toleratePO; Peritonitis; Failed outpatientmanagement

CLINICAL VIGNETTES

45-year-old G4P5 who just delivered twinsfollowed by two whole placentas now hascopious vaginal bleeding; PE: 800 cc bloodin 5 minutes with boggy uterus

19-year-old female with no PMH presentsvia EMS with a syncopal episode, patienthas now regained consciouness andmentions she was treated for an STD 2years ago; pelvic: cervical motiontenderness; Labs: positive pregnancy test

37-year-old G2P1 at 10 weeks presents withsevere nausea and emesis along withvaginal bleeding; pregnancy test: β-hCG>100,000 mIU/mL; U/S: no fetal activityand a snowstorm appearance

67-year-old female with PMH of HTN,CAD, and DM presents with painlessvaginal bleeding, but otherwise has noother associated symptoms such as dysuriaor abdominal pain; vaginal exam: nocervical tenderness

19-year-old G0P0 presents with a suddenonset of left sided pelvic pain soon after herbasketball game, aside from a past historyof an ovarian cyst, is otherwise healthy

23-year-old G5P0 at 6 weeks presents withpainless vaginal bleeding, but is otherwisehealthy; pelvic: closed OS

41-year-old G2P1 at 21 weeks presents withheadache as well as lower extremityswelling; PE: BP of 150/95, +1 lowerextremity edema; Labs: significantproteinuria

Uterine atony

Ectopic pregnancy

Gestational trophoblastic disease

Endometrial cancer

Adnexal torsion

Threatened abortion

Preeclampsia

Page 254: Deja review _emergency_medicine__deja_review_

6-year-old female is brought in by hermother for a vaginal malodorousdischarge, but is otherwise healthy

34-year-old female in her third trimesterpresents after an MVC with vaginalbleeding along with painful vaginalcontractions; fetal heart monitoring: latedecelerations

23-year-old female in her postpartumperiod presents with dyspnea and chestpain that she describes as sharp and worseon inspiration; PE: unremarkable

238 Deja Review: Emergency Medicine

Foreign body

Abruptio placentae

Pulmonary embolism

Page 255: Deja review _emergency_medicine__deja_review_

C H A P T E R 1 4

Trauma

239

GENERAL APPROACH

What is the leading cause of death inpeople under the age of 45 in the UnitedStates?

Name the top three trauma-related deaths

What are the three peak times for traumaticdeath and common causes of death for each:

First peak (immediate death)

Second peak (minutes–few hours)

Third peak (days–weeks)

What constitutes the primary survey?

What is the single most importantintervention to perform on all traumapatients at the scene?

What are some techniques to secure anairway on the field?

What is the procedure of choice to securean airway on the field?

What is the most reliable method toconfirm ET placement?

Trauma: 50 million deaths occureach year, half of which requiremedical attention

1. Motor vehicle crashes (MVCs)2. Falls3. Burns and fire-related death

Laceration of the great vessels;Airway obstruction; Massive headinjury; High C-spine injury

Tension pneumothorax; Cardiactamponade; Multiple injuries leadingto hypovolemia; Ruptured spleen;Massive hemothorax

Sepsis; Pneumonia; Multiorganfailure

ABCDE: Airway, breathing circu-lation, disability (neuro), exposure

Airway control with C-spinestabilization

Endotracheal tube; Esophageal-tracheal combi tube; Laryngealmask airway (LMA)

Endotracheal intubation

Visualization of the tube passingthe cords

Copyright © 2008 by The McGraw-Hill Companies, Inc. Click here for terms of use.

Page 256: Deja review _emergency_medicine__deja_review_

Although pediatric airway management issimilar to adults, what are two differences?

What surgical technique can one use ifintubation fails?

What are some methods used to quicklyassess volume status in trauma patients?

What type of access should be done in anytrauma patient?

What is the difference between colloid andcrystalloid fluids?

Are there any advantages of colloids overcrystalloid fluids?

What is the optimal fluid type and amountthat should be used for initial resuscitation?

What is minimal amount of circulatingvolume loss to produce signs of shock?

What is the first sign of hemorrhagicshock?

What is shock?

What category of shock is most common intrauma?

What is the crystalloid to blood replacementratio (mL)?

240 Deja Review: Emergency Medicine

1. Children <9 years; use uncuffedET tube

2. Children <10 years; needle cricpreferred over surgical cric

Needle cricothyrotomy

Skin color, capillary refill, pulse,mental status

Intravenous (IV) access with twolarge-bore IVs for rapid fluidinfusion

Colloid: contains protein such asalbumin and fresh-frozen plasma;Crystalloid: little or no protein suchas normal saline (NS) or lactatedringers

Small amount of colloid can effecta large change in intravascularvolume, crystalloids are just aseffective/cheaper

2 L of lactated ringers or normalsaline

30%

Tachycardia and cutaneousvasoconstriction

Shock is a state where the oxygendemands of the body are not met

Hypovolemic shock (hemorrhage)

3:1

Hemorrhagic Shock Class I Class II Class III Class IV

Blood loss (mL) 0–750 750–1500 1500–2000 >2000Blood volume loss (%) 0–15 15–30 30–40 >40Pulse rate <100 >100 >120 >140Blood pressure Normal Normal Decreased DecreasedPulse pressure Normal Decreased Decreased DecreasedFluid replacement Crystalloid Crystalloid Crystalloid Crystalloid

and blood and bloodMental status Anxious Anxious Confused Lethargic

Page 257: Deja review _emergency_medicine__deja_review_

Trauma 241

Name five potential spaces where life-threatening bleeding can occur?

What clinical index is widely used toassess neurological function?

Name the three components of GCS

What GCS score is indicative of severeneurological impairment?

Which component of the GCS has thehighest prognostic factor?

What are some examples of blunt trauma?

What are some major factors determineseverity of injury in an MVC?

Do lateral impacts or frontal impacts carrya higher mortality in a MVC?

What is the mortality rate of a fall from30 feet?

What is the basic pattern of injury in fallswhere victims land on their feet?

What are some examples of penetratingtrauma?

What are some major determinants of injuryin gunshot wounds (GSW)?

1. Chest2. Abdomen3. Pelvis4. Bilateral femoral fractures5. External wounds

Glasgow Coma Score (GCS)

1. Eye opening2. Verbal response3. Motor response

8 or less—“eight and it’s too late”

Motor response

MVC, falls, assaults, and pedestrian-automobile accidents

Ejection from vehicle; Size andweight of vehicle; Location ofvictim in vehicle; Use of restraints;Direction of impact; Speed of car atimpact

Lateral impacts

50%

Calcaneous fracture; Acetabularfracture; L1-L2 compressionfracture

Guns, knifes, arrows, swords

Mass of projectile; Muzzle velocity;Location and trajectory of projectile

HEAD INJURY

What is the most common cause of deathfrom trauma?

What is the most common mechanism ofinjury?

Central nervous system (CNS)injury

MVC

Page 258: Deja review _emergency_medicine__deja_review_

What are the five layers of the scalp?

What is the thinnest region of the skullthat is most vulnerable to injury?

What are the three layers of meninges?

Name the regions of the brain.

What portion of the brainstem controls thereticular activating system?

What portion of the brainstem controls thecardiorespiratory system?

What is the Monroe-Kellie doctrine?

Why is this significant in head injury?

242 Deja Review: Emergency Medicine

Skin

Connective tissue

Aponeurosis

Loose areolar tissue

Pericranium

Temporal region

1. Dura mater2. Arachnoid membrane3. Pia mater

Cerebrum

Cerebellum

Brainstem

Midbrain

Pons

Medulla

Midbrain; Pons

Medulla

The total volume in the intracranialcompartment is constant

The intracranial space does nottolerate increases in pressure verywell such as tumors, bleeding, orbrain swelling and has limitedability to compensate

What is the threshold of intracranial pressureat which compression or ischemia can occur?

What is the goal for the management ofICP?

20 mmHg

Maintaining ICP less then 20 mmHgand consider the placement ofventriculostomy catheter (can drainand monitor ICP)

Intracranial Pressure

Normal <10 mm HgHigh >20 mm HgSevere >40 mm Hg

Page 259: Deja review _emergency_medicine__deja_review_

What are some indications for ICPmonitoring?

What is the hallmark of brain injury?

Which head-injured patients requirea head CT?

Trauma 243

GCS of less than 8 or abnormal CTsuggest of ICP

Altered level of consciousness

All but the most minor head-injured patients

What is the most important evaluation todo in a person suspected of traumatic braininjury (TBI)?

What are the three categories of TBI andprevalence?

Categories of TBI:

Mild TBI

Moderate TBI

Severe TBI

What physical finding is indicative of TBI?

What are some key points in the managementof TBI?

During a physical exam, what particularfindings should one look for?

What is the initial diagnostic test of choicein the setting of TBI?

What are five key features to look for onhead CT?

What is the period of risk highest forposttraumatic seizure?

Serial GCS evaluation

1. Mild: 80%2. Moderate: 10%3. Severe: 10%

GCS of 13–15 with brief loss ofconsciousness (LOC)

GCS of 9–12 and may be confusedwith possible focal neuro deficits

GCS of 8 or less: can have mortalityup to 40% and most survivors havesignificant disabilities

LOC

Rapidly diagnose any mass lesionsfollowed by evacuation; Treat anyextracranial lesions; Avoid anysecondary brain injuries such ahypotension, hypoxity orhypoglycemia

GCS, pupillary changes, extremitymovement, and ability to answerquestions

Noncontrast CT of the head

“Blood Can Be Very Bad”

Blood

Cistern

Brain

Ventricles

Bone

First week after head trauma

Traumatic Brain Injury

Page 260: Deja review _emergency_medicine__deja_review_

What are the risk factors for a posttrau-matic seizure?

Does anticonvulsant prophylaxis play a role?

What is the general deposition of thosewith mild head injury?

What role does serial neuroassessmenthave in mild head injury?

What factors are considered when decidingif a patient with mild head injury canreturn to play sports?

What is a major risk factor for sustaininghead injuries?

What is a cerebral concussion?

What are some other clinical features ofa head concussion?

What is the typical finding on a head CT?

What is a cerebral contusion?

What are some clinical features of a cerebralcontusion?

What regions of the brain are typicallyinjured in a cerebral contusion?

What are some findings on a head CT?

What is an important delayed complicationof cerebral contusions?

What are some key points in themanagement of cerebral contusions?

244 Deja Review: Emergency Medicine

Cortical contusions, subduralhematoma, penetrating head injury,epidural, and depressed skullfractures

Some recommend that phenytoinbe given in the first week

Most can be safely observed anddischarged if normal neuro function

Patients with mild head injury canstill develop posttraumatic intra-cerebral hematomas and brainswelling

If LOC and amnesia occurred

History of head injuries

Head injury that typically results inbrief loss of neurologic functionsuch as LOC or amnesia

Nausea, vomiting, and confusionthat often resolve rapidly

Usually normal

Similar to a concussion, but withmore pronounced neurologicfindings

More severe neurologic findingssuch as obtundation or coma

Frontal and temporal regions

Lesions at the site of impact (coupcontusion) and site opposite theimpact (contrecoup contusion)

Cerebral hematoma or edema

Typically admit for observation;Monitor for signs of greater intra-cranial pressure; If suspect compli-cation, repeat head CT

Page 261: Deja review _emergency_medicine__deja_review_

What is diffuse axonal injury (DAI)?

What are some clinical features of DAI?

What is the initial CT for patients who endup with DAI?

What are some later CT findings for DAI?

What are some key points in themanagement of DAI?

Trauma 245

Serious diffuse brain injury as aresult of traumatic decelerationfrequently causing a persistentvegetative state in patients

Prolonged coma often with posturingand autonomic dysfunction (poorprognosis)

Normal in most cases

Intraventricular hemorrhage;Hemorrhage within the corpuscallosum; Small focal areas of lowdensity

Admission with neurosurgeryconsultation

Penetrating Head Injuries

Distinguish between high-velocity andlow-velocity injuries.

Is there a difference in prognosis betweenhigh- and low-velocity injuries?

Why are high-velocity injuries moredestructive?

What is the initial treatment for high-velocity injuries to the head?

Injury to which part of the brain carries thehighest mortality?

What is the primary factor that determinesprognosis in low-velocity injuries?

What is the initial management fora protruding object in the head such asknife or arrow?

High velocity: bullets; Lowvelocity: arrows and knives

Yes: high-velocity projectiles carrya very high mortality

Kinetic energy of the projectiledestroys surrounding tissues

IV antibiotics and anticonvulsants

Basal ganglia, brainstem, andposterior fossa

Location of the brain injury

Leave it alone! The risk of hemor-rhage mandates removal in the OR

Skull Fractures

Where do linear skull fractures mostcommonly occur?

What is the most important complication tomonitor in skull fractures?

Temporal bone

Intracranial hematoma

Page 262: Deja review _emergency_medicine__deja_review_

What are the treatment guidelines for thefollowing types of skull fractures:

Open skull fractures

Depressed skull fractures

Linear skull fractures (nondepressed)

Is surgery generally required for depressedskull fractures?

When is surgery typically indicated indepressed skull fractures?

What are the physical findings associatedwith basilar skull fractures?

Why are CSF leaks significant?

Is there a role for prophylactic antibioticuse in CSF leak?

246 Deja Review: Emergency Medicine

Operative intervention

Operative intervention to raisefragment

None

No

Cerebrospinal (CSF) leak or cosmeticpurposes

CSF leak (rhinorrhea/otorrhea);Periorbital ecchymosis (Raccoon’seye); Hemotympanum;Retroauricular ecchymosis (Battle’ssign)

Increased risk of meningitis

It can actually increase mortality(can use in consultation withneurosurgery)

What is the most common artery involvedin a epidural hematoma?

What is the classic clinical scenario for anepidural hematoma?

What are some clinical features of anepidural hematoma?

What is the classic CT finding of anepidural hematoma?

What are some key points in themanagement of an epidural hematoma?

What is the mechanism by which subduralhematomas occur?

What are some groups that are moresusceptible to subdural hematomas?

Middle meningeal artery

Initially LOC followed by a lucidperiod then a coma (only in 1/3 ofcases)

Mass effect on brain: contralateralhemiparesis with a fixed dilatedpupil on the side of the hematoma

Biconvex lesion; Associatedtemporal/parietal skull fracture

Immediate neurosurgical consul-tation; Often requires surgicaldecompression; Consider use ofmannitol to decrease ICP

The bridging veins often tearresulting in intrinsic bleeding andmass effect

Alcoholics; Elderly (smaller brainvolume)

Hemorrhage

Page 263: Deja review _emergency_medicine__deja_review_

What are some clinical features of a subduralhematoma?

What is the classic CT finding of a subduralhematoma?

What are some key points in themanagement of a subdural hematoma?

Trauma 247

Mass effect: range from headacheto lethargy and coma

Crescent-shaped lesion

Immediate neurosurgical interven-tion; Distinguish from chronicsubdural, which may not requireimmediate surgery

NECK TRAUMA

Why are penetrating neck injuries sodangerous?

Name some important structures in the neck:

Vascular

Nerves

Others

What is the mortality rate of a missed neckinjury?

Which muscle of the neck, if not violated,can neck injuries be managed non-operatively?

What is the first concern in any penetratingneck injury?

What are some factors that determine ifa patient should be managed operatively ornonoperatively?

What are some examples of hard signs?

What are some soft signs?

The high density of vascular,neurologic, and visceral structures

Carotid, jugular, vertebral, andgreat vessels

Vagus, phrenic, sympathetic trunk,and cranial nerve (CN) V

Esophagus, trachea, thoracic duct,and lung apices

10–15%

Platysma

Airway injury

Stability, presence of hard signs,and location of the injury (zones)

Stridor, bleeding, and expandinghematoma

Hoarseness, dysphonia, hemoptysis,dysphagia, and odynophagia

Three Zones of the Neck

Zone I Clavicles to cricothyroid membraneZone II Cricothyroid membrane to angle of mandibleZone III Above the angle of the mandible

Page 264: Deja review _emergency_medicine__deja_review_

What mandates exploration?

What is the standard diagnostic approachin a stable patient who has a neck injury?

What are the three most common mech-anisms of blunt injury to the neck?

What are some common causes of airwayloss?

What are some contraindications toorotracheal intubation in neck injury?

What are the clinical features of a missedesophageal injury?

What diagnostic test should be done ina patient who has an abnormal GCS witha normal CT in the setting of a neck injury?

248 Deja Review: Emergency Medicine

Zone II injury with hard signs or anunstable patient

Angiography, EGD/bariumswallow, and tracheobronchoscopy

1. Direct impact (car/all-terrainvehicle)

2. Excessive flexion/extension3. Compression (hanging)

Expanding hematoma, thyroidfracture, tracheal fracture, andaspirations

Obvious pharynx, larynx, tracheal,or facial injury

Fever, tachycardia, and sepsis

Four-vessel angiogram

What potential injuries are associated withan oral-maxillofacial (OMF) injury?

What is the first consideration when doingthe primary survey?

What are some considerations in an oral-maxillofacial injury?

What percentage of OMF injuries domandibular fractures make-up?

What is the most common mechaninsm ofinjury in mandibular injury?

What part of the mandible are mostsusceptible to injury?

How can airway obstruction occur in thesetting of OMF injury

What are the most common maxillofacialinjuries that occur in blunt trauma?

What is the most common physical findingof mandibular fractures?

Cervical injury

Airway obstruction

Search for life-threatening bleedingin the thoracic, abdominal, head,and extremities

2/3

Blunt trauma from assaults

Condylar, angle, and symphysis

Dentures/avulsed teeth andaspiration of blood

Nasal and mandibular fractures

Malocclusion of the teeth

BONY ORAL-MAXILLOFACIAL INJURY

Page 265: Deja review _emergency_medicine__deja_review_

Trauma 249

What are some physical finding of amandibular fracture?

What is important to remember aboutmandibular fractures?

What is the diagnostic test of choice formandibular fractures?

What are some key points in themanagement of mandibular fractures?

What are some common causes ofmandibular dislocations?

What are some clinical features ofa mandibular dislocation?

What is commonly done for a mandibulardislocation?

What is the main reason to obtain an x-rayevaluation?

What areas define the midface?

What is the typical mechanism of injury tothe midface?

What does mobility of the maxillarydentition indicate?

What physical finding is most common inmidface fractures?

What physical maneuver can confirma suspected midface fracture?

What specific exam should be done for anyorbital/zygomatic complex?

What diagnosis is suspected when one findsa firm fixed point of limitation in gaze?

Do anterior or posterior epistaxis bleedmore?

What fractures are CSF leaks associatedwith?

What is the radiographic test of choice formidface fractures?

Malocclusion, trismus, pain, ecchy-mosis of the floor of the mouth,and deviation opening the mouth

Fracture in two or more places >50%

Dental panoramic view (Panorex)

Consultation with ENT forreduction/fixation; Open fracturestypically require antibiotics; Updatetetanus status

Excessive opening of mouth (i.e.,laughing) and trauma

Jaw displaced to unaffected side,difficulty talking/eating, andanterior open bite

Manual reduction

Rule out fractures

Orbital-zygomatic-maxillary complex

Blunt trauma from MVC andassault

Maxillary fracture

Malocclusion

Grab anterior maxillary teeth andcheck for mobility of the hard palate

Check pupils, globes, and visualacuity

Entrapment of extraocular muscles

Posterior

Midfacial, frontal sinus, and basilarskull fractures

CT scan with facial cut

Page 266: Deja review _emergency_medicine__deja_review_

When does osseous healing begin to occur?

What are the four stability points of thezygoma?

What is the general physical finding inzygomaticomaxillary (ZMC) fractures?

What is the goal of the treatment of ZMCfractures?

What is an orbital blowout fracture?

What is the weakest section of the orbitalcomplex?

What are some clinical features of anorbital blowout fracture?

What is the mechanism by which extraoculareye movement dysfunction occurs?

What is the radiographic test of choice?

What are some key points in the manage-ment of an orbital blowout fracture?

What are maxillary fractures commonlydue to?

How are maxillary fractures commonlyclassified?

LeFort I

LeFort II

LeFort III

What are some clinical features ofmaxillary fractures?

What is the preferred imaging modality formaxillary fractures?

What are some key points in themanagement of maxillary fractures?

250 Deja Review: Emergency Medicine

7 days

1. Frontal bone2. Maxilla3. Temporal bones4. Frontozygomatic structure

Depression at the site of trauma,pain on mandibular opening, orlimited opening

Surgical reduction without internalfixation

Fractures of any of the orbital wallssecondary to direct impact of theglobe

The medial wall and floor of theorbit

Enophthalmos, upward gaze palsy,diplopia, pain on eye movement,and V2 parasthesia

Extraocular muscle entrapment

Modified-Waters view

Patients should get ophthalmologyf/u; Persistent entrapment =surgery; Consider antibiotics ifsinus involvement

Direct trauma to the face (largeforce)

Palate-facial

Pyramidal

Craniofacial

Midface mobility, malocclusion,CSF rhinorrhea, and soft-tissueswelling

CT

ABCs; CT to delineate the extentof fracture; Antibiotics if sinusinvolvement

Page 267: Deja review _emergency_medicine__deja_review_

Trauma 251

SPINAL TRAUMA

Name three common mechanisms of spinalcord injury (SCI).

What is the average age and gender ofthose who sustain spinal cord injury?

What is the percentage of patients with SCIwho also have other significant injuries?

What fraction of SCI involves the cervicalspine?

What is the general treatment for spinalcoloumn injury?

Describe the general composition of thespinal column.

Is the thoracic column flexible?

Is the lumbar column flexible?

Why is this important?

What are the three main spinal cordpathways and what fibers are carried?

What is the “three columns of the spine”theory?

Name the boundaries of the three columnsof the spine:

Anterior column

Middle column

Posterior column

1. MVC2. Violence3. Falls

Males with an average age of 30

50%

50%

Treatment centers on preventingfurther injury through fixation(internal or external)

7 cervical vertebrae, 12 thoracicvertebrae, 5 lumbar vertebrae, and5 fused sacral vertebrae

No, it is relatively stiff due to theorientation of facets and interactionwith ribs

Yes

The point where the thoraciccolumn and lumbar column meetcreates a point where shear stressoccurs making T12-L1 a site ofcommon spinal trauma

1. Dorsal column pathway: position/vibration

2. Spinothalamic pathway: pain/temperature

3. Corticospinal pathway: movement

A way to visualize thebiomechanical stability of the spine

Anterior 2/3 vertebral body andanterior longitudinal ligament

Posterior 1/3 of the vertebral bodyand posterior longitudinal ligament

Facets and posterior ligaments

Page 268: Deja review _emergency_medicine__deja_review_

How many of the columns must becompromised in order for the spine to beconsidered unstable?

What is the consequence of an unstablevertebral column?

Does spinal column injury equate to spinalcord injury?

What are some examples of different typesof mechanisms that can cause spinal injury?

What is complete spinal cord injury?

What is incomplete spinal cord injury?

What are some examples of incompletespinal cord injury:

Posterior cord injury

Anterior cord injury

Central cord injury

Brown-séquard injury

What presumption must be made with anytenderness along the spinal column?

For which patient population should onehave a higher index of suspicion for spinalinjury?

What is SCIWORA?

Why is this more common in children?

Why is this more common in the elderly?

When should a cervical spine injury besuspected?

What are the most commonly missedfractures in the cervical spine?

252 Deja Review: Emergency Medicine

2 out of 3

Spinal cord injury with possibleparalysis

Not necessarily

Axial loading; Hyperflexion/extension; Rotational injuries

Irreparable damage with nodiscernible motor, sensory, orelectrical function

Some preservation of sensory and/or motor function

Loss of vibration and position

Loss of bilateral motor, temperature,and pain

Loss of pain and temperature;Mortor loss (arms > legs)

Ipsilateral loss of position/vibration/motor; contralateral loss of pain/temperature

There is vertebral fracture andligamentous injury

Elderly, children, patients withosteoporosis, and history ofmetastatic bone cancer

Spinal cord injury without radio-graphic abnormality

Elasticity of their ligaments

Underlying cervical stenosis

High-speed MVC; Fall >15 feet;Any injury above the clavicle;Diving accidents; Electrical injury

C1-C2 and C7-T1

Page 269: Deja review _emergency_medicine__deja_review_

What is the Nexus criteria?

List the Nexus criteria.

What are the three views recommended toassess cervical injury?

Which view is commonly obtained?

True or False: As long as all cervicalvertebrae are visualized, the film isadequate.

What are the ABCS of assessing lateralfilms?

Alignment

Bones

Cartilage

Soft tissue

When is a CT of the cervical spineindicated?

When is an MRI indicated?

What is a flexion-extension film useful for?

What is a Jefferson fracture?

What is an odontoid fracture?

Type I

Type II

Type III

Trauma 253

It is a set of criteria that help toidentify those patients with a lowprobability of injury to the cervicalspine

Normal alertness; Not intoxicated;No cervical midline tenderness; Nofocal neurologic deficits; No distrac-ting injuries

1. Lateral2. AP3. Open mouth (odontoid view)

Lateral alone is adequate in 90% ofcases

False. C7-T1 must be visualized

Alignment

Bone

Cartilage

Soft tissue

Anterior/posterior/spinolaminarlines

Check vertebral body heights

Intervertebral spaces and facets

Look for soft tissue swelling,especially C2-C3

Inadequate plain films; Fracture onfilms; Unconscious patients

Neurological deficits

A flexion-extension film is typicallyused to assess ligamentous injury

Axial loading injury that results ina C1 burst fracture with C2involvement

Involves the tip of the dens of C2

Transverses the dens at the junctionof the body of C2

Involves C2 vertebral body

Page 270: Deja review _emergency_medicine__deja_review_

Which odontoid fracture carries the worseprognosis?

What is a clay shoveler’s fracture?

What is a hangman’s fracture?

What is the most common site of injury inthe thoracolumbar injury?

When are AP and lateral films indicated?

When is a CT indicated?

When is an MRI indicated?

What is a compression fracture?

What is a burst fracture?

What is a chance fracture?

What are some key points in themanagement of spinal injury?

254 Deja Review: Emergency Medicine

Type II

Avulsion of the spinous process ofC6 – T3 typically the result of flexioninjury or direct trauma

Bipeduncular fracture of C2 due toexcessive extension

T12-L1

If a patient complains of pain in theregion or if the mechanism of injuryis suggestive

If there is a fracture noted on plainfilms, film is inadequate, or patientcannot respond

Neurological deficits

Anterior vertebral body fracture

Vertebral body is crushed in alldirections

Fracture due to excessive flexionsuch as an MVC where a seatbelt isused

Protect the cord by stabilization; CTscan if plain films are indeterminate

THORACIC TRAUMA

What fraction of patients who sustaininjury to the chest require thoracotomy?

What findings are indicative of seriouschest injury?

If a patient with penetrating thoracic injuryloses vital signs in the ED, what procedureis indicated?

If a patient with blunt thoracic injury losesvitals in the ED, would one still do athoracotomy?

What are some primary indications forurgent thoracotomy or sternotomy?

10–25%

JVD, subcutaneous emphysema,and tracheal deviation

Emergent thoracotomy

No—the mortality rate approaches100%

Massive hemothorax; Cardiactamponade; Aortic tear; Esophagealdisruption or perforation; Openpneumothorax

Page 271: Deja review _emergency_medicine__deja_review_

Name the six immediate life-threatsassociated with thoracic trauma.

What are the six potential life-threateninginjuries to the thoracic region?

Trauma 255

1. Airway obstruction2. Tension pneumothorax3. Massive hemothorax4. Open pneumothorax5. Flail chest6. Cardiac tamponade

1. Blunt cardiac injury2. Traumatic rupture of the aorta3. Major tracheobronchial injury4. Diaphragmatic injury5. Esophageal perforation6. Pulmonary contusion

What is the most common cause?

What size is considered a large defect?

What are some clinical features of an openpneumothorax?

Should the wound be fully closed witha dressing?

What is the standard treatment?

Penetrating injuries

>3 cm

Hypoxia; Hypoventilation;Tachypnea; Chest pain

No! It can convert to tensionpneumothorax

Tube thoracostomy on the affectedside

Open Pneumothorax

Tension Pneumothorax

What is the pathogenesis of tensionpneumothorax?

What are some clinical features of tensionpneumothorax?

What immediate action is required fortension pneumothorax?

Where do you insert the needle for needledecompression?

What is the consequence ofdecompression?

Air is able to enter, but not leavethe pleural space

Decreased breath sound on one side;Tracheal deviation (late finding);Subcutaneous emphysema;Hypotension

Needle decompression followed bytube thoracostomy

Second intercostal space mid-clavicular line or fifth intercostalspace in anterior axillary line

Converts tension pneumothoraxinto simple pneumothorax

Hemothorax

What is a common cause of a hemothorax? Damage to the primary orsecondary pulmonary vessels

Page 272: Deja review _emergency_medicine__deja_review_

How much blood can each hemothoraxcontain?

What will the chest x-ray show?

How much fluid is required before anupright CXR can detect it?

What are some clinical features ofa hemothorax?

Do all hemothorax need surgicalintervention?

What are some indications for surgicalintervention for a hemothorax?

256 Deja Review: Emergency Medicine

Upto 3 L

Total opacity of the affected side“white out”

200 mL

Dullness to percussion, diminishedbreath sounds, and decreasedtactile fremitus

No—most are self-limited

Initial chest tube output is >1500 mL;50% hemothorax; Chest tube outputis >200 mL/hour over 4–6 hours

What are some clinical features of a flailchest?

What are some common radiographicfindings in a patient with flail chest?

What is the patient at high risk for?

What is the test of choice?

What are some key points in the managementof a flail chest?

What are some indications to intubate?

Paradoxical movement of the flailsegments with spontaneousbreathing

Two or more consecutive ribfractures with pulmonary contusions

Pneumothorax and hemothorax

CXR (CT more accurate)

Low threshold for ET intubation; Paincontrol; Pulmonary physiotherapy

PaCO2 >55 mmHg; Respiratoryfatigue; PaO2 <60 mmHg

What is cardiac tamponade?

What is the mechanism by which cardiactamponade commonly occurs?

What is the most common site of perforationthat leads to cardiac tamponade?

What is Beck’s Triad?

How common does Beck’s triad present?

Build-up of fluid in the pericardialspace that obstructs effective cardiacpumping

Penetrating injuries

Right atrium

Hypotension; Muffled heart sounds;Jugular venous distension (JVD)

1/3 of cases

Flail Chest

Cardiac Tamponade

Page 273: Deja review _emergency_medicine__deja_review_

Trauma 257

Where is the site where the aorta mostcommonly tears?

What is the most common mechanism bywhich a aortic rupture occurs?

About how many patients who sustain atraumatic aortic rupture die at the scene?

What are some clinical features of a tramauticaortic ruptures?

How is the diagnosis of a aortic ruptureusually made?

What are some findings on a CXR that maybe suggestive of a aortic rupture?

What are two confirmative tests that can beused to help diagnose a aortic rupture?

What are some key points in themanagement of aortic ruptures?

Ligamentum arteriosum

Sudden deceleration (i.e., falls andMVCs)

Up to 90%

Retrosternal pain; Pulse deficits;Dyspnea; Upper extremity hyper-tension with decreased femoralpulses

History is very important, but anabnormal CXR along with confir-mative studies can confirm thediagnosis

Superior mediastinum widening,indistinct aortic knob, rib fractures,left hemothorax, and left apicalpleural cap

1. CT2. Transesophageal echocardio-

graphy (TEE)

Immediate surgical repair; RegulateBP to minimize tear

Tramautic Aortic Rupture

Blunt Cardiac Injury

How does a blunt cardiac injury (BCI)commonly occur?

What is the spectrum of BCIs?

How do myocardial concussions occur?

Commonly occurs in a high-speedMVC where the chest strikes thesteering wheel

Myocardial concussion; Myocardialcontusion; Tamponade; Cardiacrupture

Typically the heart will strike thechest wall with no permanent celldamage

What is a characteristic ECG finding ofcardiac tamponade?

What are some key points in themanagement of cardiac tamponade?

Electrical alternans

Ultrasound can rapidly diagnosetamponade; Pericardiocentesis istemporizing until an open thoraco-tomy can be done in the OR; IV fluids

Page 274: Deja review _emergency_medicine__deja_review_

What are some possible complications ofmyocardial concussions?

What are some key points in themanagement of a myocardial concussion?

What is a myocardial contusion?

What ventricle is more commonly injuredin a myocardial contusion?

What are some commonly used tests todistinguish low-risk from high-riskpatients?

What are some key points in themanagement of a myocardial contusion?

258 Deja Review: Emergency Medicine

Hypotension; Dysrhythmias

Most will resolve without treatment;ACLS for dysrhythmias (i.e., asystole)

More forcible injury to the myocar-dium from impaction against thechest wall

Right ventricle

ECG; Echocardiography

Observation for low-risk patients(normal vitals, asymptomatic, etc.);Admit patients with conductionabnormalities

What is a very common mechanism bywhich a pulmonary contusion occurs?

What is an important point to know abouta pulmonary contusion?

What are some common clinical features ofa pulmonary contusion?

What are some common CXR findings ina pulmonary contusion?

What are some potential complications ofa pulmonary contusion?

What are some key points in themanagement of a pulmonary contusion?

Deceleration (MVCs or falls)

Most common potential lethal chestinjury

Dyspnea, tachycardia, tachypneawith chest wall tenderness

Typically show patchy alveolarinfiltrate to consolidation, usuallywithin 6 hours of injury

Pneumothorax; Pneumonia (mostsignificant)

Adequate ventilation to allowhealing; Low-threshold for intuba-tion; Liberal pain control to allowadequate breathing/coughing

Pulmonary Contusion

Diaphragmatic Injury

Which side of the diaphragm is mostinjured in blunt trauma?

Which side of the diaphragm is mostinjured in penetrating trauma?

What is the operative approach fordiaphragmatic repair?

Left, presumably due to an inherentweakness on that side

Left, since most assailants are right-handed

Celiotomy

Page 275: Deja review _emergency_medicine__deja_review_

Trauma 259

How should any abdominal injury bedivided into?

What are three common causes of blunttrauma?

What are two common causes of penetratingtrauma?

Name three regions of the body to considerin abdominal trauma?

What is the general management foranyone who is hemodynamically unstableor has peritoneal signs?

What are the goals of exploratorylaparotomy?

What are some signs of hypotension?

What are peritoneal signs caused by?

What are some peritoneal signs?

What percentage with hemoperitoneumwill have acute findings?

What is the most important thing to do ina suspected abdominal injury with an initialbenign exam?

What are other factors in a trauma situationthat is associated with abdominal injury?

What is the most commonly injured solidorgan?

Blunt versus penetrating trauma

1. MCV2. Falls3. Assaults

1. Gunshot wounds2. Knives

1. Peritoneal cavity2. Retroperitoneal cavity3. Pelvis

To the OR for laparotomy

Immediate hemostatic control;Control any GI contamination;Operative repair

Tachycardia, obtundation, cool skin,poor capillary refill

Irritation to the peritoneal liningcaused by leaking of blood, bile, orgastric juices

Guarding, rigid abdomen, orrebound tenderness

80%

Serial abdominal exams

Chest injury, pelvic fracture,hypotension, and lap belt contusion

In blunt trauma: spleen; In pene-trating trauma: liver

ABDOMINAL TRAUMA

What are some sequela of a diaphragmaticrupture?

True or False: most diaphragmatic tearswill spontaneously heal.

Herniation of viscous that canresult in SBO, incarceration, andcompression of the heart/lungs(these can present years later)

False: most ruptures will requireoperative repair

Page 276: Deja review _emergency_medicine__deja_review_

What is the most commonly injured holloworgan?

What are the three diagnostic tests toconsider in any trauma to the abdomen?

Diagnostic peritoneal lavage (DPL):

What is it

Indications

Accuracy

Advantages

Disadvantages

Criteria for positive DPL

FAST exam:

What is it

Indications

Accuracy

Advantages

Disadvantages

CT:

What is it?

Indications?

Accuracy?

260 Deja Review: Emergency Medicine

Small bowel

1. Diagnostic peritoneal lavage (DPL)2. Focused abdominal sonography

for trauma (FAST exam)3. CT

Catheter placement in the peritonealcavity to see if there is any initialreturn of fluid. If nothing, place literof warm saline and drain

Hemodynamically unstable withquestionable abdominal injury

Sensitivity and specificity is 95%

Fast, accurate, and inexpensive

Invasive, nontherapeutic rate of20%, inability to pick up retro-peritoneal and isolated diaphrag-matic injuries

10 ml of gross blood; >100 kRBC/mm3; >500 WBC/mm3;Bacteria, bile, and food particles

Use of sonagraphy to rapidly detecthemoperitoneum

Hemodynamically unstable withquestionable abdominal injury

Sensitivity and specificity is between70–90% and poor at detecting solidorgan damage

Fast, accurate, and inexpensive

Poor at detecting solid organ damageand small amounts of blood, requirestraining

CT is used to evaluate solid organinjury and detect fluid/air in cavity

Hemodynamically stable patientsthat require abdominal evaluation

92–98%

Page 277: Deja review _emergency_medicine__deja_review_

Trauma 261

What is the first thing to assess in blunttrauma to the abdomen?

What is the most common abdominalorgan injured in blunt trauma?

If the patient is unstable and has obviousperitoneal signs, what is the next step?

What is the test of choice in a stable patientwith suspected abdomen injury?

What are the major forces involved withblunt trauma?

Name the possible organ injury with thefollowing:

Right lower rib fracture

Left lower rib fracture

Epigastric contusion

Anterior pelvis fracture

ABC! Airway with proper venti-lation and assess hemodynamicstability

Spleen followed by liver

Proceed directly to exploratorylaprotomy

CT

Crushing, shearing, and stretching

Liver and gallbladder

Spleen and left kidney

Duodenum, pancreas, and mesentery

Bladder and urethra

Blunt Abdominal Injury

Penetrating Abdominal Injury

What percentage of those with GSWrequire operative repair?

What percentage of those with knifewounds require operative repair?

What abdominal organ is most commonlyinjured in penetrating injuries?

Is CT useful in GSW?

What percentage of those with anteriorstab wounds have peritoneal violation?

Of those with peritoneal violation, howmany require operative management?

Up to 90%

1/4

Liver

Exploratory laprotomy is diagnosticand therapeutic

2/3

1/2

Advantages?

Disadvantages?

Noninvasive, evaluates solid organinjury, and evaluates retroperitonealinjuries

Expensive, time, variable indetecting hollow viscus injury

Page 278: Deja review _emergency_medicine__deja_review_

What is the cause of most genitourinary(GU) injuries?

What is a key marker of GU injury?

What are the possible locations of GUinjury?

What should be done with macroscopichematuria?

What percentage of renal injuries will haveno hematuria?

Is initial return of blood on catheterplacement concerning?

What should be done with microscopichematuria?

What are some diagnostic tests utilized?

Urethrogram

Cystogram

CT

US

Intravenous pyelogram

What percentage of renal injury is fromblunt trauma?

What percentage of those with blunt renaltrauma will lose a kidney?

What is the general management of thosewith renal trauma that is stable?

What is the indication of operativemanagement?

How common is post-injury hypertension?

What is the cause of most bladder injury?

262 Deja Review: Emergency Medicine

Blunt trauma

Hematuria

Upper: kidney and ureter; Lower:bladder and urethra

Further evaluation

15%

No—is usually catheter-related

Further imaging if mechanism ofinjury is suggestive

In any suspected urethral injury

Important to fully inflate bladder todetect small injuries and done post-void

Test of choice for renal trauma

Useful for detecting renalparenchyma injury

Largely replaced by CT for staging

80%

5%

Nonoperative management

Unstable, hilar/pedicle damage,and significant blood in urine

15%

Blunt trauma

GENITOURINARY TRAUMA

What are some indications for Ex Lap ina knife wound?

What is recommended in a stable patientwith a knife wound?

Hemodynamically unstable, peri-toneal signs, obvious evisceration

Local wound exploration

Page 279: Deja review _emergency_medicine__deja_review_

What percentage of blunt trauma isextraperitoneal?

What are the indications of a cystogram?

What are extraperitoneal injuriesassociated with?

What are intraperitoneal injuries associatedwith?

What is the general treatment for bladderrupture?

How are most extraperitoneal bladderinjuries managed?

What is the cause of most ureteral injury?

What are the diagnostic tests of choice?

What is the general treatment?

What is the cause of most urethral injury?

What are posterior urethral injuriesassociated with?

What are anterior urethral injuriesassociated with?

What is the diagnostic test of choice?

Trauma 263

80%

Gross hematuria; Seatbelt contu-sions; Pelvic fractures

Fractures of superior and inferiorpubic rami

Seatbelt injuries with a full bladder

Ex Lap followed by primary repair

Bladder drainage alone

Penetrating trauma

Intravenous pyelogram (IVP) andCT

Primary repair and stenting

Blunt trauma

Pelvic fracture

Penetrating trauma

Urethrogram

What is a dislocation?

What is a subluxation?

What is a fracture:

Open fracture

Closed fracture

What are some important descriptions forbone fractures?

Match the possible nerve injury:

Anterior shoulder dislocation

Humeral shaft

Total loss of articulation contact

Partial loss of articular congruity

Break (partial or complete) incontinuity of the bone

Fracture that results in opencommunication

Fracture with intact skin

Pattern, morphology, location, openversus closed, and neurovascularstatus

Axillary nerve injury

Radial nerve injury

ORTHOPEDIC TRAUMA

Page 280: Deja review _emergency_medicine__deja_review_

Posterior hip dislocation

Proximal fibular fracture

What percentage of fractures are missed inthose with multiple injuries?

What are important components of thephysical exam?

What is the initial diagnostic test of choice?

What is the initial treatment in anyfracture?

Are antibiotics recommended in openfractures?

What is the purpose of splinting?

Should open fractures be splinted?

What is important to assess after splintingof open fractures?

What is the gold standard of splinting?

What is the mangled severity scoringsystem (MSSS)

What are the primary components of theMSSS?

What is the most important factor whendeciding amputation versus salvage?

What is the primary issue in any openfracture?

What is an important management issue inaddition to antibiotics?

What is the initial treatment for openfractures?

What is a typically antibiotic regiment?

Is operative management indicated foropen fractures?

264 Deja Review: Emergency Medicine

Sciatic nerve injury

Peroneal nerve injury

10–15%

Inspection, palpation, range ofmotion, and neurovascular status

Plain films with at least two views,above and below the injury

Reduction; Splint; Irrigate if open;Update tetanus status

Yes

Immobilization to help controlbleeding, pain, and preventsecondary injuries

Splint as they are

Neurovascular status

Plaster of paris

A scoring system to help guidewhether a severely mangled limbshould be salvaged versusamputated

Skeletal/soft tissue injury; Limbischemia; Shock; Age

Neurologic status

Infection (osteomyelitis)

Adequate debridement

Early irrigation; Early splinting

First generation cephalosporin/aminoglycoside; Penicillin if theinjury is barnyard related; Tetanustoxoid

Yes—take to OR within 6–8 hours

Page 281: Deja review _emergency_medicine__deja_review_

Hip dislocations:

Are anterior or posterior dislocationsmore common?

What is a common cause of posteriorhip dislocations?

What percentage of hip dislocationsresult in sciatic nerve injury?

What is the most concerningcomplication?

What is done to avoid AVN?

Femoral neck/shaft fractures:

What is a common cause of a femoralneck fracture in children/adults?

What is a common cause of a femoralneck fracture in elderly patients?

What is a particular concern?

What is the typical treatment?

What is important to rule-out in femoralshaft fractures?

What is the typical treatment for femoralshaft fractures?

Knee dislocations:

What is a common cause of knee injury?

How often is the popliteal artery injured?

What is typically done to assess thepopliteal artery?

What nerve injuries are typicallyassociated with knee dislocations?

What is the initial management in kneedislocations?

Tibial shaft fractures:

What is a common cause of tibial shaftfractures?

What syndrome are tibial fracturesassociated with?

What is the typical treatment for tibialshaft fractures?

Trauma 265

Posterior

MVC

10–15%

Avascular necrosis (AVN)

Immediate reduction (closed or open)

High-energy impacts (i.e., MVC)

Low-energy impacts (i.e., falls)

AVN

Open reduction internal fixation(ORIF)

Femoral neck fractures

Intramedullary nailing

Any high-force impact

20%

Arteriography

Tibial and peroneal nerve

Urgent reduction

High-energy impacts (i.e., MVC)

Compartment syndrome

ORIF

Page 282: Deja review _emergency_medicine__deja_review_

Pelvic fractures:

What is the primary concern in anypelvic fracture?

How many liters of blood can the pelvisaccommodate?

What do pelvic fractures have a highassociation with?

What is the mortality rate of open pelvicfractures?

What is the mortality rate of majorvascular disruption secondary to pelvicfractures?

What is the initial management insuspected pelvic fractures?

What type of physical exam is importantto perform in a pelvic fracture?

Hand trauma:

What is important to know about handinjuries?

What assumption must be made if thereis a laceration, swelling, and ecchymosis?

What is the Allen test used for?

What is the function of the radial nerve?

What is the function of the mediannerve?

What is the function of the ulnar nerve?

What is compartment syndrome?

What is the common cause of compartmentsyndrome?

What percentages of compartmentsyndrome do fractures account for?

What fractures are highly associated withcompartment syndrome?

What factors are associated withcompartment syndrome?

What is a very common physical findingon exam?

266 Deja Review: Emergency Medicine

Life-threatening bleeding

5 L

Head, thoracic, and abdomentrauma

50%

75% (it is rare)

External fixation of the pelvis

Detailed lower neurovascular exam

It is the most injured part of thebody

Neurovascular damage

To test patency of both the radialand ulnar artery

Extension of the wrist

Flexion of the wrist and oppositionof thumb

Assist in flexion of wrist

A significant increase in pressurewithin a confined space (fascia)

Any injury that leads to swellingwithin a confined space

50%

Tibial fractures

Reperfusion after 4–6 hours ofswelling; Significant crush injury;Combined arterial and venous injury

Pain out of proportion followed byparaesthesia

Page 283: Deja review _emergency_medicine__deja_review_

What are some common signs on exam?

What is the first sign of compartmentsyndrome?

What is a late finding of compartmentsyndrome?

What is the primary treatment forcompartment syndrome?

What is the typical pressure reading forfasciotomy?

What is rhabdomyolysis?

What is the most feared complication ofrhabdomyolysis?

What is the most sensitive marker formuscle damage?

What is the most common cause ofrhabdomyolysis in trauma?

What is the most common cause ofrhabdomyolysis in non-trauma situation?

What is the pathogenesis of rhabdomyolysis?

What is the primary objective in treatment?

What is another concern in rhabdomyolysis?

What is the standard treatment to treathyperkalemia?

What is the prognosis of rhabdomyolysis?

Trauma 267

Swelling with pain on passivestretching

Loss of function

Loss of pulses

Fasciotomy

Greater than 30 mm Hg or 20–30mm Hg with symptoms

It is any type of significant muscleinjury that results in release of toxins

Kidney failure

Serum creatine phosphokinase(CPK)

Anything that causes muscle deathsuch as crush injuries

Neuroleptic malignant syndrome;Malignant hyperthermia

Fe: forms toxic oxygen metabolites;Myoglobin: forms casts to clogrenal tubules

Adequate fluids to ensure renalperfusion

Hyperkalemia

Sodium bicarbonate and insulin todrive potassium into cells; Calciumto stabilize the heart; Kayexalate tobind potassium

Generally good with most patientsreturning to baseline kidney functionin 3–4 weeks

TRAUMA IN PREGNANCY

What are some important points abouttrauma in pregnancy?

Most common cause of nonobstetricdeath; Fundamentally treating twopatients; Management centers aroundmother; “What is good for themother is good for the child”

Page 284: Deja review _emergency_medicine__deja_review_

What are some important caveats about theairway management of pregnant traumapatients?

What are some important points in regardsto circulatory status in pregnant traumapatients?

What is supine hypotension syndrome?

What is the optimal position to laya pregnant trauma patient?

What are some important components ofthe obstetric evaluation?

When is the fetus considered viable?

What is the most common cause of fetaldeath following blunt trauma?

What are some clinical features of placentalabruption?

What is the most important preventativemeasure in MVCs?

268 Deja Review: Emergency Medicine

Continuous 100% oxygen (esp. fetalHb); Pulse oximetry monitoring;RSI as required with normal medica-tions; Thoracostomy at third orfourth ICS

Increased HR/low BP may reflectnormal pregnancy, not shock; Avoidsupine position; LR is preferred overNS; Blood transfusion if failure toimprove after 2 L of crystalloid

When the gravid uterus compressesthe IVC, decreasing preload and COwhen in supine position

Lateral decubitus position

Uterine contractions; Fetal heart rate(ensure between 120–160); Fundalheight and tenderness; Fetal move-ment; Pelvic and rectal examination

Gestational age >24 weeks

Placental abruption

Uterine tenderness, fetal distress,abdominal cramps, and signs ofshock

Properly worn seatbelts

CLINICAL VIGNETTES

18-year-old male is brought over by hisparents due to concern for a head injury aftera football game where the patient ran headfirst into another player, patient mentions he“blacked-out” but otherwise feels fine; PE: nofocal neurologic deficits; CT of head: normal

81-year-old male with a history of afib wasseen in the ED 3 days ago after falling andhitting his head, had a negative CT of thehead at that time, but now is presenting withconfusion; PE: unremarkable neuro exam;CT of head: now shows a crescent-shapedlesion

Concussion

Subdural hematoma

Page 285: Deja review _emergency_medicine__deja_review_

19-year-old male who was at a divingcompetition is brought in by EMS incervical precautions. Patient dove froma very high platform and mentions he couldnot extend his arms in time; Cervical films:C1 ring is fractured in multiple places

23-year-old female involved in a knifefight is being evaluated in the trauma bayand is currently complaining of dyspnea;PE: decreased breath sounds and hyper-resonance to percussion on the right chest

18-year-old female is emergently broughtin by helicopter to the trauma bay to beevaluated for a gunshot wound to thechest, patient is intubated and suddenlybecomes hypotensive; PE: jugular venousdistension and muffled heart sound

You arrive at a scene involving a caraccident, a patient was just extricated andis in obvious respiratory distress withsuspected cervical spine injury. Is nasotra-cheal intubation the procedure of choice?

16-year-old male who was involved ina gang fight and hit squarely in the backwith lead pipe is now complaining of backpain; PE: remarkable tenderness of hisupper back; thoracic plain film: avulsionfracture of the spinous process of T2

57-year-old alcoholic male is brought intothe ED by EMS after being knockedunconscious in a bar fight, patient wasawake and demanding to go home, butnow is unconscious; PE: unremarkable;CT of head: biconvex lesion near thetemporal bone

61-year-old female is brought into the EDby paramedics after being extricated ina high-speed car collision, she is unconsciousand unresponsive; PE: posturing; CT ofhead: widely scattered neuronal damage

34-year-old male is brought in by EMSfrom a high speed MVC where the patientwas extricated and his side passenger wasfound dead; PE: fractured left femur andmultiple scalp lacerations; CXR: fracture ofthe first rib and 9-cm superior mediastinumalong with an indistinct aortic knob

Trauma 269

Jefferson fracture

Simple pneumothorax

Pericardial tamponade

No—orotracheal intubation is stillthe procedure of choice

Clay shoveler’s fracture

Epidural hematoma

Diffuse axonal injury (DAI)

Traumatic aortic rupture

Page 286: Deja review _emergency_medicine__deja_review_

41-year-old female is brought in by EMSafter being hit by a car and was thrown15-ft across the street, patient is currentlyhypotensive and unresponsive to fluids;FAST exam: blood in Morrison’s pouch

17-year-old female with no PMH is cominginto the ED with an injury to her left eye.Patient mentions she was hit squarely inher left eye with a softball and now hasdouble vision; PE: inability for the left eyeto gaze upward; modified Waters view: airfluid level in maxillary sinus

8-year-old female is brought into the ED byher mother after being kicked in the chestby a horse at the ranch, the patient ishaving difficulties breathing and insignificant pain; CXR: frank consolidationon the right lung

19-year-old male is brought into the ED byEMS after a diving accident where thepatient dove head first and lostconsciousness, patient is now in cervicalprecautions and is A&O x4; PE: clear fluidis slowly dripping down his left ear

67-year-old male with no PMH presentsafter an MVC where his chest hit thesteering wheel and is coming into the EDwith complaints of chest pain; PE:tenderness with palpation of the anteriorchest wall; ECG: sinus tachycardia; Labs:normal cardiac enzymes

25-year-old male with a gunshot wound tothe chest is currently being evaluated inthe trauma bay when he suddenly becomeshypotensive and in respiratory distresswith distended neck veins

270 Deja Review: Emergency Medicine

Abdominal injury requiringlaprotomy 15-ft

Orbital floor fracture

Pulmonary contusion

Basilar skull fracture

Myocardial concussion

Tension pneumothorax

Page 287: Deja review _emergency_medicine__deja_review_

C H A P T E R 1 5

EnvironmentalExposures

271

BURNS

What are some important causes of burns?

What are some elements in the history toobtain in a patient who presents withburns?

What is the “rule of nine”?

Head and neck

Each arm

Anterior trunk

Posterior trunk

Each leg

Perineum

Can this be applied to infants and youngchildren?

What are some clinical features to know foreach of the following types of burns:

Superficial (First degree)

Partial thickness (Second degree)

Thermal; Chemical; Radiation;Electricity

Any signs of respiratory distress?;Any toxic substance at the site ofinjury?; Did the burn occur withina closed space?

It is used to estimate the bodysurface area burned, which guidestreatment

9%

9%

18%

18%

18%

1%

No—they have proportionallylarger heads

Confined to superficial layer of skin;Erythema and pain, but no blisters;Sunburn most common cause; Healsin a week (does not scar)

Epidermal and top dermis involved;Blister formation is the hallmark;Thermal liquids most common cause;Heals in 2 weeks (some scarring)

Copyright © 2008 by The McGraw-Hill Companies, Inc. Click here for terms of use.

Page 288: Deja review _emergency_medicine__deja_review_

Full thickness (Third degree)

Musculoskeletal (Fourth degree)

What are some risk factors that makes a burnpatient more predisposed to complications?

What are some important basic managementskills any bystander can utilize?

What are some signs of an endangeredairway in patient with thermal burn?

What are some key points in the initialmanagement of burn patients?

What total body surface area (TBSA) willtypically require aggressive fluidresuscitation?

What is the Parkland’s formula?

How is the Parkland’s formula used in thefirst 24 hours?

What are some ways to measure fluidresuscitation?

What are some important complications toconsider in burns?

272 Deja Review: Emergency Medicine

Epidermal and full dermisinvolvement; Charred with leatherappearance; Full skin and nervepermanently destroyed; Healingwill only occur with grafting/surgery

Involvement of muscle/fascia/bone;Necrosis is common; Melted metalis common cause; Debridement/amputation is common

Immunocompromised; Extremesof ages; Associated head injury;Concomitant inhalation injury(i.e., CO)

Remove patient from area; Also stopburning process; Apply a dry/clean/sterile dressing

Respiratory problems (i.e., stridor);Carbonaceous sputum; Singed hair;Oropharyngeal swelling

Very close monitoring of airway,breathing, circulation (ABCs); Lowthreshold for intubation; Aggressivefluid resuscitation in all but the mostsuperficial of burns

TSBA >20%

Used to calculate the amount offluid to give in the first 24 hours formoderate to severe burns

LR at 4 mL � kg � percentage burnwith the half given over the first 8 hours and the rest given over 16 hours

Heart rate (<100 beats/min); Urinaryoutput (0.5–1 mL/kg/hour);Mentation

Carbon monoxide and cyanidepoisoning; Circumferential burns;Infections (late complication)

Page 289: Deja review _emergency_medicine__deja_review_

Environmental Exposures 273

What are some other managementguidelines to remember with burns?

Where are circumferential burns mostdangerous?

List some important burn-care guidelinesfor minor burns?

What are some commonly used topicalantibacterial agents for minor burns?

Morphine is commonly used forpain control; Prophylactic antibioticsfor select patients; Tetanusprophylaxis; Contact burn centersfor major burns; Escharotomy withcircumferential burns

Thoracic chest (compromise breath-ing); Extremities (compartmentsyndrome)

Debride any lost tissue/brokenblisters; Blisters on sole/palms canbe left as is; Cool compresses forburn area; Remove all jewelry;Topical antibacterial agent; Dischargewith pain medicine and follow-up

Bacitracin; Polymyxin B; Silversulfadiazine

ELECTRICAL, LIGHTNING, AND CHEMICAL INJURIES

Electrical Injuries

What is important to know about electricalinjuries?

What types of electrical injuries are there?

What are some mechanisms of injuries dueto electricity?

What are some factors that contribute tothe severity of electrical injuries?

What are some features of exposure to ACcurrents?

Leading cause of occupation-relateddeath; It is more frequent in malesbetween ages 20 and 40 years; Up to45% of severe electrical injuries arefatal

Low voltage (<1000 volts); Highvoltage (>1000 volts); Lightning

Direct effects of electrical current;Blunt injury (falls, muscle contrac-tions, etc.); Conversion of electricityto thermal energy

Amount of current flowing via thebody; Voltage; Resistance; Type ofcurrent (AC versus DC); Durationof currents

Repetitive stimulation of muscles(spasms); Prolonged contact withelectricity; AC current preventsself-release from source; Vfib mostcommon dysrhythmia

Page 290: Deja review _emergency_medicine__deja_review_

274 Deja Review: Emergency Medicine

What are some features of exposure to DCcurrents?

What is the most common mechanism ofinjury in the following:

Low voltage

High Voltage

Lightning

What is the most common cause of death inelectrical injuries?

What are some important baseline studiesto consider?

What are some other complicationsassociated with electrical injuries?

What are some key points in the managementof electrical burns?

Single muscle spasms (typicallythrown); Increased risk of traumadue to being thrown; Asystole mostcommon dysrhythmia

Working on electrical circuits orappliances; Biting into cords(infants); Electrical weapons (taser)

Conductive object contact withhigh voltage overhead lines

In open field or near a tall object

Cardiac arrhythmias; Respiratoryarrest (paralysis of diaphragm)

CBC/Chem-7/Coag; ECG; U/A;Urine myoglobin; CK-MB; CPK

Burns; Rhabdomyolysis;Myoglobinuria; Autonomicdysfunction; Vascular injuries;Cataracts

Electrical burns treated like thermalburns; Aggressive fluid replacement;Cardiac monitoring in severeinjuries; Monitor for compartmentsyndrome; Also monitor for rhab-domyolysis (i.e., ARF); Tetanusprophylaxis

Lightning Injuries

What are some important points to knowabout lightning injuries?

What are some common mechanisms bywhich lighting can cause injury?

What are some common clinical features ofa lightning strike?

High-intensity bursts of shortduration; Direct current (up to1.5–2 billion volts!); Rarely causesdeep tissue burns; Fluid loss israrely an issue

Thermal burns; Blunt trauma fromblast impact; Direct lightning strike;Lightning strikes nearby object

Missing clothes/shoes, stunned,evidence of burns (not always),unconsciousness, headache, vision/hearing problems, and often havemild tachycardia/ hypertension

Page 291: Deja review _emergency_medicine__deja_review_

What are important injuries to consider inthe following organ systems:

Central nervous system

Cardiovascular system

Eyes and ears

What are some important laboratory anddiagnostic tests to consider?

What are some key points in themanagement of lightning injuries?

Environmental Exposures 275

Seizures; Loss of consciousnesswith amnesia; Peripheral nervedamage

Dysrhythmias (systole most com-mon); Pericardial tamponade;Respiratory arrest

Ruptured tympanic membrane iscommon; Corneal damage; Cataractformation

CBC/Chem-7/Coag; Cardiacenzymes; U/A; ECG; Cervical filmsfor suspected spinal injury; CT foraltered mental status

ABCs; Treat lightning burns likeregular burns; Tetanus prophylaxis;Patients should be admitted withcardiac monitoring

What are some important points for thefollowing types of chemical burns:

Acids

Bases

What are some factors that determine theseverity of an acid/base burn?

What are some commonly encounteredacids?

What are some commonly encounteredbases?

What are some diagnostic tests to considerin chemical burns?

Acids are proton donors;Coagulation necrosis by denaturingproteins; Acid burns are typicallymore superficial

Bases are proton acceptors; Severeinjury (i.e., liquefaction necrosis);Bases tend to penetrate deeper intotissue

Length of contact of the agent; pHof the agent; Concentration of theagent; Volume of the agent

Hydrochloric acid; Sulfuric acid;Hydrofluoric acid

Sodium hydroxide; Ammonia;Sodium and calcium hypochlorite

Usually none in minor chemicalburns; CBC/Chem-7/Coag in severeburns; Endoscopy for ingestions;CXR for ingestions as well

Chemical Injuries

Page 292: Deja review _emergency_medicine__deja_review_

What is near-drowning?

What are some important points to knowabout near-drowning?

What are some risk factors of near-drowning?

What are some complications of near-drowning?

What is important about hypothermia inthe setting of near-drowning?

What are some major pulmonarycomplications?

What are some major neurologiccomplications?

What are the most common arrhythmias?

What is the major factor in death due todrowning?

What are some poor prognostic factors innear-drowning?

What are some pre-hospital managementissues?

276 Deja Review: Emergency Medicine

Survival after suffocation in a liquidmedium

Common cause of accidental death;Incidence highest in males betweenages 1 and 5 years; Drowning ismuch more common in summer

Inability to swim; Use of illicit drugsor alcohol; Poor adult supervision;Risk-taking behavior

Hypothermia; Acute respiratorydistress syndrome; Bradycardia;Hypoxia

It has a neuroprotective effect whichmay allow prolonged resuscitationwithout permanent sequelae

Surfactant washout; Adult respiratorydistress syndrome (ARDS);Pulmonary edema

Cerebral edema; Hypoxia; Seizure

Atrial fibrillation; Sinus bradycardia

Cerebral hypoxia

Submersion >10 minutes; Timeto CPR >10 minutes; Water temp>10 C°; GCS <8; Resuscitation>25 minutes

CPR; Possible cervical injuryshould be suspected; Pulses aredifficult to palpate in hypothermia;Remove wet clothing; Considervarious rewarming techniques

What is the general treatment for spilledchemicals?

Copious irrigation is the mainstay;Wipe off any dry chemicals prior toirrigation; Alkaline burns requirelonger irrigation; Ocular involvementalso copious irrigation

NEAR-DROWNING

Page 293: Deja review _emergency_medicine__deja_review_

What are some in-hospital managementissues?

Environmental Exposures 277

Treat organ-specific damage;Prevent secondary neurologicdamage; Correct fluid/electrolyteimbalance; Permissive hypercapniato avoid barotrauma

HYPOTHERMIA

What are the classifications ofhypothermia?

Mild hypothermia

Moderate hypothermia

Severe hypothermia

What is the physiological response tohypothermia?

Is a standard thermometer useful tomeasure the degree of hypothermia?

What is the most reliable method tomeasure temperature in hypothermia?

What are other less invasive methods tomeasure temperature in hypothermia?

What are some causes of hypothermia?

What are some clinical features ofhypothermia:

Mild hypothermia

Moderate hypothermia

Severe hypothermia

What are some complications of hypother-mia?

Core temperature 32–35°C

Core temperature 28–32°C

Core temperature below 28°C

Shivering; Increased adrenalactivity; Increased thyroid activity;Peripheral vasoconstriction

No—cannot measure below 34.4°C

Pulmonary artery probe

Rectal probe, tympanic membraneprobe, and bladder probe

Environmental exposure;Malnutrition; Sepsis; Medications(i.e., general anesthetics);Hypothyroidism; Hypopituitarism

Shivering; Hypertension; Confusion;Atrial fibrillation; Tachycardia

Decreasing level of consciousness;Loss of shivering mechanism;Bradycardia; Cold diuresis; Dilatedpupils

Coma; Oliguria; Asystole at <20°C;Pulmonary edema

Lactic acidosis; Bleeding diathesis;Rhabdomyolysis; Bladder atony;Frostbite

Page 294: Deja review _emergency_medicine__deja_review_

What are the characteristic ECG findings ofhypothermia?

What are some examples of passive externalrewarming?

In what degree of hypothermia is passiveexternal rewarming used?

What are some advantages of passiveexternal rewarming?

What is a disadvantage of passive externalrewarming?

What are some examples of active externalrewarming?

At what degree of hypothermia is activeexternal rewarming used?

What are some advantages of active externalrewarming?

What is a disadvantage of active externalrewarming?

What are some examples of active internalrewarming?

At what degree of hypothermia is activeinternal rewarming used?

What are some advantages of activeinternal rewarming?

What is a disadvantage of active externalrewarming?

278 Deja Review: Emergency Medicine

Prolongation of all intervals;Osborne wave (J-point elevation)

Blankets; Humidified heatedoxygen by mask; Removing wetclothing

Mild hypothermia

Intense monitoring is not needed;Noninvasive

Slow process

Radiant heat; Electric heat blanket;Warm bath

Mild and moderate hypothermia

Intense monitoring is not needed;Noninvasive; Can be combinedwith passive external rewarming

May cause iatrogenic burns

Warmed intravenous fluids;Peritoneal dialysis; Extracorporealblood rewarming; Closed thoraciclavage

Moderate and severe hypothermia

Fastest modality to raise coretemperature; Most effective; Can beused if hemodynamically unstable

Invasive; Intense monitoring

HYPERTHERMIA

What is hyperthermia?

What are four ways that the body losesheat?

It is an elevation of core temperatureabove 37°C due to failure of thermo-regulation

1. Convection2. Conduction3. Radiation4. Evaporation

Page 295: Deja review _emergency_medicine__deja_review_

What are some important causes ofhyperthermia?

What are some risk factors forhyperthermia?

Describe the types of heat stroke

Classic heatstroke

Exertional heatstroke

What are some common findings in thevarious types of hyperthermia:

Heat exhausation

Heat stroke

Malignant hyperthermia

Neuroleptic malignant syndrome

What are some complications of the varioustypes of hyperthermia:

Heat stroke

Malignant hyperthermia

Neuroleptic malignant syndrome

What are the key points in management ofhyperthermia?

Environmental Exposures 279

Heat stroke; Malignant hyperther-mia; Neuroleptic malignantsyndrome; Drugs (i.e., cocaine);Metabolic (i.e., DKA)

Poor physical fitness; Obesity; Druguse; Dehydration

Occurs commonly in elderly and thesick; Compromised thermoreg-ulation; Cardiovascular andendocrine disorders

Common in young athletes;Typically massive exogenous heat;Exertional heat production

Mild hyperpyrexia; Nausea andvomiting; Signs of dehydration

Temperature >105.8�F; Tachypnea;Rales; Excessive bleeding

Muscle rigidity; Hypercarbia; Sinustachycardia; Marked hyperthermia

Altered mental status; Autonomicinstability; Muscle rigidity;Hyperthermia

Renal and hepatic failure; Acuterespiratory distress syndrome;Disseminated intravascularcoagulation; Seizures

Rhabdomyolysis; Disseminatedintravascular coagulation;Hypertension; Hyperkalemia

Dysrhythmias; Pulmonary edema;Renal failure

Lower core temperature to less then38.8�C; Accurate core temperaturemeasurements; Dantrolene in malig-nant hyperthermia; Bromocriptinefor neuroleptic malignant syndrome(NMS); Treat metabolicderangements

Page 296: Deja review _emergency_medicine__deja_review_

When is the optimal time to begin cooling?

What are some techniques in cooling?

What are important points in patienteducation to avoid hyperthermia?

280 Deja Review: Emergency Medicine

Immediately within the hour—golden hour

Immersion therapy (ice water bath);Evaporation (cool spray withfanning); Cold pack to axillary areasand groin

Lifestyle change (i.e., limit drug use);Caution when in hot weather; Waysto keep cool

ALTITUDE SICKNESS

What are some examples of altitudesickness?

Define various heights of altitude:

High

Very high

Extremely high

What are some important points to knowabout AMS?

What are some clinical features of AMS?

What are some risk factors for AMS?

What is the most effective method to avoidAMS?

What are some key points in the manage-ment of AMS?

Name two commonly used drugs that helpprevent AMS.

Name some other medications used forAMS?

Acute mountain sickness (AMS);High-altitude pulmonary edema;High-altitude cerebral edema

8,000–12,000 feet

12,000–18,000 feet

>18,000 feet

Mild form of altitude sickness; Itcan occur at altitudes >6,000 feet;Three-fourth experience AMS at10,000 feet

Headache (most common); Dyspnea;nausea; Edema; Insomnia; Decreasedurine output

History of altitude sickness; Rate ofascent; Duration of stay at highaltitude; Actual elevation

Slow ascent

Never ascend with symptoms ofAMS; Stop ascend or descend ifsymptomatic; Most cases are self-limiting; Low-flow oxygen

1. Acetazolamide2. Ginkgo biloba

Dexamethasone; Promethazine;Prochlorperazine

Page 297: Deja review _emergency_medicine__deja_review_

What are some methods to help preventAMS prior to the ascend?

What are some important points to knowabout in high-altitude pulmonary edema(HAPE)?

What are some clinical features of HAPE?

What are some key points in the managementof HAPE?

What is high-altitude cerebral edema(HACE)?

What are some clinical features of HACE?

What are some key points in its management?

Environmental Exposures 281

First camp at <8,000 feet; Avoiddirect ascend >9,000 feet at one time;Well-hydration; Avoid narcotics,EtOH, and sleeping medicines;Pretreatment medication

Major cause of death in altitudesickness; More common in ascentsabove 12,000 feet; Sudden presen-tation common; Children are moresusceptible; More common in fityoung climbers

Dry cough, dyspnea, fatigue,tachycardia, chest tightness, andperiodic breathing

Descend as soon as possible;Supplemental oxygen; Nifedipineprior to ascend; Descent in severecases; Portable hyperbaric chamberuse

Believed to be hypoxic-inducedincrease in cerebral blood flowalong with decreased integrity ofthe blood-brain barrier

Ataxia (most common); Decreasemental status; Papilledema; Retinalhemorrhage; Seizure; And rapiddeath from brain herniation issevere cases

Slow ascent whenever possible;When it occurs, immediate descent;Dexamethasone may be effective

DIVING INJURIES

What are some important points to knowabout diving injuries?

What are some specific elements to obtainin a diving history?

There are more then 1k divinginjuries per year; Up to 10% ofdiving injuries are fatal

Activities prior to diving (esp. flying);Location (i.e., ocean); Dive times;Equipment used and gases breathed;Maximum depth, time spent, andrate of ascent; Dive problems

Page 298: Deja review _emergency_medicine__deja_review_

What are some complications associatedwith diving injuries?

What is the most common form of divinginjury?

What is barotrauma?

Name some examples of barotrauma.

What is one of the most feared compli-cations of diving?

What are two serious sequelae of AGE?

What are some clinical features of AGE?

What are some key points in the managementof AGE?

What is another feared complication ofdiving?

What is DCS?

What are two groups of DCS?

What are some clinical features of Type 1DCS?

What are some key points in the managementof Type 1 DCS?

What are three forms of Type 2 DCS?

What are some clinical features of cerebralDCS?

What are some clinical features of spinalDCS?

282 Deja Review: Emergency Medicine

Hypothermia; Submersion injuries(drowning); Decompression sickness;Nitrogen narcosis; Barotrauma

Barotrauma

It is injury in air-filled spaces dueto under-pressurization or over-pressurization during descent orascent, respectively

Pulmonary barotraumas;Pneumomediastinum;Pneumothorax; Ear barotraumas

Air gas embolism (AGE)

1. Myocardial infarction2. Stroke

Dysrhythmia; Arrest; Change inmental status; Visual disturbances

100% oxygen; Recompressionchamber; Ground transport tochamber

Decompression sickness (DCS)

It is the release of bubbles fromsolution due to rapid reduction inpressure. Typically nitrogen bubblesare produced

1. Type 1 DCS (musculoskeletal)2. Type 2 DCS (neurologic)

Pain to the arms or legs that rangesfrom mild discomfort to severe painor may present as pruritus alone

Recompression; Watch for theprogression to Type 2 DCS

1. Cerebral DCS (common inaviators)

2. Spinal DCS (common in divers)3. Pulmonary DCS

Seizures, visual disturbances (blurry,diplopia, etc.), and hemiplegia

Paresthesia, bladder paralysis, andincontinence

Page 299: Deja review _emergency_medicine__deja_review_

What are some clinical features ofpulmonary DCS?

What are some key points in themanagement of Type 2 DCS?

Environmental Exposures 283

Cough, dyspnea, and chest pain

Reduce size of bubbles via recom-pression 100% oxygen to wash outnitrogen; Admission for observa-tion; Further recompression if newsymptoms

BITES

Cat Bites

What is the most common organism in catbites?

What are some other organisms that arecommonly associated with cat bites?

What population demographics are mostcommonly bitten by cats?

What are some important elements in thehistory to obtain with regards to cat bites?

What are some important aspects of thephysical to focus on?

What are the essentials in the treatment ofcat bites?

What are some key points with respect towound closure?

What are some indications for antibioticprophylaxis?

What are some common antibioticregiments used?

What are two other importantconsiderations?

What is cat-scratch disease?

Pasteurella multocida

Fusobacterium; Staphylococcus;Actinomyces

Elderly women (men are mostcommonly bitten by dogs)

Domestic or wild cat; Vaccine status;Tetanus status of patient

Neurovascular status; Bony injury;Tendon involvement; Joint spaceinvolvement; Foreign bodies

Inspection of the wound;Debridement; Irrigation; Closurewhere indicated

Puncture wounds should be leftopen; Facial wounds can be closedprimarily; Most other delayedprimary closure

Immunocompromised patient;Hand wounds; Puncture wounds;Prosthetic valves

Ampicillin-clavulanate; TMP/SMX;Ciprofloxacin

Rabies and tetanus status

Regional lymphadenitis of the armsor legs that is typically unilateraland commonly affects children andadolescents

Page 300: Deja review _emergency_medicine__deja_review_

What is the causative agent in cat-scratchdisease?

What is the typical incubation period?

What is the treatment for cat-scratchdisease?

284 Deja Review: Emergency Medicine

Bartonella henselae

Roughly 1 week

Most cases are self-limited and maytake months to resolve, severe casesmay require antibiotics

What are some common pathogensinvolved with dog bites?

Which organism can potentially causea lethal infection in immunocompromisedpatients?

What are some clinical features of patientswho may be infected with Capnocytophagacanimorsus?

What are the key points in the managementof recent lacerations (<12 hours)?

What is typically done for dog bites thatare older than 12 hours or on the hand?

What else is important to consider withdog bites?

What are some indications for antibioticprophylaxis with dog bites?

What are some commonly used antibioticregiments?

What are some indications for admissionand use of IV antibiotics?

Pasteurella; Klebsiella; Streptococcus

Capnocytophaga canimorsus

Sepsis; Acute renal failure;Endocarditis

Inspection; Debridement; Irrigation;Closure

Left open after irrigation then closed3–5 days after (delayed primaryclosure)

Rabies; Tetanus

Same as cat bites

Ampicillin-clavulanate; TMP/SMX;Ciprofloxacin

Injury to tendons, bones, and joints

Systemic infections

Dog Bites

Human Bites

What are the three most commonorganisms involved with human bites?

What are some important points to keep inmind about the wound care of human bites?

S. aureus; Streptococcus; Fusibacterium

Inspect the wound carefully(i.e., tooth); The surrounding skincleansed thoroughly; X-ray hand torule out fractures and fracture belt(FB)

Page 301: Deja review _emergency_medicine__deja_review_

What particular area of the body should beleft open initially?

What areas of the body can typically besutured?

What are some high-risk features whereantibiotics may be indicated?

What are some commonly used antibioticregiments?

What are some indications for the use of IVantibiotics?

What are some other issues to consider inhuman bites?

Environmental Exposures 285

Hands

Face, head, and neck

Immunocompromised patient; Areawith poor blood supply; Handwounds

Second or third-generationcephalosporin; Macrolide withclindamycin or TMP-SMX

Obvious infection (older wound);Tendon, bone, and joint spaceinvolvement; Patient showing signsof systemic infection

Tetanus; HIV (although rarelytransmissible)

Snakes

What are two families of snakes thataccount for the majority of venomoussnake bites in the United States?

What are some important points in thefield management of snake bites?

What are some characteristics of venomousrattlesnakes?

What are some characteristics of venomouscoral snakes?

What are some methods not recommendedfor snake bites (were commonly used)?

About what percentage of venomous snakebites result in significant envenomation?

What are some complications of snakebites?

What are some other issues to consider aswith all bites?

1. Elapidae family (i.e., coral snakes)2. Crotalidae family (i.e., rattlesnake)

Injured area should be immobi-lized and raised above the heart;Thoroughly clean the wound;Attempt to identify the snake;Immediate transportation tohospital

Triangular head; Elliptical pupils;Retractable fangs

Their banding pattern “red andyellow; kill a fellow...red and black;friend of jack”

Mechanical suction devices; Incisionand oral suction; Tourniquet; Ice

3/4

Coagulopathy; Thrombocytopenia;Local tissue damage; Neurotoxicity(i.e., respiratory depression)

Update tetanus status; Snakevenom is sterile

Page 302: Deja review _emergency_medicine__deja_review_

What are some commonly encounteredsnakes within the Crotalidae family?

What are the most prominent clinicalfeatures of bites from within the Crotalidaefamily?

What is the treatment of choice forCrotalidae bites?

Should all patients with bites from snakeswithin the Crotalidae family be admitted?

What is the most prominent clinical featuresof bites from within the Elapidae family?

What are the earliest clinical features ofenvenomation from within the Elapidaefamily?

What is the most feared complication ofbites from within the Elapidae family?

What is the treatment of choice for Elapidaeenvenomation?

What is the most frequent reaction for bothantivenins?

How many hours can the effects ofElapidae envenomation be delayed up to?

Should all patients with bites from withinthe Elapidae family be admitted?

286 Deja Review: Emergency Medicine

Copperhead; Rattlesnakes; Cotton-mouth

Local tissue destruction andswelling; Thrombocytopenia;Coagulopathy

Polyvalent Crotalidae ovine immuneFab (i.e., Crofab)

Patients with no signs of enveno-mations and normal laboratoryvalues can usually be dischargedafter 10–12 hours

Neurotoxicity (i.e., total paralysis)

Cranial nerve dysfunction; Mentalstatus change

Respiratory arrest

Horse serum based antivenin

Serum sickness (delayed up to2 weeks), but it is far more commonin horse-based antivenin

12 hours

Yes

RABIES

What is rabies?

What is the causative agent of rabies?

What is the most common source of rabies?

What wildlife animals are major reservoirsof rabies?

It is a viral infection of the CNSwith an incubation period of up to2 months that is transmitted via thesaliva of infected animals

Rhabdovirus

Wildlife animals (i.e., bats), not verycommon in domestic animals

Raccoon; Skunk

Page 303: Deja review _emergency_medicine__deja_review_

What are some clinical features in each ofthe following stage of rabies:

Incubation

Prodrome

Acute neurologic syndrome

Coma and death

How many patients with rabies who havenot received the rabies vaccine survive?

What are some clinical features ofencephalitic involvement?

What are some basic wound caremanagement issues?

What are things to know about vaccineselection for rabies?

What should be done with the wild animalthat bit the patient, if captured?

Environmental Exposures 287

Incubation from 1–3 months;Canincubate for up to 7 years; Bitescloser to brain progress faster

Nonspecific flu-like symptoms;Ranges from a few days to a week;May get pain and pruritus at bite site

Encephalitic or paralytic presen-tation; Lasts for 2–7 days

Generalized flaccid paralysis;Respiratory and vascular collapse;Most die within 2 weeks once comasets in

Only one has ever survived

Persistent fever; Painful pharyngealor inspiratory spasms; Seizures;Hyperactivity

Thoroughly clean the wound;Tetanus prophylaxis if needed;Rabies vaccine as indicated

Active immunization for bites fromanimals in a suspected group withHDCV; Passive immunization withIG for bites from animals with rabieswith HRIG; Typically both will beused postexposure

Sacrificed and tested

TETANUS

What is the pathogen responsible fortetanus?

Can the pathogen enter healthy tissue?

What is the pathophysiology of tetanus?

Clostridium tetani

No—it requires an anaerobicenvironment such as a wound

Toxins are released after the sporesconvert to vegetative forms in ananaerobic environment, whichprevents release of inhibitoryhormones (spinal cord) and resultsin generalized spasms

Page 304: Deja review _emergency_medicine__deja_review_

What are some important risk factors forthe development of tetanus?

Although rare in developed countries,what are some high-risk groups fortetanus?

What is the incubation period for tetanus?

What are some forms of tetanus?

What is the common presenting symptomof tetanus?

What are some other clinical features oftetanus?

What is local tetanus?

What is cephalic tetanus?

What is neonatal tetanus?

What are some other important diagnosisto consider in patients who present withgeneralized spasms?

What are some key points in themanagement of tetanus?

What are some wounds that predispose totetanus?

288 Deja Review: Emergency Medicine

Devitalized tissue; Any injury withinoculation of the spores; Coinfectionwith other bacteria; A foreign body

Elderly; Intravenous drug abusers(IVDA); Patients with dentalinfections; Diabetic patients withinfected ulcers

Ranges from a few days to months

Generalized—most common form;Neonatal; Local; Cephalic

Trismus (“lockjaw”)

Tonic and periodic muscular spasmsthat are generalized and often resultin periods of apnea with no impair-ment of consciousness

Tonic/spasmic muscular contractionthat is confined to one extremity orregion that often progresses to gene-ralized form

Typically in patients with headinjuries involvement of the cranialnerves, usually the facial nerve andoften progress to generalized form

Common cause of neonatal death indeveloping countries due to aseptichandling of umbilical stump whichleads to spasms, seizures, and death

Infections (i.e., meningitis);Hypocalcemic tetany; Dystonic reac-tions to neuroleptics; Drug with-drawal (i.e., narcotics); Strychninetoxicity

ABCs—esp airway; Spasms can bemanaged with benzos; Surgicaldebridement for wounds; Givehuman tetanus immune globulin

Burns; Penetrating wound;Contaminated wounds

Page 305: Deja review _emergency_medicine__deja_review_

What are some things to keep in mind inregard to tetanus prophylaxis?

Environmental Exposures 289

Offer to all patients if have notbeen immunized within 10 years;Immigrants and elderly patientsshould get the complete vaccineseries if not sure

What are the three major concerns for anyinsect bite or sting?

What is the most common reaction frominsect bites?

What are some common insects thatbite/sting humans?

Name three insects that are commonlyassociated with systemic allergic reactions?

1. Anaphylaxis 2. Upper airway obstruction3. Toxic reactions from multiple

stings

Local inflammatory reaction

Ants; Bees; Wasps; Spiders

1. Horseflies2. Blackflies3. Deerflies

INSECT BITES

Spider Bites

How many poisonous species of spidersare there in the United States?

Does this mean all spiders are dangerous?

What are the most dangerous species ofspiders in the United States?

What type of spiders are becoming popularas pets and although rarely bite, have a badreputation as being aggressive?

Are Tarantula bites poisonous?

What are three reactions to spider bites?

What are some clinical features of localreactions to spiders bites?

Over 2500

No—most are either too small orare unable to penetrate skin

Loxosceles species (Brown spiders);Latrodectus species (Black widow);Agelenidae and Atrax species

Tarantulas

No—although they do have uticarialhair that can induce local reactions/anaphylaxis

1. Local reactions2. Systemic reaction3. Allergic reaction

Commonly have fang markingswith redness with no blisters if it isnon-necrotizing and will last abouta week

Page 306: Deja review _emergency_medicine__deja_review_

290 Deja Review: Emergency Medicine

What is the primary feature of necrotizingwounds from spiders bites?

What are some species of spiders thatcharacteristically cause necrotic lesions?

What are some clinical features of systemicreactions to spider bites?

What are some important things to knowabout black widow spiders?

What are some clinical features of a localreaction from a black widow spider bite?

What are some clinical features of a systemicreaction to a black widow spider bite?

What is latrodectism?

What are some key points in themanagement of black widow spider bites?

What role does antivenin play in thetreatment of black widow spider bites?

What are some indications for the use ofantivenin in black widow spider bites?

What are some important things to knowabout brown spiders?

What is viscerocutaneous loxoscelism?

Characterized initially by rednesswhich expands upto 14 cm, followedby a blister which forms withina day and ulcers that can leavenecrotic tissue

Loxosceles species (brown spiders);Chiracanthium

Myalgias, fever, fatigue, and canrarely cause hemolysis andcoagulopathy

Located in warm regions of the earth;The female is far more poisonous;The poison is a potent neurotoxin

Dull muscle crampings that oftenwax and wane, chest pain (due toradiation from upper extremity),rectal spasms, and can even mimican acute abdomen

Primarily neurologic: hypertension,diffuse pain, tachycardia, profusesweating, and difficulty in speaking

It is a systemic reaction to blackwidow spider bites that results innausea, emesis, and severe spasmthat can result in death

Thoroughly clean wound with soap/water; Tetanus status update;Observation for 4 hours if a blackwidow; Narcotics for pain is themainstay; Benzos for severe musclespasms; Nitroprusside for severehypertension

Not commonly used—bites fromblack widows are rarely fatal

Patients with refractory pain;Pregnant; Severe hypertension

Live in human dwellings; Hideduring the day in various spots(cracks); Distributed in theMidwest and south central regionof the United States

It is a systemic response to a brownrecluse spider bite that results insevere intravascular hemolyticsyndrome

Page 307: Deja review _emergency_medicine__deja_review_

Environmental Exposures 291

What is the feared complication of brownspider bites?

What are some key points in themanagement of brown spider bites?

Local tissue necrosis that mayrequire surgical correction

Thoroughly clean wound withsoap/water; Tetanus statusupdated; Use ice to help decreaseinflammation; Consider use ofDapsone to treat local effects—although used historically

Tick Bites

What are some important tick-transmittedhuman diseases?

What is the infecting organism in RockyMountain spotted fever (RMSF)?

What is the tick that commonly spreadsR. Rickettsii?

What is the incubation period of RMSF?

What is the peak incidence of RMSF?

Is the disease most commonly reported inthe rocky mountain region (i.e., Montana)?

What is clinical hallmark of RMSF?

What are some other clinical features ofRMSF?

What is the diagnostic test of choice forRMSF?

Are there tests you can use to detectRMSF?

What is the most common cause of fatalityin RMSF (although deaths are rare)?

Lyme disease; Rocky Mountainspotted fever; Babesiosis; Ehrlichiae;Relapsing fever

Rickettsia Rickettsii

Female Dermacentor tick

A couple of days to 2 weeks

In the spring and summer withyoung children being mostcommonly infected

No—more common in east region(i.e., Virginia)

Rash—an erythematous blanchingrash with 2–4 mm macules thatappear initially on the flexor surfaceof wrist/ankles that spreads to palm/soles, which then moves centrally

Fever (usually high grade >39�),severe headaches, myalgias, and GIsymptoms which are then typicallyfollowed by the hallmark rash

It is a clinical diagnosis (fever,headache, and rash in spring/summer) and if you suspect,initiate treatment

Serology (typically negative early on);Skin biopsy; Indirect fluorescentantibody

Delayed treatment

Page 308: Deja review _emergency_medicine__deja_review_

List three antibiotics commonly used totreat RMSF?

What is the concern when usingtetracycline antibiotics?

What are some important adverse reactionsto keep in mind about chloramphenicol?

What are some indications for the use ofchloramphenicol?

What is the most common tick-borneillness in the United States?

What is the organism responsible for Lymedisease?

What is the tick that harbors thisorganism?

What is the peak incidence of Lyme disease?

What are the three phases of Lyme disease?

What is the hallmark of early localizeddisease?

What are some other clinical features oflocalized disease?

What are some clinical features ofdisseminated disease?

What are some common neurologicabnormalities of disseminated disease?

What are some clinical features of chronicdisease?

What is the most important diagnostic testfor Lyme disease?

292 Deja Review: Emergency Medicine

1. Doxycycline2. Tetracycline3. Chloramphenicol

Teeth staining in younger children

Aplastic anemia; Bone marrowsuppression; Gray baby syndrome

Pregnancy; Children <8 years;Severe illness

Lyme disease

Borrelia burgdorferi

Ixodes tick

Spring and summer

1. Localized2. Dissemination3. Chronic infection

Erythema migrans (EM)

Usually occurs within a month, willdevelop fever, myalgia, headache,malaise, and fatigue with EM (canbe absent in up to 20% of cases)

Occur days-months after a tick bite that can be characterized byconjunctivitis, myocarditis, varying degrees of AV block, andneurologic abnormalities being the predominate feature

Cranial neuropathy (Bell’s palsycommon); Peripheral neuropathy;Meningoencephalitis

Occurs months-years after tick bite;where musculoskeletal complaintsmost common, peripheral neuro-pathy, encephalopathy, and neuro-cognitive dysfunction also canoccur

Clinical suspicion is the mostimportant

Page 309: Deja review _emergency_medicine__deja_review_

Environmental Exposures 293

What are some tests that can be done forLyme disease?

What are some commonly used antibioticsfor early Lyme disease?

What is the treatment for severe CNSmanifestations or carditis of Lyme disease?

Serology testing; CSF

Doxycycline; Amoxicillin;Clarithromycin

Ceftriaxone or penicillin

24-year-old male who works as a cookpresents with a burn to the left hand fromspilling soup on it; PE: skin is red andpainful with blisters

32-year-old male is brought in unconsciousby EMS from a frozen lake, patient isunresponsive and does not have evidenceof falling via the lake; ECG: Osborn wavesand prolongation of all intervals

18-year-old healthy male complains ofnumbness, leg cramps, and paresthesias of lower extremities a few hours afterhiking for about 4 hours via rivers; PE:unremarkable except for feet that are paleand insensitive to touch

18-year-old male presents with complaintsof pain on the back of his skin, he mentionshe was tanning the day before; PE: skin onthe back is red and tender to touch, butdoes not have blisters

76-year-old with a recent history of headsurgery now comes to the ER complainingof facial spasms and inability to open herjaw, but otherwise no other complaints; PE:unremarkable except for trismus of the jaw

12-year-old male presents with a severeheadache and high fever, patient mentionshe developed these symptoms abouta week ago with a rash developingyesterday; PE: erythematous blanchingrash on flexor surface of wrists/ankles

Serving as the team physician on a mountainexpedition, what is the most likely diagnosisbased on the symptoms for each of thefollowing members:

Second degree burn

Hypothermia

Immersion foot (i.e., trench foot)

First degree burn

Cephalic tetanus

Rocky Mountain spotted fever

CLINICAL VIGNETTES

Page 310: Deja review _emergency_medicine__deja_review_

Four days into the ascend, a membercomplains of increasing dyspnea,fatigue, headache, cough; PE: rales andcyanosis

A few days into the ascend, a member iscomplaining of a bad headache, whichis worse in the morning and has hadtrouble sleeping

A member is beginning to display oddbehavior, seeing things that are notthere and often acting confused

7-year-old child is brought in by her motherdue to a dog bite to the hand about an hourago. Patient is otherwise doing well withno other complaints; PE: normal exam ofthe hand; x-ray of hand: no foreign body orair; should you suture the hand?

42-year-old construction worker is broughtin by a coworker in severe spasms of hisentire body and screaming out in pain; PE:most noticeable for stiffness of the jaw(trismus) and evidence of an old puncturewound on his left foot

35-year-old male is brought into the EDvia EMS for profuse sweating along withnausea while he was jogging at the beachseveral hours ago; PE: tachycardia,hypotension, normal temperature

34-year-old female is brought in by herhusband with a week’s history of the “flu”but now is having periods of confusionand hallucinations. Husband mentionsthat the “flu” started soon after comingback from camping; PE: only remarkablefor an old bite near the left calf

21-year-old female presents in distress withdrooping of her eyelid and the corner ofher mouth, she only recalls a brief bout ofthe “flu” and a funny rash a week aftercamping; PE: Bell’s palsy; ECG: firstdegree AV block

294 Deja Review: Emergency Medicine

High-altitude pulmonary edema(HAPE)

Acute-mountain sickness (AMS)

High-altitude cerebral edema(HACE)

No—all hand wounds should beleft open initially

Generalized tetanus

Heat exhaustion

Rabies (excitement phase)

Lyme disease

Page 311: Deja review _emergency_medicine__deja_review_

34-year-old diver comes in to the EDcomplaining of dizziness and extremenausea whenever he stands up, hementions this occurred after a dive wherehe underwent a rapid descent

17-year-old female presents with severemuscle cramps in her calves during trackpractice in hot and humid weather; PE:normal vitals

Environmental Exposures 295

Barotitis interna

Heat cramps

Page 312: Deja review _emergency_medicine__deja_review_

This page intentionally left blank

Page 313: Deja review _emergency_medicine__deja_review_

297

C H A P T E R 1 6

ToxicologicalEmergencies

GENERAL APPROACH

What is the first course of action for any Airway, breathing, circulation (ABC)patient who presents with suspectedpoisoning?

What are some other actions to take once O2 saturation for hypoxia; Finger stickABCs have been established? for glucose; Assess vitals; Accurate

history

What two organ systems should the exam 1. Cardiovascular system (CVS)concentrate on? 2. Central nervous system (CNS)

Why concentrate on the CNS and CVS The most lethal adverse affects ofduring the exam? toxicological ingestion typically affect

the CVS and CNS

What are some interventions for any Treatment is primarily supportive; poisoned person? Always consider other ingestions; Call

poison center for recommendations

What is gastric decontamination? The use of various techniques toeither remove the toxin or expeditepassage through the GI tract to limitabsorption

Name five methods of gastric 1. Ipecacdecontamination. 2. Activated charcoal (AC)

3. Whole bowel irrigation4. Intestinal evacuants5. Orogastric lavage

Are gastric decontamination methods No—while historically commonly routinely used in acute poisonings? used, gastric decontamination is

now used in select cases

Copyright © 2008 by The McGraw-Hill Companies, Inc. Click here for terms of use.

Page 314: Deja review _emergency_medicine__deja_review_

What are important things to know about Derived from plant alkaloids;ipecac? Single dose produces emesis in over

90% of patients; Emesis typicallyoccurs around 30 minutes

What are some indications for the use of Considered where AC binds poorly ipecac? to toxins; In acute ingestions (<1 hour);

If removal of small amount hassignificant impact on outcome; Patientshould have intact gag reflex

What are some contraindications of ipecac? Prior significant emesis; Avoid ifunconsciousness/altered mental state;Nontoxic ingestions; Avoid if ingestedcaustic substances

What are some complications with the Aspiration; Lethargy; Mallory-Weissuse of ipecac? tear; Intractable emesis

What is gastric lavage? Orogastric lavage with a large-bore tube to lavage with adequate volumesuntil clear while removing anyremaining toxins

What are some indications for the use of Consider if ingestion occurs withingastric lavage? an hour; Preferred for patients who

have no gag; Consider where a rapiddeterioration is expected (i.e., TCAs)

What are some contraindications for the Any caustic ingestions; If drug isuse of gastric lavage? most likely not in the stomach; Any

large foreign bodies or sharp objects

What are some complications of gastric Aspiration; Esophageal/gastriclavage? perforation; Tension pneumothorax

What is activated charcoal (AC)? Fine black powder produced by burning carbonaceous material that will result in a substance in a huge surface area to bind many substances

While AC will bind many substances, Strong acids and bases; Metals (i.e.,what are some substances that AC does iron and lithium); Alcoholsnot bind well?

What substances does multiple-dose Theophylline; Digoxin; Phenytoin;activated charcoal (MDAC) prove Carbamazepineeffective in?

What are some contraindications of AC? Any perforation; Loss of airway reflex

What are some complications of AC? Small bowel obstruction (very rare);Aspiration

298 Deja Review: Emergency Medicine

Page 315: Deja review _emergency_medicine__deja_review_

What is whole bowel irrigation? Use of large volumes of fluid to cleanse the entire GI tract that will clear most matter (i.e., toxin) within a few hours

What substance is commonly used in Polyethylene glycol (PEG)whole bowel irrigation?

What are some indications for whole Toxic substance not well absorbedbowel irrigation? by AC; Toxins with prolonged

absorption; GI drug concealment

What are some contraindications of whole Bowel obstruction and perforation; bowel irrigation? Hemodynamic instability; Evidence

of no bowel activity

What are some toxins where hemodialysis MEAL(HD) is commonly indicated in severe cases? Methanol; Ethylene glycol;

Aspirin; Lithium

What are some indications where HD Sign of end-organ damage; Absoluteshould be considered? level; Inability to metabolize

What are some complications of HD? Blood loss; Hypotension; Coagulopathyfrom heparin; Decrease in platelets

What is urinary alkalinization? A method of enhanced elimination by alkalinization of urine (via bicarb) to enhance ion trapping and elimination via urine

What are some substances where urine Aspirin; Chlorpropamide;alkalinization is indicated? Methotrexate; Phenobarbital

OVER-THE-COUNTER DRUGS

Acetaminophen

What are some important things to know APAP is found in over 100 drug about acetaminophen (APAP)? preparations; Leading cause of liver

failure requiring transplantation;Leading drug involved in ingestion

What is the normal metabolism of APAP? >90% conjugated to glucuronide/sulfate conjugates (eliminated bykidney after); 2% excreted by kidneyunchanged; 5% oxidized to N-acetyl-para-benzoquinoneimine (NAPQI)

What is the primary toxic metabolite of NAPQIAPAP that is responsible for liver necrosis?

Toxicological Emergencies 299

Page 316: Deja review _emergency_medicine__deja_review_

What is the body’s method to detoxify Glutathione binds to NAPQINAPQI under normal circumstances? preventing hepatocyte necrosis

What happens when there is an APAP Conjugation and sulfation pathwaysoverdose? are saturated which means more

NAPQI is produced and overwhelmsglutathione stores

What is the toxic dose of APAP in acute 150 mg/kg (7.5 g) in an adult setting? (24 hours)

What is the time course of APAP toxicity:

Phase 1 (0–24 hours) Anorexia, nausea, emesis, and elevatedtransaminases

Phase 2 (24–72 hours) Right upper quadrant (RUQ) pain,bilirubin and PT elevate, andtransaminases begin to peak

Phase 3 (72–96 hours) Hepatic necrosis begins (may getencephalopathy, jaundice, and death)

Phase 4 (96 hours–2 weeks) Healing of liver if acute fulminantliver failure did not occur in phase 3

What is the Rumack-Matthew normogram? Predicts the risk of toxicity assuming a one time ingestion with complete absorption

Based on the normogram, what is the 150 mg/L (in the United States) cut-off level in deciding to treat or not? at 4 hours

What are some limitations of applying the Does not apply to multiple ingestions;normogram to APAP overdose? Not applicable to chronic ingestions;

Typically will end up over treating

What is the antidote for APAP toxicity? N-acetylcysteine (NAC)

What is the mechanism by which NAC Precursor to cysteine then to works? glutathione; Enhance sulfation of

APAP; Can act as free radicalscavenger; Glutathione substitute

When is the optimal time to give NAC Within 8 hours (100% protective)following APAP overdose?

How is NAC administered? Oral; IV (if difficult to ingest due to smell/taste)

What are adverse reactions to IV NAC? Anaphylactoid reaction;Hypotension and death (very rare);Elevates PT

300 Deja Review: Emergency Medicine

Page 317: Deja review _emergency_medicine__deja_review_

What are some poor prognostic factors pH <7.30; Creatinine >3.3;after APAP overdose? Grade III/IV encephalopathy

Salicylates

What are some of the therapeutic Antipyretic; Analgesic; Anti-properties of aspirin (ASA)? inflammatory

What are some important things to know A significant source of poisoning;about ASA? ASA can produce substantial

toxicity/death; There are more than200 products with ASA

What are some sources of ASA? Oil of wintergreen; Arthritis/decongestants/cold preparations;Keratolytics; Pepto-Bismol

What is the toxic level of ingestion for 300-400 mg/kg produces serious acute ASA poisoning? toxicity; 100 mg/kg/day for over

2 days will produce chronic toxicity

Do symptoms correlate well with serum Symptoms correlate better with levels? CSF levels, treatment should be

based on clinical picture

What are the two primary acid-base 1. Respiratory alkalosisdisturbances of ASA toxicity? 2. Metabolic acidosis

What is the mechanism by which ASA Uncouples ox-phos to produce fever;toxicity occurs? Stimulates respiratory drive for

tachypnea; Directly causes metabolicacidosis; Acidosis will increase the Vd

What are some clinical features of acute Primarily GI: nausea, vomiting, ASA toxicity? tinnitus, agitation, delirium, seizure,

and coma

What are some clinical features of chronic Nonspecific: altered mental status, ASA toxicity? lethargy, dehydration, and metabolic

acidosis

Serum Salicylate Level

50 mg/dL Moderate toxicity75 mg/dL Severe toxicity100 mg/dL Potentially lethal

Toxicological Emergencies 301

Page 318: Deja review _emergency_medicine__deja_review_

What is the primary way in which death CNS overstimulation (seizure/occurs in ASA overdose? hyperthermia); Cardiovascular

collapse; Pulmonary edema

What are some important diagnostic tests Serum ASA level (serial levels more to consider in ASA overdose? useful); ABG (for acid-base distur-

bances); Potassium; Renal function

Is there any use for AC in ASA overdose? Yes—AC binds ASA well

What are some key points in the ABCs is the first priority; Care ismanagement of ASA overdose? primarily supportive; Aggressive

rehydration; Sodium bicarbonate foracidosis

What is the function of sodium bicarbonate Alkalinize the urine (enhance in ASA toxicity? elimination); Essentially traps salicylic

acid to be excreted; Treats severe aci-dosis; Alkalinize serum to decrease Vd

What is the role of hemodialysis in ASA Used in severely ill patients overdose? where immediate removal of salicylic

acid is needed as well as correcting metabolic and fluid derangements

List some indications for hemodialysis in Acute ASA level of >100 mg/dL;ASA overdose. Chronic ASA level >60 mg/dL; Renal

insufficiency; Severe metabolicacidosis (pH <7.1)

Iron

What are some important things to know Unintentional ingestion mostly about iron overdose? from children; Iron is potentially very

toxic; Most sources from vitaminsand iron pills

What are the three most common 1. Ferrous gluconate 12%preparations of iron and their elemental 2. Ferrous sulfate 20%iron content? 3. Ferrous fumarate 33%

What are some key points in the 10–35 % is absorbed; Iron crossespharmacokinetics of iron absorption? absorbed in the ferrous state; Iron is

rapidly cleared and taken up by cells

What is the general toxic dose of iron Toxic overdose >60 mg/kg of ele-overdose? mental iron; Generally asymptomatic

<20 mg/dL

What are some of the toxic effects of iron? Inhibition of the Krebs cycle;Uncoupling of oxidative phospho-rylation; Mucosal cell necrosis; Freeradical production

302 Deja Review: Emergency Medicine

Page 319: Deja review _emergency_medicine__deja_review_

What three organ systems are most affected 1. GI epitheliumby iron overdose (primarily from free- 2. Heartradical production)? 3. Liver

What is the most common cause of death Circulatory shockin iron overdose?

What are the four phases of iron toxicity: Clinical picture is more important then trying to categorize patients

Phase 1: GI (0–12 hours) Direct injury to the GI mucosa:abdominal pain, diarrhea, emesis,hematemesis, etc.; Severity rangesfrom mild to shock

Phase 2: Latent (6–24 hours) Period of apparent recovery. Patientsin this phase are usually stable, butthey are not asymptomatic. Risk ofdeveloping life-threatening hypov-olemia and acidosis

Phase 3: Metabolic phase Clinical manifestations of the metabolic(24 hours–4 days) phase include fever, pallor, cyanosis,

jaundice, renal failure, lethargy,coma, shock, and bleeding. Potentialfor death is highest here

Phase 4: Delayed phase (2–8 weeks) Characterized by late complications,usually intestinal scarring with GIobstruction

What are important serum iron concentrations to be aware of?

50–150 µg/dL Normal levels

350 µg/dL Risk for toxicity

500 µg/dL Significant toxicity likely

>1000 µg/dL Considerable morbidity

What are some laboratory tests to CBC/Chem-7/Coags; ABG forconsider? moderate-severe cases; Iron studies

(i.e., TIBC, Fe, etc.)

What role does an abdominal radiograph While a KUB may be able to (KUB) play in iron toxicity? detect opacities (Fe) on film, its

absence does not rule out ingestion

Is gastric decontamination effective with Gastric lavage, ipecac, and AC rela-iron overdose? tively ineffective with iron ingestion

What is the antidote commonly used for Deferoxamine (DFO)iron toxicity?

Toxicological Emergencies 303

Page 320: Deja review _emergency_medicine__deja_review_

What are some functions of DFO? Chelation of iron; DFO can removeiron bound to transferrin; DFO canalso remove iron from cells

When is the general serum iron level in Generally 500 ug/mL or greaterwhich to administer DFO?

What are some adverse reactions with Acute renal failure; Septicemia fromadministration of DFO? Y. enterocolitica; ARDS; Hypotension

PRESCRIPTION MEDICATIONS

Anticoagulants

What are the two main categories of anticoagulants and some examples of each:

Indanedione anticoagulants Pindone; Diphacinone; Valone

Hydroxycourmarin anticoagulants Brodifacoum; Warfarin; Fumarin

What are some scenarios where overdose Accidental ingestion by children;of anticoagulants can occur? Drug interactions; Suicidal ingestion;

Homicidal attempts (i.e., rat poison)

What is the mechanism of action of Inhibits the synthesis of vitamin warfarin? K-dependent factors (II, VII, IX, X,

and protein C and S), so that once the existing factors degrade, no more is made

What are common sites of bleeding with GI tract and genitourinary tract;anticoagulant overdose? Epistaxis and hemoptysis can be

common

What is the most feared complication of Intracranial bleedinganticoagulant overdose?

What are the typical abnormal labs with Elevated PT/PTT time; Plateletsanticoagulants overdose? and LFT are usually normal

What drugs interaction typically lead Cimetidine, erythromycin, metron-to excessive anticoagulation? idazole, and ciprofloxacin typically

lead to excess anticoagulation

What are some distinguishing features of Very long-acting anticoagulants; Half-superwarfarins? life that exceed 3–4 months; Vitamin K

therapy may require months; Typicallyonly found in rat poison

What are some key points in the If asymptomatic, typically observe;management of accidental ingestion? Coags/GI decontamination not

needed; Advise to watch for any signsof bleeding

304 Deja Review: Emergency Medicine

Page 321: Deja review _emergency_medicine__deja_review_

What are some key points in the Careful montoring, especially if management of intentional ingestion? active bleeding; ABCs—active bleed-

ing can obstruct airway; CBC andcoags should be done serially; Know ifpatient needs to be anticoagulated

What are some treatment options for a For severe bleeding: FFP or whole patient who is actively bleeding from blood; Most other cases: vitamin Kanticoagulants?

How often should PT be monitored? Initially every 6–8 hours, PT takesdays to normalize

What are some routes of vitamin K Oral, IM, or IVadministration?

What are adverse reactions of giving IV Anaphylactoid reaction (rare);vitamin K? Cerebral thrombosis

What is the mechanism of action for Inhibits ATIII that results in prolonged unfractionated heparin (UFH)? PTT

What are some adverse reactions of UFH? Heparin-induced thrombocytope-nia (HIT); Hyperkalemia (inhibitsaldosterone)

What are low-molecular-weight heparins Derivatives of commercial heparin,(LMWH)? LMWH inactivate factor Xa, but have

a lesser effect on thrombin

What are three LMWH approved for use 1. Enoxaparinin the United States? 2. Ardeparin

3. Dalteparin

What are some advantages of LMWH Longer duration of action; Laboratoryover UFH? monitoring is not necessary; They are

much less likely to induce HIT; LMWHcan be given outpatient

What is the treatment of choice for Discontinue heparin as it has a very heparin overdose? short half-life. Protamine sulfate can

be given for serious bleeding as a result of heparin or LMWH

Oral Hypoglycemics

What are some important points regarding Normally maintained between maintenance of plasma glucose levels? 70–150 mg/dL; Glycogenolysis/

gluconeogensis help maintain normallevels; Adult liver has 70 grams ofglycogen

Toxicological Emergencies 305

Page 322: Deja review _emergency_medicine__deja_review_

What is important to know about the Uses about 60% of glucose; First organbrain and its use of glucose? to be affected by hypoglycemia;

Hypoglycemia will activate sympa-thetic axis

What are clinical features of hypoglycemia? Diaphoresis, tachycardia, tremor, altered mental status, seizure, coma, and rarely focal neurologic deficits that mimic TIAs

What is the general principle for initial Give dextrose then feed the patienttreatment for all hypoglycemic agents?

Name some commonly used oral agents Sulfonylurea; Alpha-glucosidasein the treatment of non-insulin-dependent inhibitors; Thiazolidinedionesdiabetes mellitus (NIDDM)? Biguanides

What are some commonly used Glyburide; Glipizide; Tolazamidesulfonylureas?

What is the primary mechanism of action They cause insulin release from of sulfonylureas? remaining pancreatic cells via cell

depolarization and also improve sensitivity to insulin

How soon can hypoglycemia occur after Can vary anywhere from 30 minutes the ingestion of a sulfonylurea? to many hours after ingestion.

Administration of dextrose can maskhypoglycemia

What are some key points in the Carbohydrate-rich meal for awake management of hypoglycemia secondary patients; 50% dextrose for patientsto sulfonylureas? with altered MS; Glucagon is not

effective in hypoglycemia; Observefor at least 8–12 hours with frequentaccuchecks

What are two agents available for 1. Diazoxiderefractory hypoglycemia secondary to 2. Octreotide (main treatment)sulfonylureas?

What are some important things to know Inhibits insulin secretion; Causesabout diazoxide? hypotension and hyponatremia; Can

cause sodium and fluid retention

What are some important things to know Somatostatin analogue; More effectiveabout octreotide? then diazoxide; Inhibits secretion of

insulin; Generally very well tolerated

What are some key points in the A single ingestion by child = admis-management of hypoglycemia from sion; Interactions may enhance toxicity;sulfonylureas? Any patients who present hypo-

glycemic = admission for observation

306 Deja Review: Emergency Medicine

Page 323: Deja review _emergency_medicine__deja_review_

What are some other techniques to prevent Activated charcoalabsorption and enhance elimination?

What is the mechanism of action of Decreases hepatic gluconeogenesis;biguanides (i.e., metformin)? Increases uptake of glucose; Increases

utilization of glucose into lactate

What adverse effect is particularly Lactic acidosis important to monitor with metformin?

Who are at increased risk of lactic acidosis Patients with impaired renal clearancesecondary to metformin use?

What is the treatment of lactic acidosis Treatment is supportive with correc-due to metformin use? tion of acid-base disturbance and

rehydration

Does metformin cause hypoglycemia? Rarely

Cardiac Glycosides

What are cardiac glycosides? Drugs with a steroid ring, one-four sugars attached to them, and unsatu-rated lactone ring

What are the primary indications for CHF; Control of rapid ventriculardigoxin? response from afib and aflutter

What are some sources of cardiac Foxglove; Bufo toads; Milkweeds; glycosides? Oleander

What is the mechanism of action of cardiac Inhibit sodium-potassium exchange glycosides? pump that will increase intracellular

calcium

What are some effects of cardiac Increased vagal tone; Increasedglycosides? automaticity; Increased contractility

What is the time course for toxicity to Drugs must first move into cells, develop after an overdose? symptoms generally do not occur for

several hours

Time toOnset Peak Effect

Oral 1.5–6 hour 4–6 hoursIV 5–20 minutes 1–3 hours

Toxicological Emergencies 307

Page 324: Deja review _emergency_medicine__deja_review_

What are some clinical features of acute Nausea and emesis are almost firstoverdose? symptoms with confusion and

weakness, can also develop heartblock/bradycardia

What are some clinical features of chronic Anorexia, nausea, and emesis commonoverdose? with headaches, confusion, and

lethargy

Which patients commonly present with Commonly an elderly person with chronic digoxin toxicity? underlying heart disease who presents

with nonspecific GI/neuro complaints(usually with precipitating factor suchas dehydration)

How does chronic toxicity commonly Drug interactions that increase levels;develop? Worsening renal function; Diuretics

or infection that lead to dehydration

What are the some possible ECG findings Prolonged PR interval; Short Q-T;in digoxin overdose? ST scooping and depression (esp.

laterally); Decreased T-waves

What is another feared complication of Life-threatening hyperkalemia digoxin overdose?

What are some key points in the AC may be considered; HD is notmanagement of acute digoxin overdose? effective; Follow potassium closely—

treat accordingly; Avoid the use ofcalcium—greater arrhythmias; Treatany dysrhythmias accordingly, butavoid type IA/IC antiarrhythmics

What is the standard treatment for acute Digoxin-specific Fab antibody digoxin overdose? fragment (Digibind)

What are some indications for the use of Serious dysrhymias; Bradycardiadigibind? refractory to atrophine; Hyperkalemia

(>5.5)

Beta-Blockers

What are indications for the use of Hypertension; Prevent reinfarctionbeta-blockers? and s/p MI; Dysrhythmias; Glaucoma;

Migraine headaches

What are some important things to know There are many preparations; Agentsabout beta-blockers? may be selective or nonselective;

With overdose, selectivity is loss

What are some commonly used Metoprolol; Carvedilol; Labetalol;beta-blockers? Timolol

308 Deja Review: Emergency Medicine

Page 325: Deja review _emergency_medicine__deja_review_

What is the function of B1 receptors? Heart (increase HR/inotrophy/automaticty); Eye (increase aqueoushumor production); Kidney (increaserenin production)

What is the function of B2 receptors? Liver (gluconeogensis); Smoothmuscle relaxation; Skeletal muscle(glycogenolysis)

What is the function of B3 receptors? Adipose tissue (lipolysis)

How soon after ingestion of beta-blockers Usually within 6 hoursdo patients manifest symptoms?

What are some important clinicalmanifestation based on systems:

Cardiovascular system Bradycardia, hypotension, CHF,QRS/QT prolongation (rare)

Respiratory system Apnea, respiratory depression, andbronchospasms

CNS Seizure, delirium, and coma (mostlyin the setting of hypotension)

Endocrine system Children are particularly susceptibleto hypoglycemia

What type of beta-blockers are Ones that are lipophilic, have sodium considered the most dangerous? channel activity, and have potassium

channel activity

Which beta-blocker causes a Propranolol disproportionate amount of deaths?

What are some basic therapeutic measures Supportive care with fluids; Cardiacfor overdose with mild symptoms? monitoring

What are some key points in the ABCs; Cardiac monitoring/fluids/management of beta-blocker overdose atropine; Glucagon is the drug offor moderate-severe sick patients? choice; Catecholamines in severe

cases

What are indications for admission? History of sustained-release overdose;Children should be admitted; Anysymptoms/ECG changes within6 hours

Calcium Channel Blockers

What are some important features of Block slow calcium channels in calcium channel blockers (CCB)? myocardium and vascular smooth

muscle; Decrease myocardialinotrophy/conduction; Vasodilationin peripheral vasculature

Toxicological Emergencies 309

Page 326: Deja review _emergency_medicine__deja_review_

What are some indications for the use of Hypertension; Angina; Dysrhythmias;calcium channel blockers? Migraines

What are the three most commonly used 1. Diltiazem (benzothiazapine)CCBs? 2. Verapamil (phenylalklamine)

3. Nifedipine (dihydropyridine)

How soon after ingestion do symptoms Depending on the formulation, of CCBs overdose appear? can range from the first hour to

24 hours

What is the mechanism of death in CCB Profound cardiogenic shock with overdose? peripheral vasodilation

What are some important clinicalmanifestation based on:

CVS Hypotension, dysrhythmias, brady-cardia, and cardiogenic shock

Respiratory system ARDS

CNS Dizziness, seizures, altered MS, and stroke

Endocrine system Hyperglycemia

What are some key points in the manage- Prevent and correct hypotension;ment of CCB overdose? ABCs; AC; Bradydysrhythmias treated

with advanced life support (ACLS)

What are important therapeutic IVF bolus; Calcium; Glucagon andmaneuvers to reverse hypotension? catecholamines; High insulin therapy

Which patients can safely be discharged Typically those who show no after CCB overdose? symptoms or ECG changes after

6 hours and did not ingest anysustained-release formulation

PSYCHIATRIC MEDICATIONS

Selective Serotonin Reuptake Inhibitors

Generic Name Brand Name

Citaloprim CelexaFluoxetine ProzacFluvoxamine FluvoxParoxetine PaxilSertraline ZoloftVenlafaxine Effexor

310 Deja Review: Emergency Medicine

Page 327: Deja review _emergency_medicine__deja_review_

What are some important things to know SSRIs are generally safe; Designedabout selective serotonin reuptake to answer TCAs side effect profileinhibitors (SSRIs)?

What is the mechanism of action of SSRIs? They inhibit presynaptic neuronal reuptake of serotonin

What are some clinical features of SSRIs Nausea, emesis, sedation, lethargy, overdose? and rarely seizures

What is important to know about Large overdoses can cause seizure citalopram? and QT prolongation

What are some key points in the Treatment is primarily supportive;management of SSRI overdose? Important to rule out other overdoses

What are some general indications to Monitor for 6 hours and if no changes,medically clear a patient following SSRI can clear with exception of citaloprimoverdose? and buproprion

What is serotonin syndrome? Excessive stimulation of serotonin receptors typically due to ingestion of serotonergic medication

What are some mechanisms by which Prevent breakdown of 5-HT; Enhanceexcessive serotonin can occur? 5-HT release (i.e., ecstasy); Block

reuptake (i.e., cocaine)

What are some clinical features of Mental status change, hyperreflexia, serotonin syndrome? hyperthermia, agitation, myoclonus,

and seizure

What are some key points in the Benzodiazepines, cooling, andmanagement of serotonin syndrome? hydration; Sedation/intubation in

refractory cases

Other Antidepressants

What are key points of drug overdose forthe following antidepressants:

Trazadone and Nefazodone Inhibits reuptake of 5-HT; Overdosemay cause sedation

Amoxapine Cyclic antidepressant; Works on DAand NE receptors; High incidence ofseizures; Not associated with ECGabnormalities

Buproprion Prevents reuptake of DA and NE;Indicated for smoking cessation; Cancause seizures

Toxicological Emergencies 311

Page 328: Deja review _emergency_medicine__deja_review_

Tricyclic Antidepressants

What are some important things to know Higher frequency of adverse effects;about tricyclic antidepressants (TCAs)? Have low therapeutic index;

Significant sedative/anticholinergiceffect

Why are TCAs fairly toxic in overdose? Primarily due to their nonspecificblockage of reuptake of variousneurotransmitters

What are some of the adverse effectswhen taken in overdose:

Anticholinergic Dry skin, hallucinations, delirium, hyperthermia, tachycardia, andmydriasis

Alpha-adrenergic blockage Peripheral vasodilation withhypotension

Sodium channel blockage Inhibit fast sodium channels(quinidine-like effect) with widenedQRS complex

What is a useful diagnostic test to obtain ECGto further evaluate TCA overdose?

What is the most sensitive indicator of QRS widthtoxicity on ECG?

What is the QRS width where seizures and QRS width >120 msecdysrhythmias may occur?

What are some key points in the ABCs with cardiac monitoring ismanagement of TCA overdose? crucial; AC should also be given

<1 hour

What is the drug of choice for TCA Sodium bicarbonateoverdose which manifest QRS widening?

When is it generally safe to discharge No signs of toxicity and continuouspatients from the ED after TCA overdose? monitoring for 6 hours

Tricyclic Antidepressants

ImipramineAmitriptylineDesipramineNortriptyline

312 Deja Review: Emergency Medicine

Page 329: Deja review _emergency_medicine__deja_review_

Monoamine Oxidase Inhibitors

What are some important things to know Were among the first class used forabout monoamine oxidase inhibitors MDD; Were later largely replaced by(MAOIs)? TCAs; They now have limited indi-

cations for use

What is the mechanism of action MAO is an enzyme that breaks downof MAOIs? monoamines, so its inhibition will

increase the concentration of NE, DA,and 5-HT

What are some clinical features of MAOI Tachycardia, hypertension, agitation,overdose? and diaphoresis; may get cardiovas-

cular as well as neurological collapsein severe overdose

How late can symptoms appear following Effects can be delayed for over an MAOI overdose? 24 hours

What are some late complications DIC, rhabdomyolysis, and pulmonaryof MAOI overdose? edema

What are some key points in the ABCs; Promptly treat severe hyper-management of MAOI overdose? tension; ACLS for dysrhythmias which

may occur; Aggressive fluid bolusfor hypotension

Can patients be safely discharged after MAOIs are the exception to thebeing asymptomatic for 6 hours? “6-hour rule” and should be moni-

tored for at least 24 hours

What other drugs can interact with MAOIs Any sympathomimetic such as cocaineto produce toxicity? or dopamine can produce toxicity

MAOIs are well-known to produce toxicity These foods contain tyramine, whichwhen ingested with “wine and cheese,” acts as an indirect sympathomimeticwhy? to precipitate toxicity

What are some other foods that produce Aged meat; Soy sauce; Sauerkrauttoxicity when ingested with MAOIs?

Generic Name Brand Name

Isocarboxazid MarplanPhenelzine NardilSelegiline DeprenylTranylcypromine Parnate

Toxicological Emergencies 313

Page 330: Deja review _emergency_medicine__deja_review_

NEUROLEPTICS

What are neuroleptics? Originally known as antipsychoticsand tranquilizers, this class of drugsis commonly used for a variety ofanxiety and psychotic states

What are some indications for the use of Psychosis; Delirium; Agitation;neuroleptics? Nausea

What are some examples of positive Mediated primarily by central D2

symptoms and the receptor that receptor: Delusions; Thoughtmediates them? disorders; Hallucinations

What are some examples of negative Mediated primarily by 5-HT2A

symptoms and the receptor that receptor: Apathy; Social withdrawal;mediates them? Blunted effect

What are some adverse effects from More common with typical neurolep-normal use of neuroleptics? tics: Acute dystonia; Neuroleptic

malignant syndrome; Glucosedysregulation

What are some common extrapyramidal Akathisia; Parkinsonism; Dystonicsymptoms seen with neuroleptics? reactions

What are some adverse reactions when Reduced seizure threshold;taken in acute overdose? Hypotension/reflex tachycardia;

Hyper- or hypothermia; CNSdepression or coma (large doses);Quinidine-like effect

What are some key points in the ABCs with IV access; Treat dystoniamanagement of neuroleptic overdose? (i.e., diphenhydramine); Treat

hypotension (i.e., fluids); Treatcardiotoxicity like TCAs (i.e., bicarb)

What are some commonly used medications Benztropine; Diphenhydramine;to treat acute dystonic reactions? Diazepam

When can a patient be medically clear No signs and symptoms for 6 hoursafter a neuroleptic ingestion?

What idiosyncratic reaction affects a small Neuroleptic malignant syndromepercentage of patients on neuroleptics that (NMS)is potentially fatal?

What are some clinical features of NMS? Autonomic instability (i.e., change inHR and BP), profound hyperthermia,mental status change, and rigidity

What are some key points in the Rapid cooling (i.e., spray mist/ice);management of NMS? Use of benzos (paralytics if severe);

Discontinue the offending agent

314 Deja Review: Emergency Medicine

Page 331: Deja review _emergency_medicine__deja_review_

Lithium

What are interesting things to know about Alkali metal with a long history oflithium? use; Used in the past for gout and

CHF; Up to 90% will have some signof toxicity

While the exact mechanism of lithium’s May substitute for sodium in neurons;antimanic effects are not fully understood, Increase GABA transmission; Affectwhat are some of its proposed mechanisms? protein kinases (i.e., C and G)

What are some common preparations Immediate release: 300 mg tiq or qid;available? Sustained release: 300 mg bid;

Controlled release: 450 mg bid

What are some important pharmacokinetic 95% of lithium is renally cleared;properties of lithium? Lithium is absorbed preferentially to

sodium; Any volume-depleted statewill result in increased reabsorptionof lithium

What are some side effects of lithium at Fine tremors, polyuria, diabetestherapeutic doses? insipidus, weight gain, leukocytosis

and cog-wheeling rigidity

What is an important question to ask when Acute versus chronic toxicity or is ita patient presents with a question of acute on chroniclithium toxicity?

What are some clinical features of acute Initial symptoms will be GI-related:lithium toxicity? nausea, emesis, and diarrhea followed

by neurologic symptoms such astremors, lethargy, and seizure or coma

Is acute lithium overdose directly While ECG may show nonspecificcardiotoxic? T-wave changes, it is not directly

cardiotoxic

What are some clinical features of chronic Primarily neurologic: tremors, nystag-lithium toxicity? mus, seizure, lethargy, and coma

What are some common causes of chronic Dehydration; Incorrect dosing;toxicity? Renal insufficiency; Interaction with

other drugs (i.e., NSAIDs)

Therapeutic Level

Maintenance 0.5–0.8 mEq/LAcute mania 0.7–1.2 mEq/L

Toxicological Emergencies 315

Page 332: Deja review _emergency_medicine__deja_review_

What are some long-term sequelae of Personality changes; Memory deficits;lithium use? Diabetes insipidus; Cerebellar dys-

function (i.e., ataxia)

What are some key points in the Follow lithium level; Chem-7 (esp. formanagement of lithium toxicity? renal function); Check for other drug

interactions

What are some indications for the use of Renal failure (will not be able to clearhemodialysis with lithium overdose? lithium); Severe neurological

symptoms

Why is it important to check lithium levels Patients will get rebound lithium level6 hours after HD? as lithium redistributes from tissues

DRUGS OF ABUSE

Opioids

What is the definition of opioids? Natural and synthetic substances withmorphine-like activity, opioids haveanalgesic and central nervous systemdepressant effects, as well as thepotential to cause euphoria

What are endorphins? Endogenous peptides that producepain relief (i.e., dynorphins/beta-endorphins)

What are some major opioid receptors Kappa, delta, and Mufound in the human body?

What is the primary opioid receptor that Mumediates euphoria/analgesia/respiratorydepression?

What are some other clinical features of Most classic finding is miosis, alteredopioid overdose? mental status that can range from

lethargy to coma, and respiratorydepression

What is the most important adverse reaction Respiratory depressionto monitor with opioid overdose?

What are some other adverse effects of Noncardiogenic pulmonary edemaopioid overdose? (NCPE); Cardiotoxicity (i.e., penta-

zocine); Quinidine-like effect (i.e., QRSwidening); Seizures (i.e., meperidine)

What is the mechanism of noncardiogenic Maybe involves loss of consciousnesspulmonary edema? with respiratory depression and

hypoxia

316 Deja Review: Emergency Medicine

Page 333: Deja review _emergency_medicine__deja_review_

What is the agent of choice to reverse Naloxone (Narcan)opioid overdose?

Like alcohol withdrawal (i.e., life- Nothreatening), is opioid withdrawallife-threatening as well?

What are some indications for admission Anyone who requires a naloxone drip;following opioid overdose? Evidence of NCPE; Little improvement

after naloxone; Life-threateningco-ingestion

Sedatives-Hypnotics

What is the definition of a sedative? Medication that reduces anxiety andinduces relaxation

What is the definition of a hypnotic? Medication that induces sleep

Is there really a difference between Not really, the two terms are usedthe two? interchangeably

What are some examples of sedative- Benzodiazepines; Barbiturates; hypnotics? Buspirone; Zolpidem

What are some common indications for Anxiety; Seizures; Muscle spasms;sedative-hypnotics? Insomnia; Alcohol withdrawal

What is the mechanism of action They enhance the activity of GABAof barbiturates? receptors by increasing the duration

by which chloride channels open asopposed to benzos, which increase thefrequency of chloride channel opening

What are some key points in the Airway support is crucialmanagement of sedative-hypnoticsoverdose?

Are barbiturates generally safer than No—benzodiazepines are generallybenzodiazepines? safer as they produce less respiratory

depression and minimal cardiac sideeffects

What are some clinical features of a CNS effects ranging from sedation tobenzodiazepine overdose? coma and respiratory depression in

large overdoses

What is the antidote of choice for Flumazenilbenzodiazepine overdose?

What is the mechanism of action of Nonspecific competitive antagonistflumazenil?

Toxicological Emergencies 317

Page 334: Deja review _emergency_medicine__deja_review_

Is it always safe to give flumazenil in No—particularly in multiple druga suspected benzodiazepine overdose? ingestions where benzos can have

a seizure protective effect with drugssuch as TCA or if the patient is onchronic use, as it may inducewithdrawal

Is benzodiazepine withdrawal dangerous? Yes—it is similar to alcohol with-drawal (i.e., hyperthermia, hyper-tension, seizure, etc.) and can bepotentially fatal

Toxic Alcohols

Name the two alcohols that can be 1. Methanolpotentially fatal? 2. Ethylene glycol

What is an important fact to note about All can increase the plasma osmolalthese two alcohols? gap; Methanol and ethylene glycol

lead to high anion gap metabolicacidosis

What are some characteristics of methanol? Colorless clear flammable liquid thathas a slight alcohol odor

What are some common sources De-icing solutions; Shellac; Varnish;of methanol? Windshield washer fluid

What is the toxic dose of methanol? Less then 1 mL/mg can lead toblindness or severe toxicity

What is the major toxic metabolite of Alcohol dehydrogenase metabolism tomethanol? formaldehyde (causes metabolic

acidosis) and formic acid (optic nervetoxin)

What are some clinical features of methanol Inebriation, nausea, abdominal pain,toxicity? gastritis, and early visual disturbance

such as blurriness and photophobia

What are some severe symptoms of Coma, seizure, blindness, hypotension,methanol toxicity? cardiac failure, and pulmonary edema

Methanol Levels

<20 mg/dL Generally asymptomatic>50 mg/dL Acidosis>100 mg/dL Visual symptoms>150 mg/dL Generally fatal

318 Deja Review: Emergency Medicine

Page 335: Deja review _emergency_medicine__deja_review_

What are important laboratory tests An osmolal gap and anion gap;to obtain? Methanol level

What are some key points in the ABCs; Aggressive early therapy ismanagement of methanol toxicity? key, especially before the onset of

symptoms; Sodium bicarbonate foracidosis; While ethanol can be given(and is effective) 4-methylpyrazole iscommonly used; Folic acid mayincrease metabolism of formic acid

What is the mechanism of action of Inhibits alcohol dehydrogenase 4-methylpyrazole? preventing the formation of toxic

metabolites

When should hemodialysis be started? High [methanol] >50 mg/dL;Presence of metabolic acidosis; Severesymptoms such as visual changes

What are some common sources of Nail-polish remover; Glues; Rubbingisopropyl alcohol? alcohol

What is the metabolism of isopropyl About 50% excreted in urinealcohol? unchanged; The rest is converted to

acetone

Is acetone dangerous? It is not toxic, but can lead to a ketosiswith no acidosis (hallmark of isopropylalcohol)

How is acetone excreted? Primarily through the kidney and lung

What is the typical lab finding in Increased osmolal gap with no acidosisisopropyl alcohol?

What is the treatment for isopropyl alcohol Supportive care; Respiratory caretoxicity?

What are some common sources of Brake fluid; Automobile coolantethylene glycol? systems

What is the toxic dose of ethylene glycol? >15 mL/kg

What are the toxic metabolites of Glycoaldehyde; Glycolic acid; Oxalateethylene glycol?

What are some effects of oxalate? Combines with calcium (calciumoxalate crystals) that damage thekidney and can also damage organssuch as liver and brain, in addition,can cause hypocalcemia

What are some ECG findings associated ECG can show findings of hypocal- with ethylene glycol? cemia such as a prolonged QT or

manifestations of hyperkalemia dueto ARF

Toxicological Emergencies 319

Page 336: Deja review _emergency_medicine__deja_review_

What is the typical lab finding in ethylene Elevated osmolal gap; Elevated anionglycol toxicity? gap acidosis

What are common urinary findings in Hematuria, proteinuria, and crystal-ethylene glycol toxicity? luria, which is a diagnostic finding

Is gastric decontamination effective? Ipecac, cathartics, and gastricaspiration have little role here andAC poorly absorbs ethylene glycol

What are some key points in the Aggressive early therapy is key;management of ethylene glycol? Correct any acidosis immediately;

While ethanol can be given (and iseffective); Fomepizole is the standardof care now; Hemodialysis in severecases

What are some indications of HD in Severe metabolic acidosis; Renalethylene glycol toxicity? dysfunction (i.e., ARF); Levels

>50 mg/dL

Cocaine

What is the mechanism of action Cocaine enhances monoamine neuro-of cocaine? transmitter activity in the central and

peripheral nervous systems byblocking the presynaptic reuptakepumps for these neurotransmitters

What is a secondary effect of cocaine that Blocks voltage-gated membraneis unique among other stimulants? sodium ion channels:

Local anesthetic effects;Dysrhythmias

What are two forms of cocaine? 1. Base (form that can be smoked)2. Salt (form that can be ingested or

injected)

What are some clinical features of acute Euphoria, increased energy, alertness;intoxication? decreased appetite, need for sleep,

and fatigue

Pharmacokinetics Onset Duration

Intravenous seconds 15–30 minutesInhalation seconds 15–30 minutesIntranasal 20 minutes 1 hourGastrointestinal 90 minutes 3 hours

320 Deja Review: Emergency Medicine

Page 337: Deja review _emergency_medicine__deja_review_

What are some adverse effects of cocaine Panic attacks, paranoia, cocaine-intoxication? induced psychosis, impaired

judgment, and dysphoric mood

What are the effects of cocaine on specificorgans:

CVS Increases heart rate, blood pressure,and systemic vascular resistance;cardiac arrhythmias, sudden death,and AMI; cardiomyopathy andmyocarditis with chronic use

CNS Seizures, cerebral vasoconstriction,cerebrovascular disease, and stroke;acute dystonic reactions (i.e., akathisia)

Respiratory system Perforation of the nasal septum andchronic rhinitis from snorting; SOB,wheezing, pneumothorax, andpulmonary edema from smoking

What are some important causes of AMI (most likely); Pneumothorax;chest pain to consider in a patient who Aortic dissection; Pulmonarypresents shortly after cocaine use? infarction

What is the drug of choice for acute Benzodiazepinescocaine toxicity?

What are some key points in the manage- Supportive care (i.e., ABCs); Liberalment of acute cocaine intoxication? use of benzos; CT for any question of

stroke; Cardiac workup if suspectedAMI; Prevent/treat rhabdo andhyperthermia

What is particularly worrisome about Can still have continued seizures thatintubating a patient with acute cocaine can lead to permanent brain damageintoxication (i.e., having intractable (must have EEG monitoring in place)seizures)?

What are some clinical features of cocaine Anhedonia, cocaine craving, anxiety,withdrawal? and depression (it is not life-

threatening)

What is the difference between a body-stuffer and a body-packer?

Body-stuffer Swallow small packs to avoid policecapture; Typically mild and transientadverse affects; Tx is observation andAC admistration

Body-packer Smuggle large quantities of drugs(cocaine); Often swallow 100+pre-packed drugs; Potentially fatal ifthey rupture; Surgical intervention ifbags rupture/obstruct

Toxicological Emergencies 321

Page 338: Deja review _emergency_medicine__deja_review_

Phencyclidine

What are some common street names Angel dust, crystal, peep, hog, andfor phencyclidine (PCP)? PCP

What are some available forms of PCP? Powder, tablet, crystal, liquid, andcapsule

What are some important things to know Frequently found as mixture in otherabout PCP? drugs; Often produce brief dissociative

reactions; Effects are often unpre-dictable (part of the appeal for many)

What is particularly important about the Well absorbed by any routepharmacokinetics of PCP?

What is the clinical hallmark of PCP Vertical nystagmusintoxication that allows it to be distin-guished from other street drugs?

What are some clinical features of low Confusion, ataxia, dysphoria, catatonicto moderate PCP intake? behavior, dystonia, violent behavior,

and frank psychosis in rare cases

What are some adverse effects of high Hypertension, seizure, anddose PCP intake? hyperthermia

What are some key points in the Care is supportive (i.e., ABCs); Benzosmanagement of PCP? for seizure and agitation; Reduce

external stimuli; Physical/chemicalrestraint if violent; Severe HTN shouldbe treated to avoid CVA; Prevent/treat rhabdo and hyperthermia

Amphetamines

What are amphetamines? Stimulant agents with sympath-omimetic properties (like cocaine) thatact on the CNS and PNS that stim-ulate both beta and alpha receptors

What are some common amphetamine Methamphetamine (i.e., crank, meth,derivatives? glass); Methylphenidate (i.e., Ritalin);

3,4-Methylenedioxyamphetamine(i.e., Ecstasy)

What are some major routes of Oral, intravenous, and inhalationamphetamine administration?

What are the two organ systems of concern 1. CNSwith amphetamine intoxication? 2. CVS

322 Deja Review: Emergency Medicine

Page 339: Deja review _emergency_medicine__deja_review_

What are some neurologic symptoms of Anxiety, aggression, seizure, delirium,amphetamine intoxication? euphoria, stroke, and cerebral edema

What are some cardiovascular symptoms Tachycardia, hypertension, chest pain,of amphetamine intoxication? dysrhythmias, AMI, and sudden

death

What are some other complications of Renal failure, rhabdomyolysis,amphetamine intoxication? hyperthermia, anorexia, and

complications associated with IVDA

What are some clinical features of Anxiety, drug craving, irritability,amphetamine withdrawal? insomnia, mood swing, and paranoia

What are some key points in the manage- Primarily supportive (i.e., ABCs);ment of amphetamine intoxication? Prevent/treat rhabdo and hyper-

thermia; Benzos for seizure andagitation

Lysergic Acid Diethylamide

What are some commonly used Lysergic acid diethylamide (LSD);hallucinogens? Psilocybin; Ketamine; Mushrooms;

Mescaline

What is the mechanism of action of Drugs that induce hallucinations,hallucinogens? where a user perceives a sensory

experience that is not actually there,although in many cases many drugsjust distort sensory input (i.e.,illusions)

Give some examples of common illusions Trail: objects in visual field “leave aproduced by LSD? trail”; Feelings of depersonalization;

Synesthesia: “see sound” or “hearcolors”

What are some common clinical features Altered perception is the hallmarkof LSD intoxication? along with hypertension, pupillary

dilation, sweating, palpitations,blurred vision, incoordination andtremors

What is the hallmark of acute LSD “Bad trip” where the user experi-intoxication? ences fear, paranoia, feelings of

depersonalization

What is the optimal way to handle Reassurance and “talking the patienta patient with a bad trip? down” until the drug wears off and

consider use of benzos

Toxicological Emergencies 323

Page 340: Deja review _emergency_medicine__deja_review_

What are some long-term complications Primarily psychiatric: flashbacksof LSD use? (reliving the perceptual distortions),

depression, psychosis, and personalitychange

Is death from LSD common? LSD generally does not directly causedeath, but indirectly via self-injury ordepression/suicide

METALS, CHEMICALS, AND GASES

General Information

What is important to know about the acute Most metals bind to sulfhydral groupstoxicity of metals? of enzymes found throughout the

body so have multisystem effects

What are some common clinical features ofacute toxicity of most metals:

Gastrointestinal system The hallmark of acute metal toxicities:Nausea, emesis, and diarrhea

CVS Can range from symptoms of volumedepletion (i.e., tachycardia) to frankheart failure or dysrhythmias

Renal system Loss of protein and amino acids inurine, can also get acute tubularnecrosis

Nervous system Peripheral neuropathy is common aswell as altered mental status

What are some clinical features of chronictoxicity of most metals:

Nervous system CNS and PNS disturbances are moreprominent than GI symptoms

Renal system Varying degrees of renal insufficiencyis usually noted

Hematology/Oncology Anemias and neoplasm can be found

Dermatology Rashes and colored lines of gums/nails often noted

What are some important aspects of the History, occupation, lifestyles, hobbies,evaluation to focus on with suspected use of herbal remedies, and travelsexposure to metals?

What particular area of the exam should Neurologic examone focus on?

324 Deja Review: Emergency Medicine

Page 341: Deja review _emergency_medicine__deja_review_

What are some appropriate laboratory tests CBC with a peripheral smear; Chem-7to obtain? (assess renal function); Liver function

tests; Urinanalysis; Abdominal films;Blood and urine metal tests

Arsenic

What group is more likely to get arsenic Industrial workers(As) exposure?

What are some important things to know Over 1 million workers are exposedabout arsenic? to As; Commonly found in pesticides/

herbicides; Main route of exposure isinhalation; Also become exposed viasmelting of ore

What are other common sources of As? Shellfish; Combustion of fuel; Metalalloys/glass/ceramics

What are some forms of As? Inorganic (arsenates, elementalarsenic); Organic (arsine)—generallynontoxic

Which form is generally more toxic? Inorganic trivalent forms (i.e., arsenite)

What makes As particularly attractive as Resembles sugar and tastelessa poison?

What are two primary routes of As 1. Inhalationexposure? 2. Ingestion

What is the primary mechanism by which Uncouples oxidative phosphorylation;As exerts its toxicities? Inhibits mitochondrial enzymes;

Binds to globin portion of hemoglobin

What are some clinical features of acute Nausea, emesis, diarrhea, ECGAs due to inorganic salts? changes, dysrhythmias, shock,

hematuria, seizure, coma, bonemarrow suppression, and peripheralneuropathies

What are some clinical features of chronic Cirrhosis, hematopoietic malignancies,toxicity due to As? dermatitis, stocking-glove sensory

neuropathy, and cancer

What are some methods to detect As? Blood levels (<5 mcg/L normal);Difficult to differentiate organic versusinorganic; Urine “spot” testing

What are some key points in the Supportive care; Appropriate labmanagement of acute As toxicity? testing; Consider use of chelating

agent

What are some chelating agents used? Dimercaprol; D-penicillamine;Succimer

Toxicological Emergencies 325

Page 342: Deja review _emergency_medicine__deja_review_

What are some functions of chelating Bind to metal to facilitate excretion;agents? Deplete tissues of metals

Lead

Which populations are at the greatest risk Adults through occupationalof lead poisoning? exposures; Children through lead-

based paints

What are some common sources of lead? Ammunitions; Car radiators; Ceramicware with lead glazes; Batteries;Paints; Moonshine

What are major routes of absorption Ingestion; Dermal absorption;of lead? Inhalation

What is the primary site of lead absorp- Bones (>90% in adults compared totion in the body? 75% in children)

What are the long-term cognitive deficits Learning, behavioral disorders,associated with elevated lead levels? and decreased intelligence

How is lead typically absorbed in the Lead initially attaches to red bloodbody? cells and then distributes to various

locations such as the brain, kidneyand bones

What are some clinical features of Abdominal pain, nausea, emesis,acute lead toxicity? lethargy, fatigue, seizure, and coma

What are some clinical features of Nephritis, peripheral neuropathy,chronic lead toxicity? myalgias, anemia, and motor

weakness

What are some other diagnostic tests X-ray fluorescence; Nerve conductionto consider? velocity testing; Neurobehavioral

testing

What are the classic laboratory findings Basophilic stippling; Anemia;of lead poisoning? Hemolysis

What is a normal lead level? <10 ug/dL

What are some key points in the manage- Removal of lead source (i.e., stripment of lead poisoning? paint); Chelating agents

What are some commonly used chelating EDTA; Succimer; Dimercaprolagents?

What are some functions of chelating They bind inorganic metals andagents? enhance excretion via the kidneys

and GI tract; They can also depletelevels from soft tissues to be excreted

326 Deja Review: Emergency Medicine

Page 343: Deja review _emergency_medicine__deja_review_

Hydrocarbons

What are some important things to know Common cause of mortality inabout hydrocarbons? children; Hydrocarbons are

ubiquitous; Hydrocarbons commonlyingested/aspirated

What are some common sources of Gasoline; Motor oils; Petroleum jelly;hydrocarbons? Laxatives; Solvents

What are two primary routes of 1. Ingestionhydrocarbon toxicity? 2. Inhalation

What are some hydrocarbons with systemic Aromatic hydrocarbons; Halogenatedeffects? hydrocarbons

What are some clinical features of Drowsiness, seizures, coma, nausea,hydrocarbon ingestions? emesis, and in cases where there is

aspiration of hydrocarbons, patientswill exhibit respiratory involvementsuch as dyspnea, coughing, distress,and even hypoxia/cyanosis

How do most patients do after hydrocarbon Most are asymptomatic after ingestioningestion?

What is an important complication of Aspirationhydrocarbon ingestion?

What are some physical properties that Greater volatility; Lesser viscosity;predict the aspiration potential of Surface tensionhydrocarbons?

What are some signs that aspiration may Typically patients will cough, gag, andhave occurred? exhibit dyspnea on exertion

What are some indications for patients Symptomatic after 6 hours; Abnormalwith hydrocarbon ingestion of admission? CXR suggestive of aspiration

What does “sniffing,” “bagging,” or Inhalation of volatile hydrocarbons“huffing” imply? with the intention of getting high

What are some clinical features of inhaling Euphoria, agitation, seizure, stupor,hydrocarbons? and delusions

What is the most feared complication of Sudden death (fatal dysrhythmias)inhaling halogenated hydrocarbons?

What is the mechanism by which Heart is sensitized to circulating halogenated hydrocarbons can cause catecholamines, so any suddenfatal dysrhythmias? increase in sympathetic response can

cause fatal dysrhythmias

What are some hydrocarbons that may Asphalt; Tarcause thermal burns?

Toxicological Emergencies 327

Page 344: Deja review _emergency_medicine__deja_review_

What are some key points in the Supportive care is the mainstaymanagement of hydrocarbon toxicity? (i.e., ABC); Monitor carefully for

respiratory involvement; Avoidemetic agents (i.e., ipecac); AC is notparticularly useful; Standard ACLSfor dysrhythmias

Methemoglobin

What is methemoglobin? Abnormal hemoglobin (Hg) that is inthe ferric state (Fe 3+) rather then theferrous state (Fe 2+) that renders itunable to accept oxygen or carbondioxide

What are some of the physiologic effects Reduces the oxygen-carrying capacity;of methemoglobin on oxygen-carrying Left shift of the dissociation curvecapacity?

What is the normal level of methemoglobin <1% of total hemoglobinin a healthy adult?

What are the two primary mechanisms by 1. NADH electron donation of ferricwhich methemoglobin is eliminated? to ferrous

2. NADPH (accounts for smallportion)

What are two common causes of congenital 1. NADH methemoglobin reductasemethemoglobinemia? deficiency

2. Hemoglobin M

What is the most common cause of Acquired methemoglobinemiamethemoglobinemia?

What is the mechanism by which acquired Commonly occurs due to drugs ormethemoglobinemia occur? toxins that oxidize ferrous iron

List some common causes of acquired Local anesthetics (most commonmethemoglobinemia? cause); Nitrites; Sulfonamide; Dapsone

What is the hallmark of methemo- Cyanosis that fails to improve withglobinemia? high-flow oxygen

What are some clinical features of Largely dependent on level ofmethemoglobinemia? methemoglobin: fatigue, anxiety,

dizziness, tachycardia, mental statuschange, and dysrhythmias/acidosisat higher levels

What is the methemoglobin level at which Methemoglobin levels of 15%central cyanosis appears?

328 Deja Review: Emergency Medicine

Page 345: Deja review _emergency_medicine__deja_review_

Aside from persistent cyanosis, what are Chocolate brown appearance of bloodsome other diagnostic clues of methemo- on filter paper; Normal partial pres-globinemia? sure of oxygen on ABG; MetHb level

determined by cooximetry

What is the treatment of choice for Methylene bluemethemoglobinemia?

What is the mechanism of action of Increases erythrocyte reduction ofmethylene blue? methemoglobin to oxyhemoglobin

What are some adverse reactions to Hemolysis in G6PD deficiency;methylene blue? Methemoglobinemia at high doses;

False low pulse ox readings

Carbon Monoxide

What are some important things to know Leading cause of poisoning in theabout carbon monoxide (CO) poisoning? United States; Majority of cases due

to fires; Suicide contributes to a goodportion of cases; CO is odorless andcolorless

List some sources of CO. Incomplete combustion of carbona-ceous material (i.e., engine exhaust);Degradation of heme; Verticaltransmission (maternal-to-fetal);Halogenated hydrocarbons

What is the pathophysiology of CO CO binds with Hb forming carboxy-poisoning? hemoglobin (COHb that decreases

oxygen content of blood and will alsoshift O2-Hb dissociation curve to theleft (decease oxygen delivery to tissue)

What two organ systems are most 1. CNS profoundly affected by CO poisoning? 2. CVS

Acute Symptoms associated with CO levels

COHb Level Symptoms

10–20% Flu-like symptoms such as headache and nausea20–30% Severe headache, irritability, and impaired judgment40–50% Loss of consciousness and confusion60–70% Unconsciousness, cardiovascular collapse, seizure>80% Rapidly fatal

Toxicological Emergencies 329

Page 346: Deja review _emergency_medicine__deja_review_

What are some important points to know Smokers can have levels as highabout COHb levels? as 10%; Does not predict neurologic

sequelae

What are some clinical features of COpoisoning in the following organ systems:

CNS Headaches, dizziness, blurred vision,ataxia, seizure, coma, and even death

CVS Signs of demand ischemia (i.e., chestpain), hypotension, and dysrhythmias

Respiratory Pulmonary edema and ARDS

Renal ARF (2° to rhabdomyolysis)

Dermal Characteristic cherry-red color (moreso after massive exposure and death)

What is an important neurologic Delayed neurologic sequelae (DNS)complication after CO poisoning?

What is DNS? Neurologic deterioration after a lucidperiod of around 2 weeks

What are some clinical features of DNS? Ataxia, tremor, amnesia, memoryimpairment, paralysis, and dementia

When do the symptoms of DNS resolve? Range from 1 month to 1 year depend-ing on severity

What is the concern of the fetus with Fetal Hb binds CO more avidly thanregards to CO poisoning? maternal Hb, which can result in

anoxic brain injury and death ofthe fetus

What are some key points in the Remove from source as soon asmanagement of CO poisoning? possible; Administer 100% O2

immediately; Check COHb by co-oximetry; ABG/ECG when indicated;Hyperbaric oxygen when indicated

What are some indications for the use of Evidence of end-organ damagehyperbaric oxygen (HBO) in CO poisoning? (i.e., LOC); COHb levels >25%;

COHb >15% for pregnant women/child

Persistent symptoms after 1 atm O2

FIO2 COHb T1/2

Room air 2–6 hours100% at 1 atm 90 minutes100% at 3 atm 30 minutes

330 Deja Review: Emergency Medicine

Page 347: Deja review _emergency_medicine__deja_review_

Cyanide and Hydrogen Sulfide

What are some important sources of Combustion of many types of material;cyanide (CN)? Smoking; Food sources (i.e., amyg-

dalin); Ingestion of cyanide salts (i.e.,homicide)

What is the pathophysiology of CN Inhibition of cytochrome oxidasetoxicity? (essential for oxidative phospho-

rylation) that results in cellularhypoxia leading to increased anaerobicmetabolism (lactic acidosis)

Name three routes of exposure for CN. 1. Parental2. Inhalation3. Ingestion

What are some clinical features of acute Headache, confusion, lethargy,CN toxicity? hypotension, abdominal pain, nausea,

vomiting, traditional cherry-red skin,and severe metabolic acidosis

When should one suspect CN toxicity? A fire victim with a coma and acidosis;Bitter almond odor; Unexplainedcoma/acidosis (i.e., in laboratory orindustrial work)

What role does CN levels play in the acute They cannot be obtained rapidly, somanagement of cyanide? must use clinical judgement

What is a common laboratory finding in Severe metabolic acidosis with greateracute CN toxicity? anion gap

What is the initial management for patients Supportive care (i.e., establish airway);with suspected CN toxicity? Sodium bicarbonate for acidosis; Treat

associated conditions (i.e., CO);Consider use of antidote

What is the antidote typically given for Cyanide antidote kiteCN toxicity? Sodium nitrite; Sodium thiosul-

fate; Amyl nitrite pearls

What is the mechanism by which nitrite Induces a methemoglobinemia, foradministration works? which CN has a greater affinity

What antihypertensive is known to contain NitroprussideCN?

What other toxin produces effects similar Hydrogen sulfideto CN?

What are some sources of hydrogen Natural sources (i.e., sulfur springs);sulfide? Industrial sources; Decay of sulfur-

containing products (i.e., fish)

Toxicological Emergencies 331

Page 348: Deja review _emergency_medicine__deja_review_

What is the pathophysiology of Similar to CN, but binds to the samehydrogen sulfide? enzyme with greater affinity then CN

and also causes mucous membraneirritation

What are some clinical features of Hypoxia, irritation to areas such ashydrogen sulfide toxicity? eyes, throat, and nasal passage, and

severe metabolic acidosis

When should the diagnosis of hydrogen Rapid loss of consciousness; Odor ofsulfide be suspected? rotten eggs; Rescue from an enclosed

space; Multiple victims

What is a common laboratory finding in Severe metabolic acidosishydrogen sulfide poisoning?

What is the initial management in Remove the patient from the source;patients with suspected hydrogen Supportive care; Nitrite may be ofsulfide toxicity? some use; Consider HBO therapy

Pesticides

What is a pesticide? Agent commonly used to destroy orrepel pests such as insects or rodents

What is the mechanism of organophosphate Bind to cholinesterases, especiallytoxicity? acetylcholinesterases, preventing the

breakdown of acetylcholine (ACh)

What is the mechanism of toxicity of Cholinergic poisoning due to excessiveorganophosphates? accumulation of ACh

What are the clinical effects primarily Excessive ACh at the nicotinic receptorsdue to? (autonomic ganglia and skeletal

muscle) and muscarinic receptors

What are some factors that determine the Route of exposure; Lipid solubility;clinical effects? Dose

What is “SLUDGE” syndrome? Clinical effects due to excessiveACh at the muscarinic receptors

Salivation

Lacrimation

Urination

Diarrhea

GI cramps

Emesis

What are some other clinical features of Bronchoconstriction, bronchorrhea,excessive muscarinic activation? miosis, and bradycardia

332 Deja Review: Emergency Medicine

Page 349: Deja review _emergency_medicine__deja_review_

What are some CNS effects of excessive Agitation, confusion, coma, andACh activity? seizure

What are the nicotinic effects of excessive Fasciculations, muscle weakness, andACh acitivty? paralysis

What is the initial management of Supportive care; Decontamination oforganophosphate toxicity? patient; Consider use of an antidote

What are two antidotes that can be used in 1. Atropineorganophosphate toxicity? 2. Pralidoxime (2-PAM)

What is the mechanism of atropine? Competitive inhibition of ACh only atmuscarinic receptors

What is the endpoint of atropine therapy? Drying of secretions

What is the mechanism of pralidoxime? Regenerates organophosphate-boundacetylcholinesterase complex, regener-ating its ability to metabolize ACh

TOXICOLOGY SUPPLEMENT

Toxicological Emergencies 333

Toxin Antidote

Acetaminophen N-AcetylcysteineAnticholinergics PhysostigmineArsenic D-penicillamine/

DimercaprolBenzodiazepines FlumazenilBeta-blockers Glucagon

Black widow spider Latrodectus antiveninBotulism Botulinum antitoxinBrown recluse spider Loxosceles antiveninCalcium channel blockers Glucagon and calciumCoral snake bite Elapid antivenin

Cyanide Amyl nitrite, sodiumnitrite, sodiumthiosulfate

Digitalis glycosides Digoxin-specific FABEthylene glycol Ethanol or fomepizoleHeparin ProtamineHydrogen sulfide Sodium nitrite

Page 350: Deja review _emergency_medicine__deja_review_

Toxidromes Temp HR RR BP Pupil Diaphoresis MS

Anticholinergic ↑ ↑ +/− − ↑ ↓ DeliriumCholinergic − +/− +/− +/− +/− ↑ NormalSympathomimetic ↑ ↑ ↑ ↑ ↑ ↑ AgitatedSedative-hypnotics ↓ ↓ ↓ ↓ +/− − Depressed

or ethanolOpioids ↓ ↓ ↓ ↓ ↓ − DepressedWithdrawal from − ↑ − ↑ ↑ ↑ Normal

opioids anxiousWithdrawal from ↑ ↑ ↑ ↑ ↑ ↑ Agitated

sedative- confusedhypnoticsor ethanol

334 Deja Review: Emergency Medicine

(Continued)

Toxin Antidote

Hypoglycemic agents DextroseIron DeferoxamineIsoniazid Pyridoxine (B6)Lead DimercaprolMethanol Ethanol or fomepizoleMethemoglobin Methylene blueMethotrexate Leucovorin and folateOpiates NaloxoneOrganophosphates AtrophineRattlesnake bites Crotalidae antivenin

(crofrib)Tricyclics Sodium bicarbonateWarfarin Vitamin K

Page 351: Deja review _emergency_medicine__deja_review_

335

C H A P T E R 1 7

BehavioralEmergencies

MEDICAL EVALUATION AND CLINICAL APPROACH

What are some important things to consider Is the patient a danger to self orin the clinical approach to patients with others?; Are physical symptoms psychiatric problems? a manifestation of a psychiatric

disorder?; Psychiatric disorders maybe exacerbated by a physical condition;Patients may present with a medicalproblem caused by a psychiatricdisorder

What are some features for each of thethe following triage categorization forpsychiatric patients (as well as for allother patients):

Emergent Patient has active suicidal ideation;Patient has homicidal ideation; Acutelyintoxicated; Life-threatening injury(i.e., myocardial infarction [MI]);Abnormal vital signs

Urgent Suicidal ideation; Agitation/anxiety;Incoherent patient

Nonurgent Does not meet criteria for the first two;Patient requests psychiatric help

What is a very important thing to keep in All psychiatric patients should receive mind when evaluating a psychiatric both a thorough psychiatric andpatient? medical evaluation

How do you deal with a patient who may Restraints or seclusionhave uncontrolled behavioral problems?

What are some warning signs that a patient Abrupt changes in behavior; with a psychiatric problem may require Threatening violent behavior; Patientrestraints? states in fear of losing control

Copyright © 2008 by The McGraw-Hill Companies, Inc. Click here for terms of use.

Page 352: Deja review _emergency_medicine__deja_review_

What are some characteristics that Safety foremost; Continuous obser-a seclusion room should have? vation; Low stimulation (i.e., low

lights); Security staff

What are some key points for thefollowing types of restraints used:

Verbal restraint Should be attempted in a calmapproach; Encourage the patient totalk about any concerns and offerreassurance; Physical/chemicalrestraints may be needed

Physical restraint Commonly used for intoxicated,demented/delirious, and violentpatients; Has minimal side effectsand immediately reversible; Removerestraints when patient is not a dan-ger to self or others

Chemical restraint Behavioral control once full evaluationdone; Haldol and lorazepam drug ofchoice; Less intrusive than physicalrestraint; Benzodiazepines mayworsen dementia and delirium

What are some characteristics of a patient Patient may regard behavior aspresenting with a psychiatric disorder? normal; History of behavioral

problems; Often will have normalvitals and laboratory test results; Canhave hallucinations (i.e., auditory)

What are some medical conditions that Toxicological emergencies; Urinarymay present as behavioral emergencies? tract infection; Drug withdrawal (i.e.,

EtOH); Myocardial infarction (MI);Diabetic ketoacidosis; Chronic renaldisease; Thyroid dysfunction

What are some laboratory tests to consider Glucose; Complete blood count (CBC)in evaluation of a psychiatric patient? Urinalysis; Lytes (also calcium);

Toxicology screen; Carboxyhemoglobinlevel

What are some elements in the medical Contact current and past primaryhistory to consider when evaluating doctors; Obtain all medical anda patient with a psychiatric problem? psychiatric records; List of medica-

tions, especially sedatives/psych/pain medications; Always ask aboutalcohol and drug use

What are the key components of themental status examination (MSE):

Level of consciousness Alert; Fluctuating; Somnolent

336 Deja Review: Emergency Medicine

Page 353: Deja review _emergency_medicine__deja_review_

General appearance Overall appearance (i.e., hygiene);Movement: chores, tics, tremors,etc.; Activity level (i.e., agitation)

Orientation Person, place, time, and event

Memory Immediate, STM, and LTM; Threeword recall

Mood Stability; Quality (i.e., moodyvs. anger)

Speech Fluency, rate, and rhythm; Illogicalversus logical

Thought content Perception (i.e., hallucination);Bizarre thoughts; Delusions

Insight and judgment History can usually infer this

Cognitive function Ask to perform task such as spellinga word backward or serial 7s

DEPRESSION AND SUICIDE

What are the symptoms of major Five or more of the followingdepressive disorder? symptoms for 2 weeks or greater:

Anhedonia; Depressed mood;Fatigue; Sleep disturbance; Changein appetite; Inability to concentrate;Sense of worthlessness; Suicidalthoughts

What are some important points in the Medications (i.e., beta-blockers);history to consider when evaluating History of drug use; Neurologica patient with depression? conditions (i.e., CNS tumor);

Endocrine conditions; Infectious dis-ease (i.e., HIV); Any previous psychi-atric history; Suicidal or homicidalideation; Any recent life changes;Evaluation of social structure (i.e.,family)

What is the primary goal when evaluating His/her potential for suicidea patient with depression?

What are some factors to consider when Previous attempts at suicide; Socialdeciding to admit a patient with support; Younger or older males aredepression? more at risk; Plan and means to carry

out suicide; Excessive use of drugs oralcohol

Behavioral Emergencies 337

Page 354: Deja review _emergency_medicine__deja_review_

What are some discharge criteria to If a support environment exists; Agreesconsider in a depressed patient? to return if depression worsens; Not

demented, delirious, or intoxicated;Close follow-up

Should antidepressants be started in the No—antidepressants take up toED prior to discharge of the patient? 4 weeks to work and will not acutely

treat depression in the ED

How many people who attempt suicide are For every 20 attempts, 1 is successfulsuccessful?

When do suicide attempts most commonly During a crisis marked by an acuteoccur? personal loss

What are some common psychiatric Depression; Schizophrenia; illnesses associated with completed Personality disorders; Panic disorderssuicides?

What role does gender play in suicide? Females attempt suicide three timesmore often; Males are successful threetimes more often

Does drug abuse play a role in suicide? Yes—one fourth of successful suicidesinvolve drugs and alcohol and up tohalf in adolescent suicides

List some risk factors associated with Underlying psychiatric illnesses;suicide attempts? Age (rate highest in elderly); Chronic

pain (i.e., cancer); Marital status(marriage is protective); Presence oflethal means; Family history

What is the most common cause of death Firearmsin suicide in all age groups?

What are some warning signs of suicide? Recent life changes; Depression

Will asking a patient directly about suicide No—one should always askintent put ideas into his/her head?

What are key questions to ask a patient Ask if they are suicidal; Ask if theywho expresses suicidal intention? have a plan; Assess if they have the

means

What is “silent suicide?” Killing oneself slowly via nonviolentmeans such as not taking medication

What age group is “silent suicide” most Elderlycommon in?

Can a suicidal patient leave American If they are found to be incompetent or Medical Association (AMA)? dangerous, they cannot leave AMA

338 Deja Review: Emergency Medicine

Page 355: Deja review _emergency_medicine__deja_review_

ACUTE PSYCHOSIS

What is the definition of acute psychosis? Break in reality often characterized bydelusions, hallucinations, anddisorganized speech/movement

Define the following terms:

Hallucinations False perception of a sensory modalitythat is not there with auditory stimulibeing the most common

Delusions Fixed falsely held belief that is notaccepted by a given cultural groupand is held despite an evidence to thecontrary

Catatonia Apparent detachment from theenvironment typically characterizedby frozen rigid posture or violentagitation

What are some examples of negative Poverty of speech; Loss of volition; symptoms? Flat affect

What is a major psychiatric disorder that Schizophreniacan present as an acute psychotic episode?

What is the prevalence of schizophrenia 1% regardless of race or genderin the general population?

When is the onset of schizophrenia? Commonly by late adolescence toearly adulthood

What does the diagnosis of schizophrenia Severe impairment in the level ofrequire? functioning; Duration of >6 months;

At least two symptoms of acutepsychosis for greater than a month;Exclusion of medical conditions ascause of symptoms

What are some features for each of thefollowing psychiatric disorders that maypresent as acute psychosis:

Schizoaffective disorder Psychosis that is chronic; It is oftenassociated with mood disorders;Psychotic features can occur withoutmood symptoms

Schizophreniform disorder Psychosis that lasts <6 months; Doesnot occur during a mood disorder

Behavioral Emergencies 339

Page 356: Deja review _emergency_medicine__deja_review_

Brief psychotic disorder Psychosis that lasts <1 month; Doesnot occur during a mood disorder

Major depression with psychotic Psychosis that occurs during features a depressive episode

What is the most important thing to do Establishing safetywhen evaluating a patient who is psychotic?

What are some things to do to ensure safety Search for weapons; Use restraintswhen evaluating a patient with acute if necessary; Avoid having the patientpsychosis? between you and an exit point

Should all patients with acute psychosis be No, but patients who are a danger toadmitted? others or themselves should probably

be admitted

MANIA

What defines a manic episode? Three or more of the following forover 1 week:

Impulsivity; Distractibility;Pressured speech; Grandiosity;Decreased need for sleep; Agitation;Flight of ideas

Can a patient with a manic episode also Yes—often with paranoia orhave acute psychosis? grandiosity

What are some medical conditions that can CNS tumors; Hyperthyroidismcause mania?

What are some medications/drugs that are Phencyclidine; Steroids; EtOH;known to cause mania? Psychostimulants

What are some elements of the history to Current medications; History ofattain when evaluating a patient with illicit drug use; Any prior psychiatric mania? history; Any homicidal or suicidal

ideation; Any recent life stressors

Are antimanic medications such as lithium No—take days to weeks to take effector carbamazepine useful for an acuteepisode of mania?

What class of drugs are useful for an acute Antipsychotic medicationsepisode of mania? (i.e., haloperidol)

What are some factors when deciding if Impulsivity leads to danger to self ora patient with acute mania should be others; Poor support structure; Activeadmitted? delusions that are dangerous

340 Deja Review: Emergency Medicine

Page 357: Deja review _emergency_medicine__deja_review_

PANIC ATTACKS

What are some clinical features of Tremor; Shortness of breath;a panic attack? Paresthesias; Derealization; Chest pain;

Tachycardia; Sense of impendingdoom

Are patients with panic disorder at Yes—up to 18 times more than theincreased risk of suicide? general population

Are patients who present with a panic No—during a panic attack, the patientattack just overreacting? truly feels threatened and commonly

needs reassurance

What are some medical conditions that Asthma; Chronic obstructive pul-may mimic a panic attack? monary disease (COPD); Metabolic

disturbances; Dysrhythmias; Hypoxia

What are some characteristics of a panic Typically begins suddenly; Lasts forattack? about 15 minutes; Can occur without

provocation

What are some elements of the history to Current medications; Any priorattain when evaluating a patient with psychiatric history; Excessive caffeinemania? use; Any recent life stressors

What class of drugs are useful for the Benzodiazepinesshort-term management of a panic attack?

What is the most useful intervention Reassurance and communicationfor patients with a panic attack?

EATING DISORDERS

What are two eating disorders commonly 1. Bulimia Nervosa (BN)seen in the emergency department? 2. Anorexia Nervosa (AN)

What is bulimia nervosa? Chronic eating disorder that oftenwaxes wanes, typically exacerbatesduring times of stress characterizedby “binge and purge”

Describe the typical bulimic patient? A normal-appearing female aroundthe age of 18–24

How prevalent is bulimia? 5% of young adult females

What are some characteristic features of Most patients with bulimia binge,a binge? that is characterized by excessive

consumption of calories (up to14,000 Kcal!), concealing from friendsand family

Behavioral Emergencies 341

Page 358: Deja review _emergency_medicine__deja_review_

Are bulimics typically underweight? No—often have normal weight

Is binge eating typically from hunger? Not necessarily—commonly describedas as a feeling of loss of control

What is purging? Inappropriate compensatory responseto binging often characterized by self-induced emesis

What are some medical complicationsof bulimia:

Ipecac use Dermatomyositis; Cardiomyopathy

Diuretic use Electrolyte imbalance (i.e.,hypokalemia); Dehydration

Laxative use Constipation; Hypokalemia;Dehydration

Self-induced emesis Electrolyte imbalance; Dental prob-lems (i.e., erosions); Submandibular/parotid gland enlargement; May getesophageal tear or rupture

What are some clues during the history Loss of dental enamel; Unexplainedand physical exam that may point hypokalemia; Large fluctuations into bulimia? weight; Excessive exercise; Esophageal

problems (i.e., bleeding)

What are some indications for admission Metabolic complications (i.e., hypoten-for a patient who presents with bulimia? sion); Suicidal ideation; Persistent

emesis

What is anorexia nervosa (AN)? An eating disorder characterized bya preoccupying fear of obesity regard-less of weight loss

What are four diagnostic criteria of AN? 1. Preoccupying fear of gainingweight

2. Weight loss >15% of ideal bodyweight

3. Amenorrhea greater than threeconsecutive cycles

4. Distorted body image

What is the mortality rate of AN at Almost approaches 10%10 years?

What is characteristic of patients with Notorious for resistance to treatmentAN who are in treatment? and unmotivated

What are some clues during the history Excessive exercise; Unexplainedand physical exam that may point to AN? weight loss or growth problems;

Activity or occupation (i.e., dancer)

342 Deja Review: Emergency Medicine

Page 359: Deja review _emergency_medicine__deja_review_

What are some key points in the Correct any underlying metabolicmanagement of patients with AN? problems; Initial evaluation may

require psychiatric involvement;Determine if outpatient treatment ispossible

DEMENTIA AND DELIRIUM

Why are dementia and delirium important Patients with dementia or deliriumto consider? often have impaired ability to recog-

nize their condition and may be sus-ceptible to injury

What is dementia? Progressive and global impairmentof cognitive function without alter-ation in consciousness

What are some causes of irreversible Alzheimer’s disease; Vasculardementia? dementia (multi-infarct); Creutzfeldt-

Jakob diseases; Parkinson’s disease

What are some clinical features of Multiple cognitive deficits thatdementia? include memory impairment along

with either or some of the following:apraxia, aphasia, and agnosia

Is the onset of dementia typically acute? No—gradual onset with disturbancesin recent memory that can be exacer-bated by illnesses or certainmedications

What are some causes of reversible Medication; Metabolic disorders;dementia? Endocrine disorders; Depression

(pseudodementia)

What are some management points in the Eliminate medications that maytreatment of dementia? exacerbate the condition; Identify

and correct any underlying metabolicor endocrine disorder; If dementia isirreversible, consider medication thatmay slow the progression

What are some clinical features Acute onset with often diurnalof delirium? fluctuation of symptoms, cognitive

impairment, and reduced ability tofocus and sustain attention

What are some important causes of Drugs and medications; Heavy metals;delirium to consider? CNS injury; Infection; Metabolic

disturbances

What is the treatment of dementia? Identify and treat the underlyingcause; Ensure the safety of a patient

Behavioral Emergencies 343

Page 360: Deja review _emergency_medicine__deja_review_

INTOXICATION AND WITHDRAWAL

What is intoxication? Ingestion of a drug or alcohol thatoften leads to impairment of judg-ment, perception, motor activity, andattention

What are some clinical features of Primarily manifests as impairmentintoxication? of judgment and motor activity

with progression to delirium,coma, seizure, or even death withincreasing amounts

How is the diagnosis of intoxication Laboratory evaluationtypically made?

What are some substances that cause Cocaine; Methamphetamine; psychostimulant intoxication? Phenylpropanolamine

What are some clinical features of Can have paranoid psychotic psychostimulant intoxication? excitation, may have signs of sympa-

thetic response, and stereopathies(i.e., nail biting)

What medication class is useful for Antipsychotics (i.e., haloperidol)patient with psychostimulant intoxication?

What are some key points in the manage- Ensure safety of patient (i.e., restraints);ment of patients with psychostimulant General supportive measures; intoxication? Treatment of the intoxicating agent;

Appropriate referral to psychiatry ifneeded

What are some clinical features of alcohol Confusion, ataxia, agitation, slurredintoxication? speech, hallucinations, and possible

violent paranoid ideation

What is an appropriate medication class Antipsychoticsif behavioral control is needed?

What is withdrawal? Clinical syndrome that occurs withthe cessation of a substance and canbe reduced when the substance istaken again

What is the most commonly encountered Alcoholwithdrawal syndrome?

What are the clinical stages of alcoholwithdrawal from the time of last drink:

6–24 hours Hypertension, tachycardia, nausea,anxiety, and sleep disturbances

344 Deja Review: Emergency Medicine

Page 361: Deja review _emergency_medicine__deja_review_

24–72 hours More severe autonomic disturbancesand hallucinations and can take upto 6 days to resolve. Seizures canalso occur during this time

3–5 days Can progress to delirium tremens

What is delirium tremens? Potentially fatal form of ethanolwithdrawal

What are some clinical features of Autonomic instability, globaldelirium tremens? confusion, tremors, incontinence,

and hallucinations with a substantialmortality if left untreated

What is the treatment of acute Establish supportive care; IV fluidsalcohol withdrawal? along with thiamine, magnesium,

and multivitamin; Generally avoidgiving glucose before thiamine asthis may precipitate Wernicke’sencephalopathy; Sedation withbenzodiazepines is key

What are some indications for a head CT Focal seizures; Status epilepticus; in an alcoholic who has seizures? New-onset seizure

PSYCHOPHARMACOLOGY

What class of medications are commonly Benzodiazepinesused for short-term control of anxietyand agitation?

What are some indications for the use of Short-term management of anxiety;benzodiazepines? Control seizures; Alcohol withdrawal;

Induce muscle relaxation

Name two benzodiazepines commonly 1. Lorazepamused in the ED setting for psychiatric 2. Diazepamemergencies?

What are some side effects of Impairment of motor coordination;benzodiazepines? Respiratory depression; Ataxia at

higher doses; Potential for addiction

Name two benzodiazepines that 1. Diazepamhave potential for abuse? 2. Alprazolam

Is it possible to die from benzodiazepine Yeswithdrawal?

What are some advantages of using Minimal cardiovascular depression;lorazepam in the acute setting for Does not inhibit or induce cytochromebehavioral emergencies? isoenzymes; No active metabolites

Behavioral Emergencies 345

Page 362: Deja review _emergency_medicine__deja_review_

What is the primary concern of using Respiratory depressionhigh-dose benzodiazepines (especiallyIV route)?

Are overdoses of benzodiazepines No—unless concomitant ingestioncommonly fatal? with other sedatives such as alcohol

What are some indications of neuroleptics? Reduces aggression; Reduces psy-chotic thinking; Helps relieve anxiety

What is the primary mechanism of action Antagonizes dopamine receptors inof neuroleptics? the mesolimbic area within the CNS

What are some side effects of neuroleptics? Reflex tachycardia; Orthostatichypotension; Can lower seizurethreshold

What are some characteristics of Minimal cardiovascular effects;haloperidol that make it an ideal Effective at reducing agitation;neuroleptic to use in the ED? Minimal sedation; Rapid onset;

Synergistic with benzodiazepines

What side effect is common with Dystonic reactionshaloperidol?

What are some characteristics of atypical Effective for psychotic patients whoneuroleptics? are refractory to typical neuroleptics;

Effective for negative symptoms;Less likely to cause tardive dyskinesia,but more likely to cause akathisia

Give some examples of atypical Olanzapine; Quetiapine; Clozapineneuroleptics?

What are some examples of extrapyramidalsymptoms seen with antipsychotics:

Parkinsonism Commonly within the first month ofuse; Characterized by cogwheelrigidity, akinesia, masked facies, andbradykinesia; Reducing the dose canhelp symptoms

Dystonias Painful clonus of voluntary muscles;Typically involves the face and neck;Commonly within the first month ofuse; Treatment is with diphenhy-dramine or benztropine

Akathisia Internal sense of motor restlessness;Most common form involves pacingand an inability to sit still; Propranololis the medication of choice

What is neuroleptic malignant Rare, but life-threatening, idiosyncraticsyndrome (NMS)? reaction to a neuroleptic medication

346 Deja Review: Emergency Medicine

Page 363: Deja review _emergency_medicine__deja_review_

What are some clinical features of NMS? Characterized by fever, muscularrigidity, altered mental status, andautonomic dysfunction

Which types of neuroleptic are commonly Although potent neuroleptics (i.e.,associated with NMS? haloperidol) are more commonly

associated with NMS, all antipsychoticagents, typical or atypical, may pre-cipitate the syndrome

What is the diagnostic criteria of NMS? High fever with severe muscle rigidityand two or more of the following:

Change in mental status;Tachycardia; Tremor; Leukocytosis;Metabolic acidosis; Labile or highblood pressure; Elevated CPK

What is the treatment of NMS? Commonly requires an ICU setting;Stop all neuroleptics; Benzodiazepinesare the mainstay

Are there any emergent indications for No—they require weeks to take effectthe use of antidepressants in the ED?

What class of antidepressants were among Tricyclic antidepressants (TCAs)the first to be used to treat depression?

Name some examples of TCAs? Nortriptyline; Amitriptyline;Imipramine

What is particular to know about TCAs? Have a very low therapeutic index

What are some side effects of TCAs? Anticholinergic, orthostatic hypoten-sion, increased seizure risk, and havevarious cardiac effects

What class of antidepressants have a high Selective serotonin reuptake inhibitorstherapeutic index and largely replaced known as SSRIsTCAs?

What are some examples of SSRIs? Sertraline; Citalopram; Paroxetine

What are some indications of SSRIs? Depression; Anxiety; Posttraumaticstress disorders; Obsessive-compulsivedisorders

What are some side effects of SSRIs? Generally mild; Notable druginteractions; Toxic in only very highdoses

What is serotonin syndrome? It is an idiosyncratic reaction that canoccur with interactions betweenserotonergic agents such as SSRIs

Behavioral Emergencies 347

Page 364: Deja review _emergency_medicine__deja_review_

What are some clinical features ofserotonin syndrome:

Gastrointestinal Nausea, emesis, and diarrhea

Central nervous system Hyperreflexia, tremor, and altered MS

Autonomic instability Hyperthemia, diaphoresis, andorthostasis

What is the treatment of serotonin Primarily supportivesyndrome?

Name a class of antidepressants associated Monamine oxidase inhibitors (MAOIs)with hypertensive crisis with the ingestionof tyramine-containing foods?

What are some tyramine-containing foods? Aged cheese; Wine; Beer; Fava beans

What are some clinical features of Hypertension, chest pain, severehypertensive crisis? headache, tachycardia, and diaphoresis

What is the treatment of choice for Phentolaminehypertensive crisis?

348 Deja Review: Emergency Medicine

Page 365: Deja review _emergency_medicine__deja_review_

AA streptococcus (GAS) necrotizing fasciitis, 194AAA. See abdominal aortic aneurysmsabdomen, trauma to

blunt, 261causes of, 259diagnostic tests for, 259–260exploratory laparotomy and, 259hypotension, 259penetrating, 259, 261peritoneal signs, 259

abdominal aortic aneurysms (AAA)aorta diameter and, 114clinical features of, 114CT contrast with contrast, 115developmental risk factors for, 114flank pain and, 148management of, 115misdiagnosis of, 114pathophysiology of, 114physical findings in, 114plain abdominal film and, 114syncope and, 44ultrasonography, 115

ABG. See arterial blood gasabortion, 230abruptio placentae, 233

clinical vignette of, 238absence seizure, clinical vignette of, 46abuse. See child abuseAC. See activated charcoalAC currents, 273acalculous cholecystitis, 128Accidental Death and Disability: The Neglected

Disease of Modern Society, 1acetaminophen (APAP), 299–301

fever and, 215overdose, 300

acetone, 319acid-balance base, 21–23

abnormal, 22disturbance, 22laboratory tests and, 22

acids, injuries and, 275ACLS. See advanced cardiac life supportacquired immune deficiency syndrome (AIDS)

human immunodeficiency virus and, 182indicators of, 182toxoplasmosis and, 183

ACS. See acute coronary syndromeactivated charcoal (AC), 298, 307active external rewarming, 278acute abdominal aneurysm, clinical vignette of, 117acute angle-closure glaucoma, 55–56

clinical vignette of, 58acute aortic regurgitation

causes of, 110clinical features of, 111CXR and, 111ECG and, 111management of, 111

acute bacterial prostatitisclinical features of, 151etiologic causes of, 151management of, 152

acute coronary syndrome (ACS), 89ADP receptor inhibitors, 92aspirin, 91beta-blockers, 92clinical features of, 89–90clinical presentations of, 89glycoprotein IIb/IIIa inhibitors, 91heparin, 92modifiable risk factors of, 89morphine, 92nitroglycerin, 92non-modifiable risk factors of, 89

acute hemorrhage, 9acute mitral regurgitation, 108acute mountain sickness (AMS), 280–281acute myocardial infarction (AMI)

complications with, 93thrombolytics in, 92

acute otitis media (AOM), 59–60clinical vignette of, 75

acute psychosis, 339–340acute pulmonary edema, precipitating factors

of, 94acute renal failure (ARF)

community-acquired, 145complications of, 146–147definition of, 145diagnostic tests for, 146dialysis and, 146–147hospital-acquired, 145treatment of, 146

acute tubular necrosis (ATN), 146acute mountain sickness (AMS), clinical vignette

for, 294acyclovir, oral herpes simplex virus and, 178, 179,

180Addison’s disease, 162

clinical vignette of, 163adenomyosis, 228adenosine, pediatric cardiopulmonary resuscita-

tion and, 200ADH. See antidiuretic hormoneadnexal torsion, 227

clinical vignette of, 237ADP receptor inhibitors, acute coronary syndrome

and, 92adrenal

catecholamines, 161cortex, 161, 162crisis, 162insufficiency, 162, 172–173medulla, 161suppression, 162

advanced cardiac life support (ACLS), 2AGE. See air gas embolismAIDS. See acquired immune deficiency

syndrome

349

Index

Copyright © 2008 by The McGraw-Hill Companies, Inc. Click here for terms of use.

Page 366: Deja review _emergency_medicine__deja_review_

air gas embolism (AGE), 282airway management

complications with, 5need for, 5

airway obstruction, 5albuterol, hyperkalemia and, 18alcohol

intoxication, 344toxic, 318–320withdrawal, 344–345

alcoholic ketoacidosis, 159aldosterone, 13alkali burns, 49allergic reactions

antihistamine and, 12asthma medications and, 12corticosteroids and, 12foods associated with, 11penicillin and, 12

alpha-adrenergic blockage, 312altitude sickness, 280–281alveolar fractures, 74Alzheimer’s disease, clinical vignette of, 45AMA. See American Medical Associationamantadine, influenza virus and, 178ambulance, cost of, 2American Medical Association (AMA), suicidal

patient and, 338AMI. See acute myocardial infarctionaminoglycoside, endocarditis and, 102amitriptyline, 312amniotic fluid embolus, pregnancy and, 235amoxapine, 311amphetamines, 322–323amphotericin B, cryptococcal central nervous

system infection and, 183AMS. See acute-mountain sicknessanal fissure

clinical features of, 138definition of, 138management of, 138recurrence rate of, 139

anal foreign bodies, 139anaphylactoid reaction, 11anaphylaxis

clinical features of, 12definition of, 11diagnosis of, 12epinephrine and, 12pathophysiology of, 12

anesthesia, dental emergencies and, 73anion gap acidosis, normal, 22anion gap metabolic acidosis, 22anisocoria, 47Anopheles mosquito, malaria and, 187anorectal abscess, 138anorexia nervosa, 342–343anticholinergic, 312

asthma exacerbation and, 210anticoagulants, 304–305antidepressants, tricyclics, 312antidiuretic hormone (ADH), 13, 14antihistamine, 12antipsychotics, 346AOM. See acute otitis mediaaortic stenosis, 206

causes of, 109clinical features of, 109

clinical vignette of, 116CXR and, 110ECG and, 110management of, 110symptoms of, 109

APAP. See acetaminophenAPGAR (activity, pulse, grimace, appearanc e,

respirations) score, 201appendiceal obstruction, causes of, 127appendicitis

causes of, 127clinical vignette of, 143definition of, 126diagnostic tests for, 127management of, 127migratory pain and, 127pathophysiology of, 127pediatric, 211

clinical vignette, 223perforation and, 127signs of, 127symptoms of, 127

ARF. See acute renal failurearrhythmias, near-drowning and, 276arsenic (As), 325–326arterial blood gas (ABG), 97As. See arsenicASA. See aspirinascending cholangitis, 128

clinical vignette of, 144aspirin (ASA), 3

fever and, 215asthma, 79–80

bronchial, clinical vignette of, 86definition of, 209exacerbation

clinical features of, 210clinical vignette, 223management of, 210patient admission and, 211patient discharge after, 211risk factors for, 210

medications for, 12pathophysiology for, 209triggers for, 209

ATN. See acute tubular necrosisatropine, 333

pediatric cardiopulmonary resuscitationand, 200

avascular necrosis, hip dislocation and, 265avulsed permanent tooth, 74azithromycin

chancroid and, 186ciprofloxacin with, granuloma inguinale

and, 187nongonococcal urethritis/cervicitis and, 185

azotemia, 145prerenal, 145, 146renal, 146

BB2-agonist, asthma exacerbation and, 210babesiosis, 291back pain, lower. See lower back pain (LBP)bacteremia, 10bacteria

cellulitis and, 189flesh-eating, 193

350 Index

Page 367: Deja review _emergency_medicine__deja_review_

invasive, 142toxin-producing, 142

bacteria-induced diarrhea, 142–143Campylobacter, 143Escherichia coli (enterotoxigenic), 142Escherichia coli serotype O157:H7, 142Salmonella, 143Shigella, 143Staphylococcus aureus, 142Vibrio cholera, 142Vibrio parahaemolyticus, 142

bacterial conjunctivitis, 52bacterial endocarditis, 191bacterial meningitis, 46

clinical vignette of, 46bacterial pneumonia, 77–78bacterial prostatitis, clinical vignette of, 154bacterial tracheitis, 207Bactrim, doxycycline with, granuloma inguinale

and, 187bagging, 327bag-valve-mask (BVM), 5balloon tamponade, gastrointestinal bleeding

and, 125barbiturates, 317barotitis interna, clinical vignette for, 295barotrauma, 282Bartonella henselae, 284bases, injuries and, 275basic life support (BLS), 2basilar skull fracture, clinical vignette of, 270BCI. See blunt cardiac injuryBeck’s triad, 105, 256behavioral emergencies. See psychiatric patientsbenign prostatic hyperplasia (BPH), clinical

vignette of, 154benzodiazepine, 317, 321, 341, 345

neonatal seizures and, 204benztropine, 314beta human chorionic gonadotrophin (β-hCG), 226beta-blockers, 308–309

acute coronary syndrome and, 92hypoglycemia and, 157

β-hCG. See beta human chorionic gonadotrophinbicarbonate-carbonic acid system, 21biguanides, 307binge, 341, 342bioprosthetic valves, 111bites. See also insect bites

from cat, 283–284from dog, 284from human, 284–285from snakes, 286

black widow, 289, 290bladder cancer, clinical vignette of, 154bladder, injuries to, 262, 263bleeding disorders

components of, 165hemophilia, 166vWF, 166–167

bloodcirculating volume of, 8infusion of, 9types of, 9

blood pressure reduction, aggressive, 115blood smear, malaria and, 188BLS. See basic life supportblunt cardiac injury (BCI), 257–258

BNP. See brain-type natriuretic peptidebodily fluids, infectious, 195body temperature, 215body water deficit (BWD), 17body weight, water and, 12body-packers, 321body-stuffing, 321Boerhaave’s syndrome, 121–122boil, 190bony oral-maxillofacial injury, 248–250Borrelia burgdorferi, 292BPH. See benign prostatic hyperplasiabrain injury. See head injury; traumatic brain

injurybrain-type natriuretic peptide (BNP), 94brief psychotic disorder, 340British antilewisite (BAL), 325brodifacoum, 304bronchial asthma, clinical vignette of, 86bronchiolitis, 208–209bronchogenic cancer, clinical vignette of, 86Broselow tape, 199brown spiders, 289, 290, 291bulimia nervosa, 340–342bullous myringitis, 60

clinical vignette of, 75bupropion, 311burns, 271–273

airway and, 272alkali, 49causes of, 271circumferential, 273clinical features of, 271–272complications and, 272full thickness (third degree), 272management of, 272, 273musculoskeletal (fourth degree), 272partial thickness (second degree), 271rule of nine, 271superficial (first degree), 271thermal, 271topical antibacterial agents for, 273total body surface area and, 272

burst fracture, 254buspirone, 317BVM. See bag-valve-maskBWD. See body water deficit

Ccalcium channel blockers (CCB), 309–310calcium gluconate, hyperkalemia and, 18calcium oxalate, 148calcium stone, 148calculous cholecystitis, 128Campylobacter, 143CA-MRSA. See community-acquired methicillin-

resistant Staphylococcus aureusCandida albicans, vulvovaginitis and, 235Capnocytophaga canimorsus, 284carbon monoxide (CO) poisoning,

329–330carbuncle, 190cardiac arrest, pediatric, 200cardiac glycosides, 307–308cardiac tamponade, 105, 256

echocardiography and, 105management of, 105

cardiogenic pulmonary edema, 93

Index 351

Page 368: Deja review _emergency_medicine__deja_review_

cardiogenic shockcause of, 10, 11clinical features of, 11definition of, 10evaluation of, diagnostic tests for, 11laboratory tests for, 11

cardiomyopathyclassifications of, 99dilated, 98–99echocardiographic evaluation for, 99hypertrophic, 100–101restrictive, 99–100

cardiopulmonary resuscitation, pediatriccricoid ring and, 199fetal, 198intrapartum, 198intubation, 198larynx and, 199maternal, 198mechanical ventilation, 199percutaneous transtracheal ventilation, 199trachea and, 199tracheal medication administration and, 200tracheal tube size and, 199vascular access and, 199–200

carvedilol, 308cat bites, 283–284catatonia, 339cat-scratch disease, 283–284cauda equina syndrome

clinical vignette of, 45lower back pain and, 43

CCB. See calcium channel blockersCD. See Crohn’s diseaseCD4+ T-cell count, 182–183, 184cefixime, gonococcal urethritis/cervicitis and, 185ceftriaxone

chancroid and, 186gonococcal urethritis/cervicitis and, 185

Celexa, 310celiotomy, 258cellulitis

bacteria and, 189children and, 189clinical features of, 189community-acquired methicillin-resistant

Staphylococcus aureus (CA-MRSA)and, 189

definition of, 189treatment of, 189

Centor criteria, 67central diabetes insipidus, 16central nervous system (CNS) disease

cryptococcal, 183human immunodeficiency virus and, 183

central pontine myelinolysis, 15central retinal artery occlusion (CRAO), 54–55

clinical vignette of, 58central retinal vein occlusion (CRVO), 55cephalic tetanus, 288

clinical vignette for, 293cerebral concussion, 244cerebral contusion, 244cerebral edema, hyponatremia and, 14cerebral malaria, 188cerebral vascular accident (CVA)

clinical vignette of, 45depressed level of consciousness and, 36

hemorrhagic stroke, 35hypertension and, 36ischemic stroke and, 35National Institutes of Heart stroke scale, 33–34occluded vessels and, 36risk factors for, 35thrombolytic therapy and, 36–37

cerebrospinal fluid rhinorrhea, 63cerebrospinal (CSF) leak, 246

oral-maxillofacial fractures and, 249cervical cancer, HPV and, 187chalazion, 54chance fracture, 254chancroid, 152, 186chemical injuries, 275–276chemoprophylaxis, meningitis and, 33CHF. See congestive heart failurechickenpox, 179–180child abuse

clinical vignette, 224laboratory tests and, 221shaken baby syndrome, 222skeletal survey for, 221types of, 221

chlamydia, 66Chlamydia psittaci, 78chlamydial infections, 185chloramphenicol, tick bites and, 292chloride-resistant alkalosis, 23chloride-sensitive alkalosis, 23chloroquine, 188cholecystitis, 128

admission criteria for, 129biliary scintiscanning and, 129clinical features of, 128CT and, 129diagnostic studies for, 129management of, 129ultrasound and, 129

cholelithiasis, developmental risk factors for, 128cholinergic crisis, 40chronic aortic regurgitation, 110chronic bronchitis, 81chronic hypertension, pregnancy and, 232chronic mitral regurgitation

causes of, 107clinical features of, 107CXR and, 108ECG and, 108management of, 108

chronic obstructive pulmonary disease (COPD),80–81

clinical vignette of, 86chronic pancreatitis, 130chronic renal failure (CRF)

causes of, 147clinical vignette of, 155definition of, 147hemodialysis complications of, 148stages of, 147treatment for, 147

chronic villus sampling (CVS), 19Chvostek’s sign, 19cimetidine, hypoglycemia and, 157ciprofloxacin

azithromycin with, granuloma inguinale and, 187chancroid and, 186gonococcal urethritis/cervicitis and, 185

352 Index

Page 369: Deja review _emergency_medicine__deja_review_

circle of Willis, 35circulation

anterior, 35posterior, 34

cirrhosis, hyponatremia and, 14citalopram, 310clay shoveler’s fracture, 254

clinical vignette of, 269clonidine, hypertensive urgencies and, 115clostridial myonecrosis, 191Clostridium difficile, 132–133Clostridium species, 191Clostridium tetani, 287clozapine, 346cluster headache (HA), 25–26

clinical vignette of, 46CMV. See cytomegalovirusCN. See cyanideCNS. See central nervous system diseaseCO. See carbon monoxide poisoningcocaine, 320–321

intoxication and, 344Combivir, human immunodeficiency virus

and, 196community-acquired acute renal failure, 145community-acquired methicillin-resistant

Staphylococcus aureus (CA-MRSA),cellulitis and, 189

community-acquired pneumonia, clinical vignetteof, 87

compartment syndrome, 266–267complete abortion, 230compression fracture, 254concussion, clinical vignette of, 268congenital heart disease, pediatric, 205–206congestive heart failure (CHF), 93

classifications of, 93hypokalemia and, 17hyponatremia and, 14oxygen and, 94pediatric, 214treatment of, 94

conjunctivitisbacterial, 52

clinical vignette of, 57viral, 51

clinical vignette of, 57contact vulvovaginitis, 236COPD. See chronic obstructive pulmonary

diseasecopious irrigation, clinical vignette of, 57coral snakes, 285cornea

abrasions of, 48–49clinical vignette of, 57

foreign bodies of, 48clinical vignette of, 57

ulcer of, 52–53clinical vignette of, 57

corneal epithelial break, 53corticosteroids, 12Coxiella burnetii, 78CRAO. See central retinal artery occlusionCRF. See chronic renal failurecribriform plate, fracture of, 63cricothyrotomy, 7Crohn’s disease (CD), 131Crotalidae family, 286

croup, 207clinical vignette, 223

CRVO. See central retinal vein occlusioncryptococcal central nervous system infection, 183CSF leak. See cerebrospinal leakCullen’s sign, 130Cushing’s disease, clinical vignette of, 163cutaneous abscesses

clinical features of, 190–191development of, 190organisms and, 190treatment of, 191

CVA. See cerebral vascular accidentCVS. See chronic villus samplingcyanide (CN), 331cyanide antidote kit, 331cyanosis

pathologic, 205physical findings of, 205“Ts” of, 205

cystine, 148cytomegalovirus (CMV), 181

foscarnet and, 181ganciclovir and, 181human immunodeficiency virus and, 183pneumonia, 181retinitis, 181, 183

DDAI. See diffuse axonal injuryD&C. See dilation and curettageDC currents, 274DCS. See decompression sicknessDeBakey Classification, of thoracic aortic

dissection, 112decompression sickness (DCS), 282deep venous thrombosis (DVT)

anticoagulants for, 96clinical features of, 95clinical presentation of, 95clinical vignette of, 116contrast venography, 96D-dimer assay, 96development of, 95duplex ultrasonography, 96MRI, 96patient admission with, 96physical exam for, 95pregnancy and, 234risk factors and, 95treatment goals for, 96

deferoxamine, 303–304delayed neurologic sequelae (DNS), 330delirium, 343delusions, 339dementia, 342dental caries, 74–75dental emergencies, 73–75

alveolar fractures, 74anesthesia and, 73avulsed permanent tooth, 74dental caries, 74–75fractures of, 74infiltration and, 73periapical abscess, 75

Deprenyl, 313depression, 337–338

major, with psychotic features, 340

Index 353

Page 370: Deja review _emergency_medicine__deja_review_

desipramine, 312DI. See diabetes insipidusdiabetes insipidus (DI), 16

clinical vignette of, 163diabetic ketoacidosis (DKA)

alcoholic, 159clinical features of, 158clinical symptoms of, 158clinical vignette of, 163complications of, 159definition of, 158diagnostic tests for, 159ketone bodies, 158laboratory results for, 159management of, 159metabolic derangements with, 158mortality and, 159potassium and, 159precipitating factors of, 158

diabetics, hypoglycemia and, 157diagnostic peritoneal lavage (DPL), 260diaphragmatic herniation, 203

clinical vignette, 222diaphragmatic injury, 258diarrhea

bacteria-induced, 142–143causes of, 141definition of, 141human immunodeficiency virus and, 184invasive bacteria, 142management of, 141parasite-induced, 141toxin-producing bacteria, 142transmission modes, 141viral-induced, 141wet mount of stool, 142

diazepam, 314, 345diazoxide, 306DIC. See disseminated intravascular

coagulationdiffuse axonal injury (DAI), 245

clinical vignette of, 269digital intubation, 7digoxin, 307dilated cardiomyopathy, 98

causes of, 99clinical features of, 99CXR and, 99ECG and, 99echocardiogram, 99management of, 99

dilation and curettage (D&C), 230diltiazem, 310dilutional coagulopathy, 9Dimercaprol, 326diphacinone, 304diphenhydramine, 314diphtheria, pharyngitis and, 66–67direct inguinal hernia, 136disaster

definition of, 3hospital preparedness, 4operation, phases of, 3, 4plan, phases of, 3

disc herniation, lower back pain and, 43dislocations, 263

of hip, 264–265of knee, 265

disseminated intravascular coagulation(DIC), 168

clinical vignette of, 174diuretics, bulimia and, 342diverticula, definition of, 134diverticular disease, complications of, 135diverticulitis

clinical features of, 135clinical vignette of, 144complications of, 135definition of, 135diagnostic tests in, 135management of, 135right lower quadrant pain and, 135

diverticulosisclinical features of, 135management of, 135

Dix-Hallpike maneuver, 37–38DKA. See diabetic ketoacidosisDNS. See delayed neurologic sequelaedog bites, 284dorsal column disorders, 39doxycycline

bactrim with, granuloma inguinale and, 187granuloma inguinale and, 152lymphogranuloma venereum and, 186nongonococcal urethritis/cervicitis and, 185syphilis and, 152, 186

doxycycline/quinine, malaria and, 189D-penicillamine, 325DPL. See diagnostic peritoneal lavagedriving injuries, 281–283drug abuse

amphetamines, 322–323cocaine, 320–321lysergic acid diethylamide, 323–324opioids, 316–317phencyclidine, 322sedatives-hypnotics, 317–318suicide and, 338toxic alcohols, 318–320

duodenal ulcers, 125durethritis, diagnosis of, 151DVT. See deep venous thrombosisdysphagia

anatomical problems from, 120clinical features of, 120definition of, 119management of, 120neuromuscular problems from, 120transfer, 120transport, 120

Eeating disorders, 341–343Eaton-Lambert syndrome, 40–41EBV. See Epstein-Barr viruseclampsia, 232ectopic pregnancy (EP)

clinical features of, 228clinical vignette of, 237diagnosis of, 229differential diagnosis of, 228discriminatory zone, 229methotrexate and, 229risk factors for, 228surgery and, 229syncope and, 44

354 Index

Page 371: Deja review _emergency_medicine__deja_review_

transabdominal ultrasound and, 229ultrasound and, 229

EDTA, 326Effexor, 310Ehrlichiae, 291Elapidae family, 286electrical injuries, 273–274electrolytes

acid-base balance, 21–23hypercalcemia, 19–20hyperchloremia, 21hyperkalemia, 17–18hypermagnesemia, 20–21hypernatremia, 16–17hypocalcemia, 19hypochloremia, 21hypokalemia, 17hypomagnesemia, 20hyponatremia, 13–16

elephantiasis nostra, 189EM. See erythema migransEmergency Medical Dispatcher, 3Emergency Medical Service (EMS)

care provided by, 2development of, 1First National Conference on, 1medical director of, 3service system types, 2systems elements, 1–2training levels, 2–3treatment refusal, 2

emergency medical technician (EMT), nationalregister for, 1

emphysema, 81emphysematous cholecystitis, 128empyema, 82–83EMS. See Emergency Medical ServiceEMT. See emergency medical technicianendocarditis

antibiotic regiment for, 102bacteria entry and, 101causes of, 101clinical features of, 102definition of, 101developmental risk factors of, 101diagnostic tests for, 102infective, 116left-sided, 102management of, 102organisms and, 101pathophysiology of, 101prophylaxis for, 103right-sided, 102

endometrial cancer, clinical vignetteof, 237

endometriosis, 227endorphins, 316endotracheal intubation, 95endotracheal tube (ETT), 5ENT (ears, nose, and throat), infections of

facial, 68–71, 71–73oral, 68–71pharyngitis, 65–68

EP. See ectopic pregnancyepididymitis, 152–153

clinical vignette of, 155epidural hematoma, 246

clinical vignette of, 45, 269

epidural mass, 39epiglottitis, 206–207

clinical vignette, 223epilepsy, 29epinephrine, 12

pediatric cardiopulmonary resuscitationand, 200

epistaxis, 64–65Epstein-Barr virus (EBV), 180, 181erysipelas, 189–190erythema migrans (EM), 292Escherichia coli (enterotoxigenic), 142Escherichia coli serotype O157:H7, 142esophageal atresia, 201–202

clinical vignette, 222esophageal carcinoma, clinical vignette of, 143esophageal perforation, causes of, 121esophageal tracheal combitube (ETC), 5esophagitis

infectious, 120inflammatory, 120management of, 120

esophagusanatomy of, 119dysphagia, 119–120gastroesophageal reflux disease, 120–121odynophagia, 119–120perforation of, 121–122rupture of, 122swallowed foreign body, 122–123

ETC. See esophageal tracheal combitubeethanol, hypoglycemia and, 157ethylene glycol, 318, 319, 320etomidate, 4ETT. See endotracheal tubeexchange transfusion, malaria and, 189extracellular compartment, 12extrinsic nervous systems, esophagus and, 119eye

anisocoria, 47anterior chamber of, 47fundus of, 47herpes simplex virus of, 52herpes zoster ophthalmicus, 52history examination of, 47hyphema, 47, 50hypopyon, 47infections of

conjunctivitis, 51–52corneal ulcer, 52–53hordeolum, 54periorbital/orbital cellulitis, 53–54

limbus, 47posterior chamber of, 47tanopen, 47trauma of, 48–51

blunt injuries, 50–51chemical injuries, 49–50corneal abrasions, 48–49corneal foreign bodies, 48subconjunctival hemorrhage, 49

vitreous humor, 47

Ffacial infections, 71–73

mastoiditis, 72–73sinusitis, 71–72

fall, trauma, death and, 241

Index 355

Page 372: Deja review _emergency_medicine__deja_review_

fasciotomy, pressure reading for, 267FAST exam. See focused abdominal sonography

for traumafebrile seizures, 203

clinical vignette, 223femoral hernia, 136femoral neck/shaft fractures, 265fentanyl, 4fever

human immunodeficiency virus and, 183pediatric, 215

first degree burns, 271clinical vignette for, 293

First National Conference, on Emergency MedicalService, 1

fistula-in-ano, 139Fitz-Hugh-Curtis syndrome, 236flail chest, 255flesh-eating bacteria, 193flu. See influenza virusFluconazole, cryptococcal central nervous system

infection and, 183flumazenil, 317–318fluoroquinolones, 79fluoxetine, 310fluvox, 310fluvoxamine, 310focused abdominal sonography for trauma (FAST

exam), 260folliculitis, 190foreign bodies

anal, 139aspiration

accidental home death and, 208clinical vignette, 223

of cornea, 48clinical vignette of, 57metallic, 48

of nose, 64clinical vignette of, 75

vulvovaginitisclinical vignette of, 238pediatrics, complete airway obstruction and, 199swallowed

clinical features of, 122clinical vignette of, 143common, 122complications of, 122management of, 123nifedipine and, 123passage of, 122, 123proximal impactions of, 122sublingual nitroglycerin and, 123surgical intervention of, 123

vulvovaginitis and, 236foreign bodies, anal, 139foscarnet, cytomegalovirus and, 181Fournier’s gangrene, 152

clinical vignette of, 154fourth degree burns, 272fractures, 263

alveolar, 74antibiotic regiment for, 222of cribriform plate, 63dental emergencies and, 74of femoral neck/shaft, 265of nose, 62–63

clinical vignette of, 75

of pelvis, 265–266splinting, 264of tibial shaft, 265treatment of, 264

Francisella tularensis, 78full renal compensation, 23full thickness (third degree) burns, 272fumarin, 304furosemide, hyperkalemia and, 18furuncle, 190

Ggallbladder disease

acalculous cholecystitis, 128ascending cholangitis, 128calculous cholecystitis, 128definition of, 128gallstone ileus, 128

gallstone ileus, 128ganciclovir, cytomegalovirus and, 181, 183Gardnerella vaginalis, vulvovaginitis and, 235GAS. See A streptococcus necrotizing fasciitis; group

A streptococcalgas gangrene

cause of, 191clinical features of, 192hallmark of, 192hematogenous spread and, 191–192incubation period of, 192nonclostridial, 192presentation of, 192treatment of, 192

gastric decontamination, 297, 320gastric lavage, 298gastric outlet obstruction, peptic ulcer disease

and, 126gastric ulcers, 125gastroesophageal reflux disease (GERD)

causes of, 121clinical features of, 121complications of, 121definition of, 120exacerbation of, 121management of, 121

gastrointestinal bleedingballoon tamponade and, 125causes of, 123diagnostic studies for, 124EGD and, 124epidemiology of, 123laboratory tests for, 124lower, 124management of, 124mortality and, 123octreotide and, 124physical exam and, 124presentation of, 124somatostatin and, 124upper, 123

gastroschisis, 202–203GCA. See giant cell arteritisGCS. See Glasgow coma scoregenital herpes, 179

vulvovaginitis and, 235genital lesions, 185genital warts, 187genitals, male, problems with, infections,

151–153

356 Index

Page 373: Deja review _emergency_medicine__deja_review_

genitourinary traumacauses of, 262cystogram, 262diagnostic tests for, 262extraperitoneal, 263hematuria and, 262hypertension and, 262locations of, 262

GERD. See gastroesophageal reflux diseasegestational trophoblastic disease (GTD), 230–231

clinical vignette of, 237GFR. See glomerular filtration rategiant cell arteritis (GCA), 26–27Glasgow coma score (GCS), 241Glipizide, 306globe, rupture of, 50

clinical vignette of, 57glomerular filtration rate (GFR), 145glyburide, 306glycoprotein IIb/IIIa inhibitors, acute coronary

syndrome and, 91gonococcal urethritis/cervicitis, 185gonorrhea, 66, 185Goodpasture’s syndrome, clinical vignette of, 154granuloma inguinale, 152, 187Grave’s disease, clinical vignette of, 162Greenfield umbrella filter, 96Grey Turner’s sign, 130group A streptococcal (GAS), 67, 68GSW. See gunshot woundsGTD. See gestational trophoblastic diseaseGuillain-Barré syndrome, clinical vignette of, 45gunshot wounds (GSW), trauma, death and, 241

HHACE. See high-altitude cerebral edemaHaemophilus influenzae type b (Hib) epiglottitis, 206hallucinations, 339haloperidol, 346Hampton’s hump, 97hand trauma, 266

clinical vignette for, 294hangman’s fracture, 254HAPE. See high-altitude pulmonary edemaHBO. See hyperbaric oxygenHbS, 169HBV. See hepatitis B virushCG. See human chorionic gonadotropinHCV. See hepatitis C virushead injury

altered level of consciousness and, 243brain regions and, 242intracranial pressure and, 242meninges layers, 242Monroe-Kellie doctrine, 242motor vehicle accident and, 241penetrating, 245scalp layers and, 242

headachescluster, 25–26

clinical vignette of, 46giant cell arteritis, 26–27migraine, 26

clinical vignette of, 46subarachnoid hemorrhage, 27–28temporal arteritis, 26–27

hearing loss, sensorineural, clinical vignetteof, 75

hearing loss, acute, 61–62heat cramps, clinical vignette for, 295heat exhaustion, 279

clinical vignette for, 294heat stroke, 279HEG. See hyperemesis gravidarumHelicobacter pylori, 125, 126Heliox, asthma exacerbation and, 211HELLP syndrome, 233hematoma

epidural, 246subdural, 246–247

hemodialysis, 299hyperkalemia and, 18

hemodynamic monitoring, 8hemophilia, 166

bleeding history of, 166clinical vignette of, 174laboratory findings of, 166

hemoptysis, 81–82hemorrhage. See also hematoma

acute, 9response to, 8–9subarachnoid, 27–28subconjunctival, 49

clinical vignette of, 57hemorrhagic stroke, 35hemorrhoids

clinical features of, 137definition of, 137developmental risk factors of, 137external, 137internal, 137

clinical vignette of, 144management of, 137surgical intervention of, 137

hemostasisbleeding time, 165components of, 165internationalized normalized ratio, 166partial thromboplastin time, 166platelets, 165prothrombin time, 165–166

hemothorax, 255–256heparin

acute coronary syndrome and, 92pulmonary embolism and, 98

hepatitis B virus (HBV), occupational postexpo-sure prophylaxis and, 195

hepatitis C virus (HCV), occupational postexpo-sure prophylaxis and, 195

herniaclinical features of, 136definition of, 136direct inguinal, 136femoral, 136incarcerated, 136indirect inguinal, 136irreducible, 136management of, 136reducible, 136strangulated, 136umbilical, 136

herpes simplex virus (HSV)chickenpox, 179–180encephalitis, 179of eye, 52of finger, 179

Index 357

Page 374: Deja review _emergency_medicine__deja_review_

herpes simplex virus (HSV) (Cont.):genital, 179herpes zoster, 180herpes zoster ophthalmicus, 180immunocompromised patient with, 179ocular, 179oral, 178, 179pathophysiology of, 178pharyngitis and, 65rash appearance, 178recurrence of, 178shingles, 180

herpes simplex virus (HSV) keratitis, clinicalvignette of, 57

herpes zoster, 180herpes zoster ophthalmicus (HZO), 52, 180herpetic whitlow, 179high-altitude cerebral edema (HACE), 281

clinical vignette for, 294high-altitude pulmonary edema

(HAPE), 281clinical vignette for, 293–294

high-flow via nonrebreather mask, 94Highway Safety Act of 1996, 1high-yield pediatric charts

rapid sequence intubation protocol, 198seizures, 198temperature conversion, 197vital signs, 197

hip dislocations, 264–265HIV. See human immunodeficiency virusHIV viral load, 182hordeolum, 54

clinical vignette of, 58Horner’s syndrome, 25hospital-acquired acute renal failure, 145HPV. See human papillomavirushuffing, 328human bites, 284–285human gonadotropin (hCG), 226human immunodeficiency virus (HIV)

acquired immune deficiency syndromeand, 182

anal intercourse and, 196CNS disease and, 183cutaneous conditions of, 184developmental risk factors for, 182diagnosis of, 182diarrhea and, 184emergency department evaluation and, 183fever and, 183intravenous drug abuse and, 183nonnucleoside reverse-transcriptase inhibitors

and, 184nucleoside reverse-transcriptase inhibitors

and, 184occupational postexposure prophylaxis

and, 195opportunistic viral disease and, 183oral/esophageal complaints and, 184percutaneous exposure and, 196Pneumocystis carinii pneumonia and, 184postexposure prophylaxis regiment for, 196presentation of, 182protease inhibitors and, 184pulmonary infections and, 184seroconversion, 182treatment goals for, 184

tuberculosis and, 184vaginal intercourse and, 195

human papillomavirus (HPV), 187Hutchinson’s sign, 180hydatidiform moles, 231hydrocarbons, 327–328hydrocephalus, normal pressure, clinical vignette

of, 46hydrogen sulfide, 332hydroxycourmarin anticoagulants, 304hyperbaric oxygen (HBO)

carbon monoxide poisoning and, 330gas gangrene and, 192

hypercalcemia, 19–20causes of, 19clinical features of, 19clinical vignette of, 174ECG changes of, 20management of, 20secondary to malignancy, 170serum calcium level in, 19

hyperchloremiacauses of, 21management of, 21serum chloride level in, 21

hyperemesis gravidarum (HEG), 231hyperkalemia, 17–18, 267

acute renal failure and, 146causes of, 18clinical features of, 18ECG changes associated with, 18serum potassium level in, 17treatment of, 18

hypermagnesemiaacute renal failure and, 146causes of, 20clinical features of, 20ECG findings from, 20management of, 21serum magnesium level in, 20

hypernatremia, 16–17causes of, 16clinical features of, 16serum sodium level in, 16treatment of, 16urine output and, 16volume-replacement for, 17

hyperosmolality, 13hypersensitivity reaction, 11hypertension

cerebral vascular accident and, 36pregnancy and, 232

hypertensive emergenciesclinical syndromes of, 115clinical vignette of, 116definition of, 115pathophysiology of, 115

hypertensive urgenciescauses of, 115clonidine and, 115definition of, 115labetalol and, 115management of, 115

hyperthermia, 278–280avoidance of, 280causes of, 279complications of, 279cooling and, 280

358 Index

Page 375: Deja review _emergency_medicine__deja_review_

malignant, 279management of, 279risk factors for, 279

hyperthyroidism, 161hypertrophic cardiomyopathy

causes of, 100clinical features of, 100clinical vignette of, 116CXR and, 100ECG and, 100echocardiogram and, 100management of, 101symptoms of, 100syncope and, 44

hyphema, 47, 50hypnotics, 317–318hypocalcemia

acute renal failure and, 146causes of, 19clinical features of, 19management of, 19serum calcium level in, 19

hypochloremiacauses of, 21management of, 21serum chloride level in, 21

hypoglycemiaclinical features of, 157clinical vignette of, 163diabetics and, 157fasting, 157hormone release during, 157management of, 158medications and, 157patient admission with, 158post-prandial, 157sugar blood level of, 157syncope and, 44

hypokalemiacauses of, 17clinical features of, 17ECG changes associated with, 17management of, 17serum potassium level in, 17

hypokalemic myopathy, 39hypomagnesemia

causes of, 20clinical features of, 20ECG findings from, 20management of, 20serum magnesium level in, 20

hyponatremia, 13–16causes of, 14clinical features of, 14complication of, 14correction rate of, 15high plasma osmolality and, 14laboratory tests and, 15management of, 15plasma osmolality in, 15serum sodium level in, 13treatment of, 16

hypo-osmolality, 13hypopyon, 47hypothalamus, 160hypothermia, 277–278

active external rewarming and, 278causes of, 277

classifications of, 277clinical features of, 277clinical vignette for, 293complications from, 277near-drowning and, 276passive external rewarming and, 278pathological response to, 277temperature measurement in, 277

hypothyroidismprimary, 160secondary, 160

hypovolemic shock, 8HZO. See herpes zoster ophthalmicus

Iiatrogenic steroid use, 162ibuprofen, fever and, 215ICP. See intracranial pressureidiopathic thrombocytopenic purpura, clinical

vignette of, 174imipramine, 312immersion foot, clinical vignette for, 293incarcerated hernia, 136

pediatric, 212incomplete abortion, 230indanedione anticoagulants, 304indirect inguinal hernia, 136inevitable abortion, 230infections

ENT (ears, nose, and throat)facial, 71–73oral, 68–71pharyngitis, 65–68

of eyeconjunctivitis, 51–52corneal ulcer, 52–53hordeolum, 54periorbital/orbital cellulitis, 53–54

infectious diseasepediatrics

bacteremia, 215–216meningitis, 217otitis media, 217–218pertussis, 219–220pneumonia, 218–219sepsis, 215–216urinary tract infection, 220–221

pregnancy, antibiotic use in, 196pregnancy categories and, 196

infectious esophagitis, 120infectious mononucleosis, 65–66, 180–181infective endocarditis, 183

clinical vignette of, 116inflammatory esophagitis, 120

clinical vignette of, 143influenza virus

amantadine and, 178antigenic shift, 177clinical features of, 177complications of, 177oseltamivir and, 178pathogenicity of, 177rimantadine and, 178seasons, 177time course of, 177vaccine, 178zanamivir and, 178

inorganic salts, 325

Index 359

Page 376: Deja review _emergency_medicine__deja_review_

INR. See internationalized normalized ratioinsect bites, 289–293

spider, 289–291tick, 291–293

insulin/glucose, hyperkalemia and, 18internationalized normalized ratio (INR), 166intestinal obstruction, pediatric, 214

clinical vignette, 224intoxication, 344–345

alcohol, 344psychostimulant, 344

intracellular compartment, 12intracranial pressure (ICP), head injury

and, 242intravenous drug abuse (IVDA), human immun-

odeficiency virus and, 183intubations, pediatric, 198intussusception

clinical vignette, 223pediatric, 213

invasive bacteria, 142ipecac, 298

bulimia and, 342iron, overdose of, 302irreducible hernia, 136ischemic stroke, 35isocarboxazid, 313isopropyl alcohol, 319Isotonic crystalloid fluids, resuscitation and, 9ITP, 167–168IVDA. See intravenous drug abuse

JJaneway lesions, 102Jefferson fracture, 253

clinical vignette of, 269Jones criteria, 68jugular venous distention (JVD), 105JVD. See jugular venous distention

KKaposi’s sarcoma, 184Kayexalate, hyperkalemia and, 18ketamine, 4

asthma exacerbation and, 211kidney stones

abdominal aortic aneurysms and, 114calcium oxalate, 148

kidneysacid excretion and, 21function, assessment of, 145physiology points about, 145

Klebsiella pneumoniae, 77knee dislocations, 265

Llabetalol, 308

hypertensive urgencies and, 115lactated ringers (LR), 9lactic acidosis, 22Lamivudine, human immunodeficiency virus

and, 196large-bore IVs, 9laryngeal mask airway (LMA), 5laxatives, bulimia and, 342LBP. See lower back painlead, 326left ventricular failure

causes of, 93clinical features of, 93

Legionella pneumophila, 78leiomyomas (fibroids), 228LES. See lower esophagus sphincterleukotriene modifiers, asthma exacerbation

and, 211LGV. See lymphogranuloma venereumlidocaine, pediatric cardiopulmonary resuscitation

and, 200lightening strike, 274lighting injuries, 274–275limbus, 47lithium, 315–316LMA. See laryngeal mask airwayLMWH. See low-molecular-weight heparinlocal tetanus, 288long narrow IVs, 9lorazepam, 345lower back pain (LBP), 41–43

cauda equina syndrome and, 43cause of, 42disc herniation and, 43management of, 42metastasis and, 42nerve root involvement and, 42resolution of, 42sciatica, 42spinal epidural abscess and, 43straight leg raising, 42vertebral fracture and, 43

lower esophagus sphincter (LES), 119lower gastrointestinal bleeding, 124low-molecular-weight heparin (LMWH), 305

pulmonary embolism and, 98LR. See lactated ringersLSD. See lysergic acid diethylamideLudwig’s angina, 69lumbar puncture

for herpes simplex virus encephalitis, 179meningitis and, 204subarachnoid hemorrhage and, 28

lungsabscess of, 83–84

clinical vignette of, 87acid excretion and, 21

Lyme disease, 291, 292, 293clinical vignette for, 294

lymphogranuloma venereum (LGV), 186lysergic acid diethylamide (LSD), 323–324

Mmacrolide, chancroid and, 152magnesium sulfate, asthma exacerbation and, 211major depressive disorder, 337malaria

cerebral, 188clinical features of, 188complications of, 188diagnosis of, 188exchange transfusion and, 189laboratory findings of, 188species responsible for, 187transmission of, 187treatment of, 188–189

malignant hyperthermia, 279malignant pericardial effusion, 173

clinical vignette of, 175

360 Index

Page 377: Deja review _emergency_medicine__deja_review_

Mallory-Weiss syndrome, 121clinical vignette of, 144upper gastrointestinal bleeding and, 123

mandibledislocation of, 249fractures of, 248–249

mangled severity scoring system (MSSS), 264mania, 340manic episode, 340MAOIs. See monoamine oxidase inhibitorsMarplan, 313masticator space abscess, 69mastoiditis, 72–73

clinical vignette of, 76maxillary branch, nerves of, 73maxillary fractures, 250mechanical valves, 111mechanical ventilation, pediatric, 199Meckel’s diverticulum, pediatric, 213–214

clinical vignette, 224meconium, 201medications

prescriptionanticoagulants, 304–305beta-blockers, 308–309calcium channel blockers, 309–310cardiac glycosides, 307–308oral hypoglycemics, 305–307

psychiatric, 310–313monoamine oxidase inhibitors, 313selective serotonin reuptake inhibitors,

310–311tricyclics antidepressants, 312

meningitis, 203–204bacterial, clinical vignette of, 46causes of, 32cerebrospinal fluid, 31chemoprophylaxis and, 33clinical features of, 32clinical vignette, 224diagnostic tests for, 32host factors predisposing to, 32management of, 32pediatric, 217risk factors for, 32steroids and, 33symptoms of, 32treatment of, 33

mental status examination (MSE), of psychiatricpatients, 336–337

merozoites, 188mesenteric ischemia

causes of, 133–134clinical features of, 134clinical vignette of, 144diagnostic tests of, 134management of, 134pathophysiology of, 133surgical intervention in, 134

metabolic acidosis, 22acute renal failure and, 146

metabolic alkalosis, 22, 23metals, 324metastasis, lower back pain and, 42metformin, 307methamphetamine, intoxication and, 344methanol, 318methemoglobin, 328–329

methotrexate, ectopic pregnancy and, 229methylene blue, 329metoprolol, 308MI. See myocardial infarctionmidazolam, 4midface injuries, 249migraine, 26

clinical vignette of, 46missed abortion, 230mitral stenosis, 107mitral valve prolapse, 109MONA (morphine, oxygen, nitro, aspirin), 91monoamine oxidase inhibitors (MAOIs), 313, 348mononucleosis, clinical vignette of, 75Monroe-Kellie doctrine, 242morphine, acute coronary syndrome and, 92morphine, oxygen, nitro, aspirin (MONA), 91motor vehicle accident (MVA)

head injury and, 241trauma, death and, 241

MS. See multiple sclerosisMSE. See mental status examinationMSSS. See mangled severity scoring systemmultiple sclerosis (MS), 39

clinical vignette of, 45Murphy’s sign, 128musculoskeletal (fourth degree) burns, 272MVA. See motor vehicle accidentmyasthenia gravis, 40myasthenic crisis, 40Mycoplasma pneumonia, 79myelopathies, 39myocardial concussion, 257

clinical vignette of, 270myocardial contusion, 258myocardial infarction (MI), 90

anterior, 91ECG and, 90inferior, 90lateral, 91right ventricular, 91

myocarditiscardiac enzymes and, 104causes of, 103clinical features of, 104clinical vignette of, 117CXR and, 103definition of, 103ECG and, 103echocardiography and, 103endomyocardial biopsy and, 104management of, 104

myopathies, 39myxedema coma, 159–160

clinical vignette of, 163

NNAC. See N-acetylcysteineN-acetylcysteine (NAC), 300N-acetyl-para-benzoquinoneimine (NAPQI),

299–300naloxone, 317NAPQI. See N-acetyl-para-benzoquinoneimineNarcan, 317Nardil, 313nasal airway, 5National Institutes of Heart (NIH) stroke

scale, 33–34

Index 361

Page 378: Deja review _emergency_medicine__deja_review_

near-drowning, 276–277neck trauma, 247–248necrotizing cellulitis, 193necrotizing enterocolitis, 202

clinical vignette, 222necrotizing fascitis, 193needle sticks, 195nefazodone, 311neglect, 221neonatal tetanus, 288nephrolithiasis

clinical features of, 148clinical vignette of, 154definition of, 148diagnostic tests for, 149laboratory tests for, 148–149management of, 149patient admission with, 149

nephrotoxic agents, 146neuroleptic malignant syndrome (NMS), 279, 314,

346–347neuroleptics, 314, 346neuromuscular junction, disorders of, 40–41neuropathies, 41Nexus criteria, 253NIDDM. See non-insulin-dependent diabetes

mellitusnifedipine, 310

hypertensive urgencies and, 115swallowed foreign body and, 123

911, 3nitroglycerin, acute coronary syndrome and, 92nitroprusside, 331NMS. See neuroleptic malignant syndromenonclostridial gas gangrene, 192non-cyanotic congenital heart defects, 206nongonococcal urethritis/cervicitis, 185nonhemorrhagic hypovolemia, 8non-insulin-dependent diabetes mellitus

(NIDDM), 306noninvasive positive pressure ventilation (NPPV),

80, 94non-ST-elevation MI (NSTEMI), clinical vignette

of, 116normal saline (NS), 9nortriptyline, 312nose

epistaxis, 64–65foreign bodies of, 63

clinical vignette of, 75fracture of, 62–63

clinical vignette of, 75saddle-nose deformity, 63septal hematoma of, 63trauma to, 62–63

nosebleeds, 64–65clinical vignette of, 76

NPPV. See noninvasive positive pressureventilation

NS. See normal salineNSTEMI. See non-ST-elevation MI

Ooccult bacteremia, pediatric, 216occupational exposure prophylaxis, 195–196octreotide, 306

gastrointestinal bleeding and, 124ocular herpes simplex virus, 179

odontoid fracture, 253–254odynophagia

causes of, 120clinical features of, 120definition of, 119

olanzapine, 346omphalocele, 202–203open pneumothorax, 255open reduction internal fixation (ORIF), femoral

neck/shaft fractures and, 265ophthalmic branch, nerves of, 73opioids, 316–317optic neuritis, 56oral airway, 5oral hypoglycemics, 305–307oral infections, 68–71

abscess and, 68masticator space abscess, 69peripharyngeal abscess and, 71peritonsillar abscess, 71prevertebral abscess, 70retropharyngeal abscess, 69–70

oral-maxillofacial fractures and, cerebrospinal(CSF) leak, 249

orbital blowout fracture, 51, 250orbital cellulitis, 53–54orbital floor fracture, clinical vignette of, 270orchitis

causes of, 151clinical features of, 151management of, 151mumps-induced, 151

ORIF. See open reduction internal fixationorogenital sex, 66orthopedic trauma, 263–267

compartment syndrome, 266–267dislocation, 263femoral neck/shaft fractures, 265fracture, 263hand trauma, 266hip dislocations, 264–265knee dislocations, 265nerve injury, 263pelvic fractures, 265–266physical exam and, 264subluxation, 263tibial shaft fractures, 265

oseltamivir, influenza virus and, 178Osler’s nodes, 102osmolal gap, 13osmolality, 13osmosis, 12otitis externa, 60–61

clinical vignette of, 76otitis media, pediatric, 217–218

clinical vignette, 223ovarian cysts, 227overdose

of acetaminophen, 300of aspirin, 302of cardiac glycosides, 307–308of iron, 302of opioids, 316–317

over-the-counter drugs, 299–304acetaminophen, 299–301iron, 302–304salicylates, 301–302

oxalate, 319

362 Index

Page 379: Deja review _emergency_medicine__deja_review_

Ppancreatitis

causes of, 129chronic, 130clinical features of, 130clinical vignette of, 144complications of, 130definition of, 129diagnostic tests for, 130

panic attacks, 341paramedic, 3paraphimosis, 152parasite-induced diarrhea, 141Parkland’s formula, 272Parnate, 313paroxetine, 310partial thickness (second degree) burns, 271partial thromboplastin time (PTT), 166passive external rewarming, 278Pasteurella multocida, 283Paxil, 310PCP. See phencyclidine; Pneumocystis carinii

pneumoniaPE. See pulmonary embolismPeak expiratory flow rate (PEFR), 210pediatric(s)

appendicitis in, 211breathing rate, 199cardiac arrest, 200congenital heart disease, 205–206congestive heart failure in, 214incarcerated hernia, 212intestinal obstruction, 214intussusception, 213mechanical ventilation, 199Meckel’s diverticulum, 213–214meningitis, 217otitis media, 217–218pertussis, 219–220pneumonia, 218–219pyloric stenosis, 212urinary tract infection, 220–221volvulus, 214

pediatric cardiopulmonary resuscitationcricoid ring and, 199fetal, 198intrapartum, 198intubations, 198larynx and, 199maternal, 198mechanical ventilation, 199percutaneous transtracheal ventilation, 199trachea and, 199tracheal medication administration and, 200tracheal tube size and, 199vascular access and, 199–200

pediatric charts, high-yieldrapid sequence intubation protocol, 198seizures, 198temperature conversion, 197vital signs, 197

PEFR. See peak expiratory flow ratepelvic fractures, 265–266pelvic inflammatory disease (PID)

causes of, 236clinical features of, 236complications of, 236definition of, 236

diagnostic criteria for, 236management of, 237pathophysiology of, 236patient admission with, 237pelvic exam findings in, 236risk factors of, 236

pelvic/abdominal pain, 227–228penetrating head injuries, 245penetrating trauma, 241penicillin, 12

syphilis and, 152, 186penile ulcers, causes of, 152penis

blood supply of, 150cylindrical bodies of, 150

pentamidine isethionate, Pneumocystis cariniipneumonia and, 184

peptic ulcer disease (PUD)causes of, 125clinical features of, 125clinical vignette of, 144complications of, 126definition of, 125developmental factors of, 125diagnostic tests for, 126gastric outlet obstruction and, 126management of, 126perforation and, 126upper gastrointestinal bleeding and, 123upper GI bleeding and, 126

percutaneous transtracheal ventilation, 199perforation

appendicitis and, 127peptic ulcer disease, 126

perianal abscess, 191periapical abscess, 75pericardial disease

cardiac enzymes, 105causes of, 104CBC and, 105clinical features of, 104clinical vignette of, 117complications of, 105CXR and, 105ECG and, 105ECG findings for, 104echocardiography and, 105ESR/C-protein, 105management of, 105patient admission with, 105pericardial friction rub and, 104presentation of, 104

pericardial friction rub, 104pericardial tamponade, clinical vignette of, 269pericarditis. See pericardial diseaseperiorbital cellulitis, 53

clinical vignette of, 58peripartum cardiomyopathy, pregnancy

and, 235peripharyngeal abscess, 71peripheral diabetes insipidus, 16peripheral neurologic lesions

motor function, 38muscle function, 38myelopathies, 39myopathies, 39neuromuscular junction, 40–41neuropathies, 41

Index 363

Page 380: Deja review _emergency_medicine__deja_review_

peritonsillar abscess, 71pertussis, pediatric, 219–220pesticides, 332–333pharyngitis, 65–68

bacterial, 67causes of, 65clinical features of, 65definition of, 65diphtheria and, 66–67herpes simplex virus and, 65immunocompromised patients and, 68streptococcal, clinical vignette of, 76

phencyclidine (PCP), 322, 340phenelzine, 313phenobarbital, neonatal seizures and, 204phenylpropanolamine, intoxication and, 344phenytoin, neonatal seizures and, 204phimosis, 152physical abuse, 221, 222PID. See pelvic inflammatory diseasepilonidal sinus, 141Pindone, 304placenta previa, 233placental abruption, 233plasma, circulating volume of, 9plaster of paris, 264platelet disorders

disseminated intravascular coagulation, 168ITP and, 167–168thrombocytopenia, 167thrombocytopenic purpura, 167, 168

pleural effusion, 82–83clinical vignette of, 86

plexuses, esophagus and, 119PNA. See pneumoniaPneumocystis carinii pneumonia (PCP), 78

human immunodeficiency virus and, 184pneumonia (PNA)

atypical, 79bacterial, 77–78community-acquired, clinical vignette

of, 87Mycoplasma, 79pediatric, 218–219

pneumothoraxopen, 255spontaneous, 85–86

clinical vignette of, 87tension, 255

poisoning, 297polymyositis, 39postherpetic neuralgia, 180postpartum hemorrhage, pregnancy and, 234postpartum infection, pregnancy and, 235post-prandial hypoglycemia, 157poststreptococcal glomerulonephritis, clinical

vignette of, 154PPD test. See purified protein derivative testpralidoxime, 333preeclampsia, 232

clinical vignette of, 237pregnancy

abruptio placentae, 233amniotic fluid embolus, 235cardiovascular system and, 225deep venous thrombosis during, 234dermatologic system and, 226ectopic

clinical features of, 228diagnosis of, 229differential diagnosis of, 228discriminatory zone, 229methotrexate and, 229risk factors for, 228surgery and, 229transabdominal ultrasound and, 229ultrasound and, 229

endocrine system and, 225first trimester bleeding and, 230gastrointestinal system and, 225genitourinary system and, 225gestational trophoblastic disease, 230–231hematology system and, 225hepatitis B virus and, 195hyperemesis gravidarum, 231hypertensive emergencies during, 232–233infectious disease and

antibiotic use in, 196categories of, 196

peripartum cardiomyopathy, 235placenta previa, 233postpartum emergencies, 234–235postpartum hemorrhage, 234postpartum infection, 235premature rupture of membranes,

233–234preterm labor, 234pulmonary embolism and, 234respiratory system and, 225spontaneous abortion and, 230test, 228–229

positive, 226trauma in, 225–226uterus and, 226

pregnancy-related death, 229–230Prehn’s sign, 153premature rupture of membranes (PROM),

233–234prerenal azotemia, 145, 146prescription medications

anticoagulants, 304–305beta-blockers, 308–309calcium channel blockers, 309–310cardiac glycosides, 307–308oral hypoglycemics, 305–307

preterm labor (PTL), 234prevertebral abscess, 70primaquine, 188primary adrenal insufficiency, 162primary polydipsia, 14proctitis

causes of, 140clinical features of, 140clinical vignette of, 144complications of, 140definition of, 140management of, 140

proctosigmoidoscopy, 140PROM. See premature rupture of membranespropanolol, 309propofol, 4prosthetic valves

bioprosthetic, 111complications of, 111mechanical, 111types of, 111

364 Index

Page 381: Deja review _emergency_medicine__deja_review_

proton pump inhibitor, upper gastrointestinalbleeding and, 125

Prozac, 310pseudomembranous enterocolitis

antidiarrheal drugs and, 133clinical vignette of, 144Clostridium difficile, 132definition of, 132management of, 133

psychiatric patientsapproach to, 335characteristics of, 336chemical restraint, 336emergent, 335evaluation of, 335laboratory tests for, 336medical history of, 336mental status examination of, 336–337nonurgent, 335physical restraint, 336restraints, 335, 336seclusion and, 335, 336urgent, 335verbal restraint, 336

psychopharmacology, 345–348psychostimulants, 340PTL. See preterm laborPTT. See partial thromboplastin timePUD. See peptic ulcer diseasepulmonary contusion, 258

clinical vignette of, 270pulmonary embolism (PE)

anticoagulants for, 98clinical features of, 97clinical vignette of, 117development risk factors of, 96ECG findings for, 97epidemiology of, 96heparin and, 98low-molecular-weight heparin and, 98management of, 98pregnancy and, 234

clinical vignette of, 238pulmonary angiography, 98radiographic abnormalities in, 97screening tests for, 97source of, 96spiral CT angiography, 97symptom of, 97thrombolytics and, 98treatment goals for, 98triad of, 97ventilation-perfusion scan, 98

pupil, dilation of, 51purging, 342purified protein derivative (PPD) test, 84–85pyelonephritis

classification of, 150clinical features of, 150clinical vignette of, 154complications of, 150definition of, 150developmental risk factors of, 150management of, 150patient admission with, 150

pyloric stenosis, pediatric, 212clinical vignette, 223–224

pyuria, 150

Qquetiapine, 346

Rrabies, 286–287

clinical vignette for, 294racemic epinephrine aerosol, croup and, 207raised hypothalamic set point, 215Ranson’s criteria, 130rapid sequence intubation (RSI), 6rapid sequence intubation protocol, in pediatrics, 198rattlesnakes, 285rectal prolapse, 140rectum, exam of, 151red blood cells, circulating volume of, 9Red Desaturation Test, 56reducible hernia, 136renal azotemia, 146renal infarct, clinical vignette of, 153respiratory acidosis, 22, 23respiratory alkalosis, 22, 23restraints, for psychiatric patients, 335, 336restrictive cardiomyopathy

causes of, 100clinical features of, 100CXR and, 100ECG and, 100echocardiogram and, 100management of, 100

resuscitationgas gangrene and, 192lactated ringers and, 9normal saline and, 9pediatric, 201

retrograde intubation, 7retropharyngeal abscess, 69–70, 208

clinical vignette, 223clinical vignette of, 76

Reye’s syndrome, 215–216rhabdomyolysis, 267rheumatic fever, 68

clinical vignette of, 76Rickettsia Rickettsii, 291rifampin

endocarditis and, 102meningitis and, 33

right ventricular failurecauses of, 94physical exam findings of, 94

rimantadine, influenza virus and, 178Rinne test, 62RIPE (rifampin, isoniazid, pyrazinamide,

ethambutol), 85RMSF. See Rocky Mountain spotted feverRocky Mountain spotted fever (RMSF), 291–292

clinical vignette for, 293rocuronium, 4, 7root, of tooth, 73, 74RSI. See rapid sequence intubationrule of nine, 271Rumack-Matthew normogram, 300rust rings, metallic foreign bodies and, 48

Ssaddle-nose deformity, 63SAH. See subarachnoid hemorrhagesalicylates. See also aspirin

hypoglycemia and, 157

Index 365

Page 382: Deja review _emergency_medicine__deja_review_

salmonella, 143schizoaffective disorder, 339schizophrenia, 339schizophreniform disorder, 339sciatica, 42SCIWORA (spinal cord injury without radiograph-

ic abnormality), 252scopolamine, 49SE. See status epilepticussecond degree burns, 271

clinical vignette for, 293secondary adrenal insufficiency, 162sedatives, 317–318seizures

absence, clinical vignette of, 46causes of, 29complex, 29–30definition of, 28differential diagnosis of, 30epilepsy, 29febrile, 203generalized, 29management of, 30neonatal, 203, 204partial, 29in pediatrics, 198physical exam and, 30recurrence of, 29status epilepticus, 31subtherapeutic anticonvulsant level and, 31syncope and, 44tonic-clonic, 204

clinical vignette of, 46selective serotonin reuptake inhibitors (SSRIs),

310–311, 347selegiline, 313self-induced emesis, bulimia and, 342Sellick maneuver, 6semipermeable membrane, 13sepsis, 9

clinical features of, 10management of, 10in newborn, 202pediatric, 216

septal hematoma, of nose, 63septal myomectomy, 101septic shock, 10serosa, lack of, 119serotonin syndrome, 311, 348–349sertraline, 310serum calcium level

in hypercalcemia, 19in hypocalcemia, 19

serum chloride levelin hyperchloremia, 21in hypochloremia, 21

serum magnesium levelin hypermagnesemia, 20in hypomagnesemia, 20

serum osmolality, normal, 13serum potassium level

in hyperkalemia, 17in hypokalemia, 17

serum sodium levelin hypernatremia, 16in hyponatremia, 13

sex, orogenital, 66sexual abuse, 66, 221

sexually transmitted diseases (STDs), patientevaluation of, 185

shaken baby syndrome, 222shigella, 143shingles, 180shock

autonomic responses and, 7, 8categories of, 7definition of, 7hemodynamic monitoring and, 8hypovolemic, 8metabolic derangements and, 8trauma and, 240vasoactive hormone, release of, 8

SIADH. See syndrome of inappropriate ADHsecretion

sickle cell anemia, 169sickle cell trait, 169silent suicide, 338simple pneumothorax, clinical vignette of, 269simple triage and rapid treatment (START)

method, 4sinusitis, 71–72

clinical vignette of, 76skull fractures, 245–246slit-lamp exam, corneal foreign bodies and, 48SLUDGE (salivation, lacrimation, urination, diarrhea,

GI cramps, emesis) syndrome, 332snakes bites, 285–286sniffing, 327sodium bicarbonate, hyperkalemia and, 18sodium channel blockage, 312soft tissue infections

cellulitis, 189–190cutaneous abscesses, 190–191

somatostatin, gastrointestinal bleeding and, 124spider bites, 289–291spinal column, 251–252

composition of, 251spinal cord compression, 171–172

clinical vignette of, 174spinal cord injury

burst fracture, 254causes of, 238cervical, 252, 253cervical spine and, 251chance fracture, 254clay shoveler’s fracture, 254complete, 238compression fracture, 254CT of, 253, 254flexion-extension film, 253hangman’s fracture, 254incomplete, 238Jefferson fracture, 253management of, 254mechanisms of, 251MRI of, 253, 254Nexus criteria, 253odontoid fracture, 253–254tenderness and, 238thoracolumbar, 254treatment of, 251

spinal epidural abscess, lower back pain and, 43splinting, 264spontaneous abortion, 230spontaneous pneumothorax, 85–86

clinical vignette of, 87

366 Index

Page 383: Deja review _emergency_medicine__deja_review_

sporozoites, 188sports, traumatic brain injury and, 244SSRIs. See selective serotonin reuptake inhibitorsstable angina, 89Stanford classification, of thoracic aortic

dissection, 112staphylococcus aureus, 142START. See simple triage and rapid treatment

methodstatus epilepticus (SE), 31STDs. See sexually transmitted diseasesST-elevation MI (STEMI), 89, 90, 92

clinical vignette of, 117STEMI. See ST-elevation MIsternotomy, 254steroid myopathy, 39steroids, 340

asthma exacerbation and, 211meningitis and, 33

strangulated hernia, 136streptococcal toxic shock syndrome, 194stress ulcers, 125struvite stone, 148stye. See hordeolumsubarachnoid hemorrhage (SAH), 27–28subconjunctival hemorrhage, 49

clinical vignette of, 57subdural hematoma, 246–247

clinical vignette of, 268sublingual nitroglycerin, swallowed foreign body

and, 123subluxation, 263Succimer, 325, 326succinylcholine, 4, 6, 7suicide, 337–338sulfonylureas, 306superficial (first degree) burns, 271superior vena cava syndrome (SVCS), 172

clinical vignette of, 175supine hypotension syndrome, 268surgical debridement, gas gangrene and, 192SVCS. See superior vena cava syndromeswallowed foreign body, clinical vignette of, 143swimmer’s ear. See otitis externasyncope

abdominal aortic aneurysm and, 44causes of, 43definition of, 43ectopic pregnancy and, 44evaluation of, 44hypertrophic cardiomyopathy and, 44hypoglycemia and, 44patient admission with, 44physical exam and, 44seizure and, 44vasovagal and, 44

syndrome of inappropriate ADH secretion(SIADH), 14, 171

syphilis, 152, 185, 186syringomyelia, 39systemic inflammatory response syndrome, 10

TTAD. See thoracic aortic dissectiontanopen, 47tarantulas, 289TB. See tuberculosisTBI. See traumatic brain injury

TBSA. See total body surface areaTCAs. See tricyclics antidepressantsteeth. See dental emergenciestemperature conversion, pediatric, 197temporal arteritis, 26–27tension pneumothorax, 255

clinical vignette of, 270testicular torsion, 153

clinical vignette of, 154testis

investing layers of, 151size of, 150

tetanus, 287–289generalized, clinical vignette for, 294

tetracyclinesyphilis and, 152tick bites and, 292

tetralogy of Fallot, 205third degree burns, 272thoracic aortic dissection (TAD)

chest pain and, 112classification of, 112clinical vignette of, 117CXR and, 113Debakey classification of, 112development risk factors for, 112diagnostic studies for, 113dissection propagation and, 112, 113ECG and, 113epidemiology of, 111management of, 113mortality rate of, 112pathophysiology of, 112physical findings and, 113silent, 113Stanford classification of, 112treatment of, 114

thoracic trauma, 254–258blunt cardiac injury, 257–258cardiac tamponade, 256diaphragmatic injury, 258flail chest, 256hemothorax, 255–256open pneumothorax, 255pulmonary contusion, 258tension pneumothorax, 255traumatic aortic rupture, 257

thoracotomy, 254threatened abortion, 230

clinical vignette of, 237thrombocytopenia, 167thrombocytopenic purpura (TTP), 167, 168thrombolytics

in acute myocardial infarction, 92pulmonary embolism and, 98

thyroidhypothyroid, 159–160myxedema coma, 159–160storm, 161

TIA. See transient ischemic attacktibial shaft fractures, 265tick bites, 291–293

chronic disease, 292disseminated disease, 292localized disease, 292

timolol, 308TLS. See tumor lysis syndromeTMP-SMX. See trimethoprim-sulfamethoxazole

Index 367

Page 384: Deja review _emergency_medicine__deja_review_

Todd paralysis, 30Tolazamide, 306tonic-clonic seizure, clinical vignette of, 46tonic-clonic seizures, 204TORCHS (toxoplasmosis, rubella, cytomegalovirus,

herpes, syphilis) infections, 204total body surface area (TBSA), 272toxic shock syndrome (TSS)

clinical features of, 194management of, 194risk groups of, 194streptococcal, 194

toxicology supplement, 333–334toxin-producing bacteria, 142toxoplasmosis, acquired immune deficiency syn-

drome and, 183trachea, cardiopulmonary resuscitation and,

199–200tracheoesophageal fistula, 202tracheotomy, 7transfer dysphagia, 120transient hypertension, pregnancy and, 232transient ischemic attack (TIA), 35transport dysphagia, 120transportation, patient

methods of, 2refusal of, 2

tranylcypromine, 313trauma, 239. See also specific trauma

of abdomen, 259blunt, 241colloid fluids, 240crystalloid fluids, 240endotracheal tube placement and, 239fall and, 241genitourinary, 262–263gunshot wounds and, 241intervention of, 239life-threatening bleeding and, 241motor vehicle accident and, 241needle cricothyrotomy and, 240neurological function and, 241orthopedic, 263–267peak times for, 239penetrating, 241in pregnancy, 225–226primary survey, 239resuscitation and, 240secure airway and, 239, 240shock and, 240volume status, 240

traumatic aortic rupture, 257clinical vignette of, 269

traumatic brain injury (TBI)anticonvulsant prophylaxis and, 244categories of, 243cerebral concussion, 244cerebral contusion, 244diagnostic tests for, 243diffuse axonal injury, 245Glasgow coma score and, 241loss of consciousness and, 243management of, 243neuroassessment and, 244physical exam and, 243posttraumatic seizure and, 243–244sports and, 244

trazodone, 311

trench foot, clinical vignette for, 293triage

categories of, 4definition of, 4

Trichomonas vaginalis, vulvovaginitis and, 235tricuspid regurgitation

causes of, 106clinical features of, 106CXR and, 106ECG and, 107management of, 107

tricuspid stenosiscauses of, 106clinical features of, 106CXR and, 106dysrhythmias and, 106ECG and, 106management of, 106

tricyclics antidepressants (TCAs), 312, 347trimethoprim-sulfamethoxazole (TMP-SMX)

granuloma inguinale and, 152human immunodeficiency virus and, 183Pneumocystis carinii pneumonia and, 184

Trousseau’s sign, 19TSS. See toxic shock syndromeTTP. See thrombocytopenic purpuratuberculosis (TB), 84–85

clinical vignette of, 87human immunodeficiency virus and, 184RIPE, 85

tumor lysis syndrome (TLS), 170clinical vignette of, 174

UUC. See ulcerative colitisUES. See upper esophagus sphincterUFH. See unfractionated heparinulcerative colitis (UC), 132umbilical hernia, 136unfractionated heparin (UFH), 305unstable angina, 89

clinical vignette of, 117upper esophagus sphincter (UES), 119upper gastrointestinal bleeding, 123

esophageal varix, 123gastric varix, 123Mallory-Weiss syndrome and, 123peptic ulcer disease and, 123, 126proton pump inhibitor and, 125

urethra, injuries to, 263urethritis, 151uric acid stone, 148urinary alkalinization, 299urinary tract infection (UTI)

clinical features of, 149clinical vignette of, 153complications of, 149definition of, 149differential diagnosis of, 149management of, 150organisms associated with, 149pediatric, 220–221pyuria and, 150

urine, sample of, 150urine osmolality, 15urine sodium concentration, 15uterine atony, clinical vignette of, 237UTI. See urinary tract infection

368 Index

Page 385: Deja review _emergency_medicine__deja_review_

Vvaginal bleeding

causes of, 227physical exam and, 227sexual history and, 226types of, 226

Valone, 304valvular disease

acute aortic regurgitation, 110–111acute mitral regurgitation, 108aortic stenosis, 109–110chronic aortic regurgitation, 110chronic mitral regurgitation, 107–108mitral stenosis, 107mitral valve prolapse, 108–109prosthetic valves, 111tricuspid regurgitation, 106–107tricuspid stenosis, 106

vancomycin, endocarditis and, 102varicella-zoster virus (VZV), 180vaso-occlusive crisis, clinical vignette of, 174vasovagal, syncope and, 44vecuronium, 4, 7venlafaxine, 310ventricular septal defect (VSD), 206verapamil, 310vertebral fracture, lower back pain and, 43vertigo

central, 37, 38clinical vignette of, 46

definition of, 37peripheral, 37

vibrio cholera, 142vibrio parahaemolyticus, 142viral encephalitis, 179viral-induced diarrhea, 141Virchow’s triad, 125viscerocutaneous loxoscelism, 290visual loss, acute

acute angle-closure glaucoma, 55–56central retinal artery occlusion, 54–55central retinal vein occlusion, 55optic neuritis, 56

vital signs, in pediatrics, 197vitamin K, administration of, 305volvulus, pediatrics, 214

von Willebrand’s disease, clinical vignetteof, 174

VSD. See ventricular septal defectvulvovaginitis

candida albicans, 235clinical features of, 235contact, 236definition of, 235differential diagnosis of, 235foreign body, 236Gardnerella vaginalis, 235genital herpes, 235Trichomonas vaginalis, 235

vWF, 166–167VZV. See varicella-zoster virus

Wwarfarin, 304water

body weight and, 12loss, 13, 16regulation, 12–13

Waterhouse-Friderichsen syndrome, 162Weber’s test, 62Wernicke’s encephalopathy, clinical vignette

of, 46Westermark’s sign, 97wet mount of stool, 142Whipple’s triad, 157whole body irrigation, 299whole bowel irrigation, 299whooping cough. See pertussiswithdrawal, 344–345

Xxanthine stone, 148xanthochromia, 28

Zzanamivir, influenza virus and, 178zidovudine, human immunodeficiency virus

and, 196Zoloft, 310zolpidem, 317zygoma, 250zygomaticomaxillary (ZMC) fractures, 250

Index 369